Как пСрСвСсти Π΄Π΅Ρ†ΠΈΠ±Π΅Π»Ρ‹ Π² Ρ€Π°Π·Ρ‹: Π§Ρ‚ΠΎ Ρ‚Π°ΠΊΠΎΠ΅ Π΄Π΅Ρ†ΠΈΠ±Π΅Π» (Π΄Π‘)? ΠŸΠ΅Ρ€Π΅Π²ΠΎΠ΄ ΠΈΠ· Π΄Π΅Ρ†ΠΈΠ±Π΅Π» Π² Ρ€Π°Π·Ρ‹.

Π‘ΠΎΠ΄Π΅Ρ€ΠΆΠ°Π½ΠΈΠ΅

Π§Ρ‚ΠΎ Ρ‚Π°ΠΊΠΎΠ΅ Π΄Π΅Ρ†ΠΈΠ±Π΅Π» (Π΄Π‘)? ΠŸΠ΅Ρ€Π΅Π²ΠΎΠ΄ ΠΈΠ· Π΄Π΅Ρ†ΠΈΠ±Π΅Π» Π² Ρ€Π°Π·Ρ‹.

ΠŸΠ΅Ρ€Π΅Π²ΠΎΠ΄ ΠΈΠ· Π΄Π΅Ρ†ΠΈΠ±Π΅Π» Π² Ρ€Π°Π·Ρ‹ ΠΈ ΠΎΠ±Ρ€Π°Ρ‚Π½ΠΎ

Π”ΠΎΠ²ΠΎΠ»ΡŒΠ½ΠΎ часто Π² популярной радиотСхничСской Π»ΠΈΡ‚Π΅Ρ€Π°Ρ‚ΡƒΡ€Π΅, Π² описании элСктронных схСм употрСбляСтся Π΅Π΄ΠΈΠ½ΠΈΡ†Π° измСрСния – Π΄Π΅Ρ†ΠΈΠ±Π΅Π» (Π΄Π‘ ΠΈΠ»ΠΈ dB).

ΠŸΡ€ΠΈ ΠΈΠ·ΡƒΡ‡Π΅Π½ΠΈΠΈ элСктроники Π½Π°Ρ‡ΠΈΠ½Π°ΡŽΡ‰ΠΈΠΉ Ρ€Π°Π΄ΠΈΠΎΠ»ΡŽΠ±ΠΈΡ‚Π΅Π»ΡŒ ΠΏΡ€ΠΈΠ²Ρ‹ΠΊ ΠΊ Ρ‚Π°ΠΊΠΈΠΌ Π°Π±ΡΠΎΠ»ΡŽΡ‚Π½Ρ‹ΠΌ Π΅Π΄ΠΈΠ½ΠΈΡ†Π°ΠΌ измСрСния ΠΊΠ°ΠΊ АмпСр (сила Ρ‚ΠΎΠΊΠ°), Π’ΠΎΠ»ΡŒΡ‚ (напряТСниС ΠΈ Π­Π”Π‘), Ом (элСктричСскоС сопротивлСниС) ΠΈ ΠΌΠ½ΠΎΠ³ΠΈΠΌ Π΄Ρ€ΡƒΠ³ΠΈΠΌ, с ΠΏΠΎΠΌΠΎΡ‰ΡŒΡŽ ΠΊΠΎΡ‚ΠΎΡ€Ρ‹Ρ… ΠΎΠ±ΠΎΠ·Π½Π°Ρ‡Π°ΡŽΡ‚ количСствСнно Ρ‚ΠΎΡ‚ ΠΈΠ»ΠΈ ΠΈΠ½ΠΎΠΉ элСктричСский ΠΏΠ°Ρ€Π°ΠΌΠ΅Ρ‚Ρ€ (Ρ‘ΠΌΠΊΠΎΡΡ‚ΡŒ, ΠΈΠ½Π΄ΡƒΠΊΡ‚ΠΈΠ²Π½ΠΎΡΡ‚ΡŒ, частоту).

ΠΠ°Ρ‡ΠΈΠ½Π°ΡŽΡ‰Π΅ΠΌΡƒ Ρ€Π°Π΄ΠΈΠΎΠ»ΡŽΠ±ΠΈΡ‚Π΅Π»ΡŽ, ΠΊΠ°ΠΊ ΠΏΡ€Π°Π²ΠΈΠ»ΠΎ, Π½Π΅ составляСт особого Ρ‚Ρ€ΡƒΠ΄Π° Ρ€Π°Π·ΠΎΠ±Ρ€Π°Ρ‚ΡŒΡΡ, Ρ‡Ρ‚ΠΎ Ρ‚Π°ΠΊΠΎΠ΅ Π°ΠΌΠΏΠ΅Ρ€ ΠΈΠ»ΠΈ Π²ΠΎΠ»ΡŒΡ‚. Π’ΡƒΡ‚ всё понятно, Π΅ΡΡ‚ΡŒ элСктричСский ΠΏΠ°Ρ€Π°ΠΌΠ΅Ρ‚Ρ€ ΠΈΠ»ΠΈ Π²Π΅Π»ΠΈΡ‡ΠΈΠ½Π°, ΠΊΠΎΡ‚ΠΎΡ€ΡƒΡŽ Π½ΡƒΠΆΠ½ΠΎ ΠΈΠ·ΠΌΠ΅Ρ€ΠΈΡ‚ΡŒ. Π•ΡΡ‚ΡŒ Π½Π°Ρ‡Π°Π»ΡŒΠ½Ρ‹ΠΉ ΡƒΡ€ΠΎΠ²Π΅Π½ΡŒ отсчёта, ΠΊΠΎΡ‚ΠΎΡ€Ρ‹ΠΉ принимаСтся ΠΏΠΎ ΡƒΠΌΠΎΠ»Ρ‡Π°Π½ΠΈΡŽ Π² Ρ„ΠΎΡ€ΠΌΡƒΠ»ΠΈΡ€ΠΎΠ²ΠΊΠ΅ Π΄Π°Π½Π½ΠΎΠΉ Π΅Π΄ΠΈΠ½ΠΈΡ†Ρ‹ измСрСния. Π•ΡΡ‚ΡŒ условноС ΠΎΠ±ΠΎΠ·Π½Π°Ρ‡Π΅Π½ΠΈΠ΅ этого ΠΏΠ°Ρ€Π°ΠΌΠ΅Ρ‚Ρ€Π° ΠΈΠ»ΠΈ Π²Π΅Π»ΠΈΡ‡ΠΈΠ½Ρ‹ (A, V). И Π²ΠΏΡ€Π°Π²Π΄Ρƒ, ΠΊΠ°ΠΊ Ρ‚ΠΎΠ»ΡŒΠΊΠΎ ΠΌΡ‹ Ρ‡ΠΈΡ‚Π°Π΅ΠΌ надпись 12 V, Ρ‚ΠΎ ΠΌΡ‹ ΠΏΠΎΠ½ΠΈΠΌΠ°Π΅ΠΌ, Ρ‡Ρ‚ΠΎ Ρ€Π΅Ρ‡ΡŒ ΠΈΠ΄Ρ‘Ρ‚ ΠΎ напряТСнии, Π°Π½Π°Π»ΠΎΠ³ΠΈΡ‡Π½ΠΎΠΌ, Π½Π°ΠΏΡ€ΠΈΠΌΠ΅Ρ€, Π½Π°ΠΏΡ€ΡΠΆΠ΅Π½ΠΈΡŽ Π°Π²Ρ‚ΠΎΠΌΠΎΠ±ΠΈΠ»ΡŒΠ½ΠΎΠΉ аккумуляторной Π±Π°Ρ‚Π°Ρ€Π΅ΠΈ.

Но ΠΊΠ°ΠΊ Ρ‚ΠΎΠ»ΡŒΠΊΠΎ встрСчаСтся надпись, ΠΊ ΠΏΡ€ΠΈΠΌΠ΅Ρ€Ρƒ: напряТСниС ΠΏΠΎΠ²Ρ‹ΡΠΈΠ»ΠΎΡΡŒ Π½Π° 3 Π΄Π‘ ΠΈΠ»ΠΈ ΠΌΠΎΡ‰Π½ΠΎΡΡ‚ΡŒ сигнала составляСт 10 Π΄Π‘ΠΌ (10 dBm), Ρ‚ΠΎ Ρƒ ΠΌΠ½ΠΎΠ³ΠΈΡ… Π²ΠΎΠ·Π½ΠΈΠΊΠ°Π΅Ρ‚ Π½Π΅Π΄ΠΎΡƒΠΌΠ΅Π½ΠΈΠ΅. Как это? ΠŸΠΎΡ‡Π΅ΠΌΡƒ упоминаСтся напряТСниС ΠΈΠ»ΠΈ ΠΌΠΎΡ‰Π½ΠΎΡΡ‚ΡŒ, Π° Π·Π½Π°Ρ‡Π΅Π½ΠΈΠ΅ указываСтся Π² ΠΊΠ°ΠΊΠΈΡ…-Ρ‚ΠΎ Π΄Π΅Ρ†ΠΈΠ±Π΅Π»Π°Ρ…?

ΠŸΡ€Π°ΠΊΡ‚ΠΈΠΊΠ° ΠΏΠΎΠΊΠ°Π·Ρ‹Π²Π°Π΅Ρ‚, Ρ‡Ρ‚ΠΎ Π½Π΅ ΠΌΠ½ΠΎΠ³ΠΈΠ΅ Π½Π°Ρ‡ΠΈΠ½Π°ΡŽΡ‰ΠΈΠ΅ Ρ€Π°Π΄ΠΈΠΎΠ»ΡŽΠ±ΠΈΡ‚Π΅Π»ΠΈ ΠΏΠΎΠ½ΠΈΠΌΠ°ΡŽΡ‚, Ρ‡Ρ‚ΠΎ ΠΆΠ΅ Ρ‚Π°ΠΊΠΎΠ΅ Π΄Π΅Ρ†ΠΈΠ±Π΅Π». ΠŸΠΎΠΏΡ‹Ρ‚Π°Π΅ΠΌΡΡ Ρ€Π°Π·Π²Π΅ΡΡ‚ΡŒ нСпроглядный Ρ‚ΡƒΠΌΠ°Π½ Π½Π°Π΄ Ρ‚Π°ΠΊΠΎΠΉ таинствСнной Π΅Π΄ΠΈΠ½ΠΈΡ†Π΅ΠΉ измСрСния ΠΊΠ°ΠΊ Π΄Π΅Ρ†ΠΈΠ±Π΅Π».

Π§Ρ‚ΠΎ Ρ‚Π°ΠΊΠΎΠ΅ Π΄Π΅Ρ†ΠΈΠ±Π΅Π»?

Π•Π΄ΠΈΠ½ΠΈΡ†Ρƒ измСрСния ΠΏΠΎΠ΄ Π½Π°Π·Π²Π°Π½ΠΈΠ΅ΠΌ Π‘Π΅Π» стали Π²ΠΏΠ΅Ρ€Π²Ρ‹Π΅ ΠΏΡ€ΠΈΠΌΠ΅Π½ΡΡ‚ΡŒ ΠΈΠ½ΠΆΠ΅Π½Π΅Ρ€Ρ‹ Ρ‚Π΅Π»Π΅Ρ„ΠΎΠ½Π½ΠΎΠΉ Π»Π°Π±ΠΎΡ€Π°Ρ‚ΠΎΡ€ΠΈΠΈ Π‘Π΅Π»Π»Π°. Π”Π΅Ρ†ΠΈΠ±Π΅Π» являСтся дСсятой Ρ‡Π°ΡΡ‚ΡŒΡŽ Π‘Π΅Π»Π° (1 Π΄Π΅Ρ†ΠΈΠ±Π΅Π» = 0,1 Π‘Π΅Π»). На ΠΏΡ€Π°ΠΊΡ‚ΠΈΠΊΠ΅ ΡˆΠΈΡ€ΠΎΠΊΠΎ ΠΈΡΠΏΠΎΠ»ΡŒΠ·ΡƒΠ΅Ρ‚ΡΡ ΠΊΠ°ΠΊ Ρ€Π°Π· Π΄Π΅Ρ†ΠΈΠ±Π΅Π».

Как ΡƒΠΆΠ΅ Π³ΠΎΠ²ΠΎΡ€ΠΈΠ»ΠΎΡΡŒ, Π΄Π΅Ρ†ΠΈΠ±Π΅Π», это особСнная Π΅Π΄ΠΈΠ½ΠΈΡ†Π° измСрСния. Π‘Ρ‚ΠΎΠΈΡ‚ ΠΎΡ‚ΠΌΠ΅Ρ‚ΠΈΡ‚ΡŒ, Ρ‡Ρ‚ΠΎ Π΄Π΅Ρ†ΠΈΠ±Π΅Π» Π½Π΅ являСтся Ρ‡Π°ΡΡ‚ΡŒΡŽ ΠΎΡ„ΠΈΡ†ΠΈΠ°Π»ΡŒΠ½ΠΎΠΉ систСмы Π΅Π΄ΠΈΠ½ΠΈΡ† БИ. Но, нСсмотря Π½Π° это, Π΄Π΅Ρ†ΠΈΠ±Π΅Π» ΠΏΠΎΠ»ΡƒΡ‡ΠΈΠ» ΠΏΡ€ΠΈΠ·Π½Π°Π½ΠΈΠ΅ ΠΈ занял ΠΏΡ€ΠΎΡ‡Π½ΠΎΠ΅ мСсто наряду с Π΄Ρ€ΡƒΠ³ΠΈΠΌΠΈ Π΅Π΄ΠΈΠ½ΠΈΡ†Π°ΠΌΠΈ измСрСния.

ВспомнитС, ΠΊΠΎΠ³Π΄Π° ΠΌΡ‹ Ρ…ΠΎΡ‚ΠΈΠΌ ΠΎΠ±ΡŠΡΡΠ½ΠΈΡ‚ΡŒ ΠΊΠ°ΠΊΠΎΠ΅-Π»ΠΈΠ±ΠΎ ΠΈΠ·ΠΌΠ΅Π½Π΅Π½ΠΈΠ΅, ΠΌΡ‹ Π³ΠΎΠ²ΠΎΡ€ΠΈΠΌ, Ρ‡Ρ‚ΠΎ, Π½Π°ΠΏΡ€ΠΈΠΌΠ΅Ρ€, стало ярчС Π² 2 Ρ€Π°Π·Π°. Или, Π½Π°ΠΏΡ€ΠΈΠΌΠ΅Ρ€, напряТСниС ΡƒΠΏΠ°Π»ΠΎ Π² 10 Ρ€Π°Π·. ΠŸΡ€ΠΈ этом ΠΌΡ‹ устанавливаСм ΠΎΠΏΡ€Π΅Π΄Π΅Π»Ρ‘Π½Π½Ρ‹ΠΉ ΠΏΠΎΡ€ΠΎΠ³ отсчёта, ΠΎΡ‚Π½ΠΎΡΠΈΡ‚Π΅Π»ΡŒΠ½ΠΎ ΠΊΠΎΡ‚ΠΎΡ€ΠΎΠ³ΠΎ ΠΈ ΠΏΡ€ΠΎΠΈΠ·ΠΎΡˆΠ»ΠΎ ΠΈΠ·ΠΌΠ΅Π½Π΅Π½ΠΈΠ΅ Π² 10 ΠΈΠ»ΠΈ 2 Ρ€Π°Π·Π°. Π‘ ΠΏΠΎΠΌΠΎΡ‰ΡŒΡŽ Π΄Π΅Ρ†ΠΈΠ±Π΅Π» Ρ‚Π°ΠΊΠΆΠ΅ ΠΈΠ·ΠΌΠ΅Ρ€ΡΡŽΡ‚ эти β€œΡ€Π°Π·Ρ‹β€, Ρ‚ΠΎΠ»ΡŒΠΊΠΎ Π² логарифмичСском ΠΌΠ°ΡΡˆΡ‚Π°Π±Π΅.


Π“Ρ€Π°Ρ„ΠΈΠΊ логарифмичСской зависимости

НапримСр, ΠΈΠ·ΠΌΠ΅Π½Π΅Π½ΠΈΠ΅ Π½Π° 1 Π΄Π‘, соотвСтствуСт измСнСнию энСргСтичСской Π²Π΅Π»ΠΈΡ‡ΠΈΠ½Ρ‹ Π² 1,26 Ρ€Π°Π·Π°. ИзмСнСниС Π½Π° 3 Π΄Π‘ соотвСтствуСт измСнСнию энСргСтичСской Π²Π΅Π»ΠΈΡ‡ΠΈΠ½Ρ‹ Π² 2 Ρ€Π°Π·Π°.

Но Π·Π°Ρ‡Π΅ΠΌ Ρ‚Π°ΠΊ Π·Π°ΠΌΠΎΡ€Π°Ρ‡ΠΈΠ²Π°Ρ‚ΡŒΡΡ с Π΄Π΅Ρ†ΠΈΠ±Π΅Π»Π°ΠΌΠΈ, Ссли ΠΎΡ‚Π½ΠΎΡˆΠ΅Π½ΠΈΡ ΠΌΠΎΠΆΠ½ΠΎ ΠΈΠ·ΠΌΠ΅Ρ€ΡΡ‚ΡŒ Π² Ρ€Π°Π·Π°Ρ…? На этот вопрос Π½Π΅Ρ‚ ΠΎΠ΄Π½ΠΎΠ·Π½Π°Ρ‡Π½ΠΎΠ³ΠΎ ΠΎΡ‚Π²Π΅Ρ‚Π°. Но ΡƒΠΆ, ΠΏΠΎΡΠΊΠΎΠ»ΡŒΠΊΡƒ, Π΄Π΅Ρ†ΠΈΠ±Π΅Π»Ρ‹ Π°ΠΊΡ‚ΠΈΠ²Π½ΠΎ ΠΏΡ€ΠΈΠΌΠ΅Π½ΡΡŽΡ‚ΡΡ, Ρ‚ΠΎ навСрняка это ΠΎΠΏΡ€Π°Π²Π΄Π°Π½ΠΎ.

ΠŸΡ€ΠΈΡ‡ΠΈΠ½Ρ‹ для использования Π΄Π΅Ρ†ΠΈΠ±Π΅Π» всё-Ρ‚Π°ΠΊΠΈ Π΅ΡΡ‚ΡŒ. ΠŸΠ΅Ρ€Π΅Ρ‡ΠΈΡΠ»ΠΈΠΌ ΠΈΡ….

Частично ΠΎΡ‚Π²Π΅Ρ‚ Π½Π° этот вопрос кроСтся Π² Ρ‚Π°ΠΊ Π½Π°Π·Ρ‹Π²Π°Π΅ΠΌΠΎΠΌ Π·Π°ΠΊΠΎΠ½Π΅ Π’Π΅Π±Π΅Ρ€Π°-Π€Π΅Ρ…Π½Π΅Ρ€Π°. Π­Ρ‚ΠΎ эмпиричСский психофизиологичСский Π·Π°ΠΊΠΎΠ½, Ρ‚.Π΅ основан ΠΎΠ½ Π½Π° Ρ€Π΅Π·ΡƒΠ»ΡŒΡ‚Π°Ρ‚Π°Ρ… Ρ€Π΅Π°Π»ΡŒΠ½Ρ‹Ρ…, Π° Π½Π΅ тСорСтичСских экспСримСнтов. Π‘ΡƒΡ‚ΡŒ Π΅Π³ΠΎ Π·Π°ΠΊΠ»ΡŽΡ‡Π°Π΅Ρ‚ΡΡ Π² Ρ‚ΠΎΠΌ, Ρ‡Ρ‚ΠΎ Π»ΡŽΠ±Ρ‹Π΅ измСнСния ΠΊΠ°ΠΊΠΈΡ…-Π»ΠΈΠ±ΠΎ Π²Π΅Π»ΠΈΡ‡ΠΈΠ½ (яркости, громкости, вСса) ΠΎΡ‰ΡƒΡ‰Π°ΡŽΡ‚ΡΡ Π½Π°ΠΌΠΈ ΠΏΡ€ΠΈ условии, Ссли эти измСнСния носят логарифмичСский Ρ…Π°Ρ€Π°ΠΊΡ‚Π΅Ρ€.


Π“Ρ€Π°Ρ„ΠΈΠΊ зависимости ощущСния громкости ΠΎΡ‚ силы (мощности) Π·Π²ΡƒΠΊΠ°. Π—Π°ΠΊΠΎΠ½ Π’Π΅Π±Π΅Ρ€Π°-Π€Π΅Ρ…Π½Π΅Ρ€Π°

Π’Π°ΠΊ, Π½Π°ΠΏΡ€ΠΈΠΌΠ΅Ρ€, Ρ‡ΡƒΠ²ΡΡ‚Π²ΠΈΡ‚Π΅Π»ΡŒΠ½ΠΎΡΡ‚ΡŒ чСловСчСского ΡƒΡ…Π° ΡƒΠΌΠ΅Π½ΡŒΡˆΠ°Π΅Ρ‚ΡΡ с ростом уровня громкости Π·Π²ΡƒΠΊΠΎΠ²ΠΎΠ³ΠΎ сигнала. ИмСнно поэтому, ΠΏΡ€ΠΈ Π²Ρ‹Π±ΠΎΡ€Π΅ ΠΏΠ΅Ρ€Π΅ΠΌΠ΅Π½Π½ΠΎΠ³ΠΎ рСзистора, ΠΊΠΎΡ‚ΠΎΡ€Ρ‹ΠΉ планируСтся ΠΏΡ€ΠΈΠΌΠ΅Π½ΠΈΡ‚ΡŒ Π² рСгуляторС громкости Π·Π²ΡƒΠΊΠΎΠ²ΠΎΠ³ΠΎ усилитСля стоит Π±Ρ€Π°Ρ‚ΡŒ с ΠΏΠΎΠΊΠ°Π·Π°Ρ‚Π΅Π»ΡŒΠ½ΠΎΠΉ Π·Π°Π²ΠΈΡΠΈΠΌΠΎΡΡ‚ΡŒΡŽ сопротивлСния ΠΎΡ‚ ΡƒΠ³Π»Π° ΠΏΠΎΠ²ΠΎΡ€ΠΎΡ‚Π° Ρ€ΡƒΡ‡ΠΊΠΈ рСгулятора. Π’ этом случаС, ΠΏΡ€ΠΈ ΠΏΠΎΠ²ΠΎΡ€ΠΎΡ‚Π΅ Π΄Π²ΠΈΠΆΠΊΠ° рСгулятора громкости Π·Π²ΡƒΠΊ Π² Π΄ΠΈΠ½Π°ΠΌΠΈΠΊΠ΅ Π±ΡƒΠ΄Π΅Ρ‚ Π½Π°Ρ€Π°ΡΡ‚Π°Ρ‚ΡŒ ΠΏΠ»Π°Π²Π½ΠΎ. Π Π΅Π³ΡƒΠ»ΠΈΡ€ΠΎΠ²ΠΊΠ° громкости Π±ΡƒΠ΄Π΅Ρ‚ Π»ΠΈΠ½Π΅ΠΉΠ½ΠΎΠΉ, Ρ‚Π°ΠΊ ΠΊΠ°ΠΊ ΠΏΠΎΠΊΠ°Π·Π°Ρ‚Π΅Π»ΡŒΠ½Π°Ρ Π·Π°Π²ΠΈΡΠΈΠΌΠΎΡΡ‚ΡŒ рСгулятора громкости компСнсируСт Π»ΠΎΠ³Π°Ρ€ΠΈΡ„ΠΌΠΈΡ‡Π΅ΡΠΊΡƒΡŽ Π·Π°Π²ΠΈΡΠΈΠΌΠΎΡΡ‚ΡŒ нашСго слуха ΠΈ Π² суммС станСт Π»ΠΈΠ½Π΅ΠΉΠ½ΠΎΠΉ. ΠŸΡ€ΠΈ взглядС Π½Π° рисунок это станСт Π±ΠΎΠ»Π΅Π΅ понятно.


Π—Π°Π²ΠΈΡΠΈΠΌΠΎΡΡ‚ΡŒ сопротивлСния ΠΏΠ΅Ρ€Π΅ΠΌΠ΅Π½Π½ΠΎΠ³ΠΎ рСзистора ΠΎΡ‚ ΡƒΠ³Π»Π° ΠΏΠΎΠ²ΠΎΡ€ΠΎΡ‚Π° Π΄Π²ΠΈΠΆΠΊΠ° (А-линСйная, Π‘-логарифмичСская, Π’-ΠΏΠΎΠΊΠ°Π·Π°Ρ‚Π΅Π»ΡŒΠ½Π°Ρ)

Π—Π΄Π΅ΡΡŒ ΠΏΠΎΠΊΠ°Π·Π°Π½Ρ‹ Π³Ρ€Π°Ρ„ΠΈΠΊΠΈ зависимости сопротивлСния ΠΏΠ΅Ρ€Π΅ΠΌΠ΅Π½Π½Ρ‹Ρ… рСзисторов Ρ€Π°Π·Π½Ρ‹Ρ… Ρ‚ΠΈΠΏΠΎΠ²: А – линСйная, Π‘ – логарифмичСская, Π’ – ΠΏΠΎΠΊΠ°Π·Π°Ρ‚Π΅Π»ΡŒΠ½Π°Ρ. Как ΠΏΡ€Π°Π²ΠΈΠ»ΠΎ, Π½Π° ΠΏΠ΅Ρ€Π΅ΠΌΠ΅Π½Π½Ρ‹Ρ… рСзисторах отСчСствСнного производства указываСтся, ΠΊΠ°ΠΊΠΎΠΉ Π·Π°Π²ΠΈΡΠΈΠΌΠΎΡΡ‚ΡŒΡŽ ΠΎΠ±Π»Π°Π΄Π°Π΅Ρ‚ ΠΏΠ΅Ρ€Π΅ΠΌΠ΅Π½Π½Ρ‹ΠΉ рСзистор. На Ρ‚Π΅Ρ… ΠΆΠ΅ ΠΏΡ€ΠΈΠ½Ρ†ΠΈΠΏΠ°Ρ… основаны Ρ†ΠΈΡ„Ρ€ΠΎΠ²Ρ‹Π΅ ΠΈ элСктронныС рСгуляторы громкости.

Π’Π°ΠΊΠΆΠ΅ стоит ΠΎΡ‚ΠΌΠ΅Ρ‚ΠΈΡ‚ΡŒ, Ρ‡Ρ‚ΠΎ чСловСчСскоС ΡƒΡ…ΠΎ воспринимаСт Π·Π²ΡƒΠΊΠΈ, ΠΌΠΎΡ‰Π½ΠΎΡΡ‚ΡŒ ΠΊΠΎΡ‚ΠΎΡ€Ρ‹Ρ… различаСтся Π½Π° ΠΊΠΎΠ»ΠΎΡΡΠ°Π»ΡŒΠ½ΡƒΡŽ Π²Π΅Π»ΠΈΡ‡ΠΈΠ½Ρƒ Π² 10Β 000Β 000Β 000Β 000 Ρ€Π°Π·! Π’Π°ΠΊΠΈΠΌ ΠΎΠ±Ρ€Π°Π·ΠΎΠΌ, самый Π³Ρ€ΠΎΠΌΠΊΠΈΠΉ Π·Π²ΡƒΠΊ отличаСтся ΠΎΡ‚ самого Ρ‚ΠΈΡ…ΠΎΠ³ΠΎ, ΠΊΠΎΡ‚ΠΎΡ€Ρ‹ΠΉ ΠΌΠΎΠΆΠ΅Ρ‚ ΡƒΠ»ΠΎΠ²ΠΈΡ‚ΡŒ наш слух, Π½Π° 130 Π΄Π‘ (10Β 000Β 000Β 000Β 000 Ρ€Π°Π·).

Вторая ΠΏΡ€ΠΈΡ‡ΠΈΠ½Π° ΡˆΠΈΡ€ΠΎΠΊΠΎΠ³ΠΎ использования Π΄Π΅Ρ†ΠΈΠ±Π΅Π» являСтся простота вычислСний.

Π‘ΠΎΠ³Π»Π°ΡΠΈΡ‚Π΅ΡΡŒ, Ρ‡Ρ‚ΠΎ ΠΊΡƒΠ΄Π° ΠΏΡ€ΠΎΡ‰Π΅ ΠΏΡ€ΠΈ вычислСниях ΠΈΡΠΏΠΎΠ»ΡŒΠ·ΠΎΠ²Π°Ρ‚ΡŒ нСбольшиС числа Π²Ρ€ΠΎΠ΄Π΅ 10, 20, 60,80,100,130 (Π½Π°ΠΈΠ±ΠΎΠ»Π΅Π΅ часто ΠΈΡΠΏΠΎΠ»ΡŒΠ·ΡƒΠ΅ΠΌΡ‹Π΅ числа ΠΏΡ€ΠΈ расчётС Π² Π΄Π΅Ρ†ΠΈΠ±Π΅Π»Π°Ρ…) ΠΏΠΎ ΡΡ€Π°Π²Π½Π΅Π½ΠΈΡŽ с числами 100 (20 Π΄Π‘), 1000 (30 Π΄Π‘), 1000 000 (60 Π΄Π‘),100 000 000 (80 Π΄Π‘),10 000Β 000Β 000 (100 Π΄Π‘), 10Β 000Β 000Β 000Β 000 (130 Π΄Π‘). Π•Ρ‰Ρ‘ ΠΎΠ΄Π½ΠΈΠΌ достоинством Π΄Π΅Ρ†ΠΈΠ±Π΅Π» являСтся Ρ‚ΠΎ, Ρ‡Ρ‚ΠΎ ΠΈΡ… просто ΡΡƒΠΌΠΌΠΈΡ€ΡƒΡŽΡ‚. Если ΠΏΡ€ΠΎΠ²ΠΎΠ΄ΠΈΡ‚ΡŒ вычислСния Π² Ρ€Π°Π·Π°Ρ…, Ρ‚ΠΎ числа Π½Π΅ΠΎΠ±Ρ…ΠΎΠ΄ΠΈΠΌΠΎ ΡƒΠΌΠ½ΠΎΠΆΠ°Ρ‚ΡŒ.

НапримСр, 30 Π΄Π‘ + 30 Π΄Π‘ = 60 Π΄Π‘ (Π² Ρ€Π°Π·Π°Ρ…: 1000 * 1000 = 1000 000). Π”ΡƒΠΌΠ°ΡŽ, с этим всё ясно.

Π’Π°ΠΊΠΆΠ΅ Π΄Π΅Ρ†ΠΈΠ±Π΅Π»Ρ‹ ΠΎΡ‡Π΅Π½ΡŒ ΡƒΠ΄ΠΎΠ±Π½Ρ‹ ΠΏΡ€ΠΈ графичСском построСнии Ρ€Π°Π·Π»ΠΈΡ‡Π½Ρ‹Ρ… зависимостСй. ВсС Π³Ρ€Π°Ρ„ΠΈΠΊΠΈ Π²Ρ€ΠΎΠ΄Π΅ Π΄ΠΈΠ°Π³Ρ€Π°ΠΌΠΌ направлСнности Π°Π½Ρ‚Π΅Π½Π½, Π°ΠΌΠΏΠ»ΠΈΡ‚ΡƒΠ΄Π½ΠΎ-частотных характСристик усилитСлСй Π²Ρ‹ΠΏΠΎΠ»Π½ΡΡŽΡ‚ с ΠΏΡ€ΠΈΠΌΠ΅Π½Π΅Π½ΠΈΠ΅ΠΌ Π΄Π΅Ρ†ΠΈΠ±Π΅Π».

Π”Π΅Ρ†ΠΈΠ±Π΅Π» являСтся Π±Π΅Π·Ρ€Π°Π·ΠΌΠ΅Ρ€Π½ΠΎΠΉ Π΅Π΄ΠΈΠ½ΠΈΡ†Π΅ΠΉ измСрСния. ΠœΡ‹ ΡƒΠΆΠ΅ выяснили, Ρ‡Ρ‚ΠΎ Π΄Π΅Ρ†ΠΈΠ±Π΅Π» Π½Π° самом Π΄Π΅Π»Π΅ ΠΏΠΎΠΊΠ°Π·Ρ‹Π²Π°Π΅Ρ‚, Π²ΠΎ сколько Ρ€Π°Π· возросла, Π»ΠΈΠ±ΠΎ ΡƒΠΌΠ΅Π½ΡŒΡˆΠΈΠ»Π°ΡΡŒ какая-Π»ΠΈΠ±ΠΎ Π²Π΅Π»ΠΈΡ‡ΠΈΠ½Π° (Ρ‚ΠΎΠΊ, напряТСниС, ΠΌΠΎΡ‰Π½ΠΎΡΡ‚ΡŒ). ΠžΡ‚Π»ΠΈΡ‡ΠΈΠ΅ Π΄Π΅Ρ†ΠΈΠ±Π΅Π» ΠΎΡ‚ Ρ€Π°Π·ΠΎΠ² Π·Π°ΠΊΠ»ΡŽΡ‡Π°Π΅Ρ‚ΡΡ лишь Π² Ρ‚ΠΎΠΌ, Ρ‡Ρ‚ΠΎ происходит ΠΈΠ·ΠΌΠ΅Ρ€Π΅Π½ΠΈΠ΅ ΠΏΠΎ логарифмичСскому ΠΌΠ°ΡΡˆΡ‚Π°Π±Ρƒ. Π§Ρ‚ΠΎΠ±Ρ‹ это ΠΊΠ°ΠΊ-Ρ‚ΠΎ ΠΎΠ±ΠΎΠ·Π½Π°Ρ‡ΠΈΡ‚ΡŒ ΠΈ ΠΏΡ€ΠΈΠΏΠΈΡΡ‹Π²Π°ΡŽΡ‚ ΠΎΠ±ΠΎΠ·Π½Π°Ρ‡Π΅Π½ΠΈΠ΅

Π΄Π‘. Π’Π°ΠΊ ΠΈΠ»ΠΈ ΠΈΠ½Π°Ρ‡Π΅, ΠΏΡ€ΠΈ ΠΎΡ†Π΅Π½ΠΊΠ΅ приходится ΠΏΠ΅Ρ€Π΅Ρ…ΠΎΠ΄ΠΈΡ‚ΡŒ ΠΎΡ‚ Π΄Π΅Ρ†ΠΈΠ±Π΅Π» ΠΊ Ρ€Π°Π·Π°ΠΌ. Π‘Ρ€Π°Π²Π½ΠΈΠ²Π°Ρ‚ΡŒ с ΠΏΠΎΠΌΠΎΡ‰ΡŒΡŽ Π΄Π΅Ρ†ΠΈΠ±Π΅Π» ΠΌΠΎΠΆΠ½ΠΎ Π»ΡŽΠ±Ρ‹Π΅ Π΅Π΄ΠΈΠ½ΠΈΡ†Ρ‹ измСрСния (Π½Π΅ Ρ‚ΠΎΠ»ΡŒΠΊΠΎ Ρ‚ΠΎΠΊ, напряТСниС ΠΈ ΠΏΡ€ΠΎΡ‡.), Ρ‚Π°ΠΊ ΠΊΠ°ΠΊ Π΄Π΅Ρ†ΠΈΠ±Π΅Π» являСтся ΠΎΡ‚Π½ΠΎΡΠΈΡ‚Π΅Π»ΡŒΠ½ΠΎΠΉ, Π±Π΅Π·Ρ€Π°Π·ΠΌΠ΅Ρ€Π½ΠΎΠΉ Π²Π΅Π»ΠΈΡ‡ΠΈΠ½ΠΎΠΉ.

Если указываСтся Π·Π½Π°ΠΊ β€œ-”, Π½Π°ΠΏΡ€ΠΈΠΌΠ΅Ρ€, –1 Π΄Π‘, Ρ‚ΠΎ Π·Π½Π°Ρ‡Π΅Π½ΠΈΠ΅ измСряСмой Π²Π΅Π»ΠΈΡ‡ΠΈΠ½Ρ‹, Π½Π°ΠΏΡ€ΠΈΠΌΠ΅Ρ€, мощности, ΡƒΠΌΠ΅Π½ΡŒΡˆΠΈΠ»ΠΎΡΡŒ Π² 1,26 Ρ€Π°Π·. Если ΠΏΠ΅Ρ€Π΅Π΄ Π΄Π΅Ρ†ΠΈΠ±Π΅Π»Π°ΠΌΠΈ Π½Π΅ ставят Π½ΠΈΠΊΠ°ΠΊΠΎΠ³ΠΎ Π·Π½Π°ΠΊΠ°, Ρ‚ΠΎ Ρ€Π΅Ρ‡ΡŒ ΠΈΠ΄Ρ‘Ρ‚ ΠΎΠ± ΡƒΠ²Π΅Π»ΠΈΡ‡Π΅Π½ΠΈΠΈ, ростС Π²Π΅Π»ΠΈΡ‡ΠΈΠ½Ρ‹. Π­Ρ‚ΠΎ стоит ΡƒΡ‡ΠΈΡ‚Ρ‹Π²Π°Ρ‚ΡŒ. Иногда вмСсто Π·Π½Π°ΠΊΠ° β€œ-” говорят ΠΎ затуханиях, сниТСнии коэффициСнта усилСния.

ΠŸΠ΅Ρ€Π΅Ρ…ΠΎΠ΄ ΠΎΡ‚ Π΄Π΅Ρ†ΠΈΠ±Π΅Π» ΠΊ Ρ€Π°Π·Π°ΠΌ.

На ΠΏΡ€Π°ΠΊΡ‚ΠΈΠΊΠ΅ Ρ‡Π°Ρ‰Π΅ всСго приходится ΠΏΠ΅Ρ€Π΅Ρ…ΠΎΠ΄ΠΈΡ‚ΡŒ ΠΎΡ‚ Π΄Π΅Ρ†ΠΈΠ±Π΅Π» ΠΊ Ρ€Π°Π·Π°ΠΌ. Для этого Π΅ΡΡ‚ΡŒ простая Ρ„ΠΎΡ€ΠΌΡƒΠ»Π°:

Π’Π½ΠΈΠΌΠ°Π½ΠΈΠ΅! Π”Π°Π½Π½Ρ‹Π΅ Ρ„ΠΎΡ€ΠΌΡƒΠ»Ρ‹ ΠΏΡ€ΠΈΠΌΠ΅Π½ΡΡŽΡ‚ΡΡ для Ρ‚Π°ΠΊ Π½Π°Π·Ρ‹Π²Π°Π΅ΠΌΡ‹Ρ… β€œΡΠ½Π΅Ρ€Π³Π΅Ρ‚ΠΈΡ‡Π΅ΡΠΊΠΈΡ…β€ Π²Π΅Π»ΠΈΡ‡ΠΈΠ½. Π’Π°ΠΊΠΈΡ… ΠΊΠ°ΠΊ энСргия ΠΈ ΠΌΠΎΡ‰Π½ΠΎΡΡ‚ΡŒ.

m = 10(n / 10) ,Π³Π΄Π΅ m – ΠΎΡ‚Π½ΠΎΡˆΠ΅Π½ΠΈΠ΅ Π² Ρ€Π°Π·Π°Ρ…, n – ΠΎΡ‚Π½ΠΎΡˆΠ΅Π½ΠΈΠ΅ Π² Π΄Π΅Ρ†ΠΈΠ±Π΅Π»Π°Ρ….

НапримСр, 1Π΄Π‘ Ρ€Π°Π²Π΅Π½ 10(1Π΄Π‘ / 10)

= 1,258925…= 1,26 Ρ€Π°Π·Π°.

Аналогично,

Но, Π½Π΅ всё Ρ‚Π°ΠΊ просто. Π•ΡΡ‚ΡŒ ΠΈ ΠΏΠΎΠ΄Π²ΠΎΠ΄Π½Ρ‹Π΅ ΠΊΠ°ΠΌΠ½ΠΈ. НапримСр, Π·Π°Ρ‚ΡƒΡ…Π°Π½ΠΈΠ΅ сигнала составляСт -10 Π΄Π‘. Π’ΠΎΠ³Π΄Π°:

  • ΠΏΡ€ΠΈ -10 Π΄Π‘: 10(-10Π΄Π‘ / 10) = 0,1

    Если ΠΌΠΎΡ‰Π½ΠΎΡΡ‚ΡŒ с 5 Π’Ρ‚ ΡƒΠΌΠ΅Π½ΡŒΡˆΠΈΠ»Π°ΡΡŒ Π΄ΠΎ 0,5 Π’Ρ‚, Ρ‚ΠΎ сниТСниС мощности Ρ€Π°Π²Π½ΠΎ -10 Π΄Π‘ (ΡƒΠΌΠ΅Π½ΡŒΡˆΠ΅Π½ΠΈΡŽ Π² 10 Ρ€Π°Π·).

  • ΠΏΡ€ΠΈ -20 Π΄Π‘: 10(-20Π΄Π‘ / 10) = 0,01

    Π—Π΄Π΅ΡΡŒ Π°Π½Π°Π»ΠΎΠ³ΠΈΡ‡Π½ΠΎ. ΠŸΡ€ΠΈ сниТСнии мощности с 5 Π’Ρ‚ Π΄ΠΎ 0,05 Π’Ρ‚, Π² Π΄Π΅Ρ†ΠΈΠ±Π΅Π»Π°Ρ… ΠΏΠ°Π΄Π΅Π½ΠΈΠ΅ мощности составит -20 Π΄Π‘ (ΡƒΠΌΠ΅Π½ΡŒΡˆΠ΅Π½ΠΈΡŽ Π² 100 Ρ€Π°Π·).

Π’Π°ΠΊΠΈΠΌ ΠΎΠ±Ρ€Π°Π·ΠΎΠΌ, ΠΏΡ€ΠΈ -10 Π΄Π‘ ΠΌΠΎΡ‰Π½ΠΎΡΡ‚ΡŒ сигнала ΡƒΠΌΠ΅Π½ΡŒΡˆΠΈΠ»Π°ΡΡŒ Π² 10 Ρ€Π°Π·! ΠŸΡ€ΠΈ этом Ссли ΠΌΡ‹ ΠΏΠ΅Ρ€Π΅ΠΌΠ½ΠΎΠΆΠΈΠΌ Π½Π°Ρ‡Π°Π»ΡŒΠ½ΡƒΡŽ Π²Π΅Π»ΠΈΡ‡ΠΈΠ½Ρƒ сигнала Π½Π° 0,1 ,Ρ‚ΠΎ ΠΈ ΠΏΠΎΠ»ΡƒΡ‡ΠΈΠΌ Π·Π½Π°Ρ‡Π΅Π½ΠΈΠ΅ мощности сигнала ΠΏΡ€ΠΈ Π·Π°Ρ‚ΡƒΡ…Π°Π½ΠΈΠΈ Π² -10 Π΄Π‘. ИмСнно поэтому Π·Π½Π°Ρ‡Π΅Π½ΠΈΠ΅ 0,1 ΠΈ ΡƒΠΊΠ°Π·Π°Π½ΠΎ Π±Π΅Π· «Ρ€Π°Π·ΠΎΠ²», ΠΊΠ°ΠΊ Π² ΠΏΡ€Π΅Π΄Ρ‹Π΄ΡƒΡ‰ΠΈΡ… ΠΏΡ€ΠΈΠΌΠ΅Ρ€Π°Ρ…. Π£Ρ‡ΠΈΡ‚Ρ‹Π²Π°ΠΉΡ‚Π΅ эту ΠΎΡΠΎΠ±Π΅Π½Π½ΠΎΡΡ‚ΡŒ ΠΏΡ€ΠΈ подстановкС Π² Π΄Π°Π½Π½Ρ‹Π΅ Ρ„ΠΎΡ€ΠΌΡƒΠ»Ρ‹ Π·Π½Π°Ρ‡Π΅Π½ΠΈΠΉ Π΄Π΅Ρ†ΠΈΠ±Π΅Π» со Π·Π½Π°ΠΊΠΎΠΌ «-«.

ΠŸΠ΅Ρ€Π΅Ρ…ΠΎΠ΄ ΠΎΡ‚ Ρ€Π°Π·ΠΎΠ² ΠΊ Π΄Π΅Ρ†ΠΈΠ±Π΅Π»Π°ΠΌ ΠΌΠΎΠΆΠ½ΠΎ ΠΎΡΡƒΡ‰Π΅ΡΡ‚Π²ΠΈΡ‚ΡŒ ΠΏΠΎ ΡΠ»Π΅Π΄ΡƒΡŽΡ‰Π΅ΠΉ Ρ„ΠΎΡ€ΠΌΡƒΠ»Π΅:

  • n = 10 * log10(m) ,Π³Π΄Π΅ n – Π·Π½Π°Ρ‡Π΅Π½ΠΈΠ΅ Π² Π΄Π΅Ρ†ΠΈΠ±Π΅Π»Π°Ρ…, m – ΠΎΡ‚Π½ΠΎΡˆΠ΅Π½ΠΈΠ΅ Π² Ρ€Π°Π·Π°Ρ….

  • НапримСр, рост мощности Π² 4 Ρ€Π°Π·Π° Π±ΡƒΠ΄Π΅Ρ‚ ΡΠΎΠΎΡ‚Π²Π΅Ρ‚ΡΡ‚Π²ΠΎΠ²Π°Ρ‚ΡŒ Π·Π½Π°Ρ‡Π΅Π½ΠΈΡŽ Π² 6,021 Π΄Π‘.

  • 10 * log10(4) = 6,021 Π΄Π‘.

Π’Π½ΠΈΠΌΠ°Π½ΠΈΠ΅! Для пСрСсчёта ΠΎΡ‚Π½ΠΎΡˆΠ΅Π½ΠΈΠΉ Ρ‚Π°ΠΊΠΈΡ… Π²Π΅Π»ΠΈΡ‡ΠΈΠ½ ΠΊΠ°ΠΊ напряТСниС ΠΈ сила Ρ‚ΠΎΠΊΠ° ΡΡƒΡ‰Π΅ΡΡ‚Π²ΡƒΡŽΡ‚ Π½Π΅ΠΌΠ½ΠΎΠ³ΠΎ ΠΈΠ½Ρ‹Π΅ Ρ„ΠΎΡ€ΠΌΡƒΠ»Ρ‹:

(Π‘ΠΈΠ»Π° Ρ‚ΠΎΠΊΠ° ΠΈ напряТСниС, это Ρ‚Π°ΠΊ Π½Π°Π·Ρ‹Π²Π°Π΅ΠΌΡ‹Π΅ β€œΡΠΈΠ»ΠΎΠ²Ρ‹Π΅β€ Π²Π΅Π»ΠΈΡ‡ΠΈΠ½Ρ‹. ΠŸΠΎΡΡ‚ΠΎΠΌΡƒ ΠΈ Ρ„ΠΎΡ€ΠΌΡƒΠ»Ρ‹ ΠΎΡ‚Π»ΠΈΡ‡Π°ΡŽΡ‚ΡΡ.)

n – Π·Π½Π°Ρ‡Π΅Π½ΠΈΠ΅ Π² Π΄Π΅Ρ†ΠΈΠ±Π΅Π»Π°Ρ…, m – ΠΎΡ‚Π½ΠΎΡˆΠ΅Π½ΠΈΠ΅ Π² Ρ€Π°Π·Π°Ρ….

Если Π’Ρ‹ ΡƒΡΠΏΠ΅ΡˆΠ½ΠΎ дошли Π΄ΠΎ этих строк, Ρ‚ΠΎ считайтС, Ρ‡Ρ‚ΠΎ сдСлали Π΅Ρ‰Ρ‘ ΠΎΠ΄ΠΈΠ½ вСсомый шаг Π² освоСнии элСктроники!

Главная &raquo РадиоэлСктроника для Π½Π°Ρ‡ΠΈΠ½Π°ΡŽΡ‰ΠΈΡ… &raquo ВСкущая страница

Π’Π°ΠΊΠΆΠ΅ Π’Π°ΠΌ Π±ΡƒΠ΄Π΅Ρ‚ интСрСсно ΡƒΠ·Π½Π°Ρ‚ΡŒ:

Β 

Онлайн ΠΊΠ°Π»ΡŒΠΊΡƒΠ»ΡΡ‚ΠΎΡ€ Π΄Π΅Ρ†ΠΈΠ±Π΅Π»Ρ‹ Π² Ρ€Π°Π·Ρ‹, напряТСния Π² ΠΌΠΎΡ‰Π½ΠΎΡΡ‚ΡŒ

Π”Π΅Ρ†ΠΈΠ±Π΅Π»… Π§Ρ‚ΠΎ Π·Π° странный пассаТир? Π›Π°Π΄Π½ΠΎ Π±Ρ‹ Π΄Π΅Π±ΠΈΠ», ΠΈΠ»ΠΈ, Π½Π° Ρ…ΡƒΠ΄ΠΎΠΉ ΠΊΠΎΠ½Π΅Ρ†, ΠΈΠΌΠ±Π΅Ρ†ΠΈΠ», Ρ‚Π°ΠΊ вСдь Π½Π΅Ρ‚ — Π΄Π΅Ρ†ΠΈΠ±Π΅Π», ΠΌΠ°Ρ‚ΡŒ Π΅Π³ΠΎ.
Π’Ρ‹ΠΏΠΈΠ»ΠΈ ΠΏΠΎ Π΄Π΅Ρ†Π΅Π»Ρƒ, закусили, понимания Π½Π΅ ΠΏΡ€ΠΈΠ±Π°Π²ΠΈΠ»ΠΎ, Π΅Ρ‰Ρ‘ ΠΏΠΎ сто, ΡƒΠΆΠ΅ Π»ΡƒΡ‡ΡˆΠ΅ — Π½Π°Ρ‡Π°Π»ΠΈ Π³Π΅Π½Π΅Ρ€ΠΈΡ‚ΡŒ ΠΌΡ‹ΡΠ»ΡŽ.

И Π½Π° ΠΊΠΎΠΉ Ρ…Ρ€Π΅Π½ Π½Π°ΠΌ Π² Π±Π°Ρ‚Π°Ρ€Π΅Π΅ Ρ€Π°Π·Π²ΠΎΠ΄ΠΈΡ‚ΡŒ ΠΌΡƒΠ΄Ρ€Ρ‘Π½Ρ‹Π΅ Π²Π΅Π»ΠΈΡ‡ΠΈΠ½Ρ‹, Π΄Π° Π΅Ρ‰Ρ‘ (Π½Π΅ ΠΏΡ€ΠΈ Π±Π°Π±Π°Ρ… Π±ΡƒΠ΄Π΅Ρ‚ сказано), числСнно Ρ€Π°Π²Π½Ρ‹Π΅ дСсятичному Π»ΠΎΠ³Π°Ρ€ΠΈΡ„ΠΌΡƒ Π±Π΅Π·Ρ€Π°Π·ΠΌΠ΅Ρ€Π½ΠΎΠ³ΠΎ ΠΎΡ‚Π½ΠΎΡˆΠ΅Π½ΠΈΡ физичСской Π²Π΅Π»ΠΈΡ‡ΠΈΠ½Ρ‹ ΠΊ ΠΎΠ΄Π½ΠΎΠΈΠΌΡ‘Π½Π½ΠΎΠΉ физичСской Π²Π΅Π»ΠΈΡ‡ΠΈΠ½Π΅, ΠΏΡ€ΠΈΠ½ΠΈΠΌΠ°Π΅ΠΌΠΎΠΉ Π·Π° ΠΈΡΡ…ΠΎΠ΄Π½ΡƒΡŽ, ΡƒΠΌΠ½ΠΎΠΆΠ΅Π½Π½ΠΎΠΌΡƒ Π½Π° Π΄Π΅ΡΡΡ‚ΡŒ?
Всё Ρ€Π°Π²Π½ΠΎ — ΠΊΠ°ΠΊ отмСряли ΠΏΠΎΡ‚Π΅Ρ€ΠΈ сигнала Π² линиях ΠΊΠΈΠ»ΠΎΠΌΠ΅Ρ‚Ρ€Π°ΠΌΠΈ стандартного кабСля, Ρ‚Π°ΠΊ ΠΈ Π±ΡƒΠ΄Π΅ΠΌ ΠΎΡ‚ΠΌΠ΅Ρ€ΡΡ‚ΡŒ.

ΠžΡ‚Π²Π΅Ρ‚ Π½Π΅ слоТСн — для удобства мировосприятия.
ΠŸΡ€ΠΈΡ€ΠΎΠ΄Π° наша Ρ‚Π°ΠΊΠΎΠ²Π°, Ρ‡Ρ‚ΠΎ воздСйствиС Π½Π° ΠΎΡ€Π³Π°Π½Ρ‹ чувств ΠΌΠ½ΠΎΠ³ΠΈΡ… физичСских ΠΈ биологичСских процСссов ΠΏΡ€ΠΎΠΏΠΎΡ€Ρ†ΠΈΠΎΠ½Π°Π»ΡŒΠ½ΠΎ Π½Π΅ Π°ΠΌΠΏΠ»ΠΈΡ‚ΡƒΠ΄Π΅ Π²Ρ…ΠΎΠ΄Π½ΠΎΠ³ΠΎ воздСйствия, Π° Π»ΠΎΠ³Π°Ρ€ΠΈΡ„ΠΌΡƒ Π²Ρ…ΠΎΠ΄Π½ΠΎΠ³ΠΎ воздСйствия. ΠŸΠΎΡΡ‚ΠΎΠΌΡƒ ΠΈ ΡΠΎΠ·Π΅Ρ€Ρ†Π°Ρ‚ΡŒ отобраТСния Π±ΠΎΠ»ΡŒΡˆΠΈΡ… Π΄ΠΈΠ°ΠΏΠ°Π·ΠΎΠ½ΠΎΠ² ΠΈΠ·ΠΌΠ΅Π½ΡΡŽΡ‰ΠΈΡ…ΡΡ Π²Π΅Π»ΠΈΡ‡ΠΈΠ½ ΡƒΠ΄ΠΎΠ±Π½Π΅Π΅ всСго Π² логарифмичСском ΠΌΠ°ΡΡˆΡ‚Π°Π±Π΅.

Π˜Ρ‚Π°ΠΊ, Π΄Π΅Ρ†ΠΈΠ±Π΅Π»Ρ‹ — это ΡΠΎΠΎΡ‚Π½ΠΎΡˆΠ΅Π½ΠΈΠ΅ Π΄Π²ΡƒΡ… Π²Π΅Π»ΠΈΡ‡ΠΈΠ½, Π²Ρ‹Ρ€Π°ΠΆΠ΅Π½Π½ΠΎΠ΅ Π² логарифмичСском ΠΌΠ°ΡΡˆΡ‚Π°Π±Π΅. ΠŸΡ€ΠΈ этом ΠΎΡ‚Π½ΠΎΡˆΠ΅Π½ΠΈΠ΅ Ρ‚ΠΎΠΊΠΎΠ² ΠΈ напряТСний ΠΈΠΌΠ΅Π΅Ρ‚ коэффициСнт 20, Π° ΠΎΡ‚Π½ΠΎΡˆΠ΅Π½ΠΈΠ΅ мощностСй — коэффициСнт 10.

Для напряТСний Ρ„ΠΎΡ€ΠΌΡƒΠ»Π° ΠΏΡ€ΠΈΠΎΠ±Ρ€Π΅Ρ‚Π°Π΅Ρ‚ Π²ΠΈΠ΄ , Π° для мощностСй — .
Если Π² лСсах Π§ΡƒΡ…Π»ΠΎΠΌΡ‹ Ρƒ нас Π·Π°Ρ‚Π΅Ρ€ΡΠ»ΠΎΡΡŒ ΠΊΠ°ΠΊΠΎΠ΅-Π»ΠΈΠ±ΠΎ элСктронноС устройство, Ρ‚ΠΎ Π² качСствС ΠΎΡ‚Π½ΠΎΡˆΠ΅Π½ΠΈΡ напряТСний (Π»ΠΈΠ±ΠΎ Ρ‚ΠΎΠΊΠΎΠ², Π»ΠΈΠ±ΠΎ мощностСй) принимаСтся ΠΎΡ‚Π½ΠΎΡˆΠ΅Π½ΠΈΠ΅ Π²Ρ‹Ρ…ΠΎΠ΄Π½ΠΎΠΉ Π²Π΅Π»ΠΈΡ‡ΠΈΠ½Ρ‹ ΠΊ Π²Ρ…ΠΎΠ΄Π½ΠΎΠΉ, ΠΈ это ΠΎΡ‚Π½ΠΎΡˆΠ΅Π½ΠΈΠ΅ называСтся коэффициСнтом ΠΏΠ΅Ρ€Π΅Π΄Π°Ρ‡ΠΈ, ΠΈΠ»ΠΈ коэффициСнтом прСобразования Π΄Π°Π½Π½ΠΎΠ³ΠΎ устройства.

Пока Ρ…Π²Π°Ρ‚ΠΈΡ‚, нарисуСм Ρ‚Π°Π±Π»ΠΈΡ†Ρƒ.

Π’ΠΠ‘Π›Π˜Π¦Π ΠŸΠ•Π Π•Π’ΠžΠ”Π ΠžΠ’ΠΠžΠ¨Π•ΠΠ˜Π™ Π’Π•Π›Π˜Π§Π˜Π Π’ Π”Π•Π¦Π˜Π‘Π•Π›Π›Π«


ΠšΠΎΡΡ„Ρ„ΠΈΡ†ΠΈΠ΅Π½Ρ‚ ΠΏΠ΅Ρ€Π΅Π΄Π°Ρ‡ΠΈ, Π²Ρ‹Ρ€Π°ΠΆΠ΅Π½Π½Ρ‹ΠΉ Π² Π΄Π΅Ρ†ΠΈΠ±Π΅Π»Π°Ρ…, ΠΌΠΎΠΆΠ΅Ρ‚ ΠΈΠΌΠ΅Ρ‚ΡŒ Π·Π½Π°ΠΊ плюс ΠΈΠ»ΠΈ минус Π² зависимости ΠΎΡ‚ ΡΠΎΠΎΡ‚Π½ΠΎΡˆΠ΅Π½ΠΈΡ Π²Π΅Π»ΠΈΡ‡ΠΈΠ½ Π½Π° Π²Ρ‹Ρ…ΠΎΠ΄Π΅ ΠΈ Π²Ρ…ΠΎΠ΄Π΅ (Ссли выходная Π²Π΅Π»ΠΈΡ‡ΠΈΠ½Π° большС Π²Ρ…ΠΎΠ΄Π½ΠΎΠΉ — плюс, Ссли мСньшС — минус).

А Π’Π•ΠŸΠ•Π Π¬ ΠΠΠžΠ‘ΠžΠ ΠžΠ’, Π”Π•Π¦Π˜Π‘Π•Π›Π›Π« Π’ ΠžΠ’ΠΠžΠ¨Π•ΠΠ˜Π―


Π’ случаС Π²ΠΊΠ»ΡŽΡ‡Π΅Π½ΠΈΡ ΠΏΠΎ каскадной схСмС (ΠΏΠΎΡΠ»Π΅Π΄ΠΎΠ²Π°Ρ‚Π΅Π»ΡŒΠ½ΠΎ, Π΄Ρ€ΡƒΠ³ Π·Π° Π΄Ρ€ΡƒΠ³ΠΎΠΌ) Π½Π΅ΡΠΊΠΎΠ»ΡŒΠΊΠΈΡ… устройств — ΠΎΠ±Ρ‰ΠΈΠΉ коэффициСнт ΠΏΠ΅Ρ€Π΅Π΄Π°Ρ‡ΠΈ Π² Π΄Π΅Ρ†ΠΈΠ±Π΅Π»ΡŒΠ½ΠΎΠΌ Π²Ρ‹Ρ€Π°ΠΆΠ΅Π½ΠΈΠΈ вычисляСтся простым слоТСниСм Π·Π½Π°Ρ‡Π΅Π½ΠΈΠΉ ΠšΠΏΠ΅Ρ€.(Π΄Π‘) ΠΊΠ°ΠΆΠ΄ΠΎΠ³ΠΎ ΠΈΠ· устройств.

А Ρ‚Π΅ΠΏΠ΅Ρ€ΡŒ ΠΏΠ΅Ρ€Π΅Π²Π΅Π΄Ρ‘ΠΌ Π»ΠΎΠ³Π°Ρ€ΠΈΡ„ΠΌΠΈΡ‡Π΅ΡΠΊΡƒΡŽ ΠΌΠ΅Ρ€Ρƒ мощности, ΠΈΠ·ΠΌΠ΅Ρ€ΡΠ΅ΠΌΡƒΡŽ Π² Π΄Π‘ΠΌ (dBm — Π΄Π΅Ρ†ΠΈΠ±Π΅Π» Π½Π° ΠΌΠΈΠ»Π»ΠΈΠ²Π°Ρ‚Ρ‚) Π² ΠΌΠΎΡ‰Π½ΠΎΡΡ‚ΡŒ устройства, ΠΈΠ·ΠΌΠ΅Ρ€ΡΠ΅ΠΌΡƒΡŽ Π² ΠΏΡ€ΠΈΠ²Ρ‹Ρ‡Π½Ρ‹Ρ… Π½Π°ΡˆΠ΅ΠΌΡƒ ΠΎΡ€Π³Π°Π½ΠΈΠ·ΠΌΡƒ Π²Π°Ρ‚Ρ‚Π°Ρ….
Π€ΠΎΡ€ΠΌΡƒΠ»Π° выглядит Ρ‚Π°ΠΊ: . Β Β  Для Ρ‡Π΅Π³ΠΎ Π½Π°ΠΌ сдался этот Π΄Π‘ΠΌ?
На всякий ΠΏΠΎΠΆΠ°Ρ€Π½Ρ‹ΠΉ — Π½Π΅ΠΊΠΎΡ‚ΠΎΡ€Ρ‹Π΅ ΠΏΡ€ΠΎΠΈΠ·Π²ΠΎΠ΄ΠΈΡ‚Π΅Π»ΠΈ ΡƒΠΊΠ°Π·Ρ‹Π²Π°ΡŽΡ‚ ΠΈΠΌΠ΅Π½Π½ΠΎ этот ΠΏΠ°Ρ€Π°ΠΌΠ΅Ρ‚Ρ€, характСризуя Π±ΠΎΠ³Π°Ρ‚Ρ‹Ρ€ΡΠΊΡƒΡŽ ΠΌΠΎΡ‰ΡŒ своих ΠΈΠ·Π΄Π΅Π»ΠΈΠΉ.

Π’ΠΠ‘Π›Π˜Π¦Π ΠŸΠ•Π Π•Π’ΠžΠ”Π Π”Π‘Πœ Π’ ВАВВЫ


Π’Π°ΠΊ вСдь ΠΌΠ°Π»ΠΎ Ρ‚ΠΎΠ³ΠΎ, Ρ‡Ρ‚ΠΎ ΠΌΠΎΡ‰Π½ΠΎΡΡ‚ΡŒ усилитСлСй Π½Π°Π΄ΡƒΠΌΠ°Π»ΠΈ ΠΈΠ·ΠΌΠ΅Ρ€ΡΡ‚ΡŒ Π² Π΄Π‘ΠΌ, посягнули ΠΈ Π½Π° святоС — Π½Π° Ρ‡ΡƒΠ²ΡΡ‚Π²ΠΈΡ‚Π΅Π»ΡŒΠ½ΠΎΡΡ‚ΡŒ ΠΏΡ€ΠΈΡ‘ΠΌΠ½ΠΎΠΉ Π°ΠΏΠΏΠ°Ρ€Π°Ρ‚ΡƒΡ€Ρ‹. Π§ΡƒΠ²ΡΡ‚Π²ΠΈΡ‚Π΅Π»ΡŒΠ½ΠΎΡΡ‚ΡŒ стали ΠΎΠΏΡ€Π΅Π΄Π΅Π»ΡΡ‚ΡŒ ΠΊΠ°ΠΊ ΠΎΡ‚Π½ΠΎΡˆΠ΅Π½ΠΈΠ΅ мощности Π½Π° Π²Ρ…ΠΎΠ΄Π΅ ΠΏΡ€ΠΈΡ‘ΠΌΠ½ΠΈΠΊΠ° ΠΊ ΡƒΡ€ΠΎΠ²Π½ΡŽ мощности 1 ΠΌΠ’Ρ‚ ΠΈ Ρ‚Π°ΠΊΠΆΠ΅ Π²Ρ‹Ρ€Π°ΠΆΠ°Ρ‚ΡŒ Π² логарифмичСском ΠΌΠ°ΡΡˆΡ‚Π°Π±Π΅ Π² Π΄Π‘ΠΌ.
ΠšΡƒΠ΄Π° Π΄Π΅Π²Π°Ρ‚ΡŒΡΡ Π±Π΅Π΄Π½ΠΎΠΌΡƒ ΠΊΡ€Π΅ΡΡ‚ΡŒΡΠ½ΠΈΠ½Ρƒ? ΠŸΡ€ΠΈΠ΄Ρ‘Ρ‚ΡΡ привСсти Ρ‚Π°Π±Π»ΠΈΡ†Ρƒ ΠΈ для этого бСсчинства.

Π’ΠΠ‘Π›Π˜Π¦Π ΠŸΠ•Π Π•Π’ΠžΠ”Π Π”Π‘Πœ Π’ ΠœΠ˜ΠšΠ ΠžΠ’ΠžΠ›Π¬Π’Π«

А Π΅Ρ‰Ρ‘, ΠΈΠ½ΠΎΠ³Π΄Π° Π±Ρ‹Π²Π°Π΅Ρ‚ ΠΏΠΎΠ»Π΅Π·Π½ΠΎ Π·Π½Π°Ρ‚ΡŒ, ΠΊΠ°ΠΊΠΈΠΌ Π΄ΠΎΠ»ΠΆΠ΅Π½ Π±Ρ‹Ρ‚ΡŒ Ρ€Π°Π·ΠΌΠ°Ρ… Π²Ρ‹Ρ…ΠΎΠ΄Π½ΠΎΠ³ΠΎ напряТСния Π½Π° Π½Π°Π³Ρ€ΡƒΠ·ΠΊΠ΅, для получСния Π·Π°Π΄Π°Π½Π½ΠΎΠ³ΠΎ ΠΏΠ°Ρ€Π°ΠΌΠ΅Ρ‚Ρ€Π° мощности. НСкоторыС ΠΏΡ€ΠΈ расчётС Π²Ρ‹Ρ…ΠΎΠ΄Π½ΠΎΠΉ мощности ΠΏΠΎΠ»ΡŒΠ·ΡƒΡŽΡ‚ΡΡ простой Ρ„ΠΎΡ€ΠΌΡƒΠ»ΠΎΠΉ , подставляя вмСсто UΠ΄ — ΠΏΠΈΠΊΠΎΠ²ΠΎΠ΅ Π·Π½Π°Ρ‡Π΅Π½ΠΈΠ΅ (Π°ΠΌΠΏΠ»ΠΈΡ‚ΡƒΠ΄Π½ΠΎΠ΅ Π·Π½Π°Ρ‡Π΅Π½ΠΈΠ΅, Ρ€Π°Π²Π½ΠΎΠ΅ максимальной Π°ΠΌΠΏΠ»ΠΈΡ‚ΡƒΠ΄Π΅ ΠΏΠΎΠ»ΡƒΠ²ΠΎΠ»Π½Ρ‹ Π²Ρ‹Ρ…ΠΎΠ΄Π½ΠΎΠ³ΠΎ сигнала). Π­Ρ‚ΠΎ Π½Π΅ ΠΏΡ€Π°Π²ΠΈΠ»ΡŒΠ½ΠΎ, Π²Π΅Ρ€Π½Π΅Π΅ ΠΏΡ€Π°Π²ΠΈΠ»ΡŒΠ½ΠΎ Ρ‚ΠΎΠ»ΡŒΠΊΠΎ для сигналов ΠΏΡ€ΡΠΌΠΎΡƒΠ³ΠΎΠ»ΡŒΠ½ΠΎΠΉ Ρ„ΠΎΡ€ΠΌΡ‹. Для ΡΠΈΠ½ΡƒΡΠΎΠΈΠ΄Π°Π»ΡŒΠ½Ρ‹Ρ…, для получСния Ρ‚ΠΎΡ‡Π½ΠΎΠ³ΠΎ Ρ€Π΅Π·ΡƒΠ»ΡŒΡ‚Π°Ρ‚Π° Π½Π°Π΄ΠΎ ΠΏΠΎΠ΄ΡΡ‚Π°Π²Π»ΡΡ‚ΡŒ Π΄Π΅ΠΉΡΡ‚Π²ΡƒΡŽΡ‰Π΅Π΅ Π·Π½Π°Ρ‡Π΅Π½ΠΈΠ΅ напряТСния — .
Π›ΡƒΡ‡ΡˆΠ΅ ΠΏΠΎΠ½ΡΡ‚ΡŒ, Ρ‡Ρ‚ΠΎ Ρ‚Π°ΠΊΠΎΠ΅ Π°ΠΌΠΏΠ»ΠΈΡ‚ΡƒΠ΄Π½ΠΎΠ΅ Π·Π½Π°Ρ‡Π΅Π½ΠΈΠ΅, ΠΈ ΠΊΠ°ΠΊ Π½Π°ΠΉΡ‚ΠΈ Π΄Π΅ΠΉΡΡ‚Π²ΡƒΡŽΡ‰Π΅Π΅ для Ρ€Π°Π·Π»ΠΈΡ‡Π½Ρ‹Ρ… Ρ„ΠΎΡ€ΠΌ сигналов ΠΌΠΎΠΆΠ½ΠΎ Π½Π° страницС   ссылка Π½Π° страницу.

Π—ΠΠ’Π˜Π‘Π˜ΠœΠžΠ‘Π’Π¬ ΠΠœΠŸΠ›Π˜Π’Π£Π”Π« ΠΠΠŸΠ Π―Π–Π•ΠΠ˜Π― ОВ МОЩНОБВИ

Π—ΠΠ’Π˜Π‘Π˜ΠœΠžΠ‘Π’Π¬ МОЩНОБВИ ОВ Π’Π«Π₯ΠžΠ”ΠΠžΠ“Πž ΠΠΠŸΠ Π―Π–Π•ΠΠ˜Π―

Β 

Π§Ρ‚ΠΎ Ρ‚Π°ΠΊΠΎΠ΅ Π΄Π΅Ρ†ΠΈΠ±Π΅Π», Ρ‡Ρ‚ΠΎ ΠΎΠ±ΠΎΠ·Π½Π°Ρ‡Π°Π΅Ρ‚, ΠΊΠ°ΠΊ ΠΏΠ΅Ρ€Π΅Π²ΠΎΠ΄ΠΈΡ‚ΡŒ Π² Ρ€Π°Π·Ρ‹ ΠΈ ΠΎΠ±Ρ€Π°Ρ‚Π½ΠΎ

ΠœΡ‹ часто встрСчаСм ΡƒΡ€ΠΎΠ²Π½ΠΈ Π·Π²ΡƒΠΊΠ°, ΡƒΠΊΠ°Π·Π°Π½Π½Ρ‹Π΅ Π² Π΄Π΅Ρ†ΠΈΠ±Π΅Π»Π°Ρ… β€” Π΄Π‘ ΠΈΠ»ΠΈ dBu ΠΈ ΠΏΡ€ΠΈΠ²Ρ‹ΠΊΠ»ΠΈ ΡΡ‡ΠΈΡ‚Π°Ρ‚ΡŒ Ρ‡Ρ‚ΠΎ это Π΅Π΄ΠΈΠ½ΠΈΡ†Π° измСрСния ΡˆΡƒΠΌΠ°, Π·Π²ΡƒΠΊΠ°.Β  А Π²ΠΎΡ‚ ΠΈ Π½Π΅Ρ‚. Π­Ρ‚ΠΎ совсСм Π½Π΅ Ρ‚Π°ΠΊ. Π”Π΅Ρ†ΠΈΠ±Π΅Π» β€” это Π½Π΅ Π΅Π΄ΠΈΠ½ΠΈΡ†Π° измСрСния Π²ΠΎΠΎΠ±Ρ‰Π΅. Π­Ρ‚ΠΎ ΠΎΡ‚Π½ΠΎΡΠΈΡ‚Π΅Π»ΡŒΠ½Π°Ρ характСристика, которая ΠΌΠΎΠΆΠ΅Ρ‚ ΠΎΡ‚Ρ€Π°ΠΆΠ°Ρ‚ΡŒ напряТСниС, ΠΌΠΎΡ‰Π½ΠΎΡΡ‚ΡŒ, силу Ρ‚ΠΎΠΊΠ° ΠΈ Ρ‚.Π΄. Π§Ρ‚ΠΎ-Ρ‚ΠΎ Ρ‚ΠΈΠΏΠ° ΠΏΡ€ΠΎΡ†Π΅Π½Ρ‚ΠΎΠ², Π΄ΠΎΠ»Π΅ΠΉ, Π½ΠΎ Π² логарифмичСской зависимости. Π‘Π½Π°Ρ‡Π°Π»Π° ΠΏΠΎΠ½ΡΡ‚ΡŒ, Ρ‡Ρ‚ΠΎ Ρ‚Π°ΠΊΠΎΠ΅ Π΄Π΅Ρ†ΠΈΠ±Π΅Π», нСпросто, Π½ΠΎ Π·Π°Ρ‚Π΅ΠΌ оказываСтся, Ρ‡Ρ‚ΠΎ это ΠΎΡ‡Π΅Π½ΡŒ ΡƒΠ΄ΠΎΠ±Π½ΠΎ.

Π‘ΠΎΠ΄Π΅Ρ€ΠΆΠ°Π½ΠΈΠ΅ ΡΡ‚Π°Ρ‚ΡŒΠΈ

ΠŸΠΎΠ½ΡΡ‚ΠΈΠ΅ Π΄Π΅Ρ†ΠΈΠ±Π΅Π»Π°

ΠœΡ‹ Ρ…ΠΎΡ€ΠΎΡˆΠΎ воспринимаСм ΠΈΠ·ΠΌΠ΅Ρ€Π΅Π½ΠΈΠ΅ ΠΊΠ°ΠΊΠΈΡ…-Π»ΠΈΠ±ΠΎ ΠΏΠ°Ρ€Π°ΠΌΠ΅Ρ‚Ρ€ΠΎΠ² Π² прямых Π²Π΅Π»ΠΈΡ‡ΠΈΠ½Π°Ρ…. НапримСр, напряТСниС измСряСтся Π² Π²ΠΎΠ»ΡŒΡ‚Π°Ρ…, сила Ρ‚ΠΎΠΊΠ° β€” Π² Π°ΠΌΠΏΠ΅Ρ€Π°Ρ…, сопротивлСниС β€” Π² ΠΎΠΌΠ°Ρ… ΠΈ Ρ‚.Π΄. Когда Π³ΠΎΠ²ΠΎΡ€ΠΈΠΌ ΠΎΠ± этом, всС ясно ΠΈ понятно. Когда Π³ΠΎΠ²ΠΎΡ€ΠΈΠΌ ΠΎΠ±Β  ΡƒΠ²Π΅Π»ΠΈΡ‡Π΅Π½ΠΈΠΈ ΠΈΠ»ΠΈ сниТСнии Π² этих прямых Π΅Π΄ΠΈΠ½ΠΈΡ†Π°Ρ… измСрСния Ρ‚ΠΎΠΆΠ΅ всС понятно. НапримСр, напряТСниС с 220 Π²ΠΎΠ»ΡŒΡ‚ снизилось Π΄ΠΎ 150 Π²ΠΎΠ»ΡŒΡ‚. ВсС понятно. Π’Ρ‹Ρ€Π°ΠΆΠ΅Π½ΠΈΠ΅ Β«ΠΌΠΎΡ‰Π½ΠΎΡΡ‚ΡŒ возросла Π½Π° 50 Π²Π°Ρ‚Ρ‚Β» Ρ‚ΠΎΠΆΠ΅ вопросов Π½Π΅ Π²Ρ‹Π·Ρ‹Π²Π°Π΅Ρ‚.

Как ΠΏΠΎΠ½ΡΡ‚ΡŒ Ρ‡Ρ‚ΠΎ Ρ‚Π°ΠΊΠΎΠ΅ Π΄Π΅Ρ†ΠΈΠ±Π΅Π» ΠΈ Ρ‡Ρ‚ΠΎ ΠΎΠ½ ΠΎΠ·Π½Π°Ρ‡Π°Π΅Ρ‚

Но  ΠΈΠ½ΠΎΠ³Π΄Π° говорят ΠΎΠ± ΡƒΠ²Π΅Π»ΠΈΡ‡Π΅Π½ΠΈΠΈ ΠΈΠ»ΠΈ ΡƒΠΌΠ΅Π½ΡŒΡˆΠ΅Π½ΠΈΠΈ Ρ‚ΠΎΠ³ΠΎ ΠΆΠ΅ напряТСния ΠΈΠ»ΠΈ мощности Π½Π° 2 Π΄Π΅Ρ†ΠΈΠ±Π΅Π»Π°. Как это ΠΏΠΎΠ½ΡΡ‚ΡŒ? Π§Ρ‚ΠΎ измСряСтся Π² Π΄Π΅Ρ†ΠΈΠ±Π΅Π»Π°Ρ…? Π’Π΅Π΄ΡŒ ΠΌΠΎΡ‰Π½ΠΎΡΡ‚ΡŒ мСряСм Π² Π²Π°Ρ‚Ρ‚Π°Ρ…? Как соотнСсти Π΄Π΅Ρ†ΠΈΠ±Π΅Π»Ρ‹ с Π²Π°Ρ‚Ρ‚Π°ΠΌΠΈ ΠΈΠ»ΠΈ Π²ΠΎΠ»ΡŒΡ‚Π°ΠΌΠΈ, Π°ΠΌΠΏΠ΅Ρ€Π°ΠΌΠΈ ΠΈ Π΄Ρ€ΡƒΠ³ΠΈΠΌΠΈ Π²Π΅Π»ΠΈΡ‡ΠΈΠ½Π°ΠΌΠΈ. Π’Π΅Π΄ΡŒ Ρ‚Π°ΠΊ ΠΎΠΏΠΈΡΡ‹Π²Π°ΡŽΡ‚ ΠΌΠ½ΠΎΠ³ΠΈΠ΅ ΠΏΠ°Ρ€Π°ΠΌΠ΅Ρ‚Ρ€Ρ‹. Π’ΡƒΡ‚ Π½Π°Π΄ΠΎ Ρ€Π°Π·Π±ΠΈΡ€Π°Ρ‚ΡŒΡΡ. НС ΠΎΡ‡Π΅Π½ΡŒ просто сначала ΠΏΠΎΠ½ΡΡ‚ΡŒ, Π½ΠΎ ΠΏΠΎΡ‚ΠΎΠΌ всС становится ΠΎΡ‡Π΅Π²ΠΈΠ΄Π½Ρ‹ΠΌ.

Π§Ρ‚ΠΎ Π·Π½Π°Ρ‡ΠΈΡ‚ Π±Π΅Π» ΠΈ Π΄Π΅Ρ†ΠΈΠ±Π΅Π»

Π‘Ρ€Π°Π·Ρƒ стоит ΡƒΡΡΠ½ΠΈΡ‚ΡŒ, Ρ‡Ρ‚ΠΎ Π±Π΅Π» ΠΈ Π΄Π΅Ρ†ΠΈΠ±Π΅Π» β€” это Π½Π΅ Π΅Π΄ΠΈΠ½ΠΈΡ†Ρ‹ измСрСния Ρ‡Π΅Π³ΠΎ-Π»ΠΈΠ±ΠΎ. Π­Ρ‚ΠΎ Π½Π΅ Ρ€Π΅Π·ΡƒΠ»ΡŒΡ‚Π°Ρ‚ ΠΈΠ·ΠΌΠ΅Ρ€Π΅Π½ΠΈΠΉ. Π”Π΅Ρ†ΠΈΠ±Π΅Π» β€” это Π²Π΅Π»ΠΈΡ‡ΠΈΠ½Π°, которая ΠΏΠΎΠΊΠ°Π·Ρ‹Π²Π°Π΅Ρ‚ насколько/Π²ΠΎ сколько Ρ€Π°Π· измСнился Ρ‚ΠΎΡ‚ ΠΈΠ»ΠΈ ΠΈΠ½ΠΎΠΉ ΠΏΠ°Ρ€Π°ΠΌΠ΅Ρ‚Ρ€. Π’ΠΎ Π΅ΡΡ‚ΡŒ Π±Π΅Π» ΠΈΠ»ΠΈ Π΄Π΅Ρ†ΠΈΠ±Π΅Π» β€” это ΠΎΡ‚Π½ΠΎΡΠΈΡ‚Π΅Π»ΡŒΠ½Π°Ρ Π²Π΅Π»ΠΈΡ‡ΠΈΠ½Π°, которая высчитываСтся ΠΏΡ€ΠΈ сравнСнии Π΄Π²ΡƒΡ… ΠΈΠ·ΠΌΠ΅Ρ€Π΅Π½ΠΈΠΉ ΠΎΠ΄Π½ΠΎΠ³ΠΎ ΠΈ Ρ‚ΠΎΠ³ΠΎ ΠΆΠ΅ ΠΏΠ°Ρ€Π°ΠΌΠ΅Ρ‚Ρ€Π°.

НапримСр, Π½Π° рисункС Π΄Π°Π½ Π³Ρ€Π°Ρ„ΠΈΠΊ, ΠΊΠΎΡ‚ΠΎΡ€Ρ‹ΠΉ построили ΠΏΠΎ Ρ€Π΅Π·ΡƒΠ»ΡŒΡ‚Π°Ρ‚Π°ΠΌ измСрСния напряТСниС Π½Π° Π²Ρ‹Ρ…ΠΎΠ΄Π΅ ΠΏΡ€ΠΈΠ±ΠΎΡ€Π° ΠΏΡ€ΠΈ ΠΈΠ·ΠΌΠ΅Π½Π΅Π½ΠΈΠΈ ΠΏΠΎΠ΄Π°Π²Π°Π΅ΠΌΠΎΠΉ Π½Π° Π²Ρ…ΠΎΠ΄ частоты (АЧΠ₯). Бняты Π±Ρ‹Π»ΠΈ характСристики ΠΏΡ€ΠΈ ΡƒΡ€ΠΎΠ²Π½Π΅ сигнала 1 VΒ  (Π³Ρ€Π°Ρ„ΠΈΠΊ 1) ΠΈ 100 ΠΌV (Π³Ρ€Π°Ρ„ΠΈΠΊ 2). Если ΡΠΌΠΎΡ‚Ρ€Π΅Ρ‚ΡŒ Π½Π° Π³Ρ€Π°Ρ„ΠΈΠΊΠΈ прямых ΠΈΠ·ΠΌΠ΅Ρ€Π΅Π½ΠΈΠΉ, ΠΏΠΎΠ½ΡΡ‚ΡŒ Ρ‡Ρ‚ΠΎ-Ρ‚ΠΎ слоТно. На Π²Ρ‚ΠΎΡ€ΠΎΠΌ рисункС Π³Ρ€Π°Ρ„ΠΈΠΊ построСн Π² Π΄Π΅Ρ†ΠΈΠ±Π΅Π»Π°Ρ…. На этом Π³Ρ€Π°Ρ„ΠΈΠΊΠ΅ ΠΎΡ‡Π΅Π²ΠΈΠ΄Π½ΠΎ, Ρ‡Ρ‚ΠΎ рСакция ΠΏΡ€ΠΈΠ±ΠΎΡ€Π° одинаковая, измСнился Ρ‚ΠΎΠ»ΡŒΠΊΠΎ ΡƒΡ€ΠΎΠ²Π΅Π½ΡŒ сигнала Π½Π° Π²Ρ‹Ρ…ΠΎΠ΄Π΅, Ρ‡Ρ‚ΠΎ ΠΈ понятно.

Π”Π²Π° Π³Ρ€Π°Ρ„ΠΈΠΊΠ° ΠΈΠ·ΠΌΠ΅Ρ€Π΅Π½ΠΈΠΉ. Π›Π΅Π²Ρ‹ΠΉ β€” прямая Π·Π°Π²ΠΈΡΠΈΠΌΠΎΡΡ‚ΡŒ (напряТСния ΠΎΡ‚ частоты), ΠΏΡ€Π°Π²Ρ‹ΠΉ β€” ΠΈΠ·ΠΌΠ΅Π½Π΅Π½ΠΈΠ΅ напряТСния Π² Π΄Π‘ ΠΏΡ€ΠΈ ΠΈΠ·ΠΌΠ΅Π½Π΅Π½ΠΈΠΈ частоты

ΠŸΠ΅Ρ€Π²ΠΎΠ½Π°Ρ‡Π°Π»ΡŒΠ½ΠΎ стали ΠΈΡΠΏΠΎΠ»ΡŒΠ·ΠΎΠ²Π°Ρ‚ΡŒ Π΅Π΄ΠΈΠ½ΠΈΡ†Ρƒ Π‘Π΅Π». ΠœΠ΅ΠΆΠ΄ΡƒΠ½Π°Ρ€ΠΎΠ΄Π½ΠΎΠ΅ ΠΎΠ±ΠΎΠ·Π½Π°Ρ‡Π΅Π½ΠΈΠ΅ Π±Π΅Π»Π° β€” B, российскоС β€” Π‘ (Π½Π°ΠΏΡ€ΠΈΠΌΠ΅Ρ€, 10Π‘ ΠΈΠ»ΠΈ 10B). Но Π±ΠΎΠ»Π΅Π΅ ΡƒΠ΄ΠΎΠ±Π½Ρ‹ΠΌ оказалось ΠΏΡ€ΠΈΠΌΠ΅Π½Π΅Π½ΠΈΠ΅ ΠΎΠ΄Π½ΠΎΠΉ Π΅Π΅ дСсятой Π΄ΠΎΠ»ΠΈ β€” Π΄Π΅Ρ†ΠΈΠ±Π΅Π»Π° ΠΈΠ»ΠΈ Π΄Π‘ Π² российском ΠΎΠ±ΠΎΠ·Π½Π°Ρ‡Π΅Π½ΠΈΠΈ ΠΈ dBu Π² ΠΌΠ΅ΠΆΠ΄ΡƒΠ½Π°Ρ€ΠΎΠ΄Π½ΠΎΠΌ. Π’ΠΎ Π΅ΡΡ‚ΡŒ ΠΎΠ΄ΠΈΠ½ Π΄Π΅Ρ†ΠΈΠ±Π΅Π» β€” это 0,1 Π‘Π΅Π»Π°.

Π”Π°Π»ΡŒΡˆΠ΅, ΠΊ соТалСнию, Π±Π΅Π· ΠΌΠ°Ρ‚Π΅ΠΌΠ°Ρ‚ΠΈΠΊΠΈ Π½Π΅ ΠΎΠ±ΠΎΠΉΡ‚ΠΈΡΡŒ. ΠŸΡ€ΠΈΠ΄Π΅Ρ‚ΡΡ Π²ΡΠΏΠΎΠΌΠ½ΠΈΡ‚ΡŒ Ρ‡Ρ‚ΠΎ Ρ‚Π°ΠΊΠΎΠ΅ дСсятичный Π»ΠΎΠ³Π°Ρ€ΠΈΡ„ΠΌ. ДСсятичный Π»ΠΎΠ³Π°Ρ€ΠΈΡ„ΠΌ ΠΏΠΎΠΊΠ°Π·Ρ‹Π²Π°Π΅Ρ‚, Π² ΠΊΠ°ΠΊΡƒΡŽ ΡΡ‚Π΅ΠΏΠ΅Π½ΡŒ Π½Π°Π΄ΠΎ возвСсти число 10 Ρ‡Ρ‚ΠΎΠ±Ρ‹ ΠΏΠΎΠ»ΡƒΡ‡ΠΈΡ‚ΡŒ Ρ‚Ρ€Π΅Π±ΡƒΠ΅ΠΌΡƒΡŽ Ρ†ΠΈΡ„Ρ€Ρƒ. На рисункС Π²Ρ‹ Π²ΠΈΠ΄ΠΈΡ‚Π΅ ΡΠΎΠΎΡ‚Π½ΠΎΡˆΠ΅Π½ΠΈΠ΅, Π²ΠΎΠ·ΠΌΠΎΠΆΠ½ΠΎ Π±ΡƒΠ΄Π΅Ρ‚ понятнСС Π² Ρ‚Π°ΠΊΠΎΠΌ Π²ΠΈΠ΄Π΅.

НСсколько Π·Π½Π°Ρ‡Π΅Π½ΠΈΠΉ дСсятичных Π»ΠΎΠ³Π°Ρ€ΠΈΡ„ΠΌΠΎΠ²

Π’Π΅ΠΏΠ΅Ρ€ΡŒ, собствСнно ΠΎ Π‘Π΅Π»Π°Ρ… ΠΈ Π΄Π΅Ρ†ΠΈΠ±Π΅Π»Π°Ρ…. Если Π³ΠΎΠ²ΠΎΡ€ΠΈΡ‚ΡŒ ΠΎΠ± опрСдСлСниях, Ρ‚ΠΎ Π‘Π΅Π» β€” это дСсятичный Π»ΠΎΠ³Π°Ρ€ΠΈΡ„ΠΌ ΠΎΡ‚Π½ΠΎΡˆΠ΅Π½ΠΈΡ Π΄Π²ΡƒΡ… Π²Π΅Π»ΠΈΡ‡ΠΈΠ½. Π›ΡŽΠ±Ρ‹Ρ… Π΄Π²ΡƒΡ… Π²Π΅Π»ΠΈΡ‡ΠΈΠ½. ΠœΠΎΡ‰Π½ΠΎΡΡ‚Π΅ΠΉ, напряТСния, силы Π·Π²ΡƒΠΊΠ°, частоты ΠΈ Ρ‚.Π΄. Π”Π°Π²Π°ΠΉΡ‚Π΅ Π½Π° ΠΏΡ€ΠΈΠΌΠ΅Ρ€Π΅. Надо ΠΏΠΎΠ½ΡΡ‚ΡŒ, Ρ‡Ρ‚ΠΎ Π²Ρ‹Π΄Π°Π΅Ρ‚ ΠΏΡ€ΠΈΠ±ΠΎΡ€ Π½Π° Π²Ρ‹Ρ…ΠΎΠ΄Π΅ ΠΏΡ€ΠΈ ΠΈΠ·ΠΌΠ΅Π½Π΅Π½ΠΈΠΈ ΠΏΠ°Ρ€Π°ΠΌΠ΅Ρ‚Ρ€ΠΎΠ² Π½Π° Π²Ρ…ΠΎΠ΄Π΅. Π’Ρ‹Π±ΠΈΡ€Π°ΡŽΡ‚ ΠΊΠ°ΠΊΡƒΡŽ-Ρ‚ΠΎ Ρ‚ΠΎΡ‡ΠΊΡƒ отсчСта β€” Π±Π°Π·Ρƒ. Π—Π°Ρ‚Π΅ΠΌ ΠΈΠ·ΠΌΠ΅Π½ΡΡŽΡ‚ ΠΏΠ°Ρ€Π°ΠΌΠ΅Ρ‚Ρ€, проводят ΠΈΠ·ΠΌΠ΅Ρ€Π΅Π½ΠΈΠ΅ Ρ€Π΅Π·ΡƒΠ»ΡŒΡ‚Π°Ρ‚Π°, дСлят Π΅Π³ΠΎ Π½Π° Β«Π±Π°Π·ΡƒΒ» ΠΈ Π±Π΅Ρ€ΡƒΡ‚ дСсятичный Π»ΠΎΠ³Π°Ρ€ΠΈΡ„ΠΌ. ΠŸΠΎΠ»ΡƒΡ‡Π°ΡŽΡ‚ Ρ€Π΅Π·ΡƒΠ»ΡŒΡ‚Π°Ρ‚ измСрСния Π² Π΄Π΅Ρ†ΠΈΠ±Π΅Π»Π°Ρ…. Π’Π°ΠΊ ΠΈΠ·ΠΌΠ΅Ρ€ΡΡŽΡ‚ ΠΏΠ°Ρ€Π°ΠΌΠ΅Ρ‚Ρ€Ρ‹, ΠΏΠ΅Ρ€Π΅ΡΡ‡ΠΈΡ‚Ρ‹Π²Π°ΡŽΡ‚ Π² Π΄Π΅Ρ†ΠΈΠ±Π΅Π»Ρ‹ ΠΈ строят зависимости.

Π€ΠΎΡ€ΠΌΡƒΠ»Π°, которая поясняСт Ρ‡Ρ‚ΠΎ Ρ‚Π°ΠΊΠΎΠ΅ Π΄Π‘ (Π΄Π΅Ρ†ΠΈΠ±Π΅Π») ΠΈ ΠΊΠ°ΠΊ ΠΈΡ… ΡΡ‡ΠΈΡ‚Π°ΡŽΡ‚

На рисункС Π΄Π°Π½Ρ‹ Π΄Π²Π΅ Ρ„ΠΎΡ€ΠΌΡƒΠ»Ρ‹ β€” для вычислСния энСргСтичСских Π²Π΅Π»ΠΈΡ‡ΠΈΠ½ (ΠΏΠΎ мощности) ΠΈ Π°ΠΌΠΏΠ»ΠΈΡ‚ΡƒΠ΄Π½Ρ‹Ρ… (ΠΏΠΎ Π½Π°ΠΏΡ€ΡΠΆΠ΅Π½ΠΈΡŽ). Как Π²ΠΈΠ΄ΠΈΡ‚Π΅, ΠΎΠ½ΠΈ ΠΎΡ‚Π»ΠΈΡ‡Π°ΡŽΡ‚ΡΡ Ρ‚ΠΎΠ»ΡŒΠΊΠΎΒ  коэффициСнтом. U1 Ρ‚ΡƒΡ‚ β€” это Ρ€Π΅Π·ΡƒΠ»ΡŒΡ‚Π°Ρ‚ ΠΈΠ·ΠΌΠ΅Ρ€Π΅Π½ΠΈΠΉ, Π° Uo β€” базовая Π²Π΅Π»ΠΈΡ‡ΠΈΠ½Π°, с ΠΊΠΎΡ‚ΠΎΡ€Ρ‹ΠΉ ΡΡ€Π°Π²Π½ΠΈΠ²Π°ΡŽΡ‚ ΠΈΠ·ΠΌΠ΅Ρ€Π΅Π½ΠΈΠΉ.

ΠŸΠΎΡ‡Π΅ΠΌΡƒ ΠΈΡΠΏΠΎΠ»ΡŒΠ·ΡƒΡŽΡ‚ Π΄Π΅Ρ†ΠΈΠ±Π΅Π»Ρ‹, Π° Π½Π΅ прямыС Π²Π΅Π»ΠΈΡ‡ΠΈΠ½Ρ‹

ИспользованиС логарифмичСских зависимостСй часто Π±ΠΎΠ»Π΅Π΅ понятно ΠΈ нСсСт большС ΠΈΠ½Ρ„ΠΎΡ€ΠΌΠ°Ρ†ΠΈΠΈ, Ρ‡Π΅ΠΌ прямыС измСрСния. Π­Ρ‚ΠΎ Π²ΠΈΠ΄Π½ΠΎ Π½Π° ΠΏΡ€ΠΈΠΌΠ΅Ρ€Π΅ построСния Π³Ρ€Π°Ρ„ΠΈΠΊΠΎΠ² Π°ΠΌΠΏΠ»ΠΈΡ‚ΡƒΠ΄Π½ΠΎ-частотной характСристики. И Ρ‚Π°ΠΊΠΎΠΉ случай Π½Π΅ Π΅Π΄ΠΈΠ½ΠΈΡ‡Π½Ρ‹ΠΉ, ΠΌΠ½ΠΎΠ³ΠΈΠ΅ зависимости Π±ΠΎΠ»Π΅Π΅ ΠΈΠ½Ρ„ΠΎΡ€ΠΌΠ°Ρ‚ΠΈΠ²Π½Ρ‹ Π² логарифмичСской зависимости.

ΠšΡ€ΠΎΠΌΠ΅ Ρ‚ΠΎΠ³ΠΎ, Π΄Π΅Ρ†ΠΈΠ±Π΅Π»Ρ‹ ΠΈΡΠΏΠΎΠ»ΡŒΠ·ΡƒΡŽΡ‚ Π² Ρ‚Π΅Ρ… областях, Π³Π΄Π΅ ΠΏΠ°Ρ€Π°ΠΌΠ΅Ρ‚Ρ€Ρ‹ ΠΈΠ·ΠΌΠ΅Π½ΡΡŽΡ‚ΡΡ Π² ΠΎΡ‡Π΅Π½ΡŒ ΡˆΠΈΡ€ΠΎΠΊΠΎΠΌ Π΄ΠΈΠ°ΠΏΠ°Π·ΠΎΠ½Π΅. Π‘ΠΎΠ»Π΅Π΅ понятна Π½Π°ΠΌ ситуация со Π·Π²ΡƒΠΊΠ°ΠΌΠΈ. Π§Π΅Π»ΠΎΠ²Π΅ΠΊ Π² состоянии Π²ΠΎΡΠΏΡ€ΠΈΠ½ΠΈΠΌΠ°Ρ‚ΡŒ частоты ΠΎΡ‚ 20 Π“Ρ† Π΄ΠΎ 20 000 Π“Ρ†. НичСго сСбС разброс! Π’ тысячу Ρ€Π°Π·.

Π˜Π½Ρ‚Π΅Π½ΡΠΈΠ²Π½ΠΎΡΡ‚ΡŒ Π·Π²ΡƒΠΊΠ° ΠΈ Π΅Π³ΠΎ соотвСтствиС Π² дБА

Π‘ ΡƒΡ€ΠΎΠ²Π½Π΅ΠΌ Π·Π²ΡƒΠΊΠ° Π΅Ρ‰Π΅ ΠΊΡ€ΡƒΡ‡Π΅. НиТний ΠΏΡ€Π΅Π΄Π΅Π» восприятия β€” 10-12 Π’Ρ‚/ΠΌ, Π° ΡƒΡ€ΠΎΠ²Π΅Π½ΡŒ, ΠΏΡ€ΠΈ ΠΊΠΎΡ‚ΠΎΡ€ΠΎΠΌ Π²ΠΎΠ·Π½ΠΈΠΊΠ°Π΅Ρ‚ боль β€” 10 Π’Ρ‚/ΠΌ. Π’ΠΎ Π΅ΡΡ‚ΡŒ, Π΄ΠΈΠ°ΠΏΠ°Π·ΠΎΠ½ измСрСния Π·Π½Π°Ρ‡Π΅Π½ΠΈΠΉ β€” 13 порядков. Π­Ρ‚ΠΎ 10 000 000 000 000 Ρ€Π°Π·. ΠžΠΏΠ΅Ρ€ΠΈΡ€ΠΎΠ²Π°Ρ‚ΡŒ Ρ‚Π°ΠΊΠΈΠΌΠΈ Ρ†ΠΈΡ„Ρ€Π°ΠΌΠΈ, ΠΊΠ°ΠΊ ΠΌΠΈΠ½ΠΈΠΌΡƒΠΌ Π½Π΅ΡƒΠ΄ΠΎΠ±Π½ΠΎ. Π‘ использованиСм ΠΎΡ‚Π½ΠΎΡΠΈΡ‚Π΅Π»ΡŒΠ½Ρ‹Ρ… Π²Π΅Π»ΠΈΡ‡ΠΈΠ½ β€” Π΄Π΅Ρ†ΠΈΠ±Π΅Π» β€” Ρ†ΠΈΡ„Ρ€Ρ‹ ΠΏΠΎΠ»ΡƒΡ‡Π°ΡŽΡ‚ΡΡ Π·Π½Π°Ρ‡ΠΈΡ‚Π΅Π»ΡŒΠ½ΠΎ мСньшС, Ρ€Π°Π±ΠΎΡ‚Π°Ρ‚ΡŒ, Π²ΠΎΡΠΏΡ€ΠΈΠ½ΠΈΠΌΠ°Ρ‚ΡŒ ΠΈ Π·Π°ΠΏΠΎΠΌΠΈΠ½Π°Ρ‚ΡŒ ΠΈΡ… Π»Π΅Π³Ρ‡Π΅. НСсколько ΠΏΡ€ΠΈΠΌΠ΅Ρ€ΠΎΠ²:

  • Если ΠΏΠΎΠΊΠ°Π·Π°Ρ‚Π΅Π»ΡŒ увСличился Π² 10 Ρ€Π°Π·, говорят, Ρ‡Ρ‚ΠΎ ΠΎΠ½ увСличился Π½Π° 1 Π‘Π΅Π».
  • Если Ρ‚ΠΎΡ‚ ΠΆΠ΅ ΠΏΠΎΠΊΠ°Π·Π°Ρ‚Π΅Π»ΡŒ увСличился Π² 100 Ρ€Π°Π·, Ρ‚ΠΎ говорят ΠΎΠ± ΡƒΠ²Π΅Π»ΠΈΡ‡Π΅Π½ΠΈΠΈ Π½Π° 2 Π‘Π΅Π»Π°.
  • Π£Π²Π΅Π»ΠΈΡ‡Π΅Π½ΠΈΠ΅ Π² 100 000 тысяч Ρ€Π°Π·Β  β€” всСго Π½Π° 5 Π‘Π΅Π».

Π—Π°ΠΌΠ΅Ρ‚ΠΈΠ»ΠΈ Ρ€Π°Π·Π½ΠΈΡ†Ρƒ? ΠŸΠΎΠΊΠ°Π·Π°Ρ‚Π΅Π»ΡŒ увСличился Π² 100 Ρ€Π°Π·, Π° Π² Π±Π΅Π»Π°Ρ… увСличился Π½Π° 2 Π‘.Β  Π­Ρ‚ΠΎ ΡƒΠ΄ΠΎΠ±Π½Π΅Π΅. Π‘ΠΎΠ³Π»Π°ΡΠΈΡ‚Π΅ΡΡŒ, ΠΏΡ€ΠΎΡ‰Π΅ ΠΎΠΏΠ΅Ρ€ΠΈΡ€ΠΎΠ²Π°Ρ‚ΡŒ Π΅Π΄ΠΈΠ½ΠΈΡ†Π°ΠΌΠΈ, Ρ‡Π΅ΠΌ сотнями тысяч. Π’Π°ΠΆΠ½ΠΎ просто ΠΏΠΎΠ½ΠΈΠΌΠ°Ρ‚ΡŒ смысл сказанного. ΠŸΡ€ΠΈ возрастании прямых Π²Π΅Π»ΠΈΡ‡ΠΈΠ½ ΠΈΡ… Π½Π°Π΄ΠΎ ΡƒΠΌΠ½ΠΎΠΆΠ°Ρ‚ΡŒ Π½Π° Ρ‚ΠΎ число, Π½Π° ΠΊΠΎΡ‚ΠΎΡ€ΠΎΠ΅ ΠΏΠ°Ρ€Π°ΠΌΠ΅Ρ‚Ρ€ увСличился. ΠŸΡ€ΠΈ ΠΎΠΏΠ΅Ρ€ΠΈΡ€ΠΎΠ²Π°Π½ΠΈΠΈ Π΄Π΅Ρ†ΠΈΠ±Π΅Π»Π°ΠΌΠΈ ΠΈΡ… ΡΠΊΠ»Π°Π΄Ρ‹Π²Π°ΡŽΡ‚. Π‘ΠΎΠ³Π»Π°ΡΠΈΡ‚Π΅ΡΡŒ, это ΠΏΡ€ΠΎΡ‰Π΅.

Π§Ρ‚ΠΎ Ρ‚Π°ΠΊΠΎΠ΅ dBm, dBv, dBA (Π΄Π‘ΠΌ, Π΄Π‘Π², дБА)

Как Π²Ρ‹ ΡƒΠΆΠ΅ поняли, Π΄Π΅Ρ†ΠΈΠ±Π΅Π» β€” это ΠΎΡ‚Π½ΠΎΡΠΈΡ‚Π΅Π»ΡŒΠ½Π°Ρ Π²Π΅Π»ΠΈΡ‡ΠΈΠ½Π° ΠΈ ΠΎΡ‚Ρ€Π°ΠΆΠ°Ρ‚ΡŒ ΠΎΠ½Π° ΠΌΠΎΠΆΠ΅Ρ‚ Ρ‡Ρ‚ΠΎ ΡƒΠ³ΠΎΠ΄Π½ΠΎ. Надо Ρ‚ΠΎΠ»ΡŒΠΊΠΎ Π²Ρ‹Π±Ρ€Π°Ρ‚ΡŒ Ρ‚ΠΎΡ‡ΠΊΡƒ отсчСта, Π±Π°Π·Ρƒ, эталон, с ΠΊΠΎΡ‚ΠΎΡ€Ρ‹ΠΌ ΡΡ€Π°Π²Π½ΠΈΠ²Π°ΡŽΡ‚ всС ΠΏΠΎΡΠ»Π΅Π΄ΡƒΡŽΡ‰ΠΈΠ΅ измСнСния. Π‘Π°Π·Π° для сравнСния ΠΌΠΎΠΆΠ΅Ρ‚ Π±Ρ‹Ρ‚ΡŒ взята ΠΏΡ€ΠΎΠΈΠ·Π²ΠΎΠ»ΡŒΠ½ΠΎ. Но Ρ‚ΠΎΠ³Π΄Π° нСпонятно ΠΊΠ°ΠΊ ΡΠΎΠΎΡ‚Π½ΠΎΡΠΈΡ‚ΡŒ Ρ€Π°Π·Π½Ρ‹Π΅ измСрСния. Π’ Ρ‚Π°ΠΊΠΎΠΌ случаС, ΠΎΠ±Ρ‹Ρ‡Π½ΠΎ, ΡƒΠΊΠ°Π·Ρ‹Π²Π°ΡŽΡ‚ ΠΎΡ‚Π½ΠΎΡΠΈΡ‚Π΅Π»ΡŒΠ½ΠΎ Ρ‡Π΅Π³ΠΎ считался Π»ΠΎΠ³Π°Ρ€ΠΈΡ„ΠΌ. Π’ΠΎ Π΅ΡΡ‚ΡŒ, Ρ‡Ρ‚ΠΎ подставляли Π² Π·Π½Π°ΠΌΠ΅Π½Π°Ρ‚Π΅Π»ΡŒ (Π² Ρ„ΠΎΡ€ΠΌΡƒΠ»Π΅ Π²Ρ‹ΡˆΠ΅ это Uo).

Для элСктротСхники ΠΈ мощностСй Π±Ρ‹Π»ΠΈ Π²Ρ‹Π±Ρ€Π°Π½Ρ‹ Π±Π°Π·ΠΎΠ²Ρ‹Π΅ Ρ‚ΠΎΡ‡ΠΊΠΈ отсчСта β€” Π΄Π²Π΅ Π²Π΅Π»ΠΈΡ‡ΠΈΠ½Ρ‹ напряТСния, с ΠΊΠΎΡ‚ΠΎΡ€Ρ‹ΠΌΠΈ ΡΡ€Π°Π²Π½ΠΈΠ²Π°ΡŽΡ‚ Π±ΠΎΠ»ΡŒΡˆΡƒΡŽ Ρ‡Π°ΡΡ‚ΡŒΒ  ΠΈΠ·ΠΌΠ΅Ρ€Π΅Π½ΠΈΠΉ элСктричСских Π²Π΅Π»ΠΈΡ‡ΠΈΠ½.

  • Основная Π±Π°Π·Π° β€” это ΠΌΠΎΡ‰Π½ΠΎΡΡ‚ΡŒ Π² ΠΎΠ΄ΠΈΠ½ ΠΌΠΈΠ»Π»ΠΈΠ²Π°Ρ‚Ρ‚ (1 ΠΌΠ’Ρ‚)Β  ΠΏΡ€ΠΈ Π½Π°Π³Ρ€ΡƒΠ·ΠΊΠ΅ 600 Ом. Если ΠΏΠ΅Ρ€Π΅ΡΡ‡ΠΈΡ‚Π°Ρ‚ΡŒ, Ρ‚ΠΎ напряТСниС ΠΏΠΎΠ»ΡƒΡ‡Π°Π΅ΠΌ 0,775 Π’ΠΎΠ»ΡŒΡ‚Π°. ИмСнно эти значСния ΠΈ ΡΠ²Π»ΡΡŽΡ‚ΡΡ Ρ‚ΠΎΠΉ Π±Π°Π·ΠΎΠΉ, ΠΎΡ‚Π½ΠΎΡΠΈΡ‚Π΅Π»ΡŒΠ½ΠΎ ΠΊΠΎΡ‚ΠΎΡ€ΠΎΠΉ Π²Ρ‹ΡΡ‡ΠΈΡ‚Ρ‹Π²Π°ΡŽΡ‚ Π»ΠΎΠ³Π°Ρ€ΠΈΡ„ΠΌΡ‹. Π­Ρ‚ΠΎ принято ΠΈ Π² ΠΌΠ΅ΠΆΠ΄ΡƒΠ½Π°Ρ€ΠΎΠ΄Π½Ρ‹Ρ… измСрСниях, ΠΈ Π² отСчСствСнных. ИмСнно ΠΏΡ€ΠΈ использовании Ρ‚Π°ΠΊΠΎΠΉ Π±Π°Π·Ρ‹ ставят ΠΎΠ±ΠΎΠ·Π½Π°Ρ‡Π΅Π½ΠΈΠ΅ dBu ΠΈΠ»ΠΈ Π΄Π‘ Π² русском Π²Π°Ρ€ΠΈΠ°Π½Ρ‚Π΅. Π Π΅ΠΆΠ΅ встрСчаСтся ΠΎΠ±ΠΎΠ·Π½Π°Ρ‡Π΅Π½ΠΈΠ΅ dBm. Π­Ρ‚ΠΎ Ρ‚ΠΎΠΆΠ΅, Ρ‡Ρ‚ΠΎ dBu.
  • Иногда Π²Ρ‹Ρ…ΠΎΠ΄Π½ΠΎΠ΅ напряТСниС ΡΡ€Π°Π²Π½ΠΈΠ²Π°ΡŽΡ‚ с 1 Π’. Π’ этом случаС Ρ€Π΅Π·ΡƒΠ»ΡŒΡ‚Π°Ρ‚ ΠΏΠΎΠ΄ΠΏΠΈΡΡ‹Π²Π°ΡŽΡ‚ ΠΊΠ°ΠΊ dBv ΠΈΠ»ΠΈ Π΄Π‘Π².

На Ρ‡Ρ‚ΠΎ влияСт Ρ‚ΠΎΡ‡ΠΊΠ° отсчСта? ΠŸΡ€ΠΎΡΡ‚ΠΎ Π½Π° ΡƒΡ€ΠΎΠ²Π΅Π½ΡŒ, Π½Π° ΠΊΠΎΡ‚ΠΎΡ€ΠΎΠΌ строится Π·Π°Π²ΠΈΡΠΈΠΌΠΎΡΡ‚ΡŒ. Если ΠΆΠ΅ ΠΏΠΎ Π΄Π°Π½Π½Ρ‹ΠΌ ΠΏΠΎΡΡ‚Ρ€ΠΎΠΈΡ‚ΡŒ Π³Ρ€Π°Ρ„ΠΈΠΊ, ΠΎΠ½ Π±ΡƒΠ΄Π΅Ρ‚ ΠΈΠΌΠ΅Ρ‚ΡŒ Ρ‚Ρƒ ΠΆΠ΅ Ρ„ΠΎΡ€ΠΌΡƒ.

ΠŸΡ€ΠΈ описании Π·Π²ΡƒΠΊΠΎΠ² ΠΈ ΡˆΡƒΠΌΠΎΠ² ΡƒΠΏΠΎΡ‚Ρ€Π΅Π±Π»ΡΡŽΡ‚ дБА (dBA) ΠΈΠ»ΠΈ акустичСскиС Π΄Π΅Ρ†ΠΈΠ±Π΅Π»Ρ‹. ΠŸΡ€ΠΈ Ρ‚Π°ΠΊΠΎΠΌ исчислСнии Π·Π° Ρ‚ΠΎΡ‡ΠΊΡƒ отсчСта Π±Π΅Ρ€ΡƒΡ‚ Π½ΠΈΠΆΠ½ΠΈΠΉ ΠΏΠΎΡ€ΠΎΠ³Β  ΡΠ»Ρ‹ΡˆΠΈΠΌΠΎΡΡ‚ΠΈ ΠΈΠ»ΠΈ частоты, ΠΊΠΎΡ‚ΠΎΡ€ΡƒΡŽ Ρ€Π°Π·Π»ΠΈΡ‡Π°Π΅Ρ‚ чСловСчСскоС ΡƒΡ…ΠΎ. Π­Ρ‚ΠΎ 2Β·10-5 Па ΠΈ ΠΎΡ‚Π½ΠΎΡΠΈΡ‚Π΅Π»ΡŒΠ½ΠΎ Π½Π΅Π΅ Π²Ρ‹Ρ‡ΠΈΡΠ»ΡΡŽΡ‚ ΠΎΡ‚Π½ΠΎΡˆΠ΅Π½ΠΈΠ΅.

Как ΡΡ‡ΠΈΡ‚Π°ΡŽΡ‚ Π΄Π΅Ρ†ΠΈΠ±Π΅Π»Ρ‹

Π‘ΠΎΠ»ΡŒΡˆΠ΅ Π² Ρ…ΠΎΠ΄Ρƒ Π½Π΅ Π‘Π΅Π»Ρ‹, Π° ΠΈΡ… дСсятая Ρ‡Π°ΡΡ‚ΡŒ β€” Π΄Π΅Ρ†ΠΈΠ±Π΅Π»Ρ‹ (ΠΎΠ±ΠΎΠ·Π½Π°Ρ‡Π΅Π½ΠΈΠ΅ dB ΠΈΠ»ΠΈ Π΄Π‘). Π’Π΅Π΄ΡŒ Ρ‡Π°Ρ‰Π΅ ΡƒΠ²Π΅Π»ΠΈΡ‡Π΅Π½ΠΈΠ΅ Π½Π΅ Π² сотни ΠΈ тысячи Ρ€Π°Π·, Π° Ρ‡ΡƒΡ‚ΡŒ поскромнСС. Π’Π°ΠΊ Ρ‡Ρ‚ΠΎ ΠΎΠ±Ρ‹Ρ‡Π½ΠΎ говорят ΠΎΠ± ΡƒΠ²Π΅Π»ΠΈΡ‡Π΅Π½ΠΈΠΈ Ρ‚ΠΎΠ³ΠΎ ΠΈΠ»ΠΈ ΠΈΠ½ΠΎΠ³ΠΎ показания ΠΈΠ»ΠΈ характСристики Π½Π° 5 Π΄Π‘ ΠΈΠ»ΠΈ Π½Π° 10 Π΄Π‘.

ΠŸΡ€ΠΈΠΌΠ΅Ρ€ соотвСтствия Π΄Π΅Ρ†ΠΈΠ±Π΅Π» ΠΈ Β«Ρ€Π°Π·Β»

Но Π²Π°ΠΆΠ½ΠΎ ΠΏΠΎΠΌΠ½ΠΈΡ‚ΡŒ, Ρ‡Ρ‚ΠΎ описанная Π²Ρ‹ΡˆΠ΅ прямая Π·Π°Π²ΠΈΡΠΈΠΌΠΎΡΡ‚ΡŒ Ρ…Π°Ρ€Π°ΠΊΡ‚Π΅Ρ€Π½Π° Ρ‚ΠΎΠ»ΡŒΠΊΠΎ для энСргСтичСских Π²Π΅Π»ΠΈΡ‡ΠΈΠ½ (это Ссли ΠΌΠΎΡ‰Π½ΠΎΡΡ‚ΡŒ возросла Π² 10 Ρ€Π°Π·, Ρ‚ΠΎ ΠΎΠ½Π° ΡƒΠ²Π΅Π»ΠΈΡ‡ΠΈΠ»Π°ΡΡŒ Π½Π° 10 Π΄Π‘).Β  Для Π΄Ρ€ΡƒΠ³ΠΈΡ… Π·Π°Π²ΠΈΡΠΈΠΌΠΎΡΡ‚ΡŒ Ρ‚ΠΎΠΆΠ΅ логарифмичСская, Π½ΠΎ вычисляСтся ΠΏΠΎ Π΄Ρ€ΡƒΠ³ΠΎΠΉ Ρ„ΠΎΡ€ΠΌΡƒΠ»Π΅. И это Π½Π°Π΄ΠΎ ΠΏΠΎΠΌΠ½ΠΈΡ‚ΡŒ.

Π”Π΅Ρ†ΠΈΠ±Π΅Π»Ρ‹Π‘ΠΎΠΎΡ‚Π½ΠΎΡˆΠ΅Π½ΠΈΠ΅ ΠΌΠΎΡ‰Π½ΠΎΡΡ‚Π΅ΠΉΠ‘ΠΎΠΎΡ‚Π½ΠΎΡˆΠ΅Π½ΠΈΠ΅ Π°ΠΌΠΏΠ»ΠΈΡ‚ΡƒΠ΄
-30,50,7
-60,250,5
-100,10,3
-201,010,1
-250,030,05
-400,010,0001
-600,0010,000001

Π’ΠΎΠ·ΠΌΠΎΠΆΠ½ΠΎ, ΠΏΠΎΠΌΠΎΠΆΠ΅Ρ‚ ΠΏΠΎΠ½ΡΡ‚ΡŒ Ρ‡Ρ‚ΠΎ Ρ‚Π°ΠΊΠΎΠ΅ Π΄Π΅Ρ†ΠΈΠ±Π΅Π» ΡΠ»Π΅Π΄ΡƒΡŽΡ‰ΠΈΠ΅ сравнСниС. ΠŸΡ€Π΅Π΄ΡΡ‚Π°Π²ΠΈΠΌ, Ρ‡Ρ‚ΠΎ ΠΌΠΎΡ‰Π½ΠΎΡΡ‚ΡŒ измСняСтся Π»ΠΈΡ‚Ρ€Π°ΠΌΠΈ. Π‘ΠΎΠΎΡ‚Π½ΠΎΡˆΠ΅Π½ΠΈΠ΅ ΠΌΠ΅ΠΆΠ΄Ρƒ 0,5 Π»ΠΈΡ‚Ρ€Π° ΠΈ 1 Π»ΠΈΡ‚Ρ€ΠΎΠΌ Ρ‚Π°ΠΊΠΎΠ΅ ΠΆΠ΅ ΠΊΠ°ΠΊ ΠΈ ΠΌΠ΅ΠΆΠ΄Ρƒ 1 Π»ΠΈΡ‚Ρ€ΠΎΠΌ ΠΈ 2 Π»ΠΈΡ‚Ρ€Π°ΠΌΠΈ. Π­Ρ‚ΠΎ 0,5 ΠΈ Ρ€Π°Π²Π½ΠΎ ΠΎΠ½ΠΎ 6 Π΄Π‘. Но Ссли ΡΡ€Π°Π²Π½ΠΈΠ²Π°Ρ‚ΡŒ 0,5 ΠΈ 0,75 Π»ΠΈΡ‚Ρ€Π°, Ρ‚ΠΎ ΠΎΠ½ΠΈ относятся ΠΊΠ°ΠΊ 0,66(6) Ρ‡Ρ‚ΠΎ Π² Π΄Π΅Ρ†ΠΈΠ±Π΅Π»Π°Ρ… ΠΎΠΊΠΎΠ»ΠΎ 3,6 Π΄Π‘. ΠŸΡ€ΠΈΠΌΠ΅Ρ€Π½ΠΎ Ρ‚Π°ΠΊ.

Π”Π΅Ρ†ΠΈΠ±Π΅Π»Ρ‹ Π² акустикС

Π’Ρ‹, Π²ΠΎΠ·ΠΌΠΎΠΆΠ½ΠΎ, ΡƒΠ΄ΠΈΠ²ΠΈΡ‚Π΅ΡΡŒ, Π½ΠΎ для акустики Π΄Π΅Ρ†ΠΈΠ±Π΅Π»Ρ‹ подходят просто идСально. БобствСнно, АлСксандр Π‘Π΅Π»Π» Π²Π²Π΅Π» понятиС Π‘Π΅Π» ΠΏΡ€ΠΈ исслСдовании ΠΏΠΎΡ€ΠΎΠ³Π° ΡΠ»Ρ‹ΡˆΠΈΠΌΠΎΡΡ‚ΠΈ. Он ΠΎΠΏΡ€Π΅Π΄Π΅Π»ΠΈΠ», Ρ‡Ρ‚ΠΎ Β«Π³Ρ€ΠΎΠΌΠΊΠΎΡΡ‚ΡŒΒ» ΠΌΡ‹ воспринимаСм  Π½Π΅ ΠΏΠΎ Ρ€Π΅Π°Π»ΡŒΠ½ΠΎΠΉ мощности сигнала, Π° ΠΏΠΎ дСсятичному Π»ΠΎΠ³Π°Ρ€ΠΈΡ„ΠΌΡƒ ΠΎΡ‚ этой мощности. Как Ρ‚Π°ΠΊ? Π”Π°Π²Π°ΠΉΡ‚Π΅ рассмотрим ΠΏΡ€ΠΈΠΌΠ΅Ρ€.

Π˜ΠΌΠ΅Π΅Ρ‚ΡΡ ΡƒΡΠΈΠ»ΠΈΡ‚Π΅Π»ΡŒ, ΠΊΠΎΡ‚ΠΎΡ€Ρ‹ΠΉ Π²Ρ‹Π΄Π°Π΅Ρ‚ сигнал ΠΌΠΎΡ‰Π½ΠΎΡΡ‚ΡŒΡŽ 1 Π’Ρ‚. Π§Ρ‚ΠΎΠ±Ρ‹ ΡƒΠ²Π΅Π»ΠΈΡ‡ΠΈΡ‚ΡŒ Π΅Π³ΠΎ Π² 1,1 Ρ€Π°Π·Π°, Π΄ΠΎΠ±Π°Π²ΠΈΡ‚ΡŒ Π½Π°Π΄ΠΎ Ρ‚ΠΎΠ»ΡŒΠΊΠΎ 0,1 Π’Ρ‚. А Ссли Π½Π° Π²Ρ‹Ρ…ΠΎΠ΄Π΅ Ρƒ нас 100 Π’Ρ‚, Ρ‚ΠΎ Ρ‡Ρ‚ΠΎΠ±Ρ‹ ΡƒΠ²Π΅Π»ΠΈΡ‡ΠΈΡ‚ΡŒ ΠΌΠΎΡ‰Π½ΠΎΡΡ‚ΡŒ Π² 1,1 Ρ€Π°Π·Π° Π½Π°Π΄ΠΎ ΠΏΠΎΠ΄Π½ΡΡ‚ΡŒ ΠΌΠΎΡ‰Π½ΠΎΡΡ‚ΡŒ Π½Π° 10 Π’Ρ‚. Π£Π²Π΅Π»ΠΈΡ‡Π΅Π½ΠΈΠ΅ громкости Π² ΠΎΠ±ΠΎΠΈΡ… случаях Π±ΡƒΠ΄Π΅Ρ‚ «для ΡƒΡ…Π°Β» ΠΎΠ΄ΠΈΠ½Π°ΠΊΠΎΠ²Ρ‹ΠΌ, Π° ΡƒΠ²Π΅Π»ΠΈΡ‡Π΅Π½ΠΈΠ΅ мощности ΠΈΠΌΠ΅Π΅Ρ‚ явно Π½Π΅Π»ΠΈΠ½Π΅ΠΉΠ½Ρ‹ΠΉ Ρ…Π°Ρ€Π°ΠΊΡ‚Π΅Ρ€.

ΠœΡ‹ воспринимаСм Π½Π΅ Ρ€Π΅Π°Π»ΡŒΠ½Ρ‹ΠΉ ΡƒΡ€ΠΎΠ²Π΅Π½ΡŒ сигнала, Π° Π»ΠΎΠ³Π°Ρ€ΠΈΡ„ΠΌΠΈΡ‡Π΅ΡΠΊΡƒΡŽ Π·Π°Π²ΠΈΡΠΈΠΌΠΎΡΡ‚ΡŒ

На основании Π²ΠΎΡ‚ этого явлСния Π‘Π΅Π»Π» ΠΈ Π²Ρ‹Π²Π΅Π» Ρ‚ΠΎ самоС логарифмичСскоС ΠΎΡ‚Π½ΠΎΡˆΠ΅Π½ΠΈΠ΅. Π’ Π΅Π³ΠΎ Ρ‡Π΅ΡΡ‚ΡŒ ΠΈ Π½Π°Π·Π²Π°Π½Π° эта ΠΎΡ‚Π½ΠΎΡΠΈΡ‚Π΅Π»ΡŒΠ½Π°Ρ Π΅Π΄ΠΈΠ½ΠΈΡ†Π° ΠΈΠ·ΠΌΠ΅Ρ€Π΅Π½ΠΈΠΉ. Π§Ρ‚ΠΎ Π΅Ρ‰Π΅ это Π½Π°ΠΌ Π΄Π°Π΅Ρ‚? А Π²ΠΎΡ‚ Ρ‚Π°ΠΊΠΈΠ΅ Ρ„Π°ΠΊΡ‚Ρ‹:

  • 1 Π΄Π‘ β€” это ΠΌΠΈΠ½ΠΈΠΌΠ°Π»ΡŒΠ½Ρ‹ΠΉ ΡƒΡ€ΠΎΠ²Π΅Π½ΡŒ ΡΠ»Ρ‹ΡˆΠΈΠΌΠΎΡΡ‚ΠΈ сигнала. Π—Π²ΡƒΠΊΠΈ с Π±ΠΎΠ»Π΅Π΅ Π½ΠΈΠ·ΠΊΠΎΠΉ ΠΌΠΎΡ‰Π½ΠΎΡΡ‚ΡŒΡŽ (ΠΎ Π΄Π‘ ΠΈ Π½ΠΈΠΆΠ΅) Π±ΠΎΠ»ΡŒΡˆΠΈΠ½ΡΡ‚Π²ΠΎ людСй Π½Π΅ Π·Π°ΠΌΠ΅Ρ‡Π°ΡŽΡ‚ ΠΈ ΠΎΠΏΡ€Π΅Π΄Π΅Π»ΡΡŽΡ‚ ΠΊΠ°ΠΊ Β«Π°Π±ΡΠΎΠ»ΡŽΡ‚Π½ΡƒΡŽ Ρ‚ΠΈΡˆΠΈΠ½ΡƒΒ».
  • Если говорят ΠΎ Ρ‚ΠΎΠΌ, Ρ‡Ρ‚ΠΎ ΠΌΠΎΡ‰Π½ΠΎΡΡ‚ΡŒ сигнала/Π·Π²ΡƒΠΊΠ° возросла Π½Π° 3 Π΄Π‘, Ρ‚ΠΎ Π·Π½Π°Ρ‡ΠΈΡ‚ ΠΎΠ½Π° возросла Π² Π΄Π²Π° Ρ€Π°Π·Π°. НС ΠΏΡƒΡ‚Π°ΠΉΡ‚Π΅ с Π³Ρ€ΠΎΠΌΠΊΠΎΡΡ‚ΡŒΡŽ.
  • ΠŸΡ€ΠΈ ΡƒΠ²Π΅Π»ΠΈΡ‡Π΅Π½ΠΈΠΈ мощности Π·Π²ΡƒΠΊΠ° Π½Π° 10 Π΄Π‘, Π³Ρ€ΠΎΠΌΠΊΠΎΡΡ‚ΡŒ увСличиваСтся Π² 2 Ρ€Π°Π·Π°.
  • Π£Π²Π΅Π»ΠΈΡ‡Π΅Π½ΠΈΠ΅ напряТСния Π² Π΄Π²Π° Ρ€Π°Π·Π° β€” это 6 Π΄Π‘.

ΠŸΡ€ΠΈΠ½ΡΡ‚ΡŒ Π΄Π΅Ρ†ΠΈΠ±Π΅Π»Ρ‹ Π½Π΅ Ρ‚Π°ΠΊ Π»Π΅Π³ΠΊΠΎ. Но Π½Π°Π²Π΅Ρ€Π½ΠΎΠ΅, Π²Ρ‹ ΡƒΠΆΠ΅ поняли, Ρ‡Ρ‚ΠΎ Π² Π΄Π΅Ρ†ΠΈΠ±Π΅Π»Π°Ρ… Π³Ρ€ΠΎΠΌΠΊΠΎΡΡ‚ΡŒ Π·Π²ΡƒΠΊΠ°/ΡˆΡƒΠΌΠ° Π½Π΅ измСряСтся. Π­Ρ‚Π° Ρ†ΠΈΡ„Ρ€Π° ΠΏΠΎΠΊΠ°Π·Ρ‹Π²Π°Π΅Ρ‚ насколько измСнился сигнал ΠΎΡ‚Π½ΠΎΡΠΈΡ‚Π΅Π»ΡŒΠ½ΠΎ Β«Π½ΡƒΠ»Π΅Π²ΠΎΠΉΒ» Ρ‚ΠΎΡ‡ΠΊΠΈ восприятия. ΠŸΡ€ΠΈΠΌΠ΅Ρ€Π½ΠΎ Ρ‚Π°ΠΊ ΠΌΠΎΠΆΠ½ΠΎ это ΡΡ„ΠΎΡ€ΠΌΡƒΠ»ΠΈΡ€ΠΎΠ²Π°Ρ‚ΡŒ.

Π’Π°Π±Π»ΠΈΡ†Π° ΡƒΡ€ΠΎΠ²Π½Π΅ΠΉ ΡˆΡƒΠΌΠΎΠ²

Ну, Π° Ρ‡Ρ‚ΠΎΠ±Ρ‹ Π±Ρ‹Π»ΠΎ понятнСС, ΠΏΡ€ΠΈΠ²Π΅Π΄Π΅ΠΌ Ρ‚Π°Π±Π»ΠΈΡ†Ρƒ сравнСний ΠΏΡ€ΠΈΠ²Ρ‹Ρ‡Π½Ρ‹Ρ…, Π·Π½Π°ΠΊΠΎΠΌΡ‹Ρ… Π·Π²ΡƒΠΊΠΎΠ² ΠΈ ΠΈΡ… срСднСго уровня.

Π΄Π‘Π‘ Ρ‡Π΅ΠΌ ΠΌΠΎΠΆΠ½ΠΎ ΡΡ€Π°Π²Π½ΠΈΡ‚ΡŒΠ΄Π‘Π‘ Ρ‡Π΅ΠΌ ΠΌΠΎΠΆΠ½ΠΎ ΡΡ€Π°Π²Π½ΠΈΡ‚ΡŒ
0 Π΄Π‘ΠŸΠΎΠ»Π½Π°Ρ Ρ‚ΠΈΡˆΠΈΠ½Π°90 Π΄Π‘Π—Π²ΡƒΠΊ Ρ€Π°Π±ΠΎΡ‚Π°ΡŽΡ‰Π΅Π³ΠΎ Ρ„Π΅Π½Π°, ΠΌΠΎΡ‚ΠΎΡ†ΠΈΠΊΠ»Π°, ΠΏΠΎΠ΅Π·Π΄Π°
1 Π΄Π‘Π‘Π°ΠΌΡ‹ΠΉ Π½ΠΈΠΆΠ½ΠΈΠΉ ΠΏΠΎΡ€ΠΎΠ³ ΡΠ»Ρ‹ΡˆΠΈΠΌΠΎΡΡ‚ΠΈ100-105 Π΄Π‘Π Π΅ΠΌΠΎΠ½Ρ‚ ΠΈ Ρ€ΠΎΠΊ-ΠΊΠΎΠ½Ρ†Π΅Ρ€Ρ‚
10-24 дБШСлСст листвы110 Π΄Π‘ΠœΡƒΠ·Ρ‹ΠΊΠ° Π² Π½ΠΎΡ‡Π½ΠΎΠΌ ΠΊΠ»ΡƒΠ±Π΅
20 Π΄Π‘Π¨Π΅ΠΏΠΎΡ‚120 Π΄Π‘ΠΠ²Ρ‚ΠΎΠΌΠΎΠ±ΠΈΠ»ΡŒΠ½Ρ‹ΠΉ Π³ΡƒΠ΄ΠΎΠΊ
40 Π΄Π‘Π’Π°ΠΊ Π·Π²ΡƒΡ‡ΠΈΡ‚ доТдь130-135 Π΄Π‘Π—Π²ΡƒΠΊ Ρ€Π°Π±ΠΎΡ‚Π°ΡŽΡ‰Π΅ΠΉ Π΄Ρ€Π΅Π»ΠΈ
45 Π΄Π‘Π’ΠΈΡ…ΠΈΠΉ Ρ€Π°Π·Π³ΠΎΠ²ΠΎΡ€140 Π΄Π‘Π¨ΡƒΠΌ Ρ‚ΡƒΡ€Π±ΠΈΠ½ самолСта
60 Π΄Π‘Π“Ρ€ΠΎΠΌΠΊΠΈΠΉ Ρ€Π°Π·Π³ΠΎΠ²ΠΎΡ€160 Π΄Π‘Π—Π²ΡƒΠΊ выстрСла Π²ΠΎΠ·Π»Π΅ ΡƒΡ…Π°
80-90 дБШоссС с интСнсивным Π΄Π²ΠΈΠΆΠ΅Π½ΠΈΠ΅ΠΌ200 Π΄Π‘Π‘ΠΌΠ΅Ρ€Ρ‚Π΅Π»ΡŒΠ½Ρ‹ΠΉ ΡƒΡ€ΠΎΠ²Π΅Π½ΡŒ ΡˆΡƒΠΌΠ°

ΠšΠ°ΠΆΠ΄Ρ‹ΠΉ ΡˆΡƒΠΌ ΠΈΠ»ΠΈ Π·Π²ΡƒΠΊ ΠΈΠΌΠ΅Π΅Ρ‚ ΠΎΠΏΡ€Π΅Π΄Π΅Π»Π΅Π½Π½Ρ‹ΠΉ ΡƒΡ€ΠΎΠ²Π΅Π½ΡŒ мощности, Π½ΠΎ ΠΏΡ€ΠΎΡ‰Π΅ Π΅Π³ΠΎ ΠΎΠΏΠΈΡΡ‹Π²Π°Ρ‚ΡŒ Π² Π΄Π΅Ρ†ΠΈΠ±Π΅Π»Π°Ρ…

Π€Π°ΠΊΡ‚Ρ‹, ΠΊΠΎΡ‚ΠΎΡ€Ρ‹Π΅ позволят ΠΎΡ†Π΅Π½ΠΈΡ‚ΡŒ Π²Π°ΠΆΠ½ΠΎΡΡ‚ΡŒ акустики ΠΈ Π΄Π΅Ρ†ΠΈΠ±Π΅Π»ΠΎΠ²:

  • ΠšΠΎΠΌΡ„ΠΎΡ€Ρ‚Π½Ρ‹ΠΌ ΡƒΡ€ΠΎΠ²Π½Π΅ΠΌ ΡˆΡƒΠΌΠ° считаСтся 50-55 Π΄Π‘. Как Π²ΠΈΠ΄ΠΈΡ‚Π΅, эту Π²Π΅Π»ΠΈΡ‡ΠΈΠ½Ρƒ ΠΌΠΎΠΆΠ½ΠΎ ΡΡ€Π°Π²Π½ΠΈΡ‚ΡŒ с Ρ€Π°Π·Π³ΠΎΠ²ΠΎΡ€ΠΎΠΌ ΠΎΠ±Ρ‹Ρ‡Π½ΠΎΠΉ «громкости». ИмСнно этот ΡƒΡ€ΠΎΠ²Π΅Π½ΡŒ ΠΏΠΎ Π‘ΠΠΈΠŸΡƒ ΠΎΠΏΡ€Π΅Π΄Π΅Π»Π΅Π½ ΠΊΠ°ΠΊΒ  ΠΏΡ€ΠΈΠ΅ΠΌΠ»Π΅ΠΌΡ‹ΠΉ для Π΄Π½Π΅Π²Π½ΠΎΠ³ΠΎ Π²Ρ€Π΅ΠΌΠ΅Π½ΠΈ.
  • Π£Ρ€ΠΎΠ²Π΅Π½ΡŒ 70-90 Π΄Π‘ относится ΠΊ Β«Ρ‚Π΅Ρ€ΠΏΠΈΠΌΡ‹ΠΌΒ», Π½ΠΎ Π΄Π»ΠΈΡ‚Π΅Π»ΡŒΠ½ΠΎΠ΅ воздСйствиС ΠΌΠΎΠΆΠ΅Ρ‚ привСсти ΠΊ заболСваниям Π½Π΅Ρ€Π²Π½ΠΎΠΉ систСмы.
  • Π”Π»ΠΈΡ‚Π΅Π»ΡŒΠ½ΠΎΠ΅ воздСйствиС ΡˆΡƒΠΌΠ° Π² 100 Π΄Π‘ ΠΏΡ€ΠΈΠ²ΠΎΠ΄ΠΈΡ‚ ΠΊ сниТСнию слуха ΠΈ Π³Π»ΡƒΡ…ΠΎΡ‚Π΅.
  • Π—Π²ΡƒΠΊΠΈ ΠΌΠΎΡ‰Π½ΠΎΡΡ‚ΡŒΡŽ 130 Π΄Π‘ Π²Ρ‹Π·Ρ‹Π²Π°ΡŽΡ‚ Π±ΠΎΠ»Π΅Π²Ρ‹Π΅ ощущСния.
  • ΠœΠΎΡ‰Π½ΠΎΡΡ‚ΡŒ Π·Π²ΡƒΠΊΠ° Π² 200 Π΄Π‘ ΠΌΠΎΠΆΠ΅Ρ‚ Π±Ρ‹Ρ‚ΡŒ ΡΠΌΠ΅Ρ€Ρ‚Π΅Π»ΡŒΠ½ΠΎΠΉ.

Π’ΠΎΠΎΠ±Ρ‰Π΅, постоянноС Π½Π°Ρ…ΠΎΠΆΠ΄Π΅Π½ΠΈΠ΅ Π² ΡˆΡƒΠΌΠ½ΠΎΠΌ ΠΏΠΎΠΌΠ΅Ρ‰Π΅Π½ΠΈΠΈ сильно сниТаСт ΡΠΏΠΎΡΠΎΠ±Π½ΠΎΡΡ‚ΡŒ Π²ΠΎΡΠΏΡ€ΠΈΠ½ΠΈΠΌΠ°Ρ‚ΡŒ Π·Π²ΡƒΠΊΠΈ. Мало Ρ‚ΠΎΠ³ΠΎ, ΠΎΠ½ΠΎ ΠΏΡ€ΠΈΠ²ΠΎΠ΄ΠΈΡ‚ ΠΊ расстройствам психики, сна, Ρ‡Ρ‚ΠΎ Π½Π΅Π³Π°Ρ‚ΠΈΠ²Π½ΠΎ сказываСтся ΠΈ Π½Π° ΠΎΠ±Ρ‰Π΅ΠΌ самочувствии. ΠŸΠΎΡΡ‚ΠΎΠΌΡƒ ΡˆΡƒΠΌΠ½Ρ‹Π΅ производства β€” Π·ΠΎΠ½Π° риска. Π§Ρ‚ΠΎΠ±Ρ‹ Ρ…ΠΎΡ€ΠΎΡˆΠΎ сСбя Ρ‡ΡƒΠ²ΡΡ‚Π²ΠΎΠ²Π°Ρ‚ΡŒ, просто Π½Π΅ΠΎΠ±Ρ…ΠΎΠ΄ΠΈΠΌΠΎ врСмя ΠΎΡ‚ Π²Ρ€Π΅ΠΌΠ΅Π½ΠΈ находится Ссли Π½Π΅ ΠΏΡ€ΠΈ ΠΏΠΎΠ»Π½ΠΎΠΉ Ρ‚ΠΈΡˆΠΈΠ½Π΅, Ρ‚ΠΎ ΠΏΡ€ΠΈ Π½ΠΈΠ·ΠΊΠΎΠΌ ΡƒΡ€ΠΎΠ²Π½Π΅ ΡˆΡƒΠΌΠ°.

ΠŸΠ΅Ρ€Π΅Π²ΠΎΠ΄ Π΄Π΅Ρ†ΠΈΠ±Π΅Π»ΠΎΠ² Π² Ρ€Π°Π·Ρ‹

Π”Π°Π²Π°ΠΉΡ‚Π΅ ΠΏΠΎΠΏΡ€ΠΎΠ±ΡƒΠ΅ΠΌ ΡΡ„ΠΎΡ€ΠΌΡƒΠ»ΠΈΡ€ΠΎΠ²Π°Ρ‚ΡŒ Ρ‡Ρ‚ΠΎ Ρ‚Π°ΠΊΠΎΠ΅ Π΄Π΅Ρ†ΠΈΠ±Π΅Π» ΠΏΠΎ-Π΄Ρ€ΡƒΠ³ΠΎΠΌΡƒ. Π”Π΅Ρ†ΠΈΠ±Π΅Π» β€” это Π»ΠΎΠ³Π°Ρ€ΠΈΡ„ΠΌ ΡΠΎΠΎΡ‚Π½ΠΎΡˆΠ΅Π½ΠΈΡ Π΄Π²ΡƒΡ… Π²Π΅Π»ΠΈΡ‡ΠΈΠ½. Π­Ρ‚Π° ΠΎΡ‚Π½ΠΎΡΠΈΡ‚Π΅Π»ΡŒΠ½Π°Ρ Π²Π΅Π»ΠΈΡ‡ΠΈΠ½Π°, которая ΠΏΠΎΠΊΠ°Π·Ρ‹Π²Π°Π΅Ρ‚ Π²ΠΎ сколько ΠΎΠ΄Π½ΠΎ Π·Π½Π°Ρ‡Π΅Π½ΠΈΠ΅ большС ΠΈΠ»ΠΈ мСньшС Π΄Ρ€ΡƒΠ³ΠΎΠ³ΠΎ (Π±Π°Π·ΠΎΠ²ΠΎΠ³ΠΎ). Β«Π’ΠΎ сколько Ρ€Π°Π·Β» это Π½Π°ΠΌ понятно. ΠŸΠΎΡΡ‚ΠΎΠΌΡƒ часто приходится ΠΏΠ΅Ρ€Π΅Π²ΠΎΠ΄ΠΈΡ‚ΡŒ Π΄Π΅Ρ†ΠΈΠ±Π΅Π»Ρ‹ Π² Ρ€Π°Π·Ρ‹ ΠΈ Π½Π°ΠΎΠ±ΠΎΡ€ΠΎΡ‚. МоТно, ΠΊΠΎΠ½Π΅Ρ‡Π½ΠΎ, ΠΏΠΎΡΡ‡ΠΈΡ‚Π°Ρ‚ΡŒ, Π½ΠΎ ΠΏΡ€ΠΎΡ‰Π΅ ΠΏΠΎΠ»ΡŒΠ·ΠΎΠ²Π°Ρ‚ΡŒΡΡ Ρ‚Π°Π±Π»ΠΈΡ†Π΅ΠΉ.

Π΄Π‘Π£Π²Π΅Π»ΠΈΡ‡Π΅Π½ΠΈΠ΅ напряТСния (силы Ρ‚ΠΎΠΊΠ°) Π² Ρ€Π°Π·Ρ‹Π£Π²Π΅Π»ΠΈΡ‡Π΅Π½ΠΈΠ΅ мощности (энСргСтичСской ΡΠΎΡΡ‚Π°Π²Π»ΡΡŽΡ‰Π΅ΠΉ) Π² Ρ€Π°Π·Ρ‹Π΄Π‘Π£Π²Π΅Π»ΠΈΡ‡Π΅Π½ΠΈΠ΅ напряТСния (силы Ρ‚ΠΎΠΊΠ°) Π² Ρ€Π°Π·Ρ‹Π£Π²Π΅Π»ΠΈΡ‡Π΅Π½ΠΈΠ΅ мощности (энСргСтичСской ΡΠΎΡΡ‚Π°Π²Π»ΡΡŽΡ‰Π΅ΠΉ) Π² Ρ€Π°Π·Ρ‹
0112825,12631
11,121,262928,17794
21,261,593031,641000
31,4123135,461257
41,592,513239,841587
51,783,163344,641993
622,983448,082312
72,245,013556,823165
82,516,313663,294006
92,827,943770,925030
103,16103879,366298
113,5512,593989,297973
123,9815,854010010000
134,4719,9641112,2312596
145,0125,1242125,9415861
155,6231,6543141,2419949
166,3139,8444158,4825116
177,0848,0845177,9431663
187,9463,5946199,6039840
198,9179,3647223,7150046
201010048251,2663132
2111,22125,9449281,6979349
2212,59158,4850316,5100 000
2314,12199,60601 0001 000 000
2415,85251,2670316510 000 000
2517,79316,508010 000100 000 000
2619,96398,490316501 000 000 000
2722,37500,42100100 00010 000 000 000

Как Π²ΠΈΠ΄ΠΈΡ‚Π΅, Ρ‡Ρ‚ΠΎΠ±Ρ‹ напряТСниС ΡƒΠ²Π΅Π»ΠΈΡ‡ΠΈΠ»ΠΎΡΡŒ Π² Ρ‚Ρ€ΠΈ Ρ€Π°Π·Π°, ΠΌΠΎΡ‰Π½ΠΎΡΡ‚ΡŒ Π½Π΅ΠΎΠ±Ρ…ΠΎΠ΄ΠΈΠΌΠΎ ΠΏΠΎΠ΄Π½ΡΡ‚ΡŒ Π² 10 Ρ€Π°Π·. Π’ΠΏΠ΅Ρ‡Π°Ρ‚Π»ΡΡŽΡ‰Π°Ρ Ρ€Π°Π·Π½ΠΈΡ†Π°. Π­Ρ‚Π° Ρ‚Π°Π±Π»ΠΈΡ†Π° позволяСт Ρ‚ΠΎΡ‡Π½ΠΎ ΠΏΠΎΠ½ΡΡ‚ΡŒ связь ΠΌΠ΅ΠΆΠ΄Ρƒ этими Π²Π΅Π»ΠΈΡ‡ΠΈΠ½Π°ΠΌΠΈ.

Но сигналы ΠΈ Π²Π΅Π»ΠΈΡ‡ΠΈΠ½Ρ‹ Π½Π΅ Ρ‚ΠΎΠ»ΡŒΠΊΠΎ ΡƒΠ²Π΅Π»ΠΈΡ‡ΠΈΠ²Π°ΡŽΡ‚ΡΡ, ΠΎΠ½ΠΈ ΠΌΠΎΠ³ΡƒΡ‚ ΠΈ ΡΠ½ΠΈΠΆΠ°Ρ‚ΡŒΡΡ. Π‘Π»Π΅Π΄ΡƒΡŽΡ‰Π°Ρ Ρ‚Π°Π±Π»ΠΈΡ†Π° Π΄Π°Π½Π° для падСния Π·Π½Π°Ρ‡Π΅Π½ΠΈΠΉ ΠΎΡ‚Π½ΠΎΡΠΈΡ‚Π΅Π»ΡŒΠ½ΠΎ эталона.

Π΄Π‘Π‘Π½ΠΈΠΆΠ΅Π½ΠΈΠ΅ напряТСния (силы Ρ‚ΠΎΠΊΠ°) Π² Ρ€Π°Π·Ρ‹Π‘Π½ΠΈΠΆΠ΅Π½ΠΈΠ΅ мощности (энСргСтичСской ΡΠΎΡΡ‚Π°Π²Π»ΡΡŽΡ‰Π΅ΠΉ) Π² Ρ€Π°Π·Ρ‹Π΄Π‘Π‘Π½ΠΈΠΆΠ΅Π½ΠΈΠ΅ напряТСния (силы Ρ‚ΠΎΠΊΠ°) Π² Ρ€Π°Π·Ρ‹Π‘Π½ΠΈΠΆΠ΅Π½ΠΈΠ΅ мощности (энСргСтичСской ΡΠΎΡΡ‚Π°Π²Π»ΡΡŽΡ‰Π΅ΠΉ) Π² Ρ€Π°Π·Ρ‹
011-8,00,3980,159
-0,10,9890,977-9,00,3550,126
-0,20,9770,955-100,3160,1
-0,30,9660,933-110,2820,0794
-0,40,9550,912-120,2510,0631
-0,50,9440,891-130,2240,0501
-0,60,9330,871-140,20,0398
-0,80,9120,832-150,1780,0316
-1,00,8910,794-160,1590,0251
-1,50,8410,708-180,1260,0159
-2,00,7940,631-200,10,01
-2,50,7500,562-300,03160,001
-3,00,6680,501-400,010,0001
-3,50,6310,447-500,003160,00001
-4,00,5960,398-600,0010,000001
-4,50,5620,355-700,0003160,0000001
-5,00,5010,316-800,00010,00000001
-6,00,5010,251-900,00003160,000000001
-7,00,4470,2-1000,000010,0000000001

ОслаблСниС Ρ‚ΠΎΠ³ΠΎ ΠΈΠ»ΠΈ ΠΈΠ½ΠΎΠ³ΠΎ сигнала ΠΏΡ€ΠΎΡ‰Π΅ ΠΎΠΏΠΈΡΡ‹Π²Π°Ρ‚ΡŒ Π² Π΄Π΅Ρ†ΠΈΠ±Π΅Π»Π°Ρ…. ΠŸΡ€ΠΎΡΡ‚Ρ‹Π΅ Ρ†ΠΈΡ„Ρ€Ρ‹ Π»Π΅Π³Ρ‡Π΅ Π·Π°ΠΏΠΎΠΌΠΈΠ½Π°ΡŽΡ‚ΡΡ. Но ΠΈΠ½ΠΎΠ³Π΄Π° Π½Π°Π΄ΠΎ Π·Π½Π°Ρ‚ΡŒ ΠΈ Ρ€Π΅Π°Π»ΡŒΠ½Ρ‹ΠΉ ΡƒΡ€ΠΎΠ²Π΅Π½ΡŒ мощности. Для этого ΠΈΡΠΏΠΎΠ»ΡŒΠ·ΡƒΡŽΡ‚ Ρ‚Π°Π±Π»ΠΈΡ†Ρ‹ (ΠΏΠ΅Ρ€Π΅Π²ΠΎΠ΄ Π΄Π‘ Π² ΠΌΠΊΠ’)

ΠŸΠ΅Ρ€Π΅Π²ΠΎΠ΄ ослаблСния сигнала Π² Π΄Π‘ Π² ΠΌΠΈΠΊΡ€ΠΎΠ²ΠΎΠ»ΡŒΡ‚Ρ‹ ΠΌΠΊΠ’

ΠŸΠ΅Ρ€Π΅Π²ΠΎΠ΄ ΠΈΠ· Π΄Π± Π² Ρ€Π°Π·Ρ‹ Ρ„ΠΎΡ€ΠΌΡƒΠ»Π° | 2019

ΠŸΠ΅Ρ€Π΅Π²ΠΎΠ΄ Π²Π΅Π»ΠΈΡ‡ΠΈΠ½ ΠΈΠ· Π΄Π΅Ρ†ΠΈΠ±Π΅Π»ΠΎΠ² Π² Π°Π±ΡΠΎΠ»ΡŽΡ‚Π½Ρ‹Π΅ значСния ΠΈ ΠΌΠΎΡ‰Π½ΠΎΡΡ‚ΡŒ

ΠŸΡ€ΠΈ ΠΏΡ€ΠΎΠ²Π΅Π΄Π΅Π½ΠΈΠΈ ΠΈΠ·ΠΌΠ΅Ρ€Π΅Π½ΠΈΠΉ ΠΏΠ°Ρ€Π°ΠΌΠ΅Ρ‚Ρ€ΠΎΠ² Ρ€Π°Π΄ΠΈΠΎΠ°ΠΏΠΏΠ°Ρ€Π°Ρ‚ΡƒΡ€Ρ‹ довольно часто приходится ΠΈΠΌΠ΅Ρ‚ΡŒ Π΄Π΅Π»ΠΎ с ΠΎΡ‚Π½ΠΎΡΠΈΡ‚Π΅Π»ΡŒΠ½Ρ‹ΠΌΠΈ Π²Π΅Π»ΠΈΡ‡ΠΈΠ½Π°ΠΌΠΈ Π²Ρ‹Ρ€Π°ΠΆΠ΅Π½Π½Ρ‹ΠΌΠΈ Π² Π΄Π΅Ρ†ΠΈΠ±Π΅Π»Π°Ρ… [Π΄Π‘]. Π’ Π΄Π΅Ρ†ΠΈΠ±Π΅Π»Π°Ρ… Π²Ρ‹Ρ€Π°ΠΆΠ°ΡŽΡ‚ ΠΈΠ½Ρ‚Π΅Π½ΡΠΈΠ²Π½ΠΎΡΡ‚ΡŒ Π·Π²ΡƒΠΊΠ°, усилСниС каскада ΠΏΠΎ Π½Π°ΠΏΡ€ΡΠΆΠ΅Π½ΠΈΡŽ, Ρ‚ΠΎΠΊΡƒ ΠΈΠ»ΠΈ мощности, ΠΏΠΎΡ‚Π΅Ρ€ΠΈ ΠΏΠ΅Ρ€Π΅Π΄Π°Ρ‡ΠΈ ΠΈΠ»ΠΈ ослаблСниС сигнала, ΠΈ Ρ‚.Π΄.

Π”Π΅Ρ†ΠΈΠ±Π΅Π» β€” это ΡƒΠ½ΠΈΠ²Π΅Ρ€ΡΠ°Π»ΡŒΠ½Π°Ρ логарифмичСская Π΅Π΄ΠΈΠ½ΠΈΡ†Π°. Π¨ΠΈΡ€ΠΎΠΊΠΎΠ΅ использованиС прСдставлСния Π²Π΅Π»ΠΈΡ‡ΠΈΠ½ Π² Π΄Π‘ связано с удобством логарифмичСского ΠΌΠ°ΡΡˆΡ‚Π°Π±Π°, Π° ΠΏΡ€ΠΈ расчСтах Π΄Π΅Ρ†ΠΈΠ±Π΅Π»Ρ‹ ΠΏΠΎΠ΄Ρ‡ΠΈΠ½ΡΡŽΡ‚ΡΡ Π·Π°ΠΊΠΎΠ½Π°ΠΌ Π°Ρ€ΠΈΡ„ΠΌΠ΅Ρ‚ΠΈΠΊΠΈ β€” ΠΈΡ… ΠΌΠΎΠΆΠ½ΠΎ ΡΠΊΠ»Π°Π΄Ρ‹Π²Π°Ρ‚ΡŒ ΠΈ Π²Ρ‹Ρ‡ΠΈΡ‚Π°Ρ‚ΡŒ, Ссли сигналы ΠΈΠΌΠ΅ΡŽΡ‚ ΠΎΠ΄ΠΈΠ½Π°ΠΊΠΎΠ²ΡƒΡŽ Ρ„ΠΎΡ€ΠΌΡƒ.

БущСствуСт Ρ„ΠΎΡ€ΠΌΡƒΠ»Π° для пСрСсчСта ΠΎΡ‚Π½ΠΎΡˆΠ΅Π½ΠΈΡ Π΄Π²ΡƒΡ… напряТСний Π² число Π΄Π΅Ρ†ΠΈΠ±Π΅Π»ΠΎΠ² (аналогичная Ρ„ΠΎΡ€ΠΌΡƒΠ»Π° справСдлива ΠΈ для Ρ‚ΠΎΠΊΠΎΠ²):

НапримСр, Ссли Π²Ρ‹Ρ…ΠΎΠ΄Π½ΠΎΠΉ сигнал U2 ΠΈΠΌΠ΅Π΅Ρ‚ ΡƒΡ€ΠΎΠ²Π΅Π½ΡŒ Π²Π΄Π²ΠΎΠ΅ большС, Ρ‡Π΅ΠΌ U1, Ρ‚ΠΎ это ΠΎΡ‚Π½ΠΎΡˆΠ΅Π½ΠΈΠ΅ составит +6 Π΄Π‘ (Ig2=0,301). Если U2>U1 Π² 10 Ρ€Π°Π·, Ρ‚ΠΎ ΠΎΡ‚Π½ΠΎΡˆΠ΅Π½ΠΈΠ΅ сигналов составляСт 20 Π΄Π‘ (Ig10=1). Если U1>U2, Ρ‚ΠΎ Π·Π½Π°ΠΊ Ρƒ ΠΎΡ‚Π½ΠΎΡˆΠ΅Π½ΠΈΡ мСняСтся Π½Π° минус 20 Π΄Π‘.

Π’Π°ΠΊ, Π½Π°ΠΏΡ€ΠΈΠΌΠ΅Ρ€, Ρƒ ΠΈΠ·ΠΌΠ΅Ρ€ΠΈΡ‚Π΅Π»ΡŒΠ½ΠΎΠ³ΠΎ Π³Π΅Π½Π΅Ρ€Π°Ρ‚ΠΎΡ€Π° Π°Ρ‚Ρ‚Π΅Π½ΡŽΠ°Ρ‚ΠΎΡ€ для ослаблСния Π²Ρ‹Ρ…ΠΎΠ΄Π½ΠΎΠ³ΠΎ сигнала ΠΌΠΎΠΆΠ΅Ρ‚ ΠΈΠΌΠ΅Ρ‚ΡŒ Π³Ρ€Π°Π΄ΡƒΠΈΡ€ΠΎΠ²ΠΊΡƒ Π² Π΄Π‘. Π’ этом случаС для ΠΏΠ΅Ρ€Π΅Π²ΠΎΠ΄Π° Π²Π΅Π»ΠΈΡ‡ΠΈΠ½Ρ‹ ΠΈΠ· Π΄Π΅Ρ†ΠΈΠ±Π΅Π»ΠΎΠ² Π² Π°Π±ΡΠΎΠ»ΡŽΡ‚Π½ΠΎΠ΅ Π·Π½Π°Ρ‡Π΅Π½ΠΈΠ΅ быстрСй Π±ΡƒΠ΄Π΅Ρ‚ ΠΏΠΎΠ»ΡƒΡ‡Π΅Π½ Ρ€Π΅Π·ΡƒΠ»ΡŒΡ‚Π°Ρ‚, Ссли Π²ΠΎΡΠΏΠΎΠ»ΡŒΠ·ΠΎΠ²Π°Ρ‚ΡŒΡΡ ΡƒΠΆΠ΅ посчитанной Ρ‚Π°Π±Π». 6; 1. Она ΠΈΠΌΠ΅Π΅Ρ‚ Π΄ΠΈΡΠΊΡ€Π΅Ρ‚Π½ΠΎΡΡ‚ΡŒ 1 Π΄Π‘ (Ρ‡Ρ‚ΠΎ Π²ΠΏΠΎΠ»Π½Π΅ достаточно Π² Π±ΠΎΠ»ΡŒΡˆΠΈΠ½ΡΡ‚Π²Π΅ случаСв) ΠΈ Π΄ΠΈΠ°ΠΏΠ°Π·ΠΎΠ½ Π·Π½Π°Ρ‡Π΅Π½ΠΈΠΉ 0. -119 Π΄Π‘.

Π’Π°Π±Π». 6.1 ΠΌΠΎΠΆΠ½ΠΎ ΠΈΡΠΏΠΎΠ»ΡŒΠ·ΠΎΠ²Π°Ρ‚ΡŒ для ΠΏΠ΅Ρ€Π΅Π²ΠΎΠ΄Π° Π΄Π΅Ρ†ΠΈΠ±Π΅Π»ΠΎΠ² ослаблСния Π°Ρ‚Ρ‚Π΅Π½ΡŽΠ°Ρ‚ΠΎΡ€Π° Π² ΡƒΡ€ΠΎΠ²Π΅Π½ΡŒ Π²Ρ‹Ρ…ΠΎΠ΄Π½ΠΎΠ³ΠΎ напряТСния. Для удобства использования Ρ‚Π°Π±Π»ΠΈΡ†Ρ‹ потрСбуСтся Π½Π° Π²Ρ‹Ρ…ΠΎΠ΄Π΅ Π³Π΅Π½Π΅Ρ€Π°Ρ‚ΠΎΡ€Π° ΡƒΡΡ‚Π°Π½ΠΎΠ²ΠΈΡ‚ΡŒ ΠΏΡ€ΠΈ отсутствии ослаблСния (0 Π΄Π‘ Π½Π° Π°Ρ‚Ρ‚Π΅Π½ΡŽΠ°Ρ‚ΠΎΡ€Π΅) ΡƒΡ€ΠΎΠ²Π΅Π½ΡŒ напряТСния 1 Π’ (Π΄Π΅ΠΉΡΡ‚Π²ΡƒΡŽΡ‰Π΅Π³ΠΎ ΠΈΠ»ΠΈ Π°ΠΌΠΏΠ»ΠΈΡ‚ΡƒΠ΄Π½ΠΎΠ³ΠΎ). Π’ этом случаС ΡΠΎΠΎΡ‚Π²Π΅Ρ‚ΡΡ‚Π²ΡƒΡŽΡ‰Π΅Π΅ Π½ΡƒΠΆΠ½ΠΎΠ΅ Π·Π½Π°Ρ‡Π΅Π½ΠΈΠ΅ Π²Ρ‹Ρ…ΠΎΠ΄Π½ΠΎΠ³ΠΎ напряТСния послС установки ослаблСния находится Π½Π° пСрСсСчСнии Π³ΠΎΡ€ΠΈΠ·ΠΎΠ½Ρ‚Π°Π»ΡŒΠ½ΠΎΠΉ ΠΈ Π²Π΅Ρ€Ρ‚ΠΈΠΊΠ°Π»ΡŒΠ½ΠΎΠΉ Π³Ρ€Π°Ρ„ (значСния Π² Π΄Π΅Ρ†ΠΈΠ±Π΅Π»Π°Ρ… ΡΠΊΠ»Π°Π΄Ρ‹Π²Π°ΡŽΡ‚ΡΡ арифмСтичСски).

Π’Π΅Π»ΠΈΡ‡ΠΈΠ½Π° Π²Ρ‹Ρ…ΠΎΠ΄Π½ΠΎΠ³ΠΎ напряТСния Π² Ρ‚Π°Π±Π»ΠΈΡ†Π΅ ΡƒΠΊΠ°Π·Π°Π½Π° Π² ΠΌΠΈΠΊΡ€ΠΎΠ²ΠΎΠ»ΡŒΡ‚Π°Ρ… (1 ΠΌΠΊΠ’=10-6 Π’). I

Π’ΠΎΡΠΏΠΎΠ»ΡŒΠ·ΠΎΠ²Π°Π²ΡˆΠΈΡΡŒ Π΄Π°Π½Π½ΠΎΠΉ Ρ‚Π°Π±Π»ΠΈΡ†Π΅ΠΉ, Π½Π΅ Ρ‚Ρ€ΡƒΠ΄Π½ΠΎ Ρ€Π΅ΡˆΠΈΡ‚ΡŒ ΠΈ ΠΎΠ±Ρ€Π°Ρ‚Π½ΡƒΡŽ Π·Π°Π΄Π°Ρ‡Ρƒ β€” ΠΏΠΎ Π½Π΅ΠΎΠ±Ρ…ΠΎΠ΄ΠΈΠΌΠΎΠΌΡƒ Π½Π°ΠΏΡ€ΡΠΆΠ΅Π½ΠΈΡŽ ΠΎΠΏΡ€Π΅Π΄Π΅Π»ΠΈΡ‚ΡŒ, ΠΊΠ°ΠΊΠΎΠ΅ Π½ΡƒΠΆΠ½ΠΎ ΡƒΡΡ‚Π°Π½ΠΎΠ²ΠΈΡ‚ΡŒ ослаблСниС сигнала Π½Π° Π°Ρ‚Ρ‚Π΅Π½ΡŽΠ°Ρ‚ΠΎΡ€Π΅ Π² Π΄Π΅Ρ†ΠΈΠ±Π΅Π»Π°Ρ…. НапримСр, Ρ‡Ρ‚ΠΎΠ±Ρ‹ ΠΏΠΎΠ»ΡƒΡ‡ΠΈΡ‚ΡŒ Π½Π° Π²Ρ‹Ρ…ΠΎΠ΄Π΅ Π³Π΅Π½Π΅Ρ€Π°Ρ‚ΠΎΡ€Π° напряТСния 5 ΠΌΠΊΠ’, ΠΊΠ°ΠΊ Π²ΠΈΠ΄Π½ΠΎ ΠΈΠ· Ρ‚Π°Π±Π»ΠΈΡ†Ρ‹, Π½Π° Π°Ρ‚Ρ‚Π΅Π½ΡŽΠ°Ρ‚ΠΎΡ€Π΅ потрСбуСтся ΡƒΡΡ‚Π°Π½ΠΎΠ²ΠΈΡ‚ΡŒ ослаблСниС 100+6=106 Π΄Π‘. ΠžΡ‚Π½ΠΎΡˆΠ΅Π½ΠΈΠ΅ мощностСй Π΄Π²ΡƒΡ… сигналов Π² Π΄Π΅Ρ†ΠΈΠ±Π΅Π»Π°Ρ… вычисляСтся ΠΏΠΎ Ρ„ΠΎΡ€ΠΌΡƒΠ»Π΅:

Π€ΠΎΡ€ΠΌΡƒΠ»Π° для мощности справСдлива ΠΏΡ€ΠΈ условии, Ρ‡Ρ‚ΠΎ Π²Ρ…ΠΎΠ΄Π½ΠΎΠ΅ ΠΈ Π²Ρ‹Ρ…ΠΎΠ΄Π½ΠΎΠ΅ сопротивлСния устройства ΠΎΠ΄ΠΈΠ½Π°ΠΊΠΎΠ²Ρ‹Π΅, Ρ‡Ρ‚ΠΎ часто выполняСтся Π² высокочастотных устройствах для облСгчСния ΠΈΡ… согласования ΠΌΠ΅ΠΆΠ΄Ρƒ собой.

Для опрСдСлСния мощности ΠΌΠΎΠΆΠ½ΠΎ Π²ΠΎΡΠΏΠΎΠ»ΡŒΠ·ΠΎΠ²Π°Ρ‚ΡŒΡΡ посчитанной Ρ‚Π°Π±Π». 6.2

НСрСдко ΠΏΡ€ΠΈ практичСском использовании Π΄Π‘ Π²Π°ΠΆΠ½ΠΎ Π·Π½Π°Ρ‚ΡŒ ΠΈ Π°Π±ΡΠΎΠ»ΡŽΡ‚Π½ΠΎΠ΅ Π·Π½Π°Ρ‡Π΅Π½ΠΈΠ΅ ΡΠΎΠΎΡ‚Π½ΠΎΡˆΠ΅Π½ΠΈΡ Π΄Π²ΡƒΡ… Π²Π΅Π»ΠΈΡ‡ΠΈΠ½, Ρ‚.Π΅. Π²ΠΎ сколько Ρ€Π°Π· напряТСниС ΠΈΠ»ΠΈ ΠΌΠΎΡ‰Π½ΠΎΡΡ‚ΡŒ Π½Π° Π²Ρ‹Ρ…ΠΎΠ΄Π΅ большС, Ρ‡Π΅ΠΌ Π½Π° Π²Ρ…ΠΎΠ΄Π΅ (ΠΈΠ»ΠΈ Π½Π°ΠΎΠ±ΠΎΡ€ΠΎΡ‚). Если ΠΎΡ‚Π½ΠΎΡˆΠ΅Π½ΠΈΠ΅ Π΄Π²ΡƒΡ… Π²Π΅Π»ΠΈΡ‡ΠΈΠ½ ΠΎΠ±ΠΎΠ·Π½Π°Ρ‡ΠΈΡ‚ΡŒ: K=U2/U1 ΠΈΠ»ΠΈ К=Π 2/Π 1, Ρ‚ΠΎ ΠΌΠΎΠΆΠ½ΠΎ Π²ΠΎΡΠΏΠΎΠ»ΡŒΠ·ΠΎΠ²Π°Ρ‚ΡŒΡΡ Ρ‚Π°Π±Π». 6.3 для ΠΏΠ΅Ρ€Π΅Π²ΠΎΠ΄Π° Π²Π΅Π»ΠΈΡ‡ΠΈΠ½Ρ‹ ΠΈΠ· Π΄Π‘ Π² Ρ€Π°Π·Ρ‹ (К) ΠΈ Π½Π°ΠΎΠ±ΠΎΡ€ΠΎΡ‚.

Π’Π°ΠΊ, Π½Π°ΠΏΡ€ΠΈΠΌΠ΅Ρ€, Π°Π½Ρ‚Π΅Π½Π½Ρ‹ΠΉ ΡƒΡΠΈΠ»ΠΈΡ‚Π΅Π»ΡŒ обСспСчиваСт усилСниС сигнала ΠΏΠΎ мощности Π½Π° 28 Π΄Π‘. Из Ρ‚Π°Π±Π». 6.3 Π²ΠΈΠ΄Π½ΠΎ, Ρ‡Ρ‚ΠΎ усилСниС сигнала выполняСтся Π² 631 Ρ€Π°Π·.

Π›ΠΈΡ‚Π΅Ρ€Π°Ρ‚ΡƒΡ€Π°: И.П. ШСлСстов β€” Π Π°Π΄ΠΈΠΎΠ»ΡŽΠ±ΠΈΡ‚Π΅Π»ΡΠΌ ΠΏΠΎΠ»Π΅Π·Π½Ρ‹Π΅ схСмы, ΠΊΠ½ΠΈΠ³Π° 3.

Онлайн ΠΊΠ°Π»ΡŒΠΊΡƒΠ»ΡΡ‚ΠΎΡ€ ΠΏΠ΅Ρ€Π΅Π²ΠΎΠ΄Π° Π΄Π΅Ρ†ΠΈΠ±Π΅Π» Π² Ρ€Π°Π·Ρ‹, напряТСний Π² ΠΌΠΎΡ‰Π½ΠΎΡΡ‚ΡŒ.

Π”Π΅Ρ†ΠΈΠ±Π΅Π». Π§Ρ‚ΠΎ Π·Π° странный пассаТир? Π›Π°Π΄Π½ΠΎ Π±Ρ‹ Π΄Π΅Π±ΠΈΠ», ΠΈΠ»ΠΈ, Π½Π° Ρ…ΡƒΠ΄ΠΎΠΉ ΠΊΠΎΠ½Π΅Ρ†, ΠΈΠΌΠ±Π΅Ρ†ΠΈΠ», Ρ‚Π°ΠΊ вСдь Π½Π΅Ρ‚ β€” Π΄Π΅Ρ†ΠΈΠ±Π΅Π», ΠΌΠ°Ρ‚ΡŒ Π΅Π³ΠΎ.
Π’Ρ‹ΠΏΠΈΠ»ΠΈ ΠΏΠΎ Π΄Π΅Ρ†Π΅Π»Ρƒ, закусили, понимания Π½Π΅ ΠΏΡ€ΠΈΠ±Π°Π²ΠΈΠ»ΠΎ, Π΅Ρ‰Ρ‘ ΠΏΠΎ сто, ΡƒΠΆΠ΅ Π»ΡƒΡ‡ΡˆΠ΅ β€” Π½Π°Ρ‡Π°Π»ΠΈ Π³Π΅Π½Π΅Ρ€ΠΈΡ‚ΡŒ ΠΌΡ‹ΡΠ»ΡŽ.
И Π½Π° ΠΊΠΎΠΉ Ρ…Ρ€Π΅Π½ Π½Π°ΠΌ Π² Π±Π°Ρ‚Π°Ρ€Π΅Π΅ Ρ€Π°Π·Π²ΠΎΠ΄ΠΈΡ‚ΡŒ ΠΌΡƒΠ΄Ρ€Ρ‘Π½Ρ‹Π΅ Π²Π΅Π»ΠΈΡ‡ΠΈΠ½Ρ‹, Π΄Π° Π΅Ρ‰Ρ‘ (Π½Π΅ ΠΏΡ€ΠΈ Π±Π°Π±Π°Ρ… Π±ΡƒΠ΄Π΅Ρ‚ сказано), числСнно Ρ€Π°Π²Π½Ρ‹Π΅ дСсятичному Π»ΠΎΠ³Π°Ρ€ΠΈΡ„ΠΌΡƒ Π±Π΅Π·Ρ€Π°Π·ΠΌΠ΅Ρ€Π½ΠΎΠ³ΠΎ ΠΎΡ‚Π½ΠΎΡˆΠ΅Π½ΠΈΡ физичСской Π²Π΅Π»ΠΈΡ‡ΠΈΠ½Ρ‹ ΠΊ ΠΎΠ΄Π½ΠΎΠΈΠΌΡ‘Π½Π½ΠΎΠΉ физичСской Π²Π΅Π»ΠΈΡ‡ΠΈΠ½Π΅, ΠΏΡ€ΠΈΠ½ΠΈΠΌΠ°Π΅ΠΌΠΎΠΉ Π·Π° ΠΈΡΡ…ΠΎΠ΄Π½ΡƒΡŽ, ΡƒΠΌΠ½ΠΎΠΆΠ΅Π½Π½ΠΎΠΌΡƒ Π½Π° Π΄Π΅ΡΡΡ‚ΡŒ?
Всё Ρ€Π°Π²Π½ΠΎ β€” ΠΊΠ°ΠΊ отмСряли ΠΏΠΎΡ‚Π΅Ρ€ΠΈ сигнала Π² линиях ΠΊΠΈΠ»ΠΎΠΌΠ΅Ρ‚Ρ€Π°ΠΌΠΈ стандартного кабСля, Ρ‚Π°ΠΊ ΠΈ Π±ΡƒΠ΄Π΅ΠΌ ΠΎΡ‚ΠΌΠ΅Ρ€ΡΡ‚ΡŒ.

ΠžΡ‚Π²Π΅Ρ‚ Π½Π΅ слоТСн β€” для удобства мировосприятия.
ΠŸΡ€ΠΈΡ€ΠΎΠ΄Π° наша Ρ‚Π°ΠΊΠΎΠ²Π°, Ρ‡Ρ‚ΠΎ воздСйствиС Π½Π° ΠΎΡ€Π³Π°Π½Ρ‹ чувств ΠΌΠ½ΠΎΠ³ΠΈΡ… физичСских ΠΈ биологичСских процСссов ΠΏΡ€ΠΎΠΏΠΎΡ€Ρ†ΠΈΠΎΠ½Π°Π»ΡŒΠ½ΠΎ Π½Π΅ Π°ΠΌΠΏΠ»ΠΈΡ‚ΡƒΠ΄Π΅ Π²Ρ…ΠΎΠ΄Π½ΠΎΠ³ΠΎ воздСйствия, Π° Π»ΠΎΠ³Π°Ρ€ΠΈΡ„ΠΌΡƒ Π²Ρ…ΠΎΠ΄Π½ΠΎΠ³ΠΎ воздСйствия. ΠŸΠΎΡΡ‚ΠΎΠΌΡƒ ΠΈ ΡΠΎΠ·Π΅Ρ€Ρ†Π°Ρ‚ΡŒ отобраТСния Π±ΠΎΠ»ΡŒΡˆΠΈΡ… Π΄ΠΈΠ°ΠΏΠ°Π·ΠΎΠ½ΠΎΠ² ΠΈΠ·ΠΌΠ΅Π½ΡΡŽΡ‰ΠΈΡ…ΡΡ Π²Π΅Π»ΠΈΡ‡ΠΈΠ½ ΡƒΠ΄ΠΎΠ±Π½Π΅Π΅ всСго Π² логарифмичСском ΠΌΠ°ΡΡˆΡ‚Π°Π±Π΅.

Π˜Ρ‚Π°ΠΊ, Π΄Π΅Ρ†ΠΈΠ±Π΅Π»Ρ‹ β€” это ΡΠΎΠΎΡ‚Π½ΠΎΡˆΠ΅Π½ΠΈΠ΅ Π΄Π²ΡƒΡ… Π²Π΅Π»ΠΈΡ‡ΠΈΠ½, Π²Ρ‹Ρ€Π°ΠΆΠ΅Π½Π½ΠΎΠ΅ Π² логарифмичСском ΠΌΠ°ΡΡˆΡ‚Π°Π±Π΅. ΠŸΡ€ΠΈ этом ΠΎΡ‚Π½ΠΎΡˆΠ΅Π½ΠΈΠ΅ Ρ‚ΠΎΠΊΠΎΠ² ΠΈ напряТСний ΠΈΠΌΠ΅Π΅Ρ‚ коэффициСнт 20, Π° ΠΎΡ‚Π½ΠΎΡˆΠ΅Π½ΠΈΠ΅ мощностСй β€” коэффициСнт 10.
Для напряТСний Ρ„ΠΎΡ€ΠΌΡƒΠ»Π° ΠΏΡ€ΠΈΠΎΠ±Ρ€Π΅Ρ‚Π°Π΅Ρ‚ Π²ΠΈΠ΄ , Π° для мощностСй β€” .
Если Π² лСсах Π§ΡƒΡ…Π»ΠΎΠΌΡ‹ Ρƒ нас Π·Π°Ρ‚Π΅Ρ€ΡΠ»ΠΎΡΡŒ ΠΊΠ°ΠΊΠΎΠ΅-Π»ΠΈΠ±ΠΎ элСктронноС устройство, Ρ‚ΠΎ Π² качСствС ΠΎΡ‚Π½ΠΎΡˆΠ΅Π½ΠΈΡ напряТСний (Π»ΠΈΠ±ΠΎ Ρ‚ΠΎΠΊΠΎΠ², Π»ΠΈΠ±ΠΎ мощностСй) принимаСтся ΠΎΡ‚Π½ΠΎΡˆΠ΅Π½ΠΈΠ΅ Π²Ρ‹Ρ…ΠΎΠ΄Π½ΠΎΠΉ Π²Π΅Π»ΠΈΡ‡ΠΈΠ½Ρ‹ ΠΊ Π²Ρ…ΠΎΠ΄Π½ΠΎΠΉ, ΠΈ это ΠΎΡ‚Π½ΠΎΡˆΠ΅Π½ΠΈΠ΅ называСтся коэффициСнтом ΠΏΠ΅Ρ€Π΅Π΄Π°Ρ‡ΠΈ, ΠΈΠ»ΠΈ коэффициСнтом прСобразования Π΄Π°Π½Π½ΠΎΠ³ΠΎ устройства.

Пока Ρ…Π²Π°Ρ‚ΠΈΡ‚, нарисуСм Ρ‚Π°Π±Π»ΠΈΡ†Ρƒ.

Π’ΠΠ‘Π›Π˜Π¦Π ΠŸΠ•Π Π•Π’ΠžΠ”Π ΠžΠ’ΠΠžΠ¨Π•ΠΠ˜Π™ Π’Π•Π›Π˜Π§Π˜Π Π’ Π”Π•Π¦Π˜Π‘Π•Π›Π›Π«

ΠšΠΎΡΡ„Ρ„ΠΈΡ†ΠΈΠ΅Π½Ρ‚ ΠΏΠ΅Ρ€Π΅Π΄Π°Ρ‡ΠΈ, Π²Ρ‹Ρ€Π°ΠΆΠ΅Π½Π½Ρ‹ΠΉ Π² Π΄Π΅Ρ†ΠΈΠ±Π΅Π»Π°Ρ…, ΠΌΠΎΠΆΠ΅Ρ‚ ΠΈΠΌΠ΅Ρ‚ΡŒ Π·Π½Π°ΠΊ плюс ΠΈΠ»ΠΈ минус Π² зависимости ΠΎΡ‚ ΡΠΎΠΎΡ‚Π½ΠΎΡˆΠ΅Π½ΠΈΡ Π²Π΅Π»ΠΈΡ‡ΠΈΠ½ Π½Π° Π²Ρ‹Ρ…ΠΎΠ΄Π΅ ΠΈ Π²Ρ…ΠΎΠ΄Π΅ (Ссли выходная Π²Π΅Π»ΠΈΡ‡ΠΈΠ½Π° большС Π²Ρ…ΠΎΠ΄Π½ΠΎΠΉ β€” плюс, Ссли мСньшС β€” минус).

А Π’Π•ΠŸΠ•Π Π¬ ΠΠΠžΠ‘ΠžΠ ΠžΠ’, Π”Π•Π¦Π˜Π‘Π•Π›Π›Π« Π’ ΠžΠ’ΠΠžΠ¨Π•ΠΠ˜Π―

Π’ случаС Π²ΠΊΠ»ΡŽΡ‡Π΅Π½ΠΈΡ ΠΏΠΎ каскадной схСмС (ΠΏΠΎΡΠ»Π΅Π΄ΠΎΠ²Π°Ρ‚Π΅Π»ΡŒΠ½ΠΎ, Π΄Ρ€ΡƒΠ³ Π·Π° Π΄Ρ€ΡƒΠ³ΠΎΠΌ) Π½Π΅ΡΠΊΠΎΠ»ΡŒΠΊΠΈΡ… устройств β€” ΠΎΠ±Ρ‰ΠΈΠΉ коэффициСнт ΠΏΠ΅Ρ€Π΅Π΄Π°Ρ‡ΠΈ Π² Π΄Π΅Ρ†ΠΈΠ±Π΅Π»ΡŒΠ½ΠΎΠΌ Π²Ρ‹Ρ€Π°ΠΆΠ΅Π½ΠΈΠΈ вычисляСтся простым слоТСниСм Π·Π½Π°Ρ‡Π΅Π½ΠΈΠΉ ΠšΠΏΠ΅Ρ€.(Π΄Π‘) ΠΊΠ°ΠΆΠ΄ΠΎΠ³ΠΎ ΠΈΠ· устройств.

А Ρ‚Π΅ΠΏΠ΅Ρ€ΡŒ ΠΏΠ΅Ρ€Π΅Π²Π΅Π΄Ρ‘ΠΌ Π»ΠΎΠ³Π°Ρ€ΠΈΡ„ΠΌΠΈΡ‡Π΅ΡΠΊΡƒΡŽ ΠΌΠ΅Ρ€Ρƒ мощности, ΠΈΠ·ΠΌΠ΅Ρ€ΡΠ΅ΠΌΡƒΡŽ Π² Π΄Π‘ΠΌ (dBm β€” Π΄Π΅Ρ†ΠΈΠ±Π΅Π» Π½Π° ΠΌΠΈΠ»Π»ΠΈΠ²Π°Ρ‚Ρ‚) Π² ΠΌΠΎΡ‰Π½ΠΎΡΡ‚ΡŒ устройства, ΠΈΠ·ΠΌΠ΅Ρ€ΡΠ΅ΠΌΡƒΡŽ Π² ΠΏΡ€ΠΈΠ²Ρ‹Ρ‡Π½Ρ‹Ρ… Π½Π°ΡˆΠ΅ΠΌΡƒ ΠΎΡ€Π³Π°Π½ΠΈΠ·ΠΌΡƒ Π²Π°Ρ‚Ρ‚Π°Ρ….
Π€ΠΎΡ€ΠΌΡƒΠ»Π° выглядит Ρ‚Π°ΠΊ: . Для Ρ‡Π΅Π³ΠΎ Π½Π°ΠΌ сдался этот Π΄Π‘ΠΌ?
На всякий ΠΏΠΎΠΆΠ°Ρ€Π½Ρ‹ΠΉ β€” Π½Π΅ΠΊΠΎΡ‚ΠΎΡ€Ρ‹Π΅ ΠΏΡ€ΠΎΠΈΠ·Π²ΠΎΠ΄ΠΈΡ‚Π΅Π»ΠΈ ΡƒΠΊΠ°Π·Ρ‹Π²Π°ΡŽΡ‚ ΠΈΠΌΠ΅Π½Π½ΠΎ этот ΠΏΠ°Ρ€Π°ΠΌΠ΅Ρ‚Ρ€, характСризуя Π±ΠΎΠ³Π°Ρ‚Ρ‹Ρ€ΡΠΊΡƒΡŽ ΠΌΠΎΡ‰ΡŒ своих ΠΈΠ·Π΄Π΅Π»ΠΈΠΉ.

Π’ΠΠ‘Π›Π˜Π¦Π ΠŸΠ•Π Π•Π’ΠžΠ”Π Π”Π‘Πœ Π’ ВАВВЫ

Π’Π°ΠΊ вСдь ΠΌΠ°Π»ΠΎ Ρ‚ΠΎΠ³ΠΎ, Ρ‡Ρ‚ΠΎ ΠΌΠΎΡ‰Π½ΠΎΡΡ‚ΡŒ усилитСлСй Π½Π°Π΄ΡƒΠΌΠ°Π»ΠΈ ΠΈΠ·ΠΌΠ΅Ρ€ΡΡ‚ΡŒ Π² Π΄Π‘ΠΌ, посягнули ΠΈ Π½Π° святоС β€” Π½Π° Ρ‡ΡƒΠ²ΡΡ‚Π²ΠΈΡ‚Π΅Π»ΡŒΠ½ΠΎΡΡ‚ΡŒ ΠΏΡ€ΠΈΡ‘ΠΌΠ½ΠΎΠΉ Π°ΠΏΠΏΠ°Ρ€Π°Ρ‚ΡƒΡ€Ρ‹. Π§ΡƒΠ²ΡΡ‚Π²ΠΈΡ‚Π΅Π»ΡŒΠ½ΠΎΡΡ‚ΡŒ стали ΠΎΠΏΡ€Π΅Π΄Π΅Π»ΡΡ‚ΡŒ ΠΊΠ°ΠΊ ΠΎΡ‚Π½ΠΎΡˆΠ΅Π½ΠΈΠ΅ мощности Π½Π° Π²Ρ…ΠΎΠ΄Π΅ ΠΏΡ€ΠΈΡ‘ΠΌΠ½ΠΈΠΊΠ° ΠΊ ΡƒΡ€ΠΎΠ²Π½ΡŽ мощности 1 ΠΌΠ’Ρ‚ ΠΈ Ρ‚Π°ΠΊΠΆΠ΅ Π²Ρ‹Ρ€Π°ΠΆΠ°Ρ‚ΡŒ Π² логарифмичСском ΠΌΠ°ΡΡˆΡ‚Π°Π±Π΅ Π² Π΄Π‘ΠΌ.
ΠšΡƒΠ΄Π° Π΄Π΅Π²Π°Ρ‚ΡŒΡΡ Π±Π΅Π΄Π½ΠΎΠΌΡƒ ΠΊΡ€Π΅ΡΡ‚ΡŒΡΠ½ΠΈΠ½Ρƒ? ΠŸΡ€ΠΈΠ΄Ρ‘Ρ‚ΡΡ привСсти Ρ‚Π°Π±Π»ΠΈΡ†Ρƒ ΠΈ для этого бСсчинства.

Π’ΠΠ‘Π›Π˜Π¦Π ΠŸΠ•Π Π•Π’ΠžΠ”Π Π”Π‘Πœ Π’ ΠœΠ˜ΠšΠ ΠžΠ’ΠžΠ›Π¬Π’Π«

А Π΅Ρ‰Ρ‘, ΠΈΠ½ΠΎΠ³Π΄Π° Π±Ρ‹Π²Π°Π΅Ρ‚ ΠΏΠΎΠ»Π΅Π·Π½ΠΎ Π·Π½Π°Ρ‚ΡŒ, ΠΊΠ°ΠΊΠΈΠΌ Π΄ΠΎΠ»ΠΆΠ΅Π½ Π±Ρ‹Ρ‚ΡŒ Ρ€Π°Π·ΠΌΠ°Ρ… Π²Ρ‹Ρ…ΠΎΠ΄Π½ΠΎΠ³ΠΎ напряТСния Π½Π° Π½Π°Π³Ρ€ΡƒΠ·ΠΊΠ΅, для получСния Π·Π°Π΄Π°Π½Π½ΠΎΠ³ΠΎ ΠΏΠ°Ρ€Π°ΠΌΠ΅Ρ‚Ρ€Π° мощности. НСкоторыС ΠΏΡ€ΠΈ расчётС Π²Ρ‹Ρ…ΠΎΠ΄Π½ΠΎΠΉ мощности ΠΏΠΎΠ»ΡŒΠ·ΡƒΡŽΡ‚ΡΡ простой Ρ„ΠΎΡ€ΠΌΡƒΠ»ΠΎΠΉ , подставляя вмСсто UΠ΄ β€” ΠΏΠΈΠΊΠΎΠ²ΠΎΠ΅ Π·Π½Π°Ρ‡Π΅Π½ΠΈΠ΅ (Π°ΠΌΠΏΠ»ΠΈΡ‚ΡƒΠ΄Π½ΠΎΠ΅ Π·Π½Π°Ρ‡Π΅Π½ΠΈΠ΅, Ρ€Π°Π²Π½ΠΎΠ΅ максимальной Π°ΠΌΠΏΠ»ΠΈΡ‚ΡƒΠ΄Π΅ ΠΏΠΎΠ»ΡƒΠ²ΠΎΠ»Π½Ρ‹ Π²Ρ‹Ρ…ΠΎΠ΄Π½ΠΎΠ³ΠΎ сигнала). Π­Ρ‚ΠΎ Π½Π΅ ΠΏΡ€Π°Π²ΠΈΠ»ΡŒΠ½ΠΎ, Π²Π΅Ρ€Π½Π΅Π΅ ΠΏΡ€Π°Π²ΠΈΠ»ΡŒΠ½ΠΎ Ρ‚ΠΎΠ»ΡŒΠΊΠΎ для сигналов ΠΏΡ€ΡΠΌΠΎΡƒΠ³ΠΎΠ»ΡŒΠ½ΠΎΠΉ Ρ„ΠΎΡ€ΠΌΡ‹. Для ΡΠΈΠ½ΡƒΡΠΎΠΈΠ΄Π°Π»ΡŒΠ½Ρ‹Ρ…, для получСния Ρ‚ΠΎΡ‡Π½ΠΎΠ³ΠΎ Ρ€Π΅Π·ΡƒΠ»ΡŒΡ‚Π°Ρ‚Π° Π½Π°Π΄ΠΎ ΠΏΠΎΠ΄ΡΡ‚Π°Π²Π»ΡΡ‚ΡŒ Π΄Π΅ΠΉΡΡ‚Π²ΡƒΡŽΡ‰Π΅Π΅ Π·Π½Π°Ρ‡Π΅Π½ΠΈΠ΅ напряТСния β€” .
Π›ΡƒΡ‡ΡˆΠ΅ ΠΏΠΎΠ½ΡΡ‚ΡŒ, Ρ‡Ρ‚ΠΎ Ρ‚Π°ΠΊΠΎΠ΅ Π°ΠΌΠΏΠ»ΠΈΡ‚ΡƒΠ΄Π½ΠΎΠ΅ Π·Π½Π°Ρ‡Π΅Π½ΠΈΠ΅, ΠΈ ΠΊΠ°ΠΊ Π½Π°ΠΉΡ‚ΠΈ Π΄Π΅ΠΉΡΡ‚Π²ΡƒΡŽΡ‰Π΅Π΅ для Ρ€Π°Π·Π»ΠΈΡ‡Π½Ρ‹Ρ… Ρ„ΠΎΡ€ΠΌ сигналов ΠΌΠΎΠΆΠ½ΠΎ Π½Π° страницС ссылка Π½Π° страницу.

Π—ΠΠ’Π˜Π‘Π˜ΠœΠžΠ‘Π’Π¬ ΠΠœΠŸΠ›Π˜Π’Π£Π”Π« ΠΠΠŸΠ Π―Π–Π•ΠΠ˜Π― ОВ МОЩНОБВИ

Π—ΠΠ’Π˜Π‘Π˜ΠœΠžΠ‘Π’Π¬ МОЩНОБВИ ОВ Π’Π«Π₯ΠžΠ”ΠΠžΠ“Πž ΠΠΠŸΠ Π―Π–Π•ΠΠ˜Π―

Π”Π΅Ρ†ΠΈΠ±Π΅Π»Ρ‹ Β«ΠΏΠΎ мощности», Β«ΠΏΠΎ Π½Π°ΠΏΡ€ΡΠΆΠ΅Π½ΠΈΡŽΒ» ΠΈ Β«ΠΏΠΎ Ρ‚ΠΎΠΊΡƒΒ»

Из ΠΏΡ€Π°Π²ΠΈΠ»Π° (см. Π²Ρ‹ΡˆΠ΅) слСдуСт, Ρ‡Ρ‚ΠΎ Π΄Π‘ Π±Ρ‹Π²Π°ΡŽΡ‚ Ρ‚ΠΎΠ»ΡŒΠΊΠΎ Β«ΠΏΠΎ мощности». Π’Π΅ΠΌ Π½Π΅ ΠΌΠ΅Π½Π΅Π΅, Π² случаС равСнства R1 = R (Π² частности, Ссли R1 ΠΈ R β€” ΠΎΠ΄Π½ΠΎ ΠΈ Ρ‚ΠΎ ΠΆΠ΅ сопротивлСниС, ΠΈΠ»ΠΈ Π² случаС, Ссли ΡΠΎΠΎΡ‚Π½ΠΎΡˆΠ΅Π½ΠΈΠ΅ сопротивлСний R1 ΠΈ R ΠΏΠΎ Ρ‚ΠΎΠΉ ΠΈΠ»ΠΈ ΠΈΠ½ΠΎΠΉ ΠΏΡ€ΠΈΡ‡ΠΈΠ½Π΅ Π½Π΅ Π²Π°ΠΆΠ½ΠΎ) говорят ΠΎ Π΄Π‘ Β«ΠΏΠΎ Π½Π°ΠΏΡ€ΡΠΆΠ΅Π½ΠΈΡŽΒ» ΠΈ Β«ΠΏΠΎ Ρ‚ΠΎΠΊΡƒΒ», подразумСвая ΠΏΡ€ΠΈ этом выраТСния:

Π΄Π‘ ΠΏΠΎ Π½Π°ΠΏΡ€ΡΠΆΠ΅Π½ΠΈΡŽ = ;

Π΄Π‘ ΠΏΠΎ Ρ‚ΠΎΠΊΡƒ = .

Для ΠΏΠ΅Ρ€Π΅Ρ…ΠΎΠ΄Π° ΠΎΡ‚ Β«Π΄Π‘ ΠΏΠΎ Π½Π°ΠΏΡ€ΡΠΆΠ΅Π½ΠΈΡŽΒ» (Β«Π΄Π‘ ΠΏΠΎ Ρ‚ΠΎΠΊΡƒΒ») ΠΊ Β«Π΄Π‘ ΠΏΠΎ мощности» слСдуСт Ρ‡Ρ‘Ρ‚ΠΊΠΎ ΠΎΠΏΡ€Π΅Π΄Π΅Π»ΠΈΡ‚ΡŒ, Π½Π° ΠΊΠ°ΠΊΠΈΡ… ΠΈΠΌΠ΅Π½Π½ΠΎ сопротивлСниях (Ρ€Π°Π²Π½Ρ‹Ρ… ΠΈΠ»ΠΈ Π½Π΅ Ρ€Π°Π²Π½Ρ‹Ρ… Π΄Ρ€ΡƒΠ³ Π΄Ρ€ΡƒΠ³Ρƒ) Ρ€Π΅Π³ΠΈΡΡ‚Ρ€ΠΈΡ€ΠΎΠ²Π°Π»ΠΈΡΡŒ напряТСниС (Ρ‚ΠΎΠΊ). Если R1 Π½Π΅ Ρ€Π°Π²Π½ΠΎ R, слСдуСт ΠΏΠΎΠ»ΡŒΠ·ΠΎΠ²Π°Ρ‚ΡŒΡΡ Π²Ρ‹Ρ€Π°ΠΆΠ΅Π½ΠΈΠ΅ΠΌ для ΠΎΠ±Ρ‰Π΅Π³ΠΎ случая (см. Π²Ρ‹ΡˆΠ΅).

НСтрудно ΠΏΠΎΠ΄ΡΡ‡ΠΈΡ‚Π°Ρ‚ΡŒ, Ρ‡Ρ‚ΠΎ, Π² частности:

ΠΏΡ€ΠΈ рСгистрации мощности измСнСнию Π½Π° +1 Π΄Π‘ (+1 Π΄Π‘ Β«ΠΏΠΎ мощности») соотвСтствуСт ΠΏΡ€ΠΈΡ€Π°Ρ‰Π΅Π½ΠΈΠ΅ мощности Π² β‰ˆ1,259 Ρ€Π°Π·Π°, измСнСнию Π½Π° βˆ’3,01 Π΄Π‘ β€” сниТСниС мощности Π² Π΄Π²Π° Ρ€Π°Π·Π°, Π² Ρ‚ΠΎ врСмя ΠΊΠ°ΠΊ

ΠΏΡ€ΠΈ рСгистрации напряТСния (силы Ρ‚ΠΎΠΊΠ°) измСнСнию Π½Π° +1 Π΄Π‘ (+1 Π΄Π‘ Β«ΠΏΠΎ Π½Π°ΠΏΡ€ΡΠΆΠ΅Π½ΠΈΡŽΒ», Β«ΠΏΠΎ Ρ‚ΠΎΠΊΡƒΒ») Π±ΡƒΠ΄Π΅Ρ‚ ΡΠΎΠΎΡ‚Π²Π΅Ρ‚ΡΡ‚Π²ΠΎΠ²Π°Ρ‚ΡŒ ΠΏΡ€ΠΈΡ€Π°Ρ‰Π΅Π½ΠΈΠ΅ напряТСния (силы Ρ‚ΠΎΠΊΠ°) Π² β‰ˆ1,122 Ρ€Π°Π·Π°, ΠΏΡ€ΠΈ ΠΈΠ·ΠΌΠ΅Π½Π΅Π½ΠΈΠΈ Π½Π° βˆ’3,01 Π΄Π‘ напряТСниС (сила Ρ‚ΠΎΠΊΠ°) снизятся ΠΈ составят β‰ˆ 0,707 ΠΎΡ‚ своСго исходного значСния.

ΠŸΡ€ΠΈΠΌΠ΅Ρ€Ρ‹ вычислСний ΠŸΠ΅Ρ€Π΅Ρ…ΠΎΠ΄ ΠΊ Π΄Π‘

ΠŸΡƒΡΡ‚ΡŒ Π·Π½Π°Ρ‡Π΅Π½ΠΈΠ΅ мощности P1 стало Π² 2 Ρ€Π°Π·Π° большС исходного значСния мощности P, Ρ‚ΠΎΠ³Π΄Π°

10 lg(P1/P) = 10 lg(2) β‰ˆ3,0103 Π΄Π‘ β‰ˆ 3 Π΄Π‘,

Ρ‚ΠΎ Π΅ΡΡ‚ΡŒ рост мощности Π½Π° 3 Π΄Π‘ ΠΎΠ·Π½Π°Ρ‡Π°Π΅Ρ‚ Π΅Ρ‘ ΡƒΠ²Π΅Π»ΠΈΡ‡Π΅Π½ΠΈΠ΅ Π² 2 Ρ€Π°Π·Π°.

ΠŸΡƒΡΡ‚ΡŒ Π·Π½Π°Ρ‡Π΅Π½ΠΈΠ΅ мощности P1 стало Π² 2 Ρ€Π°Π·Π° мСньшС исходного значСния мощности P, Ρ‚ΠΎ Π΅ΡΡ‚ΡŒ P1 = 0,5 P. Π’ΠΎΠ³Π΄Π°

Ρ‚ΠΎ Π΅ΡΡ‚ΡŒ сниТСниС мощности Π½Π° 3 Π΄Π‘ ΠΎΠ·Π½Π°Ρ‡Π°Π΅Ρ‚ Π΅Ρ‘ сниТСниС Π² 2 Ρ€Π°Π·Π°. По Π°Π½Π°Π»ΠΎΠ³ΠΈΠΈ:

рост мощности Π² 10 Ρ€Π°Π·: 10 lg(P1/P) = 10 lg(10) = 10 Π΄Π‘, сниТСниС Π² 10 Ρ€Π°Π·: 10 lg(P1/P) = 10 lg(0,1)= βˆ’10 Π΄Π‘;

рост Π² 1 ΠΌΠ»Π½ Ρ€Π°Π·: 10 lg(P1/P) = 10 lg(1 000 000) = 60 Π΄Π‘, сниТСниС Π² 1 ΠΌΠ»Π½ Ρ€Π°Π·: 10 lg(P1/P) = 10 lg(0,000001) = βˆ’60 Π΄Π‘.

ΠŸΠ΅Ρ€Π΅Ρ…ΠΎΠ΄ ΠΎΡ‚ Π΄Π‘ ΠΊ Β«Ρ€Π°Π·Π°ΠΌΒ»

ИзмСнСниС Β«Π² Ρ€Π°Π·Π°Ρ…Β» ΠΏΠΎ извСстному измСнСнию Π² Π΄Π‘ (условноС ΠΎΠ±ΠΎΠ·Π½Π°Ρ‡Π΅Π½ΠΈΠ΅ Β«dBΒ» Π² Ρ„ΠΎΡ€ΠΌΡƒΠ»Π°Ρ… Π½ΠΈΠΆΠ΅) вычисляСтся ΡΠ»Π΅Π΄ΡƒΡŽΡ‰ΠΈΠΌ ΠΎΠ±Ρ€Π°Π·ΠΎΠΌ:

для мощности: ; Ρ‚Π°ΠΊΠΈΠΌ ΠΎΠ±Ρ€Π°Π·ΠΎΠΌ, Π½Π°ΠΏΡ€ΠΈΠΌΠ΅Ρ€, Ссли ΠΈΠ·ΠΌΠ΅Π½Π΅Π½ΠΈΠ΅ мощности составило +20 Π΄Π΅Ρ†ΠΈΠ±Π΅Π», это Π·Π½Π°Ρ‡ΠΈΡ‚, Ρ‡Ρ‚ΠΎ Β«P1 большС P Π½Π° Π΄Π²Π° порядка» ΠΈΠ»ΠΈ Β«P1 большС P Π² 100 Ρ€Π°Π·Β»;

для напряТСния (силы Ρ‚ΠΎΠΊΠ°): ; Ρ‚Π°ΠΊΠΈΠΌ ΠΎΠ±Ρ€Π°Π·ΠΎΠΌ, Π½Π°ΠΏΡ€ΠΈΠΌΠ΅Ρ€, Ссли ΠΈΠ·ΠΌΠ΅Π½Π΅Π½ΠΈΠ΅ напряТСния составило +20 Π΄Π΅Ρ†ΠΈΠ±Π΅Π», это Π·Π½Π°Ρ‡ΠΈΡ‚, Ρ‡Ρ‚ΠΎ Β«U1большС U Π½Π° порядок» ΠΈΠ»ΠΈ Β«Π² 10 Ρ€Π°Π·Β».

Π§Ρ‚ΠΎ Ρ‚Π°ΠΊΠΎΠ΅ Π΄Π΅Ρ†ΠΈΠ±Π΅Π»?

ΠŸΠ΅Ρ€Π΅Π²ΠΎΠ΄ ΠΈΠ· Π΄Π΅Ρ†ΠΈΠ±Π΅Π» Π² Ρ€Π°Π·Ρ‹ ΠΈ ΠΎΠ±Ρ€Π°Ρ‚Π½ΠΎ

Π”ΠΎΠ²ΠΎΠ»ΡŒΠ½ΠΎ часто Π² популярной радиотСхничСской Π»ΠΈΡ‚Π΅Ρ€Π°Ρ‚ΡƒΡ€Π΅, Π² описании элСктронных схСм употрСбляСтся Π΅Π΄ΠΈΠ½ΠΈΡ†Π° измСрСния – Π΄Π΅Ρ†ΠΈΠ±Π΅Π» (Π΄Π‘ ΠΈΠ»ΠΈ dB).

ΠŸΡ€ΠΈ ΠΈΠ·ΡƒΡ‡Π΅Π½ΠΈΠΈ элСктроники Π½Π°Ρ‡ΠΈΠ½Π°ΡŽΡ‰ΠΈΠΉ Ρ€Π°Π΄ΠΈΠΎΠ»ΡŽΠ±ΠΈΡ‚Π΅Π»ΡŒ ΠΏΡ€ΠΈΠ²Ρ‹ΠΊ ΠΊ Ρ‚Π°ΠΊΠΈΠΌ Π°Π±ΡΠΎΠ»ΡŽΡ‚Π½Ρ‹ΠΌ Π΅Π΄ΠΈΠ½ΠΈΡ†Π°ΠΌ измСрСния ΠΊΠ°ΠΊ АмпСр (сила Ρ‚ΠΎΠΊΠ°), Π’ΠΎΠ»ΡŒΡ‚ (напряТСниС ΠΈ Π­Π”Π‘), Ом (элСктричСскоС сопротивлСниС) ΠΈ ΠΌΠ½ΠΎΠ³ΠΈΠΌ Π΄Ρ€ΡƒΠ³ΠΈΠΌ, с ΠΏΠΎΠΌΠΎΡ‰ΡŒΡŽ ΠΊΠΎΡ‚ΠΎΡ€Ρ‹Ρ… ΠΎΠ±ΠΎΠ·Π½Π°Ρ‡Π°ΡŽΡ‚ количСствСнно Ρ‚ΠΎΡ‚ ΠΈΠ»ΠΈ ΠΈΠ½ΠΎΠΉ элСктричСский ΠΏΠ°Ρ€Π°ΠΌΠ΅Ρ‚Ρ€ (Ρ‘ΠΌΠΊΠΎΡΡ‚ΡŒ, ΠΈΠ½Π΄ΡƒΠΊΡ‚ΠΈΠ²Π½ΠΎΡΡ‚ΡŒ, частоту).

ΠΠ°Ρ‡ΠΈΠ½Π°ΡŽΡ‰Π΅ΠΌΡƒ Ρ€Π°Π΄ΠΈΠΎΠ»ΡŽΠ±ΠΈΡ‚Π΅Π»ΡŽ, ΠΊΠ°ΠΊ ΠΏΡ€Π°Π²ΠΈΠ»ΠΎ, Π½Π΅ составляСт особого Ρ‚Ρ€ΡƒΠ΄Π° Ρ€Π°Π·ΠΎΠ±Ρ€Π°Ρ‚ΡŒΡΡ, Ρ‡Ρ‚ΠΎ Ρ‚Π°ΠΊΠΎΠ΅ Π°ΠΌΠΏΠ΅Ρ€ ΠΈΠ»ΠΈ Π²ΠΎΠ»ΡŒΡ‚. Π’ΡƒΡ‚ всё понятно, Π΅ΡΡ‚ΡŒ элСктричСский ΠΏΠ°Ρ€Π°ΠΌΠ΅Ρ‚Ρ€ ΠΈΠ»ΠΈ Π²Π΅Π»ΠΈΡ‡ΠΈΠ½Π°, ΠΊΠΎΡ‚ΠΎΡ€ΡƒΡŽ Π½ΡƒΠΆΠ½ΠΎ ΠΈΠ·ΠΌΠ΅Ρ€ΠΈΡ‚ΡŒ. Π•ΡΡ‚ΡŒ Π½Π°Ρ‡Π°Π»ΡŒΠ½Ρ‹ΠΉ ΡƒΡ€ΠΎΠ²Π΅Π½ΡŒ отсчёта, ΠΊΠΎΡ‚ΠΎΡ€Ρ‹ΠΉ принимаСтся ΠΏΠΎ ΡƒΠΌΠΎΠ»Ρ‡Π°Π½ΠΈΡŽ Π² Ρ„ΠΎΡ€ΠΌΡƒΠ»ΠΈΡ€ΠΎΠ²ΠΊΠ΅ Π΄Π°Π½Π½ΠΎΠΉ Π΅Π΄ΠΈΠ½ΠΈΡ†Ρ‹ измСрСния. Π•ΡΡ‚ΡŒ условноС ΠΎΠ±ΠΎΠ·Π½Π°Ρ‡Π΅Π½ΠΈΠ΅ этого ΠΏΠ°Ρ€Π°ΠΌΠ΅Ρ‚Ρ€Π° ΠΈΠ»ΠΈ Π²Π΅Π»ΠΈΡ‡ΠΈΠ½Ρ‹ (A, V). И Π²ΠΏΡ€Π°Π²Π΄Ρƒ, ΠΊΠ°ΠΊ Ρ‚ΠΎΠ»ΡŒΠΊΠΎ ΠΌΡ‹ Ρ‡ΠΈΡ‚Π°Π΅ΠΌ надпись 12 V, Ρ‚ΠΎ ΠΌΡ‹ ΠΏΠΎΠ½ΠΈΠΌΠ°Π΅ΠΌ, Ρ‡Ρ‚ΠΎ Ρ€Π΅Ρ‡ΡŒ ΠΈΠ΄Ρ‘Ρ‚ ΠΎ напряТСнии, Π°Π½Π°Π»ΠΎΠ³ΠΈΡ‡Π½ΠΎΠΌ, Π½Π°ΠΏΡ€ΠΈΠΌΠ΅Ρ€, Π½Π°ΠΏΡ€ΡΠΆΠ΅Π½ΠΈΡŽ Π°Π²Ρ‚ΠΎΠΌΠΎΠ±ΠΈΠ»ΡŒΠ½ΠΎΠΉ аккумуляторной Π±Π°Ρ‚Π°Ρ€Π΅ΠΈ.

Но ΠΊΠ°ΠΊ Ρ‚ΠΎΠ»ΡŒΠΊΠΎ встрСчаСтся надпись, ΠΊ ΠΏΡ€ΠΈΠΌΠ΅Ρ€Ρƒ: напряТСниС ΠΏΠΎΠ²Ρ‹ΡΠΈΠ»ΠΎΡΡŒ Π½Π° 3 Π΄Π‘ ΠΈΠ»ΠΈ ΠΌΠΎΡ‰Π½ΠΎΡΡ‚ΡŒ сигнала составляСт 10 Π΄Π‘ΠΌ (10 dBm), Ρ‚ΠΎ Ρƒ ΠΌΠ½ΠΎΠ³ΠΈΡ… Π²ΠΎΠ·Π½ΠΈΠΊΠ°Π΅Ρ‚ Π½Π΅Π΄ΠΎΡƒΠΌΠ΅Π½ΠΈΠ΅. Как это? ΠŸΠΎΡ‡Π΅ΠΌΡƒ упоминаСтся напряТСниС ΠΈΠ»ΠΈ ΠΌΠΎΡ‰Π½ΠΎΡΡ‚ΡŒ, Π° Π·Π½Π°Ρ‡Π΅Π½ΠΈΠ΅ указываСтся Π² ΠΊΠ°ΠΊΠΈΡ…-Ρ‚ΠΎ Π΄Π΅Ρ†ΠΈΠ±Π΅Π»Π°Ρ…?

ΠŸΡ€Π°ΠΊΡ‚ΠΈΠΊΠ° ΠΏΠΎΠΊΠ°Π·Ρ‹Π²Π°Π΅Ρ‚, Ρ‡Ρ‚ΠΎ Π½Π΅ ΠΌΠ½ΠΎΠ³ΠΈΠ΅ Π½Π°Ρ‡ΠΈΠ½Π°ΡŽΡ‰ΠΈΠ΅ Ρ€Π°Π΄ΠΈΠΎΠ»ΡŽΠ±ΠΈΡ‚Π΅Π»ΠΈ ΠΏΠΎΠ½ΠΈΠΌΠ°ΡŽΡ‚, Ρ‡Ρ‚ΠΎ ΠΆΠ΅ Ρ‚Π°ΠΊΠΎΠ΅ Π΄Π΅Ρ†ΠΈΠ±Π΅Π». ΠŸΠΎΠΏΡ‹Ρ‚Π°Π΅ΠΌΡΡ Ρ€Π°Π·Π²Π΅ΡΡ‚ΡŒ нСпроглядный Ρ‚ΡƒΠΌΠ°Π½ Π½Π°Π΄ Ρ‚Π°ΠΊΠΎΠΉ таинствСнной Π΅Π΄ΠΈΠ½ΠΈΡ†Π΅ΠΉ измСрСния ΠΊΠ°ΠΊ Π΄Π΅Ρ†ΠΈΠ±Π΅Π».

Π§Ρ‚ΠΎ Ρ‚Π°ΠΊΠΎΠ΅ Π΄Π΅Ρ†ΠΈΠ±Π΅Π»?

Π•Π΄ΠΈΠ½ΠΈΡ†Ρƒ измСрСния ΠΏΠΎΠ΄ Π½Π°Π·Π²Π°Π½ΠΈΠ΅ΠΌ Π‘Π΅Π» стали Π²ΠΏΠ΅Ρ€Π²Ρ‹Π΅ ΠΏΡ€ΠΈΠΌΠ΅Π½ΡΡ‚ΡŒ ΠΈΠ½ΠΆΠ΅Π½Π΅Ρ€Ρ‹ Ρ‚Π΅Π»Π΅Ρ„ΠΎΠ½Π½ΠΎΠΉ Π»Π°Π±ΠΎΡ€Π°Ρ‚ΠΎΡ€ΠΈΠΈ Π‘Π΅Π»Π»Π°. Π”Π΅Ρ†ΠΈΠ±Π΅Π» являСтся дСсятой Ρ‡Π°ΡΡ‚ΡŒΡŽ Π‘Π΅Π»Π° (1 Π΄Π΅Ρ†ΠΈΠ±Π΅Π» = 0,1 Π‘Π΅Π»). На ΠΏΡ€Π°ΠΊΡ‚ΠΈΠΊΠ΅ ΡˆΠΈΡ€ΠΎΠΊΠΎ ΠΈΡΠΏΠΎΠ»ΡŒΠ·ΡƒΠ΅Ρ‚ΡΡ ΠΊΠ°ΠΊ Ρ€Π°Π· Π΄Π΅Ρ†ΠΈΠ±Π΅Π».

Как ΡƒΠΆΠ΅ Π³ΠΎΠ²ΠΎΡ€ΠΈΠ»ΠΎΡΡŒ, Π΄Π΅Ρ†ΠΈΠ±Π΅Π», это особСнная Π΅Π΄ΠΈΠ½ΠΈΡ†Π° измСрСния. Π‘Ρ‚ΠΎΠΈΡ‚ ΠΎΡ‚ΠΌΠ΅Ρ‚ΠΈΡ‚ΡŒ, Ρ‡Ρ‚ΠΎ Π΄Π΅Ρ†ΠΈΠ±Π΅Π» Π½Π΅ являСтся Ρ‡Π°ΡΡ‚ΡŒΡŽ ΠΎΡ„ΠΈΡ†ΠΈΠ°Π»ΡŒΠ½ΠΎΠΉ систСмы Π΅Π΄ΠΈΠ½ΠΈΡ† БИ. Но, нСсмотря Π½Π° это, Π΄Π΅Ρ†ΠΈΠ±Π΅Π» ΠΏΠΎΠ»ΡƒΡ‡ΠΈΠ» ΠΏΡ€ΠΈΠ·Π½Π°Π½ΠΈΠ΅ ΠΈ занял ΠΏΡ€ΠΎΡ‡Π½ΠΎΠ΅ мСсто наряду с Π΄Ρ€ΡƒΠ³ΠΈΠΌΠΈ Π΅Π΄ΠΈΠ½ΠΈΡ†Π°ΠΌΠΈ измСрСния.

ВспомнитС, ΠΊΠΎΠ³Π΄Π° ΠΌΡ‹ Ρ…ΠΎΡ‚ΠΈΠΌ ΠΎΠ±ΡŠΡΡΠ½ΠΈΡ‚ΡŒ ΠΊΠ°ΠΊΠΎΠ΅-Π»ΠΈΠ±ΠΎ ΠΈΠ·ΠΌΠ΅Π½Π΅Π½ΠΈΠ΅, ΠΌΡ‹ Π³ΠΎΠ²ΠΎΡ€ΠΈΠΌ, Ρ‡Ρ‚ΠΎ, Π½Π°ΠΏΡ€ΠΈΠΌΠ΅Ρ€, стало ярчС Π² 2 Ρ€Π°Π·Π°. Или, Π½Π°ΠΏΡ€ΠΈΠΌΠ΅Ρ€, напряТСниС ΡƒΠΏΠ°Π»ΠΎ Π² 10 Ρ€Π°Π·. ΠŸΡ€ΠΈ этом ΠΌΡ‹ устанавливаСм ΠΎΠΏΡ€Π΅Π΄Π΅Π»Ρ‘Π½Π½Ρ‹ΠΉ ΠΏΠΎΡ€ΠΎΠ³ отсчёта, ΠΎΡ‚Π½ΠΎΡΠΈΡ‚Π΅Π»ΡŒΠ½ΠΎ ΠΊΠΎΡ‚ΠΎΡ€ΠΎΠ³ΠΎ ΠΈ ΠΏΡ€ΠΎΠΈΠ·ΠΎΡˆΠ»ΠΎ ΠΈΠ·ΠΌΠ΅Π½Π΅Π½ΠΈΠ΅ Π² 10 ΠΈΠ»ΠΈ 2 Ρ€Π°Π·Π°. Π‘ ΠΏΠΎΠΌΠΎΡ‰ΡŒΡŽ Π΄Π΅Ρ†ΠΈΠ±Π΅Π» Ρ‚Π°ΠΊΠΆΠ΅ ΠΈΠ·ΠΌΠ΅Ρ€ΡΡŽΡ‚ эти β€œΡ€Π°Π·Ρ‹β€, Ρ‚ΠΎΠ»ΡŒΠΊΠΎ Π² логарифмичСском ΠΌΠ°ΡΡˆΡ‚Π°Π±Π΅.


Π“Ρ€Π°Ρ„ΠΈΠΊ логарифмичСской зависимости

НапримСр, ΠΈΠ·ΠΌΠ΅Π½Π΅Π½ΠΈΠ΅ Π½Π° 1 Π΄Π‘, соотвСтствуСт измСнСнию энСргСтичСской Π²Π΅Π»ΠΈΡ‡ΠΈΠ½Ρ‹ Π² 1,26 Ρ€Π°Π·Π°. ИзмСнСниС Π½Π° 3 Π΄Π‘ соотвСтствуСт измСнСнию энСргСтичСской Π²Π΅Π»ΠΈΡ‡ΠΈΠ½Ρ‹ Π² 2 Ρ€Π°Π·Π°.

Но Π·Π°Ρ‡Π΅ΠΌ Ρ‚Π°ΠΊ Π·Π°ΠΌΠΎΡ€Π°Ρ‡ΠΈΠ²Π°Ρ‚ΡŒΡΡ с Π΄Π΅Ρ†ΠΈΠ±Π΅Π»Π°ΠΌΠΈ, Ссли ΠΎΡ‚Π½ΠΎΡˆΠ΅Π½ΠΈΡ ΠΌΠΎΠΆΠ½ΠΎ ΠΈΠ·ΠΌΠ΅Ρ€ΡΡ‚ΡŒ Π² Ρ€Π°Π·Π°Ρ…? На этот вопрос Π½Π΅Ρ‚ ΠΎΠ΄Π½ΠΎΠ·Π½Π°Ρ‡Π½ΠΎΠ³ΠΎ ΠΎΡ‚Π²Π΅Ρ‚Π°. Но ΡƒΠΆ, ΠΏΠΎΡΠΊΠΎΠ»ΡŒΠΊΡƒ, Π΄Π΅Ρ†ΠΈΠ±Π΅Π»Ρ‹ Π°ΠΊΡ‚ΠΈΠ²Π½ΠΎ ΠΏΡ€ΠΈΠΌΠ΅Π½ΡΡŽΡ‚ΡΡ, Ρ‚ΠΎ навСрняка это ΠΎΠΏΡ€Π°Π²Π΄Π°Π½ΠΎ.

ΠŸΡ€ΠΈΡ‡ΠΈΠ½Ρ‹ для использования Π΄Π΅Ρ†ΠΈΠ±Π΅Π» всё-Ρ‚Π°ΠΊΠΈ Π΅ΡΡ‚ΡŒ. ΠŸΠ΅Ρ€Π΅Ρ‡ΠΈΡΠ»ΠΈΠΌ ΠΈΡ….

Частично ΠΎΡ‚Π²Π΅Ρ‚ Π½Π° этот вопрос кроСтся Π² Ρ‚Π°ΠΊ Π½Π°Π·Ρ‹Π²Π°Π΅ΠΌΠΎΠΌ Π·Π°ΠΊΠΎΠ½Π΅ Π’Π΅Π±Π΅Ρ€Π°-Π€Π΅Ρ…Π½Π΅Ρ€Π°. Π­Ρ‚ΠΎ эмпиричСский психофизиологичСский Π·Π°ΠΊΠΎΠ½, Ρ‚.Π΅ основан ΠΎΠ½ Π½Π° Ρ€Π΅Π·ΡƒΠ»ΡŒΡ‚Π°Ρ‚Π°Ρ… Ρ€Π΅Π°Π»ΡŒΠ½Ρ‹Ρ…, Π° Π½Π΅ тСорСтичСских экспСримСнтов. Π‘ΡƒΡ‚ΡŒ Π΅Π³ΠΎ Π·Π°ΠΊΠ»ΡŽΡ‡Π°Π΅Ρ‚ΡΡ Π² Ρ‚ΠΎΠΌ, Ρ‡Ρ‚ΠΎ Π»ΡŽΠ±Ρ‹Π΅ измСнСния ΠΊΠ°ΠΊΠΈΡ…-Π»ΠΈΠ±ΠΎ Π²Π΅Π»ΠΈΡ‡ΠΈΠ½ (яркости, громкости, вСса) ΠΎΡ‰ΡƒΡ‰Π°ΡŽΡ‚ΡΡ Π½Π°ΠΌΠΈ ΠΏΡ€ΠΈ условии, Ссли эти измСнСния носят логарифмичСский Ρ…Π°Ρ€Π°ΠΊΡ‚Π΅Ρ€.


Π“Ρ€Π°Ρ„ΠΈΠΊ зависимости ощущСния громкости ΠΎΡ‚ силы (мощности) Π·Π²ΡƒΠΊΠ°. Π—Π°ΠΊΠΎΠ½ Π’Π΅Π±Π΅Ρ€Π°-Π€Π΅Ρ…Π½Π΅Ρ€Π°

Π’Π°ΠΊ, Π½Π°ΠΏΡ€ΠΈΠΌΠ΅Ρ€, Ρ‡ΡƒΠ²ΡΡ‚Π²ΠΈΡ‚Π΅Π»ΡŒΠ½ΠΎΡΡ‚ΡŒ чСловСчСского ΡƒΡ…Π° ΡƒΠΌΠ΅Π½ΡŒΡˆΠ°Π΅Ρ‚ΡΡ с ростом уровня громкости Π·Π²ΡƒΠΊΠΎΠ²ΠΎΠ³ΠΎ сигнала. ИмСнно поэтому, ΠΏΡ€ΠΈ Π²Ρ‹Π±ΠΎΡ€Π΅ ΠΏΠ΅Ρ€Π΅ΠΌΠ΅Π½Π½ΠΎΠ³ΠΎ рСзистора, ΠΊΠΎΡ‚ΠΎΡ€Ρ‹ΠΉ планируСтся ΠΏΡ€ΠΈΠΌΠ΅Π½ΠΈΡ‚ΡŒ Π² рСгуляторС громкости Π·Π²ΡƒΠΊΠΎΠ²ΠΎΠ³ΠΎ усилитСля стоит Π±Ρ€Π°Ρ‚ΡŒ с ΠΏΠΎΠΊΠ°Π·Π°Ρ‚Π΅Π»ΡŒΠ½ΠΎΠΉ Π·Π°Π²ΠΈΡΠΈΠΌΠΎΡΡ‚ΡŒΡŽ сопротивлСния ΠΎΡ‚ ΡƒΠ³Π»Π° ΠΏΠΎΠ²ΠΎΡ€ΠΎΡ‚Π° Ρ€ΡƒΡ‡ΠΊΠΈ рСгулятора. Π’ этом случаС, ΠΏΡ€ΠΈ ΠΏΠΎΠ²ΠΎΡ€ΠΎΡ‚Π΅ Π΄Π²ΠΈΠΆΠΊΠ° рСгулятора громкости Π·Π²ΡƒΠΊ Π² Π΄ΠΈΠ½Π°ΠΌΠΈΠΊΠ΅ Π±ΡƒΠ΄Π΅Ρ‚ Π½Π°Ρ€Π°ΡΡ‚Π°Ρ‚ΡŒ ΠΏΠ»Π°Π²Π½ΠΎ. Π Π΅Π³ΡƒΠ»ΠΈΡ€ΠΎΠ²ΠΊΠ° громкости Π±ΡƒΠ΄Π΅Ρ‚ Π»ΠΈΠ½Π΅ΠΉΠ½ΠΎΠΉ, Ρ‚Π°ΠΊ ΠΊΠ°ΠΊ ΠΏΠΎΠΊΠ°Π·Π°Ρ‚Π΅Π»ΡŒΠ½Π°Ρ Π·Π°Π²ΠΈΡΠΈΠΌΠΎΡΡ‚ΡŒ рСгулятора громкости компСнсируСт Π»ΠΎΠ³Π°Ρ€ΠΈΡ„ΠΌΠΈΡ‡Π΅ΡΠΊΡƒΡŽ Π·Π°Π²ΠΈΡΠΈΠΌΠΎΡΡ‚ΡŒ нашСго слуха ΠΈ Π² суммС станСт Π»ΠΈΠ½Π΅ΠΉΠ½ΠΎΠΉ. ΠŸΡ€ΠΈ взглядС Π½Π° рисунок это станСт Π±ΠΎΠ»Π΅Π΅ понятно.


Π—Π°Π²ΠΈΡΠΈΠΌΠΎΡΡ‚ΡŒ сопротивлСния ΠΏΠ΅Ρ€Π΅ΠΌΠ΅Π½Π½ΠΎΠ³ΠΎ рСзистора ΠΎΡ‚ ΡƒΠ³Π»Π° ΠΏΠΎΠ²ΠΎΡ€ΠΎΡ‚Π° Π΄Π²ΠΈΠΆΠΊΠ° (А-линСйная, Π‘-логарифмичСская, Π’-ΠΏΠΎΠΊΠ°Π·Π°Ρ‚Π΅Π»ΡŒΠ½Π°Ρ)

Π—Π΄Π΅ΡΡŒ ΠΏΠΎΠΊΠ°Π·Π°Π½Ρ‹ Π³Ρ€Π°Ρ„ΠΈΠΊΠΈ зависимости сопротивлСния ΠΏΠ΅Ρ€Π΅ΠΌΠ΅Π½Π½Ρ‹Ρ… рСзисторов Ρ€Π°Π·Π½Ρ‹Ρ… Ρ‚ΠΈΠΏΠΎΠ²: А – линСйная, Π‘ – логарифмичСская, Π’ – ΠΏΠΎΠΊΠ°Π·Π°Ρ‚Π΅Π»ΡŒΠ½Π°Ρ. Как ΠΏΡ€Π°Π²ΠΈΠ»ΠΎ, Π½Π° ΠΏΠ΅Ρ€Π΅ΠΌΠ΅Π½Π½Ρ‹Ρ… рСзисторах отСчСствСнного производства указываСтся, ΠΊΠ°ΠΊΠΎΠΉ Π·Π°Π²ΠΈΡΠΈΠΌΠΎΡΡ‚ΡŒΡŽ ΠΎΠ±Π»Π°Π΄Π°Π΅Ρ‚ ΠΏΠ΅Ρ€Π΅ΠΌΠ΅Π½Π½Ρ‹ΠΉ рСзистор. На Ρ‚Π΅Ρ… ΠΆΠ΅ ΠΏΡ€ΠΈΠ½Ρ†ΠΈΠΏΠ°Ρ… основаны Ρ†ΠΈΡ„Ρ€ΠΎΠ²Ρ‹Π΅ ΠΈ элСктронныС рСгуляторы громкости.

Π’Π°ΠΊΠΆΠ΅ стоит ΠΎΡ‚ΠΌΠ΅Ρ‚ΠΈΡ‚ΡŒ, Ρ‡Ρ‚ΠΎ чСловСчСскоС ΡƒΡ…ΠΎ воспринимаСт Π·Π²ΡƒΠΊΠΈ, ΠΌΠΎΡ‰Π½ΠΎΡΡ‚ΡŒ ΠΊΠΎΡ‚ΠΎΡ€Ρ‹Ρ… различаСтся Π½Π° ΠΊΠΎΠ»ΠΎΡΡΠ°Π»ΡŒΠ½ΡƒΡŽ Π²Π΅Π»ΠΈΡ‡ΠΈΠ½Ρƒ Π² 10 000 000 000 000 Ρ€Π°Π·! Π’Π°ΠΊΠΈΠΌ ΠΎΠ±Ρ€Π°Π·ΠΎΠΌ, самый Π³Ρ€ΠΎΠΌΠΊΠΈΠΉ Π·Π²ΡƒΠΊ отличаСтся ΠΎΡ‚ самого Ρ‚ΠΈΡ…ΠΎΠ³ΠΎ, ΠΊΠΎΡ‚ΠΎΡ€Ρ‹ΠΉ ΠΌΠΎΠΆΠ΅Ρ‚ ΡƒΠ»ΠΎΠ²ΠΈΡ‚ΡŒ наш слух, Π½Π° 130 Π΄Π‘ (10 000 000 000 000 Ρ€Π°Π·).

Вторая ΠΏΡ€ΠΈΡ‡ΠΈΠ½Π° ΡˆΠΈΡ€ΠΎΠΊΠΎΠ³ΠΎ использования Π΄Π΅Ρ†ΠΈΠ±Π΅Π» являСтся простота вычислСний.

Π‘ΠΎΠ³Π»Π°ΡΠΈΡ‚Π΅ΡΡŒ, Ρ‡Ρ‚ΠΎ ΠΊΡƒΠ΄Π° ΠΏΡ€ΠΎΡ‰Π΅ ΠΏΡ€ΠΈ вычислСниях ΠΈΡΠΏΠΎΠ»ΡŒΠ·ΠΎΠ²Π°Ρ‚ΡŒ нСбольшиС числа Π²Ρ€ΠΎΠ΄Π΅ 10, 20, 60,80,100,130 (Π½Π°ΠΈΠ±ΠΎΠ»Π΅Π΅ часто ΠΈΡΠΏΠΎΠ»ΡŒΠ·ΡƒΠ΅ΠΌΡ‹Π΅ числа ΠΏΡ€ΠΈ расчётС Π² Π΄Π΅Ρ†ΠΈΠ±Π΅Π»Π°Ρ…) ΠΏΠΎ ΡΡ€Π°Π²Π½Π΅Π½ΠΈΡŽ с числами 100 (20 Π΄Π‘), 1000 (30 Π΄Π‘), 1000 000 (60 Π΄Π‘),100 000 000 (80 Π΄Π‘),10 000 000 000 (100 Π΄Π‘), 10 000 000 000 000 (130 Π΄Π‘). Π•Ρ‰Ρ‘ ΠΎΠ΄Π½ΠΈΠΌ достоинством Π΄Π΅Ρ†ΠΈΠ±Π΅Π» являСтся Ρ‚ΠΎ, Ρ‡Ρ‚ΠΎ ΠΈΡ… просто ΡΡƒΠΌΠΌΠΈΡ€ΡƒΡŽΡ‚. Если ΠΏΡ€ΠΎΠ²ΠΎΠ΄ΠΈΡ‚ΡŒ вычислСния Π² Ρ€Π°Π·Π°Ρ…, Ρ‚ΠΎ числа Π½Π΅ΠΎΠ±Ρ…ΠΎΠ΄ΠΈΠΌΠΎ ΡƒΠΌΠ½ΠΎΠΆΠ°Ρ‚ΡŒ.

НапримСр, 30 Π΄Π‘ + 30 Π΄Π‘ = 60 Π΄Π‘ (Π² Ρ€Π°Π·Π°Ρ…: 1000 * 1000 = 1000 000). Π”ΡƒΠΌΠ°ΡŽ, с этим всё ясно.

Π’Π°ΠΊΠΆΠ΅ Π΄Π΅Ρ†ΠΈΠ±Π΅Π»Ρ‹ ΠΎΡ‡Π΅Π½ΡŒ ΡƒΠ΄ΠΎΠ±Π½Ρ‹ ΠΏΡ€ΠΈ графичСском построСнии Ρ€Π°Π·Π»ΠΈΡ‡Π½Ρ‹Ρ… зависимостСй. ВсС Π³Ρ€Π°Ρ„ΠΈΠΊΠΈ Π²Ρ€ΠΎΠ΄Π΅ Π΄ΠΈΠ°Π³Ρ€Π°ΠΌΠΌ направлСнности Π°Π½Ρ‚Π΅Π½Π½, Π°ΠΌΠΏΠ»ΠΈΡ‚ΡƒΠ΄Π½ΠΎ-частотных характСристик усилитСлСй Π²Ρ‹ΠΏΠΎΠ»Π½ΡΡŽΡ‚ с ΠΏΡ€ΠΈΠΌΠ΅Π½Π΅Π½ΠΈΠ΅ΠΌ Π΄Π΅Ρ†ΠΈΠ±Π΅Π».

Π”Π΅Ρ†ΠΈΠ±Π΅Π» являСтся Π±Π΅Π·Ρ€Π°Π·ΠΌΠ΅Ρ€Π½ΠΎΠΉ Π΅Π΄ΠΈΠ½ΠΈΡ†Π΅ΠΉ измСрСния. ΠœΡ‹ ΡƒΠΆΠ΅ выяснили, Ρ‡Ρ‚ΠΎ Π΄Π΅Ρ†ΠΈΠ±Π΅Π» Π½Π° самом Π΄Π΅Π»Π΅ ΠΏΠΎΠΊΠ°Π·Ρ‹Π²Π°Π΅Ρ‚, Π²ΠΎ сколько Ρ€Π°Π· возросла, Π»ΠΈΠ±ΠΎ ΡƒΠΌΠ΅Π½ΡŒΡˆΠΈΠ»Π°ΡΡŒ какая-Π»ΠΈΠ±ΠΎ Π²Π΅Π»ΠΈΡ‡ΠΈΠ½Π° (Ρ‚ΠΎΠΊ, напряТСниС, ΠΌΠΎΡ‰Π½ΠΎΡΡ‚ΡŒ). ΠžΡ‚Π»ΠΈΡ‡ΠΈΠ΅ Π΄Π΅Ρ†ΠΈΠ±Π΅Π» ΠΎΡ‚ Ρ€Π°Π·ΠΎΠ² Π·Π°ΠΊΠ»ΡŽΡ‡Π°Π΅Ρ‚ΡΡ лишь Π² Ρ‚ΠΎΠΌ, Ρ‡Ρ‚ΠΎ происходит ΠΈΠ·ΠΌΠ΅Ρ€Π΅Π½ΠΈΠ΅ ΠΏΠΎ логарифмичСскому ΠΌΠ°ΡΡˆΡ‚Π°Π±Ρƒ. Π§Ρ‚ΠΎΠ±Ρ‹ это ΠΊΠ°ΠΊ-Ρ‚ΠΎ ΠΎΠ±ΠΎΠ·Π½Π°Ρ‡ΠΈΡ‚ΡŒ ΠΈ ΠΏΡ€ΠΈΠΏΠΈΡΡ‹Π²Π°ΡŽΡ‚ ΠΎΠ±ΠΎΠ·Π½Π°Ρ‡Π΅Π½ΠΈΠ΅ Π΄Π‘. Π’Π°ΠΊ ΠΈΠ»ΠΈ ΠΈΠ½Π°Ρ‡Π΅, ΠΏΡ€ΠΈ ΠΎΡ†Π΅Π½ΠΊΠ΅ приходится ΠΏΠ΅Ρ€Π΅Ρ…ΠΎΠ΄ΠΈΡ‚ΡŒ ΠΎΡ‚ Π΄Π΅Ρ†ΠΈΠ±Π΅Π» ΠΊ Ρ€Π°Π·Π°ΠΌ. Π‘Ρ€Π°Π²Π½ΠΈΠ²Π°Ρ‚ΡŒ с ΠΏΠΎΠΌΠΎΡ‰ΡŒΡŽ Π΄Π΅Ρ†ΠΈΠ±Π΅Π» ΠΌΠΎΠΆΠ½ΠΎ Π»ΡŽΠ±Ρ‹Π΅ Π΅Π΄ΠΈΠ½ΠΈΡ†Ρ‹ измСрСния (Π½Π΅ Ρ‚ΠΎΠ»ΡŒΠΊΠΎ Ρ‚ΠΎΠΊ, напряТСниС ΠΈ ΠΏΡ€ΠΎΡ‡.), Ρ‚Π°ΠΊ ΠΊΠ°ΠΊ Π΄Π΅Ρ†ΠΈΠ±Π΅Π» являСтся ΠΎΡ‚Π½ΠΎΡΠΈΡ‚Π΅Π»ΡŒΠ½ΠΎΠΉ, Π±Π΅Π·Ρ€Π°Π·ΠΌΠ΅Ρ€Π½ΠΎΠΉ Π²Π΅Π»ΠΈΡ‡ΠΈΠ½ΠΎΠΉ.

Если указываСтся Π·Π½Π°ΠΊ β€œ-”, Π½Π°ΠΏΡ€ΠΈΠΌΠ΅Ρ€, –1 Π΄Π‘, Ρ‚ΠΎ Π·Π½Π°Ρ‡Π΅Π½ΠΈΠ΅ измСряСмой Π²Π΅Π»ΠΈΡ‡ΠΈΠ½Ρ‹, Π½Π°ΠΏΡ€ΠΈΠΌΠ΅Ρ€, мощности, ΡƒΠΌΠ΅Π½ΡŒΡˆΠΈΠ»ΠΎΡΡŒ Π² 1,26 Ρ€Π°Π·. Если ΠΏΠ΅Ρ€Π΅Π΄ Π΄Π΅Ρ†ΠΈΠ±Π΅Π»Π°ΠΌΠΈ Π½Π΅ ставят Π½ΠΈΠΊΠ°ΠΊΠΎΠ³ΠΎ Π·Π½Π°ΠΊΠ°, Ρ‚ΠΎ Ρ€Π΅Ρ‡ΡŒ ΠΈΠ΄Ρ‘Ρ‚ ΠΎΠ± ΡƒΠ²Π΅Π»ΠΈΡ‡Π΅Π½ΠΈΠΈ, ростС Π²Π΅Π»ΠΈΡ‡ΠΈΠ½Ρ‹. Π­Ρ‚ΠΎ стоит ΡƒΡ‡ΠΈΡ‚Ρ‹Π²Π°Ρ‚ΡŒ. Иногда вмСсто Π·Π½Π°ΠΊΠ° β€œ-” говорят ΠΎ затуханиях, сниТСнии коэффициСнта усилСния.

ΠŸΠ΅Ρ€Π΅Ρ…ΠΎΠ΄ ΠΎΡ‚ Π΄Π΅Ρ†ΠΈΠ±Π΅Π» ΠΊ Ρ€Π°Π·Π°ΠΌ.

На ΠΏΡ€Π°ΠΊΡ‚ΠΈΠΊΠ΅ Ρ‡Π°Ρ‰Π΅ всСго приходится ΠΏΠ΅Ρ€Π΅Ρ…ΠΎΠ΄ΠΈΡ‚ΡŒ ΠΎΡ‚ Π΄Π΅Ρ†ΠΈΠ±Π΅Π» ΠΊ Ρ€Π°Π·Π°ΠΌ. Для этого Π΅ΡΡ‚ΡŒ простая Ρ„ΠΎΡ€ΠΌΡƒΠ»Π°:

Π’Π½ΠΈΠΌΠ°Π½ΠΈΠ΅! Π”Π°Π½Π½Ρ‹Π΅ Ρ„ΠΎΡ€ΠΌΡƒΠ»Ρ‹ ΠΏΡ€ΠΈΠΌΠ΅Π½ΡΡŽΡ‚ΡΡ для Ρ‚Π°ΠΊ Π½Π°Π·Ρ‹Π²Π°Π΅ΠΌΡ‹Ρ… β€œΡΠ½Π΅Ρ€Π³Π΅Ρ‚ΠΈΡ‡Π΅ΡΠΊΠΈΡ…β€ Π²Π΅Π»ΠΈΡ‡ΠΈΠ½. Π’Π°ΠΊΠΈΡ… ΠΊΠ°ΠΊ энСргия ΠΈ ΠΌΠΎΡ‰Π½ΠΎΡΡ‚ΡŒ.

m = 10 (n / 10) ,Π³Π΄Π΅ m – ΠΎΡ‚Π½ΠΎΡˆΠ΅Π½ΠΈΠ΅ Π² Ρ€Π°Π·Π°Ρ…, n – ΠΎΡ‚Π½ΠΎΡˆΠ΅Π½ΠΈΠ΅ Π² Π΄Π΅Ρ†ΠΈΠ±Π΅Π»Π°Ρ….

НапримСр, 1Π΄Π‘ Ρ€Π°Π²Π΅Π½ 10 (1Π΄Π‘ / 10) = 1,258925…= 1,26 Ρ€Π°Π·Π°.

ΠΏΡ€ΠΈ 20 Π΄Π‘: 10 (20Π΄Π‘ / 10) = 100 (ΡƒΠ²Π΅Π»ΠΈΡ‡Π΅Π½ΠΈΠ΅ Π²Π΅Π»ΠΈΡ‡ΠΈΠ½Ρ‹ Π² 100 Ρ€Π°Π·)

ΠΏΡ€ΠΈ 10 Π΄Π‘: 10 (10Π΄Π‘ / 10) = 10 (ΡƒΠ²Π΅Π»ΠΈΡ‡Π΅Π½ΠΈΠ΅ Π² 10 Ρ€Π°Π·)

Но, Π½Π΅ всё Ρ‚Π°ΠΊ просто. Π•ΡΡ‚ΡŒ ΠΈ ΠΏΠΎΠ΄Π²ΠΎΠ΄Π½Ρ‹Π΅ ΠΊΠ°ΠΌΠ½ΠΈ. НапримСр, Π·Π°Ρ‚ΡƒΡ…Π°Π½ΠΈΠ΅ сигнала составляСт -10 Π΄Π‘. Π’ΠΎΠ³Π΄Π°:

ΠΏΡ€ΠΈ -10 Π΄Π‘: 10 (-10Π΄Π‘ / 10) = 0,1

Если ΠΌΠΎΡ‰Π½ΠΎΡΡ‚ΡŒ с 5 Π’Ρ‚ ΡƒΠΌΠ΅Π½ΡŒΡˆΠΈΠ»Π°ΡΡŒ Π΄ΠΎ 0,5 Π’Ρ‚, Ρ‚ΠΎ сниТСниС мощности Ρ€Π°Π²Π½ΠΎ -10 Π΄Π‘ (ΡƒΠΌΠ΅Π½ΡŒΡˆΠ΅Π½ΠΈΡŽ Π² 10 Ρ€Π°Π·).

ΠΏΡ€ΠΈ -20 Π΄Π‘: 10 (-20Π΄Π‘ / 10) = 0,01

Π—Π΄Π΅ΡΡŒ Π°Π½Π°Π»ΠΎΠ³ΠΈΡ‡Π½ΠΎ. ΠŸΡ€ΠΈ сниТСнии мощности с 5 Π’Ρ‚ Π΄ΠΎ 0,05 Π’Ρ‚, Π² Π΄Π΅Ρ†ΠΈΠ±Π΅Π»Π°Ρ… ΠΏΠ°Π΄Π΅Π½ΠΈΠ΅ мощности составит -20 Π΄Π‘ (ΡƒΠΌΠ΅Π½ΡŒΡˆΠ΅Π½ΠΈΡŽ Π² 100 Ρ€Π°Π·).

Π’Π°ΠΊΠΈΠΌ ΠΎΠ±Ρ€Π°Π·ΠΎΠΌ, ΠΏΡ€ΠΈ -10 Π΄Π‘ ΠΌΠΎΡ‰Π½ΠΎΡΡ‚ΡŒ сигнала ΡƒΠΌΠ΅Π½ΡŒΡˆΠΈΠ»Π°ΡΡŒ Π² 10 Ρ€Π°Π·! ΠŸΡ€ΠΈ этом Ссли ΠΌΡ‹ ΠΏΠ΅Ρ€Π΅ΠΌΠ½ΠΎΠΆΠΈΠΌ Π½Π°Ρ‡Π°Π»ΡŒΠ½ΡƒΡŽ Π²Π΅Π»ΠΈΡ‡ΠΈΠ½Ρƒ сигнала Π½Π° 0,1 ,Ρ‚ΠΎ ΠΈ ΠΏΠΎΠ»ΡƒΡ‡ΠΈΠΌ Π·Π½Π°Ρ‡Π΅Π½ΠΈΠ΅ мощности сигнала ΠΏΡ€ΠΈ Π·Π°Ρ‚ΡƒΡ…Π°Π½ΠΈΠΈ Π² -10 Π΄Π‘. ИмСнно поэтому Π·Π½Π°Ρ‡Π΅Π½ΠΈΠ΅ 0,1 ΠΈ ΡƒΠΊΠ°Π·Π°Π½ΠΎ Π±Π΅Π· Β«Ρ€Π°Π·ΠΎΠ²Β», ΠΊΠ°ΠΊ Π² ΠΏΡ€Π΅Π΄Ρ‹Π΄ΡƒΡ‰ΠΈΡ… ΠΏΡ€ΠΈΠΌΠ΅Ρ€Π°Ρ…. Π£Ρ‡ΠΈΡ‚Ρ‹Π²Π°ΠΉΡ‚Π΅ эту ΠΎΡΠΎΠ±Π΅Π½Π½ΠΎΡΡ‚ΡŒ ΠΏΡ€ΠΈ подстановкС Π² Π΄Π°Π½Π½Ρ‹Π΅ Ρ„ΠΎΡ€ΠΌΡƒΠ»Ρ‹ Π·Π½Π°Ρ‡Π΅Π½ΠΈΠΉ Π΄Π΅Ρ†ΠΈΠ±Π΅Π» со Π·Π½Π°ΠΊΠΎΠΌ Β«-Β«.

ΠŸΠ΅Ρ€Π΅Ρ…ΠΎΠ΄ ΠΎΡ‚ Ρ€Π°Π·ΠΎΠ² ΠΊ Π΄Π΅Ρ†ΠΈΠ±Π΅Π»Π°ΠΌ ΠΌΠΎΠΆΠ½ΠΎ ΠΎΡΡƒΡ‰Π΅ΡΡ‚Π²ΠΈΡ‚ΡŒ ΠΏΠΎ ΡΠ»Π΅Π΄ΡƒΡŽΡ‰Π΅ΠΉ Ρ„ΠΎΡ€ΠΌΡƒΠ»Π΅:

n = 10 * log10(m) ,Π³Π΄Π΅ n – Π·Π½Π°Ρ‡Π΅Π½ΠΈΠ΅ Π² Π΄Π΅Ρ†ΠΈΠ±Π΅Π»Π°Ρ…, m – ΠΎΡ‚Π½ΠΎΡˆΠ΅Π½ΠΈΠ΅ Π² Ρ€Π°Π·Π°Ρ….

НапримСр, рост мощности Π² 4 Ρ€Π°Π·Π° Π±ΡƒΠ΄Π΅Ρ‚ ΡΠΎΠΎΡ‚Π²Π΅Ρ‚ΡΡ‚Π²ΠΎΠ²Π°Ρ‚ΡŒ Π·Π½Π°Ρ‡Π΅Π½ΠΈΡŽ Π² 6,021 Π΄Π‘.

10 * log10(4) = 6,021 Π΄Π‘.

Π’Π½ΠΈΠΌΠ°Π½ΠΈΠ΅! Для пСрСсчёта ΠΎΡ‚Π½ΠΎΡˆΠ΅Π½ΠΈΠΉ Ρ‚Π°ΠΊΠΈΡ… Π²Π΅Π»ΠΈΡ‡ΠΈΠ½ ΠΊΠ°ΠΊ напряТСниС ΠΈ сила Ρ‚ΠΎΠΊΠ° ΡΡƒΡ‰Π΅ΡΡ‚Π²ΡƒΡŽΡ‚ Π½Π΅ΠΌΠ½ΠΎΠ³ΠΎ ΠΈΠ½Ρ‹Π΅ Ρ„ΠΎΡ€ΠΌΡƒΠ»Ρ‹:

(Π‘ΠΈΠ»Π° Ρ‚ΠΎΠΊΠ° ΠΈ напряТСниС, это Ρ‚Π°ΠΊ Π½Π°Π·Ρ‹Π²Π°Π΅ΠΌΡ‹Π΅ β€œΡΠΈΠ»ΠΎΠ²Ρ‹Π΅β€ Π²Π΅Π»ΠΈΡ‡ΠΈΠ½Ρ‹. ΠŸΠΎΡΡ‚ΠΎΠΌΡƒ ΠΈ Ρ„ΠΎΡ€ΠΌΡƒΠ»Ρ‹ ΠΎΡ‚Π»ΠΈΡ‡Π°ΡŽΡ‚ΡΡ.)

Для ΠΏΠ΅Ρ€Π΅Ρ…ΠΎΠ΄Π° ΠΊ Π΄Π΅Ρ†ΠΈΠ±Π΅Π»Π°ΠΌ: n = 20 * log10(m)

Для ΠΏΠ΅Ρ€Π΅Ρ…ΠΎΠ΄Π° ΠΎΡ‚ Π΄Π΅Ρ†ΠΈΠ±Π΅Π» ΠΊ Ρ€Π°Π·Π°ΠΌ: m = 10 (n / 20)

n – Π·Π½Π°Ρ‡Π΅Π½ΠΈΠ΅ Π² Π΄Π΅Ρ†ΠΈΠ±Π΅Π»Π°Ρ…, m – ΠΎΡ‚Π½ΠΎΡˆΠ΅Π½ΠΈΠ΅ Π² Ρ€Π°Π·Π°Ρ….

Если Π’Ρ‹ ΡƒΡΠΏΠ΅ΡˆΠ½ΠΎ дошли Π΄ΠΎ этих строк, Ρ‚ΠΎ считайтС, Ρ‡Ρ‚ΠΎ сдСлали Π΅Ρ‰Ρ‘ ΠΎΠ΄ΠΈΠ½ вСсомый шаг Π² освоСнии элСктроники!

ΠŸΠ΅Ρ€Π΅Π²ΠΎΠ΄ Π²Π΅Π»ΠΈΡ‡ΠΈΠ½ ΠΈΠ· Π΄Π΅Ρ†ΠΈΠ±Π΅Π»ΠΎΠ² Π² Π°Π±ΡΠΎΠ»ΡŽΡ‚Π½Ρ‹Π΅ значСния ΠΈ ΠΌΠΎΡ‰Π½ΠΎΡΡ‚ΡŒ

ΠŸΡ€ΠΈ ΠΏΡ€ΠΎΠ²Π΅Π΄Π΅Π½ΠΈΠΈ ΠΈΠ·ΠΌΠ΅Ρ€Π΅Π½ΠΈΠΉ ΠΏΠ°Ρ€Π°ΠΌΠ΅Ρ‚Ρ€ΠΎΠ² Ρ€Π°Π΄ΠΈΠΎΠ°ΠΏΠΏΠ°Ρ€Π°Ρ‚ΡƒΡ€Ρ‹ довольно часто приходится ΠΈΠΌΠ΅Ρ‚ΡŒ Π΄Π΅Π»ΠΎ с ΠΎΡ‚Π½ΠΎΡΠΈΡ‚Π΅Π»ΡŒΠ½Ρ‹ΠΌΠΈ Π²Π΅Π»ΠΈΡ‡ΠΈΠ½Π°ΠΌΠΈ Π²Ρ‹Ρ€Π°ΠΆΠ΅Π½Π½Ρ‹ΠΌΠΈ Π² Π΄Π΅Ρ†ΠΈΠ±Π΅Π»Π°Ρ… [Π΄Π‘]. Π’ Π΄Π΅Ρ†ΠΈΠ±Π΅Π»Π°Ρ… Π²Ρ‹Ρ€Π°ΠΆΠ°ΡŽΡ‚ ΠΈΠ½Ρ‚Π΅Π½ΡΠΈΠ²Π½ΠΎΡΡ‚ΡŒ Π·Π²ΡƒΠΊΠ°, усилСниС каскада ΠΏΠΎ Π½Π°ΠΏΡ€ΡΠΆΠ΅Π½ΠΈΡŽ, Ρ‚ΠΎΠΊΡƒ ΠΈΠ»ΠΈ мощности, ΠΏΠΎΡ‚Π΅Ρ€ΠΈ ΠΏΠ΅Ρ€Π΅Π΄Π°Ρ‡ΠΈ ΠΈΠ»ΠΈ ослаблСниС сигнала, ΠΈ Ρ‚.Π΄.

Π”Π΅Ρ†ΠΈΠ±Π΅Π» β€” это ΡƒΠ½ΠΈΠ²Π΅Ρ€ΡΠ°Π»ΡŒΠ½Π°Ρ логарифмичСская Π΅Π΄ΠΈΠ½ΠΈΡ†Π°. Π¨ΠΈΡ€ΠΎΠΊΠΎΠ΅ использованиС прСдставлСния Π²Π΅Π»ΠΈΡ‡ΠΈΠ½ Π² Π΄Π‘ связано с удобством логарифмичСского ΠΌΠ°ΡΡˆΡ‚Π°Π±Π°, Π° ΠΏΡ€ΠΈ расчСтах Π΄Π΅Ρ†ΠΈΠ±Π΅Π»Ρ‹ ΠΏΠΎΠ΄Ρ‡ΠΈΠ½ΡΡŽΡ‚ΡΡ Π·Π°ΠΊΠΎΠ½Π°ΠΌ Π°Ρ€ΠΈΡ„ΠΌΠ΅Ρ‚ΠΈΠΊΠΈ β€” ΠΈΡ… ΠΌΠΎΠΆΠ½ΠΎ ΡΠΊΠ»Π°Π΄Ρ‹Π²Π°Ρ‚ΡŒ ΠΈ Π²Ρ‹Ρ‡ΠΈΡ‚Π°Ρ‚ΡŒ, Ссли сигналы ΠΈΠΌΠ΅ΡŽΡ‚ ΠΎΠ΄ΠΈΠ½Π°ΠΊΠΎΠ²ΡƒΡŽ Ρ„ΠΎΡ€ΠΌΡƒ.

БущСствуСт Ρ„ΠΎΡ€ΠΌΡƒΠ»Π° для пСрСсчСта ΠΎΡ‚Π½ΠΎΡˆΠ΅Π½ΠΈΡ Π΄Π²ΡƒΡ… напряТСний Π² число Π΄Π΅Ρ†ΠΈΠ±Π΅Π»ΠΎΠ² (аналогичная Ρ„ΠΎΡ€ΠΌΡƒΠ»Π° справСдлива ΠΈ для Ρ‚ΠΎΠΊΠΎΠ²):

НапримСр, Ссли Π²Ρ‹Ρ…ΠΎΠ΄Π½ΠΎΠΉ сигнал U2 ΠΈΠΌΠ΅Π΅Ρ‚ ΡƒΡ€ΠΎΠ²Π΅Π½ΡŒ Π²Π΄Π²ΠΎΠ΅ большС, Ρ‡Π΅ΠΌ U1, Ρ‚ΠΎ это ΠΎΡ‚Π½ΠΎΡˆΠ΅Π½ΠΈΠ΅ составит +6 Π΄Π‘ (Ig2=0,301). Если U2>U1 Π² 10 Ρ€Π°Π·, Ρ‚ΠΎ ΠΎΡ‚Π½ΠΎΡˆΠ΅Π½ΠΈΠ΅ сигналов составляСт 20 Π΄Π‘ (Ig10=1). Если U1>U2, Ρ‚ΠΎ Π·Π½Π°ΠΊ Ρƒ ΠΎΡ‚Π½ΠΎΡˆΠ΅Π½ΠΈΡ мСняСтся Π½Π° минус 20 Π΄Π‘.

Π’Π°ΠΊ, Π½Π°ΠΏΡ€ΠΈΠΌΠ΅Ρ€, Ρƒ ΠΈΠ·ΠΌΠ΅Ρ€ΠΈΡ‚Π΅Π»ΡŒΠ½ΠΎΠ³ΠΎ Π³Π΅Π½Π΅Ρ€Π°Ρ‚ΠΎΡ€Π° Π°Ρ‚Ρ‚Π΅Π½ΡŽΠ°Ρ‚ΠΎΡ€ для ослаблСния Π²Ρ‹Ρ…ΠΎΠ΄Π½ΠΎΠ³ΠΎ сигнала ΠΌΠΎΠΆΠ΅Ρ‚ ΠΈΠΌΠ΅Ρ‚ΡŒ Π³Ρ€Π°Π΄ΡƒΠΈΡ€ΠΎΠ²ΠΊΡƒ Π² Π΄Π‘. Π’ этом случаС для ΠΏΠ΅Ρ€Π΅Π²ΠΎΠ΄Π° Π²Π΅Π»ΠΈΡ‡ΠΈΠ½Ρ‹ ΠΈΠ· Π΄Π΅Ρ†ΠΈΠ±Π΅Π»ΠΎΠ² Π² Π°Π±ΡΠΎΠ»ΡŽΡ‚Π½ΠΎΠ΅ Π·Π½Π°Ρ‡Π΅Π½ΠΈΠ΅ быстрСй Π±ΡƒΠ΄Π΅Ρ‚ ΠΏΠΎΠ»ΡƒΡ‡Π΅Π½ Ρ€Π΅Π·ΡƒΠ»ΡŒΡ‚Π°Ρ‚, Ссли Π²ΠΎΡΠΏΠΎΠ»ΡŒΠ·ΠΎΠ²Π°Ρ‚ΡŒΡΡ ΡƒΠΆΠ΅ посчитанной Ρ‚Π°Π±Π». 6; 1. Она ΠΈΠΌΠ΅Π΅Ρ‚ Π΄ΠΈΡΠΊΡ€Π΅Ρ‚Π½ΠΎΡΡ‚ΡŒ 1 Π΄Π‘ (Ρ‡Ρ‚ΠΎ Π²ΠΏΠΎΠ»Π½Π΅ достаточно Π² Π±ΠΎΠ»ΡŒΡˆΠΈΠ½ΡΡ‚Π²Π΅ случаСв) ΠΈ Π΄ΠΈΠ°ΠΏΠ°Π·ΠΎΠ½ Π·Π½Π°Ρ‡Π΅Π½ΠΈΠΉ 0…-119 Π΄Π‘.

Π’Π°Π±Π». 6.1 ΠΌΠΎΠΆΠ½ΠΎ ΠΈΡΠΏΠΎΠ»ΡŒΠ·ΠΎΠ²Π°Ρ‚ΡŒ для ΠΏΠ΅Ρ€Π΅Π²ΠΎΠ΄Π° Π΄Π΅Ρ†ΠΈΠ±Π΅Π»ΠΎΠ² ослаблСния Π°Ρ‚Ρ‚Π΅Π½ΡŽΠ°Ρ‚ΠΎΡ€Π° Π² ΡƒΡ€ΠΎΠ²Π΅Π½ΡŒ Π²Ρ‹Ρ…ΠΎΠ΄Π½ΠΎΠ³ΠΎ напряТСния. Для удобства использования Ρ‚Π°Π±Π»ΠΈΡ†Ρ‹ потрСбуСтся Π½Π° Π²Ρ‹Ρ…ΠΎΠ΄Π΅ Π³Π΅Π½Π΅Ρ€Π°Ρ‚ΠΎΡ€Π° ΡƒΡΡ‚Π°Π½ΠΎΠ²ΠΈΡ‚ΡŒ ΠΏΡ€ΠΈ отсутствии ослаблСния (0 Π΄Π‘ Π½Π° Π°Ρ‚Ρ‚Π΅Π½ΡŽΠ°Ρ‚ΠΎΡ€Π΅) ΡƒΡ€ΠΎΠ²Π΅Π½ΡŒ напряТСния 1 Π’ (Π΄Π΅ΠΉΡΡ‚Π²ΡƒΡŽΡ‰Π΅Π³ΠΎ ΠΈΠ»ΠΈ Π°ΠΌΠΏΠ»ΠΈΡ‚ΡƒΠ΄Π½ΠΎΠ³ΠΎ). Π’ этом случаС ΡΠΎΠΎΡ‚Π²Π΅Ρ‚ΡΡ‚Π²ΡƒΡŽΡ‰Π΅Π΅ Π½ΡƒΠΆΠ½ΠΎΠ΅ Π·Π½Π°Ρ‡Π΅Π½ΠΈΠ΅ Π²Ρ‹Ρ…ΠΎΠ΄Π½ΠΎΠ³ΠΎ напряТСния послС установки ослаблСния находится Π½Π° пСрСсСчСнии Π³ΠΎΡ€ΠΈΠ·ΠΎΠ½Ρ‚Π°Π»ΡŒΠ½ΠΎΠΉ ΠΈ Π²Π΅Ρ€Ρ‚ΠΈΠΊΠ°Π»ΡŒΠ½ΠΎΠΉ Π³Ρ€Π°Ρ„ (значСния Π² Π΄Π΅Ρ†ΠΈΠ±Π΅Π»Π°Ρ… ΡΠΊΠ»Π°Π΄Ρ‹Π²Π°ΡŽΡ‚ΡΡ арифмСтичСски).

Π’Π΅Π»ΠΈΡ‡ΠΈΠ½Π° Π²Ρ‹Ρ…ΠΎΠ΄Π½ΠΎΠ³ΠΎ напряТСния Π² Ρ‚Π°Π±Π»ΠΈΡ†Π΅ ΡƒΠΊΠ°Π·Π°Π½Π° Π² ΠΌΠΈΠΊΡ€ΠΎΠ²ΠΎΠ»ΡŒΡ‚Π°Ρ… (1 ΠΌΠΊΠ’=10-6 Π’).Β I

Π’ΠΎΡΠΏΠΎΠ»ΡŒΠ·ΠΎΠ²Π°Π²ΡˆΠΈΡΡŒ Π΄Π°Π½Π½ΠΎΠΉ Ρ‚Π°Π±Π»ΠΈΡ†Π΅ΠΉ, Π½Π΅ Ρ‚Ρ€ΡƒΠ΄Π½ΠΎ Ρ€Π΅ΡˆΠΈΡ‚ΡŒ ΠΈ ΠΎΠ±Ρ€Π°Ρ‚Π½ΡƒΡŽ Π·Π°Π΄Π°Ρ‡Ρƒ β€” ΠΏΠΎ Π½Π΅ΠΎΠ±Ρ…ΠΎΠ΄ΠΈΠΌΠΎΠΌΡƒ Π½Π°ΠΏΡ€ΡΠΆΠ΅Π½ΠΈΡŽ ΠΎΠΏΡ€Π΅Π΄Π΅Π»ΠΈΡ‚ΡŒ, ΠΊΠ°ΠΊΠΎΠ΅ Π½ΡƒΠΆΠ½ΠΎ ΡƒΡΡ‚Π°Π½ΠΎΠ²ΠΈΡ‚ΡŒ ослаблСниС сигнала Π½Π° Π°Ρ‚Ρ‚Π΅Π½ΡŽΠ°Ρ‚ΠΎΡ€Π΅ Π² Π΄Π΅Ρ†ΠΈΠ±Π΅Π»Π°Ρ…. НапримСр, Ρ‡Ρ‚ΠΎΠ±Ρ‹ ΠΏΠΎΠ»ΡƒΡ‡ΠΈΡ‚ΡŒ Π½Π° Π²Ρ‹Ρ…ΠΎΠ΄Π΅ Π³Π΅Π½Π΅Ρ€Π°Ρ‚ΠΎΡ€Π° напряТСния 5 ΠΌΠΊΠ’, ΠΊΠ°ΠΊ Π²ΠΈΠ΄Π½ΠΎ ΠΈΠ· Ρ‚Π°Π±Π»ΠΈΡ†Ρ‹, Π½Π° Π°Ρ‚Ρ‚Π΅Π½ΡŽΠ°Ρ‚ΠΎΡ€Π΅ потрСбуСтся ΡƒΡΡ‚Π°Π½ΠΎΠ²ΠΈΡ‚ΡŒ ослаблСниС 100+6=106 Π΄Π‘. ΠžΡ‚Π½ΠΎΡˆΠ΅Π½ΠΈΠ΅ мощностСй Π΄Π²ΡƒΡ… сигналов Π² Π΄Π΅Ρ†ΠΈΠ±Π΅Π»Π°Ρ… вычисляСтся ΠΏΠΎ Ρ„ΠΎΡ€ΠΌΡƒΠ»Π΅:

Π€ΠΎΡ€ΠΌΡƒΠ»Π° для мощности справСдлива ΠΏΡ€ΠΈ условии, Ρ‡Ρ‚ΠΎ Π²Ρ…ΠΎΠ΄Π½ΠΎΠ΅ ΠΈ Π²Ρ‹Ρ…ΠΎΠ΄Π½ΠΎΠ΅ сопротивлСния устройства ΠΎΠ΄ΠΈΠ½Π°ΠΊΠΎΠ²Ρ‹Π΅, Ρ‡Ρ‚ΠΎ часто выполняСтся Π² высокочастотных устройствах для облСгчСния ΠΈΡ… согласования ΠΌΠ΅ΠΆΠ΄Ρƒ собой.

Для опрСдСлСния мощности ΠΌΠΎΠΆΠ½ΠΎ Π²ΠΎΡΠΏΠΎΠ»ΡŒΠ·ΠΎΠ²Π°Ρ‚ΡŒΡΡ посчитанной Ρ‚Π°Π±Π». 6.2

НСрСдко ΠΏΡ€ΠΈ практичСском использовании Π΄Π‘ Π²Π°ΠΆΠ½ΠΎ Π·Π½Π°Ρ‚ΡŒ ΠΈ Π°Π±ΡΠΎΠ»ΡŽΡ‚Π½ΠΎΠ΅ Π·Π½Π°Ρ‡Π΅Π½ΠΈΠ΅ ΡΠΎΠΎΡ‚Π½ΠΎΡˆΠ΅Π½ΠΈΡ Π΄Π²ΡƒΡ… Π²Π΅Π»ΠΈΡ‡ΠΈΠ½, Ρ‚.Π΅. Π²ΠΎ сколько Ρ€Π°Π· напряТСниС ΠΈΠ»ΠΈ ΠΌΠΎΡ‰Π½ΠΎΡΡ‚ΡŒ Π½Π° Π²Ρ‹Ρ…ΠΎΠ΄Π΅ большС, Ρ‡Π΅ΠΌ Π½Π° Π²Ρ…ΠΎΠ΄Π΅ (ΠΈΠ»ΠΈ Π½Π°ΠΎΠ±ΠΎΡ€ΠΎΡ‚). Если ΠΎΡ‚Π½ΠΎΡˆΠ΅Π½ΠΈΠ΅ Π΄Π²ΡƒΡ… Π²Π΅Π»ΠΈΡ‡ΠΈΠ½ ΠΎΠ±ΠΎΠ·Π½Π°Ρ‡ΠΈΡ‚ΡŒ: K=U2/U1 ΠΈΠ»ΠΈ К=Π 2/Π 1, Ρ‚ΠΎ ΠΌΠΎΠΆΠ½ΠΎ Π²ΠΎΡΠΏΠΎΠ»ΡŒΠ·ΠΎΠ²Π°Ρ‚ΡŒΡΡ Ρ‚Π°Π±Π». 6.3 для ΠΏΠ΅Ρ€Π΅Π²ΠΎΠ΄Π° Π²Π΅Π»ΠΈΡ‡ΠΈΠ½Ρ‹ ΠΈΠ· Π΄Π‘ Π² Ρ€Π°Π·Ρ‹ (К) ΠΈ Π½Π°ΠΎΠ±ΠΎΡ€ΠΎΡ‚.

Π’Π°ΠΊ, Π½Π°ΠΏΡ€ΠΈΠΌΠ΅Ρ€, Π°Π½Ρ‚Π΅Π½Π½Ρ‹ΠΉ ΡƒΡΠΈΠ»ΠΈΡ‚Π΅Π»ΡŒ обСспСчиваСт усилСниС сигнала ΠΏΠΎ мощности Π½Π° 28 Π΄Π‘. Из Ρ‚Π°Π±Π». 6.3 Π²ΠΈΠ΄Π½ΠΎ, Ρ‡Ρ‚ΠΎ усилСниС сигнала выполняСтся Π² 631 Ρ€Π°Π·.

Π›ΠΈΡ‚Π΅Ρ€Π°Ρ‚ΡƒΡ€Π°: И.П. ШСлСстов — Π Π°Π΄ΠΈΠΎΠ»ΡŽΠ±ΠΈΡ‚Π΅Π»ΡΠΌ ΠΏΠΎΠ»Π΅Π·Π½Ρ‹Π΅ схСмы, ΠΊΠ½ΠΈΠ³Π° 3.

Online ΠΊΠ°Π»ΡŒΠΊΡƒΠ»ΡΡ‚ΠΎΡ€ Ρ€Π°Π·Ρ‹ ΠΈ ΠΏΡ€ΠΎΡ†Π΅Π½Ρ‚Ρ‹ Π² Π΄Π΅Ρ†ΠΈΠ±Π΅Π»Ρ‹

  • Π£Ρ€ΠΎΠΊ Ρ‚Ρ€Π΅Ρ‚ΠΈΠΉ+. Учимся Ρ€Π°Π·Π±ΠΈΡ€Π°Ρ‚ΡŒ элСктричСскиС схСмы.

    Π’Π°ΠΊ ΠΊΠ°ΠΊ Ρ‚Π΅ΠΌΠ° довольно Ρ‚Π°ΠΊΠΈ ΠΎΠ±ΡˆΠΈΡ€Π½Π°Ρ ΠΈ тСорСтичСского ΠΌΠ°Ρ‚Π΅Ρ€ΠΈΠ°Π»Π° ΠΌΠ½ΠΎΠ³ΠΎ, Π΅Π³ΠΎ сокращСниС Π±ΡƒΠ΄Π΅Ρ‚ Π»Π΅ΠΆΠ°Ρ‚ΡŒ Π½Π° ΠΏΠ»Π΅Ρ‡Π°Ρ… учитСля, проводящСго занятия. НуТно ΡƒΡ‡ΠΈΡ‚Ρ‹Π²Π°Ρ‚ΡŒ Ρ‚Π°ΠΊ ΠΆΠ΅ ΡƒΡΠ²ΠΎΡΠ΅ΠΌΠΎΡΡ‚ΡŒ ΠΌΠ°Ρ‚Π΅Ρ€ΠΈΠ°Π»Π° ΠΈ Π·Π°Ρ‚ΡΠ³ΠΈΠ²Π°Ρ‚ΡŒ Π΄Π°Π½Π½ΡƒΡŽ Ρ‚Π΅ΠΌΡƒ сильно Π½Π΅ стоит. ΠœΠΎΠΆΠ΅Ρ‚ Π΄Π°ΠΆΠ΅ ΠΈΠΌΠ΅Π΅Ρ‚ ΡΠΌΡ‹ΡΠ»ΡŒ этот ΠΌΠ°Ρ‚Π΅Ρ€ΠΈΠ°Π» Ρ€Π°Π·Π±ΠΈΡ€Π°Ρ‚ΡŒ с ΠΏΠ΅Ρ€Π΅Ρ€Ρ‹Π²Π°ΠΌΠΈ Π½Π° Π΄Ρ€ΡƒΠ³ΠΈΠ΅ Ρ‚Π΅ΠΌΡ‹ Π½Ρƒ ΠΈΠ»ΠΈ Π½Π°ΠΏΡ€ΠΈΠΌΠ΅Ρ€ ΠΏΠΎ ΠΎΠΏΡ€Π΅Π΄Π΅Π»Π½Π½Ρ‹ΠΌ дням Π½Π΅Π΄Π΅Π»ΠΈ.

    Β 

    Π’ ΠΏΠΎΠ΄Π±ΠΎΡ€Π΅ ΠΌΠ°Ρ‚Π΅Ρ€ΠΈΠ°Π»Π° я Ρ€Π΅ΡˆΠΈΠ» Π½Π΅ «ΠΈΠ·ΠΎΠ±Ρ€Π΅Ρ‚Π°Ρ‚ΡŒ вСлосипСд» ΠΈ Π²ΠΎΡΠΏΠΎΠ»ΡŒΠ·ΠΎΠ²Π°Ρ‚ΡŒΡΡ Π³ΠΎΡ‚ΠΎΠ²Ρ‹ΠΌΠΈ ΠΌΠ°Ρ‚Π΅Ρ€ΠΈΠ°Π»Π°ΠΌΠΈ ΠΎΡ‚ сайта «ΠŸΡ€Π°ΠΊΡ‚ΠΈΡ‡Π΅ΡΠΊΠ°Ρ элСктроника».

    ΠŸΠΎΠ΄Ρ€ΠΎΠ±Π½Π΅Π΅… Β 
  • Как ΡΠΎΠ·Π΄Π°Π²Π°Ρ‚ΡŒ ΠΌΠ°Ρ‚Π΅Ρ€ΠΈΠ°Π»Ρ‹ Π² JCE Ρ€Π΅Π΄Π°ΠΊΡ‚ΠΎΡ€Π΅

    Π’ Π”ΠΆΡƒΠΌΠ»Π΅ ΠΌΠ°Ρ‚Π΅Ρ€ΠΈΠ°Π»Ρ‹ Π½Π΅ ΠΊΠ°ΠΊ Π² Π±Π»ΠΎΠ³Π΅, ΠΏΠΎ Ρ…Ρ€ΠΎΠ½ΠΎΠ»ΠΎΠ³ΠΈΠΈ, Π° ΠΏΠΈΡˆΡƒΡ‚ΡΡ Π² Π±Π°Π·Ρƒ Π΄Π°Π½Π½Ρ‹Ρ…, Ρ‡Ρ‚ΠΎ Π΄Π°Ρ‘Ρ‚ Π²ΠΎΠ·ΠΌΠΎΠΆΠ½ΠΎΡΡ‚ΡŒ Π³Ρ€ΡƒΠΏΠΏΠΈΡ€ΠΎΠ²Π°Ρ‚ΡŒ ΠΈΡ… ΠΏΠΎ особому, Π²Ρ‹Π²ΠΎΠ΄ΠΈΡ‚ΡŒ Π² Π½ΡƒΠΆΠ½ΠΎΠ΅ врСмя Π² Π½ΡƒΠΆΠ½ΠΎΠ΅ мСсто ΠΈ Ρ‚.Π΄.Β  Π’ΠΎ Π΅ΡΡ‚ΡŒ Π½Π°Π΄ΠΎ ΠΏΠΎΠΌΠΈΠΌΠΎ самого тСкста ΠΈ ΠΊΠ°Ρ€Ρ‚ΠΈΠ½ΠΎΠΊ ΡƒΠΊΠ°Π·Π°Ρ‚ΡŒ Π΄Ρ€ΡƒΠ³ΠΈΠ΅, слуТСбныС ΠΏΠ°Ρ€Π°ΠΌΠ΅Ρ‚Ρ€Ρ‹.Β  Π“Π»Π°Π²Π½Ρ‹Ρ… Π΄Π²Π° — это катСгория ΠΈ ΠΎΠΏΡƒΠ±Π»ΠΈΠΊΠΎΠ²Π°Ρ‚ΡŒ ΠΈΠ»ΠΈ ΠΏΡ€ΠΈΠ΄Π΅Ρ€ΠΆΠ°Ρ‚ΡŒ Π² Ρ€Π΅Π΄Π°ΠΊΡ‚ΠΎΡ€ Π½Π° самом Π΄Π΅Π»Π΅ Π² Π‘Π”).Β  Π˜Ρ‚Π°ΠΊ Π·Π°Ρ…ΠΎΠ΄ΠΈΠΌ Π½Π° hammania.net, Π² мСню Π²Ρ‹Π±ΠΈΡ€Π°Π΅ΠΌ Π‘Ρ‚Π°Ρ‚ΡŒΠΈΡΠΎΠ·Π΄Π°Ρ‚ΡŒ ΠΌΠ°Ρ‚Π΅Ρ€ΠΈΠ°Π».Β  ПопадаСм Π½Π° этот экран. Если Π½Π΅ ΠΏΠΎΠΏΠ°Π΄Π°Π΅ΠΌ Ρ‚ΠΎ ΠΈΠ»ΠΈ Π½Π΅ ΠΏΡ€ΠΎΡˆΠ»ΠΈ Π°Π²Ρ‚ΠΎΡ€ΠΈΠ·Π°Ρ†ΠΈΡŽ, ΠΈΠ»ΠΈ Π½Π΅ Π΄Π°Π»ΠΈ ΠΏΡ€Π°Π².

    ΠŸΠΎΠ΄Ρ€ΠΎΠ±Π½Π΅Π΅… Β 
  • Π£Ρ€ΠΎΠΊ Π²Ρ‚ΠΎΡ€ΠΎΠΉ. ЭлСктричСство — ΠΏΠΎΠ΄Ρ€ΠΎΠ±Π½Π΅Π΅

    Π’ нашС повсСднСвной ΠΆΠΈΠ·Π½ΠΈ ΠΌΡ‹ часто сталкиваСмся с Ρ‚Π°ΠΊΠΈΠΌ понятиСм ΠΊΠ°ΠΊ «элСктричСский Ρ‚ΠΎΠΊΒ». Π§Ρ‚ΠΎ ΠΆΠ΅ это Ρ‚Π°ΠΊΠΎΠ΅ ΠΈ всСгда Π»ΠΈ люди Π·Π½Π°Π»ΠΈ ΠΎ Π΅Π³ΠΎ сущСствовании?

    БСйчас Π±Π΅Π· элСктричСства ΠΏΡ€Π΅Π΄ΡΡ‚Π°Π²ΠΈΡ‚ΡŒ Π½Π°ΡˆΡƒ Тизнь Π½Π΅Π²ΠΎΠ·ΠΌΠΎΠΆΠ½ΠΎ. ЭлСктричСство Π½Π°ΡΡ‚ΠΎΠ»ΡŒΠΊΠΎ Π³Π»ΡƒΠ±ΠΎΠΊΠΎ ΠΏΡ€ΠΎΠ½ΠΈΠΊΠ»ΠΎ Π² Π½Π°ΡˆΡƒ ΠΎΠ±Ρ‹Π΄Π΅Π½Π½ΡƒΡŽ Тизнь, Ρ‡Ρ‚ΠΎ ΠΌΡ‹ ΠΏΠΎΡ€ΠΎΠΉ ΠΈ Π½Π΅ задумываСмся, Ρ‡Ρ‚ΠΎ это явлСниС ΠΏΠΎΠΌΠΎΠ³Π°Π΅Ρ‚ Π½Π°ΠΌ Π²ΠΎ всСх аспСктах нашСй ΠΆΠΈΠ·Π½ΠΈ.

    ΠŸΠΎΠ΄Ρ€ΠΎΠ±Π½ΠΎΠ΅ ΠΈΠ·ΡƒΡ‡Π΅Π½ΠΈΠ΅ элСктричСского Ρ‚ΠΎΠΊΠ° ΠΌΠΎΠΆΠ½ΠΎ отнСсти ΠΊ ΠΏΠ΅Ρ€ΠΈΠΎΠ΄Ρƒ ΠΊΠΎΠ½Ρ†Π° дСвятнадцатого Π²Π΅ΠΊΠ°, Π½ΠΎ ΠΏΠ΅Ρ€Π²Ρ‹Π΅ элСктричСскиС явлСния люди наблюдали Π΅Ρ‰Ρ‘ Π² пятом Π²Π΅ΠΊΠ΅ Π΄ΠΎ нашСй эры. Они Π·Π°ΠΌΠ΅Ρ‡Π°Π»ΠΈ, Ρ‡Ρ‚ΠΎ ΠΏΠΎΡ‚Ρ‘Ρ€Ρ‚Ρ‹ΠΉ ΠΌΠ΅Ρ…ΠΎΠΌ ΠΈΠ»ΠΈ ΡˆΠ΅Ρ€ΡΡ‚ΡŒΡŽ кусок янтаря притягиваСт ΠΊ сСбС Π»Ρ‘Π³ΠΊΠΈΠ΅ Ρ‚Π΅Π»Π°, Π½Π°ΠΏΡ€ΠΈΠΌΠ΅Ρ€, ΠΏΡ‹Π»ΠΈΠ½ΠΊΠΈ. Π”Ρ€Π΅Π²Π½ΠΈΠ΅ Π³Ρ€Π΅ΠΊΠΈ Π΄Π°ΠΆΠ΅ Π½Π°ΡƒΡ‡ΠΈΠ»ΠΈΡΡŒ ΠΈΡΠΏΠΎΠ»ΡŒΠ·ΠΎΠ²Π°Ρ‚ΡŒ это явлСниС – для удалСния ΠΏΡ‹Π»ΠΈ с Π΄ΠΎΡ€ΠΎΠ³ΠΈΡ… ΠΎΠ΄Π΅ΠΆΠ΄. Π•Ρ‰Ρ‘ ΠΎΠ½ΠΈ Π·Π°ΠΌΠ΅Ρ‚ΠΈΠ»ΠΈ, Ρ‡Ρ‚ΠΎ, Ссли сухиС волосы Ρ€Π°ΡΡ‡Π΅ΡΠ°Ρ‚ΡŒ янтарным Π³Ρ€Π΅Π±Π½Π΅ΠΌ, ΠΎΠ½ΠΈ ΠΏΠΎΠ΄Π½ΠΈΠΌΠ°ΡŽΡ‚ΡΡ, ΠΎΡ‚Ρ‚Π°Π»ΠΊΠΈΠ²Π°ΡΡΡŒ Π΄Ρ€ΡƒΠ³ ΠΎΡ‚ Π΄Ρ€ΡƒΠ³Π°.

    ΠŸΠΎΠ΄Ρ€ΠΎΠ±Π½Π΅Π΅… Β 
  • ΠŸΠ΅Ρ€Π΅Ρ‡Π΅Π½ΡŒ Π·Π½Π°Π½ΠΈΠΉ ΠΈ ΡƒΠΌΠ΅Π½ΠΈΠΉ Π½Π° ECC Report 089 CEPT (ENTRY LEVEL)

    ВСматичСский ΠΏΠ΅Ρ€Π΅Ρ‡Π΅Π½ΡŒ для экзамСнационных вопросов для присвоСния ΠΊΠ°Ρ‚Π΅Π³ΠΎΡ€ΠΈΠΈΒ 

    согласно полоТСниям Ρ€Π΅ΠΊΠΎΠΌΠ΅Π½Π΄Π°Ρ†ΠΈΠΈ ECC Report 089 CEPT (ENTRY LEVEL)

    Β 1. ΠŸΡ€Π°ΠΊΡ‚ΠΈΡ‡Π΅ΡΠΊΠΈΠ΅ Ρ€Π°Π±ΠΎΡ‡ΠΈΠ΅ аспСкты

    1.1. Знакомство с ΡƒΠΏΡ€Π°Π²Π»Π΅Π½ΠΈΠ΅ΠΌ ΠΏΠ΅Ρ€Π΅Π΄Π°Ρ‚Ρ‡ΠΈΠΊΠ° ΠΈΠ»ΠΈ трансивСра

    1.1.1. Π’ΠΊΠ»ΡŽΡ‡Π΅Π½ΠΈΠ΅/Π²Ρ‹ΠΊΠ»ΡŽΡ‡Π΅Π½ΠΈΠ΅ питания, ΠΏΠ΅Ρ€Π΅ΠΊΠ»ΡŽΡ‡Π°Ρ‚Π΅Π»ΡŒ Π΄ΠΈΠ°ΠΏΠ°Π·ΠΎΠ½ΠΎΠ², настройка ΠΈ индикация частоты, Π³Ρ€ΠΎΠΌΠΊΠΎΡΡ‚ΡŒ, ΡƒΡ€ΠΎΠ²Π΅Π½ΡŒ мощности ΠΈ дисплСй, усилСниС Π·Π²ΡƒΠΊΠΎΠ²ΠΎΠ³ΠΎ сигнала с ΠΌΠΈΠΊΡ€ΠΎΡ„ΠΎΠ½Π°.

    1.2. Π Π°Π±ΠΎΡ‚Π° Π½Π° ΠΊΠΎΡ€ΠΎΡ‚ΠΊΠΈΡ… Π²ΠΎΠ»Π½Π°Ρ…

    1.2.1. Настройка Π² Ρ€Π΅ΠΆΠΈΠΌΠ΅ SSB с Π²Π΅Ρ€Ρ…Π½Π΅ΠΉ ΠΈ Π½ΠΈΠΆΠ½Π΅ΠΉ Π±ΠΎΠΊΠΎΠ²ΠΎΠΉ полосой,Β 

    1.2.2. Π’Ρ‹Π·ΠΎΠ² коррСспондСнта, ΠΎΠ±Ρ‰ΠΈΠΉ Π²Ρ‹Π·ΠΎΠ²,

    1.2.3. Π‘ΠΏΠΎΡΠΎΠ±Π½ΠΎΡΡ‚ΡŒ провСдСния радиосвязи Π² ΠΏΡ€ΠΈΠ΅ΠΌΠ»Π΅ΠΌΠΎΠΌ Ρ„ΠΎΡ€ΠΌΠ°Ρ‚Π΅, Ρ€Π°ΠΏΠΎΡ€Ρ‚Π°, ΠΎΠ±ΠΌΠ΅Π½ ΠΈΠ½Ρ„ΠΎΡ€ΠΌΠ°Ρ†ΠΈΠΈ ΠΎΠ± ΠΈΠΌΠ΅Π½ΠΈ ΠΎΠΏΠ΅Ρ€Π°Ρ‚ΠΎΡ€Π°, ΠΈΠ½Ρ„ΠΎΡ€ΠΌΠ°Ρ†ΠΈΠΈ ΠΎ станции. ДСмонстрация использования Π°ΠΏΠΏΠ°Ρ€Π°Ρ‚ΡƒΡ€Ρ‹

    ΠŸΠΎΠ΄Ρ€ΠΎΠ±Π½Π΅Π΅… Β 
  • Как ΠΌΡ‹ Π±ΡƒΠ΄Π΅ΠΌ ΡƒΡ‡ΠΈΡ‚ΡŒ

    Π£Π²Π²Π°ΠΆΠ°Π΅ΠΌΡ‹Π΅ Ρ‡ΠΈΡ‚Π°Ρ‚Π΅Π»ΠΈ. ΠœΡ‹ ΠΎΡ‚ΠΊΡ€Ρ‹Π²Π°Π΅Ρ‚ Π½Π°ΡˆΡƒ Π²ΠΈΡ€Ρ‚ΡƒΠ°Π»ΡŒΠ½ΡƒΡŽ ΡˆΠΊΠΎΠ»Ρƒ ΠΏΡ€ΠΈ Π²ΠΈΡ€Ρ‚ΡƒΠ°Π»ΡŒΠ½ΠΎΠΉ ΠΊΠΎΠ»Π»Π΅ΠΊΡ‚ΠΈΠ²Π½ΠΎΠΉ радиостанции для Ρ‚ΠΎΠ³ΠΎ Ρ‡Ρ‚ΠΎΠ±Ρ‹ Π΄Π°Ρ‚ΡŒ Π²Π°ΠΌ Π²ΠΎΠ·ΠΌΠΎΠΆΠ½ΠΎΡΡ‚ΡŒ ΠΏΡ€ΠΈΠΎΠ±Ρ‰ΠΈΡ‚ΡŒΡΡ ΠΊ ΠΎΡ‡Π΅Π½ΡŒ интСрСсному Π·Π°Π½ΡΡ‚ΠΈΡŽ — Ρ€Π°Π΄ΠΈΠΎΠ»ΡŽΠ±ΠΈΡ‚Π΅Π»ΡŒΡΡ‚Π²Ρƒ. Наши ΡƒΡ€ΠΎΠΊΠΈ Π±ΡƒΠ΄ΡƒΡ‚ ΠΎΡ‡Π½Ρ‹ΠΌΠΈ, Π·Π°ΠΎΡ‡Π½Ρ‹ΠΌΠΈ ΠΈ ΠΊΠΎΠ½Ρ‚Ρ€ΠΎΠ»ΡŒΠ½Ρ‹ΠΌΠΈ. Β ΠœΠ°Ρ‚Π΅Ρ€ΠΈΠ°Π»Ρ‹ Π±ΡƒΠ΄ΡƒΡ‚ ΠΈΠ·Π»Π°Π³Π°Ρ‚ΡŒΡΡ ΠΊΠΎΡ€ΠΎΡ‚ΠΊΠΈΠΌΠΈ тСзисами, Π½Π΅ Π±ΠΎΠ»Π΅Π΅ 50-100 строк Π·Π° Ρ€Π°Π·, ΠΎΡ‡Π΅Π½ΡŒ простым языком. По Π²Π΅Ρ‡Π΅Ρ€Π°ΠΌ наши ΠΏΡ€Π΅ΠΏΠΎΠ΄Ρ‹ (сСнсСй Π“Π΅Π½Π°, сСнсСй Баша ΠΈ сСнсСй Π“ΠΎΡˆΠ°) часто Π±ΡƒΠ΄ΡƒΡ‚ доступны Π² ΠΎΠ½Π»Π°ΠΉΠ½, Π³Π΄Π΅ ΠΏΠΎΠΏΡ‹Ρ‚Π°ΡŽΡ‚ΡΡ ΠΎΡ‚Π²Π΅Ρ‚ΠΈΡ‚ΡŒ Π½Π° ваши вопросы. Π•Ρ‰Π΅ ΡƒΠ΄ΠΎΠ±Π½Π΅Π΅ Ρ„ΠΎΡ€ΠΌΠ° общСния Π² Ρ„ΠΎΡ€ΡƒΠΌΠ΅, ΠΏΠΎΡ‚ΠΎΠΌΡƒ Ρ‡Ρ‚ΠΎ снимаСт вопрос Π²Ρ€Π΅ΠΌΠ΅Π½ΠΈ : ΠΊΠΎΠ³Π΄Π° Π²Π°ΠΌ ΡƒΠ΄ΠΎΠ±Π½ΠΎ.

    Π£Ρ€ΠΎΠΊ ΠΏΠ΅Ρ€Π²Ρ‹ΠΉ. ЭлСктричСство.

    Начнём с простого. Π‘Π°Ρ‚Π°Ρ€Π΅ΠΉΠΊΠ°. Π­Ρ‚ΠΎ «Π·Π°ΠΊΠΎΠ½ΡΠ΅Ρ€Π²ΠΈΡ€ΠΎΠ²Π°Π½Π½ΠΎΠ΅» элСктричСство. Оно находится Π²Π½ΡƒΡ‚Ρ€ΠΈ ΠΈ ΠΏΠΎ ΠΊΠΎΠΌΠ°Π½Π΄Π΅ (Π·Π°ΠΌΡ‹ΠΊΠ°Π½ΠΈΡŽ Π²Ρ‹ΠΊΠ»ΡŽΡ‡Π°Ρ‚Π΅Π»Ρ) ΠΌΠΎΠΆΠ΅Ρ‚ Π΄Π΅Π»Π°Ρ‚ΡŒ ΠΊΠ°ΠΊΡƒΡŽ-Ρ‚ΠΎ Ρ€Π°Π±ΠΎΡ‚Ρƒ: ΡΠ²Π΅Ρ‚ΠΈΡ‚ΡŒ, Π²Ρ€Π°Ρ‰Π°Ρ‚ΡŒ ΠΌΠΎΡ‚ΠΎΡ€Ρ‡ΠΈΠΊ Ρ€ΡƒΡ‡Π½ΠΎΠ³ΠΎ вСнтилятора, ΠΊΠΎΠ³Π΄Π° ΠΆΠ°Ρ€ΠΊΠΎ, Β ΠΎΠ±Π΅ΡΠΏΠ΅Ρ‡ΠΈΠ²Π°Ρ‚ΡŒ вас Π·Π²ΡƒΠΊΠΎΠΌ ΠΎΡ‚ Ρ€Π°Π±ΠΎΡ‚Π°ΡŽΡ‰Π΅Π³ΠΎ Ρ€Π°Π΄ΠΈΠΎΠΏΡ€ΠΈΡ‘ΠΌΠ½ΠΈΠΊΠ° Π½Π° пляТС…. Β  Пока ΠΊΠΎΠ½Ρ‚Π°ΠΊΡ‚Ρ‹ Π½Π΅ Π·Π°ΠΌΠΊΠ½ΡƒΡ‚Ρ‹, элСктричСство Π΅ΡΡ‚ΡŒ, Π½ΠΎ Ρ€Π°Π±ΠΎΡ‚Ρƒ Π½Π΅ Π΄Π΅Π»Π°Π΅Ρ‚. Π‘ΠΏΠΈΡ‚. Β Π­Ρ‚ΠΎ называСтся напряТСниС. Или ΠΏΠΎΡ‚Π΅Π½Ρ†ΠΈΠ°Π». Π’ΠΈΠΏΠ° ΠΌΠΎΠΆΠ΅Ρ‚ Π΄Π΅Π»Π°Ρ‚ΡŒ, Π½ΠΎ ΠΏΠΎΠΊΠ° Π½Π΅ Π΄Π΅Π»Π°Π΅Ρ‚. Β  НапряТСниС всСгда ΠΏΠΎΠ΄Π°ΡŽΡ‚ ΠΏΠΎ Π”Π’Π£Πœ ΠΏΡ€ΠΎΠ²ΠΎΠ΄Π°ΠΌ: плюс ΠΈ минус. Π’ΠΎΠΎΠ±Ρ‰Π΅-Ρ‚ΠΎ Π±Ρ‹Π²Π°Π΅Ρ‚ Π΅Ρ‰Π΅ ΠΈ ΠΏΠ΅Ρ€Π΅ΠΌΠ΅Π½Π½ΠΎΠ΅ напряТСниС, Π½ΠΎ ΠΎ Π½Ρ‘ΠΌ ΠΏΠΎΠ·ΠΆΠ΅.

    ΠŸΠΎΠ΄Ρ€ΠΎΠ±Π½Π΅Π΅…
  • Π§Ρ‚ΠΎ Ρ‚Π°ΠΊΠΎΠ΅ Π΄Π΅Ρ†ΠΈΠ±Π΅Π» | Π§Ρ‚ΠΎ ΠΈΠ·ΠΌΠ΅Ρ€ΡΡŽΡ‚ Π² Π΄Π΅Ρ†ΠΈΠ±Π΅Π»Π°Ρ…, Ρ„ΠΎΡ€ΠΌΡƒΠ»Ρ‹.

    ΠžΡ‡Π΅Π½ΡŒ часто Π½ΠΎΠ²ΠΈΡ‡ΠΊΠΈ сталкиваСтся с Ρ‚Π°ΠΊΠΈΠΌ понятиСм, ΠΊΠ°ΠΊ Π΄Π΅Ρ†ΠΈΠ±Π΅Π». МногиС ΠΈΠ· Π½ΠΈΡ… ΠΈΠ½Ρ‚ΡƒΠΈΡ‚ΠΈΠ²Π½ΠΎ Π΄ΠΎΠ³Π°Π΄Ρ‹Π²Π°ΡŽΡ‚ΡΡ, Ρ‡Ρ‚ΠΎ это Ρ‚Π°ΠΊΠΎΠ΅, Π½ΠΎ Ρƒ Π±ΠΎΠ»ΡŒΡˆΠΈΠ½ΡΡ‚Π²Π° Π΄ΠΎ сих ΠΏΠΎΡ€ Π²ΠΎΠ·Π½ΠΈΠΊΠ°ΡŽΡ‚ вопросы.

    Π§Ρ‚ΠΎ Ρ‚Π°ΠΊΠΎΠ΅ Π΄Π΅Ρ†ΠΈΠ±Π΅Π»?

    ΠžΡ‚Π½ΠΎΡΠΈΡ‚Π΅Π»ΡŒΠ½Ρ‹Π΅ логарифмичСскиС Π΅Π΄ΠΈΠ½ΠΈΡ†Ρ‹ Π‘Π΅Π»Ρ‹ (Π΄Π΅Ρ†ΠΈΠ±Π΅Π»Ρ‹) ΡˆΠΈΡ€ΠΎΠΊΠΎ ΠΈΡΠΏΠΎΠ»ΡŒΠ·ΡƒΡŽΡ‚ΡΡ ΠΏΡ€ΠΈ количСствСнных ΠΎΡ†Π΅Π½ΠΊΠ°Ρ… ΠΏΠ°Ρ€Π°ΠΌΠ΅Ρ‚Ρ€ΠΎΠ² Ρ€Π°Π·Π»ΠΈΡ‡Π½Ρ‹Ρ… Π°ΡƒΠ΄ΠΈΠΎ, Π²ΠΈΠ΄Π΅ΠΎ, ΠΈΠ·ΠΌΠ΅Ρ€ΠΈΡ‚Π΅Π»ΡŒΠ½Ρ‹Ρ… устройств. ЀизичСская ΠΏΡ€ΠΈΡ€ΠΎΠ΄Π° сравниваСмых мощностСй ΠΌΠΎΠΆΠ΅Ρ‚ Π±Ρ‹Ρ‚ΡŒ любой β€” элСктричСской, элСктромагнитной, акустичСской, мСханичСской, β€” Π²Π°ΠΆΠ½ΠΎ лишь, Ρ‡Ρ‚ΠΎΠ±Ρ‹ ΠΎΠ±Π΅ Π²Π΅Π»ΠΈΡ‡ΠΈΠ½Ρ‹ Π±Ρ‹Π»ΠΈ Π²Ρ‹Ρ€Π°ΠΆΠ΅Π½Ρ‹ Π² ΠΎΠ΄ΠΈΠ½Π°ΠΊΠΎΠ²Ρ‹Ρ… Π΅Π΄ΠΈΠ½ΠΈΡ†Π°Ρ… β€” Π²Π°Ρ‚Ρ‚Π°Ρ…, ΠΌΠΈΠ»Π»ΠΈΠ²Π°Ρ‚Ρ‚Π°Ρ… ΠΈ Ρ‚. ΠΏ. Π‘Π΅Π» Π²Ρ‹Ρ€Π°ΠΆΠ°Π΅Ρ‚ ΠΎΡ‚Π½ΠΎΡˆΠ΅Π½ΠΈΠ΅ Π΄Π²ΡƒΡ… Π·Π½Π°Ρ‡Π΅Π½ΠΈΠΉ энСргСтичСской Π²Π΅Π»ΠΈΡ‡ΠΈΠ½Ρ‹ дСсятичным Π»ΠΎΠ³Π°Ρ€ΠΈΡ„ΠΌΠΎΠΌ этого ΠΎΡ‚Π½ΠΎΡˆΠ΅Π½ΠΈΡ, ΠΏΡ€ΠΈΡ‡Π΅ΠΌ ΠΏΠΎΠ΄ энСргСтичСскими Π²Π΅Π»ΠΈΡ‡ΠΈΠ½Π°ΠΌΠΈ ΠΏΠΎΠ½ΠΈΠΌΠ°ΡŽΡ‚ΡΡ: ΠΌΠΎΡ‰Π½ΠΎΡΡ‚ΡŒ, энСргия.

    ΠšΡΡ‚Π°Ρ‚ΠΈ, эта Π΅Π΄ΠΈΠ½ΠΈΡ†Π° ΠΏΠΎΠ»ΡƒΡ‡ΠΈΠ»Π° своС Π½Π°Π·Π²Π°Π½ΠΈΠ΅ Π² Ρ‡Π΅ΡΡ‚ΡŒ АлСксандра Π‘Π΅Π»Π» (1847 – 1922) – амСриканского ΡƒΡ‡Π΅Π½ΠΎΠ³ΠΎ ΡˆΠΎΡ‚Π»Π°Π½Π΄ΡΠΊΠΎΠ³ΠΎ происхоТдСния, основополоТника Ρ‚Π΅Π»Π΅Ρ„ΠΎΠ½ΠΈΠΈ, основатСля всСмирно извСстных ΠΊΠΎΠΌΠΏΠ°Π½ΠΈΠΉ AT&T ΠΈ β€œBell Laboratories”. Π•Ρ‰Π΅ интСрСсно Π½Π°ΠΏΠΎΠΌΠ½ΠΈΡ‚ΡŒ, Ρ‡Ρ‚ΠΎ Π²ΠΎ ΠΌΠ½ΠΎΠ³ΠΈΡ… соврСмСнных ΠΌΠΎΠ±ΠΈΠ»ΡŒΠ½Ρ‹Ρ… Ρ‚Π΅Π»Π΅Ρ„ΠΎΠ½Π°Ρ… (смартфонах) ΠΎΠ±ΡΠ·Π°Ρ‚Π΅Π»ΡŒΠ½ΠΎ Π΅ΡΡ‚ΡŒ Π²Ρ‹Π±ΠΈΡ€Π°Π΅ΠΌΡ‹ΠΉ Π·Π²ΡƒΠΊ Π·Π²ΠΎΠ½ΠΊΠ° (оповСщСния), Ρ‚Π°ΠΊ ΠΈ Π½Π°Π·Ρ‹Π²Π°Π΅ΠΌΡ‹ΠΉ β€œbell”. Π’ΠΏΡ€ΠΎΡ‡Π΅ΠΌ, Π‘Π΅Π» относится ΠΊ Π΅Π΄ΠΈΠ½ΠΈΡ†Π°ΠΌ, Π½Π΅ входящим Π² ΠœΠ΅ΠΆΠ΄ΡƒΠ½Π°Ρ€ΠΎΠ΄Π½ΡƒΡŽ систСму Π΅Π΄ΠΈΠ½ΠΈΡ† (БИ), Π½ΠΎ Π² соотвСтствии с Ρ€Π΅ΡˆΠ΅Π½ΠΈΠ΅ΠΌ ΠœΠ΅ΠΆΠ΄ΡƒΠ½Π°Ρ€ΠΎΠ΄Π½ΠΎΠ³ΠΎ ΠΊΠΎΠΌΠΈΡ‚Π΅Ρ‚Π° ΠΌΠ΅Ρ€ ΠΈ вСсов допускаСтся ΠΊ ΠΏΡ€ΠΈΠΌΠ΅Π½Π΅Π½ΠΈΡŽ Π±Π΅Π· ΠΎΠ³Ρ€Π°Π½ΠΈΡ‡Π΅Π½ΠΈΠΉ совмСстно с Π΅Π΄ΠΈΠ½ΠΈΡ†Π°ΠΌΠΈ БИ. Π’ основном примСняСтся Π² элСктросвязи, акустикС, Ρ€Π°Π΄ΠΈΠΎΡ‚Π΅Ρ…Π½ΠΈΠΊΠ΅.

    Π€ΠΎΡ€ΠΌΡƒΠ»Ρ‹ для вычислСния Π΄Π΅Ρ†ΠΈΠ±Π΅Π»ΠΎΠ²

    Π‘Π΅Π» (Π‘) = lg (P2/P1)

    Π³Π΄Π΅

    P1 – ΠΌΠΎΡ‰Π½ΠΎΡΡ‚ΡŒ Π΄ΠΎ усилСния, Π’Ρ‚

    P2 – ΠΌΠΎΡ‰Π½ΠΎΡΡ‚ΡŒ послС усилСния ΠΈΠ»ΠΈ ослаблСния, Π’Ρ‚

    На ΠΏΡ€Π°ΠΊΡ‚ΠΈΠΊΠ΅, оказалось, Ρ‡Ρ‚ΠΎ ΡƒΠ΄ΠΎΠ±Π½Π΅Π΅ ΠΏΠΎΠ»ΡŒΠ·ΠΎΠ²Π°Ρ‚ΡŒΡΡ ΡƒΠΌΠ΅Π½ΡŒΡˆΠ΅Π½Π½Ρ‹ΠΌ Π² 10 Ρ€Π°Π· Π·Π½Π°Ρ‡Π΅Π½ΠΈΠ΅ΠΌ Π‘Π΅Π», Ρ‚.Π΅. Π΄Π΅Ρ†ΠΈΠ±Π΅Π», поэтому:

    Π΄Π΅Ρ†ΠΈΠ‘Π΅Π» (Π΄Π‘) = 10 * lg(P2/P1)

    УсилСниС ΠΈΠ»ΠΈ ослаблСниС мощности Π² Π΄Π΅Ρ†ΠΈΠ±Π΅Π»Π°Ρ… выраТаСтся Ρ„ΠΎΡ€ΠΌΡƒΠ»ΠΎΠΉ:

    Π³Π΄Π΅

    NΠ΄Π‘ – усилСниС, Π»ΠΈΠ±ΠΎ ослаблСниС мощности Π² Π΄Π΅Ρ†ΠΈΠ±Π΅Π»Π°Ρ…

    P1 – ΠΌΠΎΡ‰Π½ΠΎΡΡ‚ΡŒ Π΄ΠΎ усилСния, Π’Ρ‚

    P2 – ΠΌΠΎΡ‰Π½ΠΎΡΡ‚ΡŒ послС усилСния ΠΈΠ»ΠΈ ослаблСния, Π’Ρ‚

    ЗначСния Π‘Π΅Π», Π΄Π΅Ρ†ΠΈΠ±Π΅Π» ΠΌΠΎΠ³ΡƒΡ‚ Π±Ρ‹Ρ‚ΡŒ со Π·Π½Π°ΠΊΠΎΠΌ β€œΠΏΠ»ΡŽΡβ€, Ссли P2 > P1 (усилСниС сигнала)Β  ΠΈ со Π·Π½Π°ΠΊΠΎΠΌ β€œΠΌΠΈΠ½ΡƒΡβ€, Ссли P2 < P1 (ослаблСниС сигнала)

    Π’ΠΎ ΠΌΠ½ΠΎΠ³ΠΈΡ… случаях, сравнСниС сигналов ΠΏΡƒΡ‚Π΅ΠΌ измСрСния мощностСй ΠΌΠΎΠΆΠ΅Ρ‚ Π±Ρ‹Ρ‚ΡŒ Π½Π΅ΡƒΠ΄ΠΎΠ±Π½Ρ‹ΠΌ ΠΈΠ»ΠΈ Π½Π΅Π²ΠΎΠ·ΠΌΠΎΠΆΠ½Ρ‹ΠΌ – ΠΏΡ€ΠΎΡ‰Π΅ ΠΈΠ·ΠΌΠ΅Ρ€ΠΈΡ‚ΡŒ напряТСниС ΠΈΠ»ΠΈ Ρ‚ΠΎΠΊ.
    Π’ этом случаС, Ссли ΠΌΡ‹ сравниваСм напряТСния ΠΈΠ»ΠΈ Ρ‚ΠΎΠΊΠΈ, Ρ„ΠΎΡ€ΠΌΡƒΠ»Π° ΠΏΡ€ΠΈΠΌΠ΅Ρ‚ ΡƒΠΆΠ΅ Π΄Ρ€ΡƒΠ³ΠΎΠΉ Π²ΠΈΠ΄:

    Π³Π΄Π΅

    NΠ΄Π‘ – усилСниС, Π»ΠΈΠ±ΠΎ ослаблСниС мощности Π² Π΄Π΅Ρ†ΠΈΠ±Π΅Π»Π°Ρ…

    U1 – это напряТСниС Π΄ΠΎ усилСния, Π’

    U2Β  – напряТСниС послС усилСния, Π’

    I1 – сила Ρ‚ΠΎΠΊΠ° Π΄ΠΎ усилСния, А

    I2 – сила Ρ‚ΠΎΠΊΠ° послС усилСния, А

    Π’ΠΎΡ‚ нСбольшая Ρ‚Π°Π±Π»ΠΈΡ‡ΠΊΠ°, Π² ΠΊΠΎΡ‚ΠΎΡ€ΠΎΠΉ ΠΏΡ€ΠΈΠ²Π΅Π΄Π΅Π½Ρ‹ основныС ΠΎΡ‚Π½ΠΎΡˆΠ΅Π½ΠΈΡ напряТСний ΠΈ ΡΠΎΠΎΡ‚Π²Π΅Ρ‚ΡΡ‚Π²ΡƒΡŽΡ‰Π΅Π΅ число Π΄Π΅Ρ†ΠΈΠ±Π΅Π»:

    Π”Π΅Π»ΠΎ Π² Ρ‚ΠΎΠΌ, Ρ‡Ρ‚ΠΎ ΠΎΠΏΠ΅Ρ€Π°Ρ†ΠΈΠΈ умноТСния ΠΈ дСлСния Π½Π°Π΄ числами Π² ΠΎΠ±Ρ‹Ρ‡Π½ΠΎΠΌ базисС, Π·Π°ΠΌΠ΅Π½ΡΡŽΡ‚ΡΡ опСрациями слоТСния ΠΈ вычитания Π² логарифмичСском базисС. НапримСр, Ρƒ нас Π΅ΡΡ‚ΡŒ Π΄Π²Π° каскадно-Π²ΠΊΠ»ΡŽΡ‡Π΅Π½Π½Ρ‹Ρ… усилитСля с коэффициСнтами усилСния K1 = 963 ΠΈ K2 = 48. Какой ΠΎΠ±Ρ‰ΠΈΠΉ коэффициСнт усилСния? ΠŸΡ€Π°Π²ΠΈΠ»ΡŒΠ½ΠΎ – ΠΎΠ½ Ρ€Π°Π²Π΅Π½ ΠΏΡ€ΠΎΠΈΠ·Π²Π΅Π΄Π΅Π½ΠΈΡŽ K = K1 * K2. Π’Ρ‹ ΠΌΠΎΠΆΠ΅Ρ‚Π΅ Π² ΡƒΠΌΠ΅ быстро Π²Ρ‹Ρ‡ΠΈΡΠ»ΠΈΡ‚ΡŒ 963*48? Π― – Π½Π΅Ρ‚. Π― ΠΌΠΎΠ³Ρƒ ΠΏΡ€ΠΈΠΊΠΈΠ½ΡƒΡ‚ΡŒ K = 1000*50 = 50 тыс., Π½Π΅ Π±ΠΎΠ»Π΅Π΅. А, Ссли Π½Π°ΠΌ извСстно, Ρ‡Ρ‚ΠΎ K1 = 59 Π΄Π‘ ΠΈ K2 = 33 Π΄Π‘, Ρ‚ΠΎ К = 59+33 = 92 Π΄Π‘ – ΡΠ»ΠΎΠΆΠΈΡ‚ΡŒ Π±Ρ‹Π»ΠΎ Π½Π΅ Ρ‚Ρ€ΡƒΠ΄Π½ΠΎ, надСюсь.

    Π’ΠΏΡ€ΠΎΡ‡Π΅ΠΌ, Π°ΠΊΡ‚ΡƒΠ°Π»ΡŒΠ½ΠΎΡΡ‚ΡŒ Ρ‚Π°ΠΊΠΈΡ… вычислСний Π±Ρ‹Π»ΠΎ Π²Π΅Π»ΠΈΠΊΠ° Π² эпоху, ΠΊΠΎΠ³Π΄Π° Π²Π²Π΅Π»ΠΈ понятиС Π‘Π΅Π» ΠΈ ΠΊΠΎΠ³Π΄Π° Π½Π΅ Π±Ρ‹Π»ΠΎ Π½Π΅ Ρ‚ΠΎ, Ρ‡Ρ‚ΠΎ Π°ΠΉΡ„ΠΎΠ½ΠΎΠ², Π½ΠΎ ΠΈ элСктронных ΠΊΠ°Π»ΡŒΠΊΡƒΠ»ΡΡ‚ΠΎΡ€ΠΎΠ².Β  БСйчас ΠΆΠ΅ достаточно ΠΎΡ‚ΠΊΡ€Ρ‹Ρ‚ΡŒ ΠΊΠ°Π»ΡŒΠΊΡƒΠ»ΡΡ‚ΠΎΡ€ Π½Π° Π²Π°ΡˆΠΈΡ… Π³Π°Π΄ΠΆΠ΅Ρ‚Π°Ρ… ΠΈ Π±Ρ‹ΡΡ‚Ρ€Π΅Π½ΡŒΠΊΠΎ ΠΏΠΎΡΡ‡ΠΈΡ‚Π°Ρ‚ΡŒ , Ρ‡Ρ‚ΠΎ Π΅ΡΡ‚ΡŒ Ρ‡Ρ‚ΠΎ. Ну ΠΈ Ρ‡Ρ‚ΠΎΠ±Ρ‹ Π½Π΅ ΠΏΠ°Ρ€ΠΈΡ‚ΡŒΡΡ ΠΊΠ°ΠΆΠ΄Ρ‹ΠΉ Ρ€Π°Π· ΠΏΡ€ΠΈ ΠΏΠ΅Ρ€Π΅Π²ΠΎΠ΄Π΅ Π΄Π‘ Π² Ρ€Π°Π·Ρ‹, ΡƒΠ΄ΠΎΠ±Π½Π΅Π΅ всСго Π½Π°ΠΉΡ‚ΠΈ Π² ΠΈΠ½Ρ‚Π΅Ρ€Π½Π΅Ρ‚Π΅ ΠΎΠ½Π»Π°ΠΉΠ½-ΠΊΠ°Π»ΡŒΠΊΡƒΠ»ΡΡ‚ΠΎΡ€. Π”Π° хотя Π±Ρ‹ Π²ΠΎΡ‚.

    Π—Π°ΠΊΠΎΠ½ Π’Π΅Π±Π΅Ρ€Π°-Π€Π΅Ρ…Π½Π΅Ρ€Π°

    ΠŸΠΎΡ‡Π΅ΠΌΡƒ ΠΈΠΌΠ΅Π½Π½ΠΎ Π΄Π΅Ρ†ΠΈΠ±Π΅Π»Ρ‹? ВсС исходит ΠΎΡ‚ Π·Π°ΠΊΠΎΠ½Π° Π’Π΅Π±Π΅Ρ€Π°-Π€Π΅Ρ…Π½Π΅Ρ€Π°, ΠΊΠΎΡ‚ΠΎΡ€Ρ‹ΠΉ Π³ΠΎΠ²ΠΎΡ€ΠΈΡ‚ Π½Π°ΠΌ, Ρ‡Ρ‚ΠΎ ΠΈΠ½Ρ‚Π΅Π½ΡΠΈΠ²Π½ΠΎΡΡ‚ΡŒ ощущСния чСловСчСских чувств прямо-ΠΏΡ€ΠΎΠΏΠΎΡ€Ρ†ΠΈΠΎΠ½Π°Π»ΡŒΠ½Π° Π»ΠΎΠ³Π°Ρ€ΠΈΡ„ΠΌΡƒ интСнсивности ΠΊΠ°ΠΊΠΎΠ³ΠΎ-Π»ΠΈΠ±ΠΎ раздраТитСля.

    Π’Π°ΠΊ ΡΠ²Π΅Ρ‚ΠΈΠ»ΡŒΠ½ΠΈΠΊ, Π² ΠΊΠΎΡ‚ΠΎΡ€ΠΎΠΌ восСмь Π»Π°ΠΌΠΏΠΎΡ‡Π΅ΠΊ, каТСтся Π½Π°ΠΌ Π½Π°ΡΡ‚ΠΎΠ»ΡŒΠΊΠΎ ΠΆΠ΅ ярчС ΡΠ²Π΅Ρ‚ΠΈΠ»ΡŒΠ½ΠΈΠΊΠ° ΠΈΠ· Ρ‡Π΅Ρ‚Ρ‹Ρ€Ρ‘Ρ… Π»Π°ΠΌΠΏΠΎΡ‡Π΅ΠΊ, насколько ΡΠ²Π΅Ρ‚ΠΈΠ»ΡŒΠ½ΠΈΠΊ ΠΈΠ· Ρ‡Π΅Ρ‚Ρ‹Ρ€Ρ‘Ρ… Π»Π°ΠΌΠΏΠΎΡ‡Π΅ΠΊ ярчС ΡΠ²Π΅Ρ‚ΠΈΠ»ΡŒΠ½ΠΈΠΊΠ° ΠΈΠ· Π΄Π²ΡƒΡ… Π»Π°ΠΌΠΏΠΎΡ‡Π΅ΠΊ. Π’ΠΎ Π΅ΡΡ‚ΡŒ количСство Π»Π°ΠΌΠΏΠΎΡ‡Π΅ΠΊ Π΄ΠΎΠ»ΠΆΠ½ΠΎ ΡƒΠ²Π΅Π»ΠΈΡ‡ΠΈΠ²Π°Ρ‚ΡŒΡΡΒ  ΠΊΠ°ΠΆΠ΄Ρ‹ΠΉ Ρ€Π°Π· Π²Π΄Π²ΠΎΠ΅, Ρ‡Ρ‚ΠΎΠ±Ρ‹ Π½Π°ΠΌ казалось, Ρ‡Ρ‚ΠΎ прирост яркости постоянСн. Π’ΠΎ Π΅ΡΡ‚ΡŒ Ссли Π΄ΠΎΠ±Π°Π²ΠΈΡ‚ΡŒ ΠΊ нашим 32 Π»Π°ΠΌΠΏΠΎΡ‡ΠΊΠ°ΠΌ Π½Π° Π³Ρ€Π°Ρ„ΠΈΠΊΠ΅ Π΅Ρ‰Π΅ ΠΎΠ΄Π½Ρƒ Π»Π°ΠΌΠΏΠΎΡ‡ΠΊΡƒ, Ρ‚ΠΎ ΠΌΡ‹ Π΄Π°ΠΆΠ΅ ΠΈ Π½Π΅ Π·Π°ΠΌΠ΅Ρ‚ΠΈΠΌ Ρ€Π°Π·Π½ΠΈΡ†Ρ‹. Для Ρ‚ΠΎΠ³ΠΎ, Ρ‡Ρ‚ΠΎΠ±Ρ‹ для нашСго Π³Π»Π°Π·Π° Π±Ρ‹Π»Π° Π·Π°ΠΌΠ΅Ρ‚Π½Π° Ρ€Π°Π·Π½ΠΈΡ†Π°, ΠΌΡ‹ Π΄ΠΎΠ»ΠΆΠ½Ρ‹ ΠΊ 32 Π»Π°ΠΌΠΏΠΎΡ‡ΠΊΠ°ΠΌ Π΄ΠΎΠ±Π°Π²ΠΈΡ‚ΡŒ Π΅Ρ‰Π΅ 32 Π»Π°ΠΌΠΏΠΎΡ‡ΠΊΠΈ, ΠΈ Ρ‚.Π΄. Или ΠΈΠ½Ρ‹ΠΌΠΈ словами, для Ρ‚ΠΎΠ³ΠΎ, Ρ‡Ρ‚ΠΎΠ±Ρ‹ Π½Π°ΠΌ казалось, Ρ‡Ρ‚ΠΎ наш ΡΠ²Π΅Ρ‚ΠΈΠ»ΡŒΠ½ΠΈΠΊ ΠΏΠ»Π°Π²Π½ΠΎ Π½Π°Π±ΠΈΡ€Π°Π΅Ρ‚ ΡΡ€ΠΊΠΎΡΡ‚ΡŒ, Π½Π°ΠΌ Π½Π°Π΄ΠΎ Π·Π°ΠΆΠΈΠ³Π°Ρ‚ΡŒ Π²Π΄Π²ΠΎΠ΅ большС Π»Π°ΠΌΠΏΠΎΡ‡Π΅ΠΊ ΠΊΠ°ΠΆΠ΄Ρ‹ΠΉ Ρ€Π°Π·, Ρ‡Π΅ΠΌ Π±Ρ‹Π»ΠΎ ΠΏΡ€Π΅Π΄Ρ‹Π΄ΡƒΡ‰Π΅Π΅ Π·Π½Π°Ρ‡Π΅Π½ΠΈΠ΅.

    ΠŸΠΎΡΡ‚ΠΎΠΌΡƒ Π΄Π΅Ρ†ΠΈΠ±Π΅Π» Π΄Π΅ΠΉΡΡ‚Π²ΠΈΡ‚Π΅Π»ΡŒΠ½ΠΎ ΡƒΠ΄ΠΎΠ±Π½Π΅Π΅ Π² Π½Π΅ΠΊΠΎΡ‚ΠΎΡ€Ρ‹Ρ… случаях, Ρ‚Π°ΠΊ ΠΊΠ°ΠΊ ΡΡ€Π°Π²Π½ΠΈΠ²Π°Ρ‚ΡŒ Π΄Π²Π΅ Π²Π΅Π»ΠΈΡ‡ΠΈΠ½Ρ‹ Π½Π°ΠΌΠ½ΠΎΠ³ΠΎ ΠΏΡ€ΠΎΡ‰Π΅ Π² ΠΌΠ°Π»Π΅Π½ΡŒΠΊΠΈΡ… Ρ†ΠΈΡ„Ρ€Π°Ρ…, Ρ‡Π΅ΠΌ Π² ΠΌΠΈΠ»Π»ΠΈΠΎΠ½Π°Ρ… ΠΈ ΠΌΠΈΠ»Π»ΠΈΠ°Ρ€Π΄Π°Ρ…. А Ρ‚Π°ΠΊ ΠΊΠ°ΠΊ элСктроника – это чисто физичСскоС явлСниС, Ρ‚ΠΎ ΠΈ Π΄Π΅Ρ†ΠΈΠ±Π΅Π»Ρ‹ Π½Π΅ обошли Π΅Π΅ стороной.

    [quads id=1]

    Π”Π΅Ρ†ΠΈΠ±Π΅Π»Ρ‹ ΠΈ АЧΠ₯ усилитСля

    Как Π²Ρ‹ ΠΏΠΎΠΌΠ½ΠΈΡ‚Π΅ Π²Β  ΠΏΡ€ΠΎΡˆΠ»ΠΎΠΌ ΠΏΡ€ΠΈΠΌΠ΅Ρ€Π΅ с ОУ, Ρƒ нас Π½Π΅ΠΈΠ½Π²Π΅Ρ€Ρ‚ΠΈΡ€ΡƒΡŽΡ‰ΠΈΠΉ ΡƒΡΠΈΠ»ΠΈΡ‚Π΅Π»ΡŒ усиливал сигнал Π² 10 Ρ€Π°Π·. Если ΠΏΠΎΡΠΌΠΎΡ‚Ρ€Π΅Ρ‚ΡŒ Π² Π½Π°ΡˆΡƒ Ρ‚Π°Π±Π»ΠΈΡ‡ΠΊΡƒ, Ρ‚ΠΎ это получаСтся 20 Π΄Π‘ ΠΎΡ‚Π½ΠΎΡΠΈΡ‚Π΅Π»ΡŒΠ½ΠΎ Π²Ρ…ΠΎΠ΄Π½ΠΎΠ³ΠΎ сигнала. Ну Π΄Π°, Ρ‚Π°ΠΊ ΠΎΠ½ΠΎ ΠΈ Π΅ΡΡ‚ΡŒ:

    Π’Π°ΠΊΠΆΠ΅ Π² Π΄Π‘ Π½Π° Π½Π΅ΠΊΠΎΡ‚ΠΎΡ€Ρ‹Ρ… Π³Ρ€Π°Ρ„ΠΈΠΊΠ°Ρ… АЧΠ₯ ΠΎΠ±ΠΎΠ·Π½Π°Ρ‡Π°ΡŽΡ‚ Π½Π°ΠΊΠ»ΠΎΠ½ характСристики АЧΠ₯. Π­Ρ‚ΠΎ ΠΌΠΎΠΆΠ΅Ρ‚ Π²Ρ‹Π³Π»ΡΠ΄Π΅Ρ‚ΡŒ ΠΏΡ€ΠΈΠΌΠ΅Ρ€Π½ΠΎ Π²ΠΎΡ‚ Ρ‚Π°ΠΊ:

    На Π³Ρ€Π°Ρ„ΠΈΠΊΠ΅ ΠΌΡ‹ Π²ΠΈΠ΄ΠΈΠΌ АЧΠ₯ полосового Ρ„ΠΈΠ»ΡŒΡ‚Ρ€Π°. ИзмСнСниС сигнала +20 Π΄Π‘ Π½Π° Π΄Π΅ΠΊΠ°Π΄Ρƒ (Π΄Π‘/Π΄Π΅ΠΊ, dB/dec) Π³ΠΎΠ²ΠΎΡ€ΠΈΡ‚ Π½Π°ΠΌ ΠΎ Ρ‚ΠΎΠΌ, Ρ‡Ρ‚ΠΎ ΠΏΡ€ΠΈ ΠΊΠ°ΠΆΠ΄ΠΎΠΌ ΡƒΠ²Π΅Π»ΠΈΡ‡Π΅Π½ΠΈΠΈ частоты Π² 10 Ρ€Π°Π·, Π°ΠΌΠΏΠ»ΠΈΡ‚ΡƒΠ΄Π° сигнала возрастаСт Π½Π° 20 Π΄Π‘. Π’ΠΎ ΠΆΠ΅ самоС ΠΌΠΎΠΆΠ½ΠΎ ΡΠΊΠ°Π·Π°Ρ‚ΡŒ ΠΈ ΠΏΡ€ΠΎ спад сигнала -20 Π΄Π‘ Π½Π° Π΄Π΅ΠΊΠ°Π΄Ρƒ. ΠŸΡ€ΠΈ ΠΊΠ°ΠΆΠ΄ΠΎΠΌ ΡƒΠ²Π΅Π»ΠΈΡ‡Π΅Π½ΠΈΠΈ частоты Π² 10 Ρ€Π°Π·, Ρƒ нас Π°ΠΌΠΏΠ»ΠΈΡ‚ΡƒΠ΄Π° сигнала Π±ΡƒΠ΄Π΅Ρ‚ ΡƒΠΌΠ΅Π½ΡŒΡˆΠ°Ρ‚ΡŒΡΡ Π½Π° -20 Π΄Π‘. Π•ΡΡ‚ΡŒ Ρ‚Π°ΠΊΠΆΠ΅ похоТая характСристика Π΄Π‘ Π½Π° ΠΎΠΊΡ‚Π°Π²Ρƒ (Π΄Π‘/ΠΎΠΊΡ‚, dB/oct). Π—Π΄Π΅ΡΡŒ ΠΏΠΎΡ‡Ρ‚ΠΈ всС Ρ‚ΠΎ ΠΆΠ΅ самоС, Ρ‚ΠΎΠ»ΡŒΠΊΠΎ ΠΈΠ·ΠΌΠ΅Π½Π΅Π½ΠΈΠ΅ сигнала происходит ΠΏΡ€ΠΈ ΠΊΠ°ΠΆΠ΄ΠΎΠΌ ΡƒΠ²Π΅Π»ΠΈΡ‡Π΅Π½ΠΈΠΈ частоты Π² 2 Ρ€Π°Π·Π°.

    Π”Π°Π²Π°ΠΉΡ‚Π΅ рассмотрим ΠΏΡ€ΠΈΠΌΠ΅Ρ€. ИмССм Ρ„ΠΈΠ»ΡŒΡ‚Ρ€ высоких частот (Π€Π’Π§) ΠΏΠ΅Ρ€Π²ΠΎΠ³ΠΎ порядка, собранного Π½Π° RC-Ρ†Π΅ΠΏΠΈ.

    Π•Π³ΠΎ АЧΠ₯ Π±ΡƒΠ΄Π΅Ρ‚ Π²Ρ‹Π³Π»ΡΠ΄Π΅Ρ‚ΡŒ ΡΠ»Π΅Π΄ΡƒΡŽΡ‰ΠΈΠΌ ΠΎΠ±Ρ€Π°Π·ΠΎΠΌ (ΠΊΠ»ΠΈΠΊΠ½ΠΈΡ‚Π΅ для ΠΏΠΎΠ»Π½ΠΎΠ³ΠΎ открытия)

    Нас сСйчас интСрСсуСт  наклонная прямая линия АЧΠ₯. Π’Π°ΠΊ ΠΊΠ°ΠΊ Ρƒ Π½Π΅Π΅ Π½Π°ΠΊΠ»ΠΎΠ½ ΠΏΡ€ΠΈΠΌΠ΅Ρ€Π½ΠΎ ΠΎΠ΄ΠΈΠ½Π°ΠΊΠΎΠ²Ρ‹ΠΉ Π΄ΠΎ частоты срСза  Π² -3Π΄Π‘, Ρ‚ΠΎ ΠΌΠΎΠΆΠ½ΠΎ Π½Π°ΠΉΡ‚ΠΈ Π΅Π΅ ΠΊΡ€ΡƒΡ‚ΠΈΠ·Π½Ρƒ, Ρ‚ΠΎ Π΅ΡΡ‚ΡŒ ΡƒΠ·Π½Π°Ρ‚ΡŒ, Π²ΠΎ сколько Ρ€Π°Π· увСличиваСтся сигнал ΠΏΡ€ΠΈ ΠΊΠ°ΠΆΠ΄ΠΎΠΌ ΡƒΠ²Π΅Π»ΠΈΡ‡Π΅Π½ΠΈΠΈ частоты Π² 10 Ρ€Π°Π·.

    Π˜Ρ‚Π°ΠΊ возьмСм ΠΏΠ΅Ρ€Π²ΡƒΡŽ Ρ‚ΠΎΡ‡ΠΊΡƒ Π½Π° частотС Π² 10 Π“Π΅Ρ€Ρ†. На частотС Π² 10 Π“Π΅Ρ€Ρ† Π°ΠΌΠΏΠ»ΠΈΡ‚ΡƒΠ΄Π° сигнала ΡƒΠΌΠ΅Π½ΡŒΡˆΠΈΠ»Π°ΡΡŒ Π½Π° 44 Π΄Π‘, это Π²ΠΈΠ΄Π½ΠΎ Π² ΠΏΡ€Π°Π²ΠΎΠΌ Π½ΠΈΠΆΠ½Π΅ΠΌ ΡƒΠ³Π»Ρƒ (out:-44)

    Π£ΠΌΠ½ΠΎΠΆΠ°Π΅ΠΌ частоту Π½Π° 10 (Π΄Π΅ΠΊΠ°Π΄Π°) ΠΈ ΠΏΠΎΠ»ΡƒΡ‡Π°Π΅ΠΌ Π²Ρ‚ΠΎΡ€ΡƒΡŽ Ρ‚ΠΎΡ‡ΠΊΡƒ Π² 100 Π“Π΅Ρ€Ρ†. На частотС Π² 100 Π“Π΅Ρ€Ρ† наш сигнал ΡƒΠΌΠ΅Π½ΡŒΡˆΠΈΠ»ΡΡ ΠΏΡ€ΠΈΠ±Π»ΠΈΠ·ΠΈΡ‚Π΅Π»ΡŒΠ½ΠΎ Π½Π° 24 Π΄Π‘

    Π’ΠΎ Π΅ΡΡ‚ΡŒ получаСтся Π·Π° ΠΎΠ΄Π½Ρƒ Π΄Π΅ΠΊΠ°Π΄Ρƒ Ρƒ нас сигнал увСличился с -44Β  Π΄ΠΎ -24 Π΄Π‘ Π½Π° Π΄Π΅ΠΊΠ°Π΄Ρƒ. Π’ΠΎ Π΅ΡΡ‚ΡŒ Π½Π°ΠΊΠ»ΠΎΠ½ характСристики составил +20 Π΄Π‘/Π΄Π΅ΠΊΠ°Π΄Ρƒ. Если +20 Π΄Π‘/Π΄Π΅ΠΊΠ°Π΄Ρƒ пСрСвСсти Π² Π΄Π‘ Π½Π° ΠΎΠΊΡ‚Π°Π²Ρƒ, Ρ‚ΠΎ получится 6 Π΄Π‘/ΠΎΠΊΡ‚Π°Π²Ρƒ.

    Достаточно часто, дискрСтныС Π°Ρ‚Ρ‚Π΅Π½ΡŽΠ°Ρ‚ΠΎΡ€Ρ‹ (Π΄Π΅Π»ΠΈΡ‚Π΅Π»ΠΈ) Π²Ρ‹Ρ…ΠΎΠ΄Π½ΠΎΠ³ΠΎ сигнала Π½Π° ΠΈΠ·ΠΌΠ΅Ρ€ΠΈΡ‚Π΅Π»ΡŒΠ½Ρ‹Ρ… ΠΏΡ€ΠΈΠ±ΠΎΡ€Π°Ρ… (особСнно Π½Π° Π³Π΅Π½Π΅Ρ€Π°Ρ‚ΠΎΡ€Π°Ρ…) ΠΏΡ€ΠΎΠ³Ρ€Π°Π΄ΡƒΠΈΡ€ΠΎΠ²Π°Π½Ρ‹ Π² Π΄Π΅Ρ†ΠΈΠ±Π΅Π»Π°Ρ…:
    0, -3, -6, -10, -20, -30, -40 Π΄Π‘. Π­Ρ‚ΠΎ позволяСт быстро ΠΎΡ€ΠΈΠ΅Π½Ρ‚ΠΈΡ€ΠΎΠ²Π°Ρ‚ΡŒΡΡ Π² ΠΎΡ‚Π½ΠΎΡΠΈΡ‚Π΅Π»ΡŒΠ½ΠΎΠΌ ΡƒΡ€ΠΎΠ²Π½Π΅ Π²Ρ‹Ρ…ΠΎΠ΄Π½ΠΎΠ³ΠΎ сигнала.

    Π§Ρ‚ΠΎ Π΅Ρ‰Π΅ ΠΈΠ·ΠΌΠ΅Ρ€ΡΡŽΡ‚ Π² Π΄Π΅Ρ†ΠΈΠ±Π΅Π»Π°Ρ…?

    Π’Π°ΠΊΠΆΠ΅ ΠΎΡ‡Π΅Π½ΡŒ часто Π² Π΄Π‘ Π²Ρ‹Ρ€Π°ΠΆΠ°ΡŽΡ‚ ΠΎΡ‚Π½ΠΎΡˆΠ΅Π½ΠΈΠ΅ сигнал-ΡˆΡƒΠΌ (signal-to-noise ratio, сокр. SNR)

    Π³Π΄Π΅

    Uc – это эффСктивноС Π·Π½Π°Ρ‡Π΅Π½ΠΈΠ΅ напряТСния сигнала, Π’

    Uш – эффСктивноС Π·Π½Π°Ρ‡Π΅Π½ΠΈΠ΅ напряТСния ΡˆΡƒΠΌΠ°, Π’

    Π§Π΅ΠΌ Π²Ρ‹ΡˆΠ΅ Π·Π½Π°Ρ‡Π΅Π½ΠΈΠ΅ сигнал/ΡˆΡƒΠΌ, Ρ‚Π΅ΠΌ Π±ΠΎΠ»Π΅Π΅ чистый Π·Π²ΡƒΠΊ обСспСчиваСтся аудиосистСмой. Для ΠΌΡƒΠ·Ρ‹ΠΊΠ°Π»ΡŒΠ½ΠΎΠΉ Π°ΠΏΠΏΠ°Ρ€Π°Ρ‚ΡƒΡ€Ρ‹ ΠΆΠ΅Π»Π°Ρ‚Π΅Π»ΡŒΠ½ΠΎ, Ρ‡Ρ‚ΠΎΠ±Ρ‹ это ΠΎΡ‚Π½ΠΎΡˆΠ΅Π½ΠΈΠ΅ Π±Ρ‹Π»ΠΎ Π½Π΅ ΠΌΠ΅Π½Π΅Π΅ 75 Π΄Π‘, Π° для Hi-Fi Π°ΠΏΠΏΠ°Ρ€Π°Ρ‚ΡƒΡ€Ρ‹ Π½Π΅ ΠΌΠ΅Π½Π΅Π΅ 90 Π΄Π‘. НС ΠΈΠΌΠ΅Π΅Ρ‚ Π·Π½Π°Ρ‡Π΅Π½ΠΈΠ΅ физичСская ΠΏΡ€ΠΈΡ€ΠΎΠ΄Π° сигнала, Π²Π°ΠΆΠ½ΠΎ, Ρ‡Ρ‚ΠΎΠ±Ρ‹ Π΅Π΄ΠΈΠ½ΠΈΡ†Ρ‹ Π±Ρ‹Π»ΠΈ Π² ΠΎΠ΄ΠΈΠ½Π°ΠΊΠΎΠ²Ρ‹Ρ… измСрСниях.

    Π’ качСствС Π΅Π΄ΠΈΠ½ΠΈΡ†Ρ‹ логарифмичСского ΠΎΡ‚Π½ΠΎΡˆΠ΅Π½ΠΈΡ Π΄Π²ΡƒΡ… ΠΎΠ΄Π½ΠΎΠΈΠΌΡ‘Π½Π½Ρ‹Ρ… физичСских Π²Π΅Π»ΠΈΡ‡ΠΈΠ½ примСняСтся Ρ‚Π°ΠΊΠΆΠ΅ Π½Π΅ΠΏΠ΅Ρ€ (Нп) β€” 1 Нп ~ 0,8686 Π‘. Π’ основС Π»Π΅ΠΆΠΈΡ‚ Π½Π΅ дСсятичный (lg), Π° Π½Π°Ρ‚ΡƒΡ€Π°Π»ΡŒΠ½Ρ‹ΠΉ (ln) Π»ΠΎΠ³Π°Ρ€ΠΈΡ„ΠΌ ΠΎΡ‚Π½ΠΎΡˆΠ΅Π½ΠΈΠΉ. Π’ настоящСС врСмя ΠΈΡΠΏΠΎΠ»ΡŒΠ·ΡƒΠ΅Ρ‚ΡΡ Ρ€Π΅Π΄ΠΊΠΎ.

    Π’ΠΎ ΠΌΠ½ΠΎΠ³ΠΈΡ… случаях, ΡƒΠ΄ΠΎΠ±Π½ΠΎ ΡΡ€Π°Π²Π½ΠΈΠ²Π°Ρ‚ΡŒ ΠΌΠ΅ΠΆΠ΄Ρƒ собой Π½Π΅ ΠΏΡ€ΠΎΠΈΠ·Π²ΠΎΠ»ΡŒΠ½Ρ‹Π΅ Π²Π΅Π»ΠΈΡ‡ΠΈΠ½Ρ‹, Π° ΠΎΠ΄Π½Ρƒ Π²Π΅Π»ΠΈΡ‡ΠΈΠ½Ρƒ ΠΎΡ‚Π½ΠΎΡΠΈΡ‚Π΅Π»ΡŒΠ½ΠΎ Π΄Ρ€ΡƒΠ³ΠΎΠΉ, Π½Π°Π·Π²Π°Π½Π½ΠΎΠΉ условно ΠΎΠΏΠΎΡ€Π½ΠΎΠΉ (Π½ΡƒΠ»Π΅Π²ΠΎΠΉ, Π±Π°Π·ΠΎΠ²ΠΎΠΉ).
    Π’ элСктротСхникС, Π² качСствС Ρ‚Π°ΠΊΠΎΠΉ ΠΎΠΏΠΎΡ€Π½ΠΎΠΉ ΠΈΠ»ΠΈ Π½ΡƒΠ»Π΅Π²ΠΎΠΉ Π²Π΅Π»ΠΈΡ‡ΠΈΠ½Ρ‹ Π²Ρ‹Π±Ρ€Π°Π½ΠΎ Π·Π½Π°Ρ‡Π΅Π½ΠΈΠ΅ мощности Ρ€Π°Π²Π½ΠΎΠ΅ 1 ΠΌΠ’Ρ‚ выдСляСмоС Π½Π° рСзисторС сопротивлСниСм 600 Ом.
    Π’ этом случаС, Π±Π°Π·ΠΎΠ²Ρ‹ΠΌΠΈ значСниями ΠΏΡ€ΠΈ сравнСнии напряТСний ΠΈΠ»ΠΈ Ρ‚ΠΎΠΊΠΎΠ² станут Π²Π΅Π»ΠΈΡ‡ΠΈΠ½Ρ‹ 0.775 Π’ ΠΈΠ»ΠΈ 1.29 мА.

    Для Π·Π²ΡƒΠΊΠΎΠ²ΠΎΠΉ мощности Ρ‚Π°ΠΊΠΎΠΉ Π±Π°Π·ΠΎΠ²ΠΎΠΉ Π²Π΅Π»ΠΈΡ‡ΠΈΠ½ΠΎΠΉ являСтся 20 ΠΌΠΈΠΊΡ€ΠΎΠŸΠ°ΡΠΊΠ°Π»ΡŒ (0 Π΄Π‘), Π° ΠΏΠΎΡ€ΠΎΠ³ +130 Π΄Π‘ считаСтся Π±ΠΎΠ»Π΅Π²Ρ‹ΠΌ для Ρ‡Π΅Π»ΠΎΠ²Π΅ΠΊΠ°:

    Π‘ΠΎΠ»Π΅Π΅ ΠΏΠΎΠ΄Ρ€ΠΎΠ±Π½ΠΎ ΠΎΠ± этом написано Π² Π’ΠΈΠΊΠΈΠΏΠ΅Π΄ΠΈΠΈ ΠΏΠΎ этой ссылкС.

    Для случаСв ΠΊΠΎΠ³Π΄Π° Π² качСствС Π±Π°Π·ΠΎΠ²Ρ‹Ρ… Π·Π½Π°Ρ‡Π΅Π½ΠΈΠΉ ΠΈΡΠΏΠΎΠ»ΡŒΠ·ΡƒΡŽΡ‚ΡΡ Ρ‚Π΅ ΠΈΠ»ΠΈ ΠΈΠ½Ρ‹Π΅ ΠΊΠΎΠ½ΠΊΡ€Π΅Ρ‚Π½Ρ‹Π΅ Π²Π΅Π»ΠΈΡ‡ΠΈΠ½Ρ‹, ΠΏΡ€ΠΈΠ΄ΡƒΠΌΠ°Π½Ρ‹ Π΄Π°ΠΆΠ΅ ΡΠΏΠ΅Ρ†ΠΈΠ°Π»ΡŒΠ½Ρ‹Π΅ обозначСния Π΅Π΄ΠΈΠ½ΠΈΡ† ΠΈΠ·ΠΌΠ΅Ρ€Π΅Π½ΠΈΠΉ:

    dbW (Π΄Π‘Π’Ρ‚) – здСсь отсчСт ΠΈΠ΄Π΅Ρ‚ ΠΎΡ‚Π½ΠΎΡΠΈΡ‚Π΅Π»ΡŒΠ½ΠΎ 1 Π’Π°Ρ‚Ρ‚Π° (Π’Ρ‚). НапримСр, ΠΏΡƒΡΡ‚ΡŒ ΡƒΡ€ΠΎΠ²Π΅Π½ΡŒ мощности составил +20 Π΄Π‘Π’Ρ‚. Π­Ρ‚ΠΎ Π·Π½Π°Ρ‡ΠΈΡ‚ Ρ‡Ρ‚ΠΎ ΠΌΠΎΡ‰Π½ΠΎΡΡ‚ΡŒ ΡƒΠ²Π΅Π»ΠΈΡ‡ΠΈΠ»Π°ΡΡŒ Π² 100 Ρ€Π°Π·, Ρ‚ΠΎ Π΅ΡΡ‚ΡŒ Π½Π° 100 Π’Ρ‚.

    dBm (Π΄Π‘ΠΌ) – здСсь Ρƒ нас отсчСт ΡƒΠΆΠ΅ ΠΈΠ΄Π΅Ρ‚ ΠΎΡ‚Π½ΠΎΡΠΈΡ‚Π΅Π»ΡŒΠ½ΠΎ 1 ΠΌΠΈΠ»Π»ΠΈΠ²Π°Ρ‚Ρ‚Π° (ΠΌΠ’Ρ‚). НапримСр, ΡƒΡ€ΠΎΠ²Π΅Π½ΡŒ мощности Π² +30Π΄Π‘ΠΌ Π±ΡƒΠ΄Π΅Ρ‚ соотвСтствСнно Ρ€Π°Π²Π΅Π½ 1 Π’Ρ‚. НС Π·Π°Π±Ρ‹Π²Π°Π΅ΠΌ, Ρ‡Ρ‚ΠΎ это Ρƒ нас энСргСтичСскиС Π΄Π΅Ρ†ΠΈΠ±Π΅Π»Ρ‹, поэтому для Π½ΠΈΡ… Π±ΡƒΠ΄Π΅Ρ‚ справСдлива Ρ„ΠΎΡ€ΠΌΡƒΠ»Π°

    Π‘Π»Π΅Π΄ΡƒΡŽΡ‰ΠΈΠ΅ характСристики – это ΡƒΠΆΠ΅ Π°ΠΌΠΏΠ»ΠΈΡ‚ΡƒΠ΄Π½Ρ‹Π΅ Π΄Π΅Ρ†ΠΈΠ±Π΅Π»Ρ‹. Для Π½ΠΈΡ… Π±ΡƒΠ΄Π΅Ρ‚ справСдлива Ρ„ΠΎΡ€ΠΌΡƒΠ»Π°

    dBV (Π΄Π‘Π’) – ΠΊΠ°ΠΊ Π²Ρ‹ догадались, ΠΎΠΏΠΎΡ€Π½ΠΎΠ΅ напряТСниС 1 Π’ΠΎΠ»ΡŒΡ‚. НапримСр, +20Π΄Π‘Π’ даст – это 10 Π’ΠΎΠ»ΡŒΡ‚

    ΠžΡ‚Β  Π΄Π‘Π’ Ρ‚Π°ΠΊΠΆΠ΅ Π²Ρ‹Ρ‚Π΅ΠΊΠ°ΡŽΡ‚ Π΄Ρ€ΡƒΠ³ΠΈΠ΅ Π²ΠΈΠ΄Ρ‹ Π΄Π΅Ρ†ΠΈΠ±Π΅Π»ΠΎΠ² с Ρ€Π°Π·Π½Ρ‹ΠΌΠΈ приставками:

    dBmV (Π΄Π‘ΠΌΠ’) – ΠΎΠΏΠΎΡ€Π½Ρ‹ΠΉ ΡƒΡ€ΠΎΠ²Π΅Π½ΡŒ 1 ΠΌΠΈΠ»Π»ΠΈΠ²ΠΎΠ»ΡŒΡ‚.

    dBuV (Π΄Π‘ΠΌΠΊΠ’) – ΠΎΠΏΠΎΡ€Π½ΠΎΠ΅ напряТСниС 1 ΠΌΠΈΠΊΡ€ΠΎΠ²ΠΎΠ»ΡŒΡ‚.

    Π—Π΄Π΅ΡΡŒ я ΠΏΡ€ΠΈΠ²Π΅Π» Π½Π°ΠΈΠ±ΠΎΠ»Π΅Π΅ ΡƒΠΏΠΎΡ‚Ρ€Π΅Π±ΠΈΠΌΡ‹Π΅ ΡΠΏΠ΅Ρ†ΠΈΠ°Π»ΡŒΠ½Ρ‹Π΅ Π²ΠΈΠ΄Ρ‹ Π΄Π΅Ρ†ΠΈΠ±Π΅Π»ΠΎΠ² Π² элСктроникС.

    Π”Π΅Ρ†ΠΈΠ±Π΅Π»Ρ‹ ΠΈΡΠΏΠΎΠ»ΡŒΠ·ΡƒΡŽΡ‚ΡΡ ΠΈ Π² Π΄Ρ€ΡƒΠ³ΠΈΡ… отраслях, Π³Π΄Π΅ ΠΎΠ½ΠΈ Ρ‚Π°ΠΊΠΆΠ΅ ΠΏΠΎΠΊΠ°Π·Ρ‹Π²Π°ΡŽΡ‚ ΠΎΡ‚Π½ΠΎΡˆΠ΅Π½ΠΈΠ΅ ΠΊΠ°ΠΊΠΈΡ…-Π»ΠΈΠ±ΠΎ Π΄Π²ΡƒΡ… измСряСмых Π²Π΅Π»ΠΈΡ‡ΠΈΠ½ Π² логарифмичСском ΠΌΠ°ΡΡˆΡ‚Π°Π±Π΅.

    Π’Π°ΠΊΠΆΠ΅ Π½Π° YouTube Π΅ΡΡ‚ΡŒ интСрСсноС Π²ΠΈΠ΄Π΅ΠΎ ΠΎ Π΄Π΅Ρ†ΠΈΠ±Π΅Π»Π°Ρ….

    ΠŸΡ€ΠΈ участии Jeer

    ΠšΠ°Π»ΡŒΠΊΡƒΠ»ΡΡ‚ΠΎΡ€

    Π΄Π‘ для расчСта коэффициСнта усилСния ΠΈ коэффициСнта затухания (ΠΏΠΎΡ‚Π΅Ρ€ΡŒ) аудиоусилитСля, Π΄Π΅Ρ†ΠΈΠ±Π΅Π», Π΄Π‘.



    ● РасчСт : УсилСниС ( усилСниС ) ΠΈ Π΄Π΅ΠΌΠΏΡ„ΠΈΡ€ΠΎΠ²Π°Π½ΠΈΠ΅ ( ΡƒΠ±Ρ‹Ρ‚ΠΎΠΊ )
    ΠΊΠ°ΠΊ
    коэффициСнт ( ΠΎΡ‚Π½ΠΎΡˆΠ΅Π½ΠΈΠ΅ ) Π΄ΠΎ ΡƒΡ€ΠΎΠ²Π΅Π½ΡŒ Π΄Π΅Ρ†ΠΈΠ±Π΅Π» ( Π΄Π‘ ) ●

    ΠΎΡ‚Π½ΠΎΡˆΠ΅Π½ΠΈΠ΅ ΠΌΠ΅ΠΆΠ΄Ρƒ Π²Π΅Π»ΠΈΡ‡ΠΈΠ½Π°ΠΌΠΈ
    Π²Ρ‹Ρ…ΠΎΠ΄Π½Ρ‹Π΅ ΠΈ Π²Ρ…ΠΎΠ΄Π½Ρ‹Π΅ сигналы.
    РСгуляторы усилСния Π½Π° усилитСлС Π² основном ΠΏΡ€Π΅Π΄ΡΡ‚Π°Π²Π»ΡΡŽΡ‚ собой нСбольшиС ΠΏΠΎΡ‚Π΅Π½Ρ†ΠΈΠΎΠΌΠ΅Ρ‚Ρ€Ρ‹ (пСрСмСнная
    рСзисторы) ΠΈΠ»ΠΈ рСгуляторы громкости, ΠΊΠΎΡ‚ΠΎΡ€Ρ‹Π΅ ΠΏΠΎΠ·Π²ΠΎΠ»ΡΡŽΡ‚ Ρ€Π΅Π³ΡƒΠ»ΠΈΡ€ΠΎΠ²Π°Ρ‚ΡŒ входящий сигнал Π½Π°
    ΡƒΡΠΈΠ»ΠΈΡ‚Π΅Π»ΡŒ Π·Π²ΡƒΠΊΠ°.

    ΠšΠΎΡΡ„Ρ„ΠΈΡ†ΠΈΠ΅Π½Ρ‚ усилСния, Ρ‚Π°ΠΊΠΆΠ΅ Π½Π°Π·Ρ‹Π²Π°Π΅ΠΌΡ‹ΠΉ усилСниСм, — это ΡΡ‚Π΅ΠΏΠ΅Π½ΡŒ, Π² ΠΊΠΎΡ‚ΠΎΡ€ΠΎΠΉ устройство ΡƒΠ²Π΅Π»ΠΈΡ‡ΠΈΠ²Π°Π΅Ρ‚ ΠΌΠΎΡ‰Π½ΠΎΡΡ‚ΡŒ сигнала.
    ΠšΠΎΡΡ„Ρ„ΠΈΡ†ΠΈΠ΅Π½Ρ‚ дСмпфирования, Ρ‚Π°ΠΊΠΆΠ΅ Π½Π°Π·Ρ‹Π²Π°Π΅ΠΌΡ‹ΠΉ потСрями, — это ΡΡ‚Π΅ΠΏΠ΅Π½ΡŒ, Π² ΠΊΠΎΡ‚ΠΎΡ€ΠΎΠΉ устройство сниТаСт ΠΌΠΎΡ‰Π½ΠΎΡΡ‚ΡŒ сигнала.

    Π’Π²Π΅Π΄ΠΈΡ‚Π΅ Π΄Π²Π° значСния ΠΈ Π½Π°ΠΆΠΌΠΈΡ‚Π΅ ΠΏΡ€Π°Π²ΡƒΡŽ полосу вычислСния Π² строкС ΠΎΡ‚ΡΡƒΡ‚ΡΡ‚Π²ΡƒΡŽΡ‰Π΅Π³ΠΎ ΠΎΡ‚Π²Π΅Ρ‚Π°.
    Π˜ΡΠΏΠΎΠ»ΡŒΠ·ΡƒΠ΅ΠΌΡ‹ΠΉ Π±Ρ€Π°ΡƒΠ·Π΅Ρ€ Π½Π΅ ΠΏΠΎΠ΄Π΄Π΅Ρ€ΠΆΠΈΠ²Π°Π΅Ρ‚ Javascript.
    ΠŸΡ€ΠΎΠ³Ρ€Π°ΠΌΠΌΠ° ΡƒΠΊΠ°Π·Π°Π½Π°, Π½ΠΎ фактичСская функция отсутствуСт.

    Π’ Π°Π½Π°Π»ΠΎΠ³ΠΎΠ²ΠΎΠΉ Π°ΡƒΠ΄ΠΈΠΎΡ‚Π΅Ρ…Π½ΠΈΠΊΠ΅ ΠΌΡ‹ ΠΈΠΌΠ΅Π΅ΠΌ Π΄Π΅Π»ΠΎ Ρ‚ΠΎΠ»ΡŒΠΊΠΎ с усилСниСм (усилСниСм) ΠΈ Π΄Π΅ΠΌΠΏΡ„ΠΈΡ€ΠΎΠ²Π°Π½ΠΈΠ΅ΠΌ (потСрями) «напряТСния».
    Π’ 1 = Π’ Π² ΠΈ Π’ 2 = Π’ Π²Π½Π΅
    Π’ 2 > Π’ 1 ΠΈΠ»ΠΈ Π’ Π²Π½Π΅ > Π’ Π² ΠΎΠ·Π½Π°Ρ‡Π°Π΅Ρ‚ усилСниС . Π—Π½Π°Ρ‡Π΅Π½ΠΈΠ΅ Π΄Π‘ ΠΏΠΎΠ»ΠΎΠΆΠΈΡ‚Π΅Π»ΡŒΠ½ΠΎΠ΅ (+).
    Π’ 2 < Π’ 1 ΠΈΠ»ΠΈ Π’ Π½Π° Π²Ρ‹Ρ…ΠΎΠ΄Π΅ < Π’ Π² ΠΎΠ·Π½Π°Ρ‡Π°Π΅Ρ‚ дСмпфирования. Π—Π½Π°Ρ‡Π΅Π½ΠΈΠ΅ Π΄Π‘ ΠΎΡ‚Ρ€ΠΈΡ†Π°Ρ‚Π΅Π»ΡŒΠ½ΠΎΠ΅ (-).
    Π’ 2 / Π’ 1 ΠΈΠ»ΠΈ Π’ Π²Π½Π΅ / Π’ дюйм ΠΎΠ·Π½Π°Ρ‡Π°Π΅Ρ‚ ΡΠΎΠΎΡ‚Π½ΠΎΡˆΠ΅Π½ΠΈΠ΅ . УсилСниС ΠΈΠ»ΠΈ Π·Π°Ρ‚ΡƒΡ…Π°Π½ΠΈΠ΅ Π² Π΄Π‘ составляСт:
    L = 20 Γ— log (ΠΎΡ‚Π½ΠΎΡˆΠ΅Π½ΠΈΠ΅ напряТСний Π’ 2 / Π’ 1 ) Π² Π΄Π‘. Π’ 1 = Π’ Π² являСтся справочным.

    Π’ Ρ„ΠΈΠ·ΠΈΠΊΠ΅ Π·Π°Ρ‚ΡƒΡ…Π°Π½ΠΈΠ΅ считаСтся ΠΏΠΎΠ»ΠΎΠΆΠΈΡ‚Π΅Π»ΡŒΠ½Ρ‹ΠΌ Π·Π½Π°Ρ‡Π΅Π½ΠΈΠ΅ΠΌ.
    Π­Ρ‚ΠΎ, СстСствСнно, ΠΏΡ€ΠΈΠ²ΠΎΠ΄ΠΈΡ‚ ΠΊ Π·Π½Π°ΠΊΠΎΠ²Ρ‹ΠΌ ошибкам ΠΏΡ€ΠΈ Π²Π²ΠΎΠ΄Π΅ чисСл.

    3 Π΄Π‘ ≑ Π’ 1,414 Ρ€Π°Π·Π° большС напряТСния (-) 3 Π΄Π‘ ≑ Π΄Π΅ΠΌΠΏΡ„ΠΈΡ€ΠΎΠ²Π°Π½ΠΈΠ΅ Π΄ΠΎ значСния 0,707
    6 Π΄Π‘ ≑ НапряТСниС Π² 2 Ρ€Π°Π·Π° большС (-) 6 Π΄Π‘ ≑ Π΄Π΅ΠΌΠΏΡ„ΠΈΡ€ΠΎΠ²Π°Π½ΠΈΠ΅ Π΄ΠΎ значСния 0.5
    10 Π΄Π‘ ≑ Π’ 3,162 Ρ€Π°Π·Π° большС напряТСния (-) 10 Π΄Π‘ ≑ Π΄Π΅ΠΌΠΏΡ„ΠΈΡ€ΠΎΠ²Π°Π½ΠΈΠ΅ Π΄ΠΎ значСния 0,316
    12 Π΄Π‘ ≑ Π² 4 Ρ€Π°Π·Π° большС напряТСния (-) 12 Π΄Π‘ ≑ Π΄Π΅ΠΌΠΏΡ„ΠΈΡ€ΠΎΠ²Π°Π½ΠΈΠ΅ Π΄ΠΎ значСния 0,25
    20 Π΄Π‘ ≑ Π’ 10 Ρ€Π°Π· большС напряТСния (-) 20 Π΄Π‘ ≑ Π΄Π΅ΠΌΠΏΡ„ΠΈΡ€ΠΎΠ²Π°Π½ΠΈΠ΅ Π΄ΠΎ значСния 0,1

    Π˜ΡΠΏΠΎΠ»ΡŒΠ·ΡƒΡ напряТСниС, ΠΏΠΎΠ»ΡƒΡ‡Π°Π΅ΠΌ: Π£Ρ€ΠΎΠ²Π΅Π½ΡŒ Π² Π΄Π‘: L = 20 Γ— log (ΠΎΡ‚Π½ΠΎΡˆΠ΅Π½ΠΈΠ΅ напряТСний)

    6 Π΄Π‘ = ΡƒΠ΄Π²ΠΎΠ΅Π½Π½ΠΎΠ΅ напряТСниС
    12 Π΄Π‘ = Π² Ρ‡Π΅Ρ‚Ρ‹Ρ€Π΅ Ρ€Π°Π·Π° большС напряТСния
    20 Π΄Π‘ = Π² Π΄Π΅ΡΡΡ‚ΡŒ Ρ€Π°Π· большС напряТСния
    40 Π΄Π‘ = Π² сто Ρ€Π°Π· большС напряТСния

    Если Π³ΠΎΠ²ΠΎΡ€ΠΈΡ‚ΡŒ ΠΎ Π·Π²ΡƒΠΊΠΎΠ²ΠΎΠΉ ΠΈΠ½ΠΆΠ΅Π½Π΅Ρ€ΠΈΠΈ, нас ΠΎΠ±Ρ‹Ρ‡Π½ΠΎ Π½Π΅ интСрСсуСт ΠΌΠΎΡ‰Π½ΠΎΡΡ‚ΡŒ.
    НС ΡΠΏΡ€Π°ΡˆΠΈΠ²Π°ΠΉΡ‚Π΅, Ρ‡Ρ‚ΠΎ ΠΎΠ·Π½Π°Ρ‡Π°Π΅Ρ‚ усилСниС мощности.
    ΠžΡΡ‚Π°Π²ΡŒΡ‚Π΅ это Ρ‚Π΅Π»Π΅Ρ„ΠΎΠ½Π½Ρ‹ΠΌ компаниям ΠΈΠ»ΠΈ ΠΏΠ΅Ρ€Π΅Π΄Π°ΡŽΡ‰ΠΈΠΌ Π°Π½Ρ‚Π΅Π½Π½Π°ΠΌ (Π°Π½Ρ‚Π΅Π½Π½Ρ‹).
    Π’ Π°ΡƒΠ΄ΠΈΠΎΡ‚Π΅Ρ…Π½ΠΈΠΊΠ΅ усилСниС мощности Π΄Π΅ΠΉΡΡ‚Π²ΠΈΡ‚Π΅Π»ΡŒΠ½ΠΎ Π½Π΅ ΠΈΡΠΏΠΎΠ»ΡŒΠ·ΡƒΠ΅Ρ‚ΡΡ.
    Π”Π΅ΠΉΡΡ‚Π²ΠΈΡ‚Π΅Π»ΡŒΠ½ΠΎ Π»ΠΈ Π½Π°ΠΌ Π½ΡƒΠΆΠ½ΠΎ усилСниС мощности (энСргии)?
    ΠŸΡ€ΠΎΡ‡Ρ‚ΠΈΡ‚Π΅ тСкст Π²Π½ΠΈΠ·Ρƒ.

    3 Π΄Π‘ ≑ ΠΌΠΎΡ‰Π½ΠΎΡΡ‚ΡŒ Π² 2 Ρ€Π°Π·Π° большС (βˆ’3) Π΄Π‘ ≑ Π΄Π΅ΠΌΠΏΡ„ΠΈΡ€ΠΎΠ²Π°Π½ΠΈΠ΅ Π΄ΠΎ значСния 0,5
    6 Π΄Π‘ ≑ Π² 4 Ρ€Π°Π·Π° большС (βˆ’6) Π΄Π‘ ≑ Π΄Π΅ΠΌΠΏΡ„ΠΈΡ€ΠΎΠ²Π°Π½ΠΈΠ΅ Π΄ΠΎ значСния 0.25
    10 Π΄Π‘ ≑ Π² 10 Ρ€Π°Π· большС (βˆ’10) Π΄Π‘ ≑ Π΄Π΅ΠΌΠΏΡ„ΠΈΡ€ΠΎΠ²Π°Π½ΠΈΠ΅ Π΄ΠΎ значСния 0,1
    12 Π΄Π‘ ≑ Π² 16 Ρ€Π°Π· большС (βˆ’12) Π΄Π‘ ≑ Π΄Π΅ΠΌΠΏΡ„ΠΈΡ€ΠΎΠ²Π°Π½ΠΈΠ΅ Π΄ΠΎ значСния 0,0625
    20 Π΄Π‘ ≑ Π² 100 Ρ€Π°Π· большС мощности (βˆ’20) Π΄Π‘ ≑ Π΄Π΅ΠΌΠΏΡ„ΠΈΡ€ΠΎΠ²Π°Π½ΠΈΠ΅ Π½Π° Π·Π½Π°Ρ‡Π΅Π½ΠΈΠ΅ 0,01

    Π˜ΡΠΏΠΎΠ»ΡŒΠ·ΡƒΡ ΠΌΠΎΡ‰Π½ΠΎΡΡ‚ΡŒ, ΠΌΡ‹ ΠΏΠΎΠ»ΡƒΡ‡Π°Π΅ΠΌ: Π£Ρ€ΠΎΠ²Π΅Π½ΡŒ Π² Π΄Π‘: L = 10 Γ— log (коэффициСнт мощности)

    3 Π΄Π‘ = удвоСнная ΠΌΠΎΡ‰Π½ΠΎΡΡ‚ΡŒ
    6 Π΄Π‘ = Π² Ρ‡Π΅Ρ‚Ρ‹Ρ€Π΅ Ρ€Π°Π·Π° большС мощности
    10 Π΄Π‘ = Π² Π΄Π΅ΡΡΡ‚ΡŒ Ρ€Π°Π· большС мощности
    20 Π΄Π‘ = Π² сто Ρ€Π°Π· большС мощности

    Если Π²Ρ‹ ΠΈΡ‰Π΅Ρ‚Π΅ коэффициСнт усилСния, учитывая Π·Π½Π°Ρ‡Π΅Π½ΠΈΠ΅ Π² Π΄Π‘,
    Π·Π°Ρ‚Π΅ΠΌ Π·Π°Ρ…ΠΎΠ΄ΠΈΠΌ Π² ΠΏΡ€ΠΎΠ³Ρ€Π°ΠΌΠΌΡƒ расчСта Π΄Π‘

    УсилСниС (усилСниС) ΠΈ Π΄Π΅ΠΌΠΏΡ„ΠΈΡ€ΠΎΠ²Π°Π½ΠΈΠ΅ (потСря)

    Π§Ρ‚ΠΎΠ±Ρ‹ ΠΈΡΠΏΠΎΠ»ΡŒΠ·ΠΎΠ²Π°Ρ‚ΡŒ ΠΊΠ°Π»ΡŒΠΊΡƒΠ»ΡΡ‚ΠΎΡ€, просто Π²Π²Π΅Π΄ΠΈΡ‚Π΅ Π·Π½Π°Ρ‡Π΅Π½ΠΈΠ΅.
    ΠšΠ°Π»ΡŒΠΊΡƒΠ»ΡΡ‚ΠΎΡ€ Ρ€Π°Π±ΠΎΡ‚Π°Π΅Ρ‚ Π² ΠΎΠ±ΠΎΠΈΡ… направлСниях Π·Π½Π°ΠΊΠ° ↔ .

    Π’ Π·Π²ΡƒΠΊΠΎΠ²ΠΎΠΉ Ρ‚Π΅Ρ…Π½ΠΈΠΊΠ΅ ΡΠ»Π΅Π΄ΡƒΡŽΡ‰Π΅Π΅ «усилСниС мощности ΠΈΠ»ΠΈ энСргии» довольно Π½Π΅ΠΎΠ±Ρ‹Ρ‡Π½ΠΎ.

    НапряТСниС / Π΄Π°Π²Π»Π΅Π½ΠΈΠ΅
    коэффициСнт усилСния
    1
    1,414
    = √2
    2
    3,16
    = √10
    4
    10
    20
    40
    100
    1000
    Π£Π²Π΅Π»ΠΈΡ‡Π΅Π½ΠΈΠ΅ x Π΄Π‘ 0 3 6 10 12 20 26 32 40 60

    ΠœΠΎΡ‰Π½ΠΎΡΡ‚ΡŒ / ΠΈΠ½Ρ‚Π΅Π½ΡΠΈΠ²Π½ΠΎΡΡ‚ΡŒ
    коэффициСнт усилСния
    1
    1.414
    = √2
    2
    3,16
    = √10
    4
    10
    20
    40
    100
    1000
    Π£Π²Π΅Π»ΠΈΡ‡Π΅Π½ΠΈΠ΅ y Π΄Π‘ 0 1,5 3 5 6 10 13 16 20 30


    На частотС отсСчки f c ΠΏΠ°Π΄Π΅Π½ΠΈΠ΅ напряТСния всСгда ΠΏΠ°Π΄Π°Π΅Ρ‚ Π΄ΠΎ значСния
    1 / √2 = 0.7071 (70,7%), Π° ΡƒΡ€ΠΎΠ²Π΅Π½ΡŒ напряТСния L дСмпфируСтся Π΄ΠΎ 20 Γ— log 10 (1 / √2) = (-) 3,0103 Π΄Π‘.

    На частотС срСза f c ΡƒΠΏΠ°Π²ΡˆΠ°Ρ ΠΌΠΎΡ‰Π½ΠΎΡΡ‚ΡŒ всСгда ΠΏΠ°Π΄Π°Π΅Ρ‚ Π΄ΠΎ значСния
    1/2 = 0,5 (50%), Π° ΡƒΡ€ΠΎΠ²Π΅Π½ΡŒ мощности L дСмпфируСтся Π΄ΠΎ 10 Γ— log 10 (Β½) = (-) 3,0103 Π΄Π‘.


    Π’Ρ‹Ρ€Π°ΠΆΠ΅Π½Π½ΠΎΠ΅ усилСниС напряТСния Π² Π΄Π‘ (усилСниС напряТСния) Π½Π° частотС срСза f c Ρ€Π°Π²Π½ΠΎ
    20 Γ— log 10 (1 / √2) = (-) 3.0103 Π΄Π‘ мСньшС максимального усилСния ΠΏΠΎ Π½Π°ΠΏΡ€ΡΠΆΠ΅Π½ΠΈΡŽ.

    Π’Ρ‹Ρ€Π°ΠΆΠ΅Π½Π½ΠΎΠ΅ усилСниС мощности Π² Π΄Π‘ (усилСниС мощности) Π½Π° частотС срСза f c Ρ€Π°Π²Π½ΠΎ
    10 Γ— log 10 (Β½) = (-) Π½Π° 3,0103 Π΄Π‘ мСньшС максимального усилСния мощности.

    Π§Ρ‚ΠΎΠ±Ρ‹ ΠΈΡΠΏΠΎΠ»ΡŒΠ·ΠΎΠ²Π°Ρ‚ΡŒ ΠΊΠ°Π»ΡŒΠΊΡƒΠ»ΡΡ‚ΠΎΡ€, просто Π²Π²Π΅Π΄ΠΈΡ‚Π΅ Π·Π½Π°Ρ‡Π΅Π½ΠΈΠ΅.
    ΠšΠ°Π»ΡŒΠΊΡƒΠ»ΡΡ‚ΠΎΡ€ Ρ€Π°Π±ΠΎΡ‚Π°Π΅Ρ‚ Π² ΠΎΠ±ΠΎΠΈΡ… направлСниях Π·Π½Π°ΠΊΠ° ↔ .

    НапряТСниС всСгда указываСтся ΠΊΠ°ΠΊ срСднСквадратичноС Π·Π½Π°Ρ‡Π΅Π½ΠΈΠ΅, Π½ΠΎ это Π½Π΅ относится ΠΊ элСктроэнСргии.

    Π˜ΠΌΠ΅Π΅Ρ‚ΡΡ Ρ‚Π°ΠΊΠΆΠ΅ эталонная ΠΌΠΎΡ‰Π½ΠΎΡΡ‚ΡŒ P 0 = 1 ΠΌΠΈΠ»Π»ΠΈΠ²Π°Ρ‚Ρ‚ ΠΈΠ»ΠΈ 0,001 Π’Ρ‚ ≑ 0 Π΄Π‘ ΠΌ

    Π£Ρ€ΠΎΠ²Π΅Π½ΡŒ Π² психоакустикС ΠΊΠ°ΠΊ ΡΡƒΠ±ΡŠΠ΅ΠΊΡ‚ΠΈΠ²Π½ΠΎ воспринимаСмая Π³Ρ€ΠΎΠΌΠΊΠΎΡΡ‚ΡŒ (Π³Ρ€ΠΎΠΌΠΊΠΎΡΡ‚ΡŒ)
    Π£ΠΊΠ°Π·Π°Π½ΠΎ смутноС чСловСчСскоС ΠΎΡ‰ΡƒΡ‰Π΅Π½ΠΈΠ΅ Π΄Π²ΠΎΠΉΠ½ΠΎΠΉ громкости (громкости)
    ΠΏΡ€ΠΈΠΌΠ΅Ρ€Π½ΠΎ ΠΎΡ‚ 6 Π΄ΠΎ 10 Π΄Π‘. Π­Ρ‚ΠΎ Π»ΠΈΡ‡Π½ΠΎΠ΅ чувство Π½Π΅ являСтся Ρ‚ΠΎΡ‡Π½ΠΎ ΠΈΠ·ΠΌΠ΅Ρ€ΠΈΠΌΠΎΠΉ Π²Π΅Π»ΠΈΡ‡ΠΈΠ½ΠΎΠΉ.


    ΠŸΡ€Π΅ΠΎΠ±Ρ€Π°Π·ΠΎΠ²Π°Π½ΠΈΠ΅ : УсилСниС G , ΠšΠΎΡΡ„Ρ„ΠΈΡ†ΠΈΠ΅Π½Ρ‚ напряТСния A Π’ ΠΈ ΠšΠΎΡΡ„Ρ„ΠΈΡ†ΠΈΠ΅Π½Ρ‚ мощности

    0 A

    1

    0 A

    1

    ΠšΠΎΡΡ„Ρ„ΠΈΡ†ΠΈΠ΅Π½Ρ‚ усилСния ΠΏΠΎ Π½Π°ΠΏΡ€ΡΠΆΠ΅Π½ΠΈΡŽ Π² Π΄Π‘


    ΠšΠΎΡΡ„Ρ„ΠΈΡ†ΠΈΠ΅Π½Ρ‚ усилСния ΠΏΠΎ мощности Π² Π΄Π‘

    ΠšΠΎΡΡ„Ρ„ΠΈΡ†ΠΈΠ΅Π½Ρ‚ усилСния = коэффициСнт усилСния (напряТСниС)
    ΠšΠΎΡΡ„Ρ„ΠΈΡ†ΠΈΠ΅Π½Ρ‚ мощности = коэффициСнт усилСния (ΠΌΠΎΡ‰Π½ΠΎΡΡ‚ΡŒ)
    Π’ 1 = Π’ Π² ΠΈ Π’ 2 = Π’ ΠΈΠ· .
    Π’ 2 > Π’ 1 ΠΈΠ»ΠΈ Π’ Π²Π½Π΅ > Π’ Π² ΠΎΠ·Π½Π°Ρ‡Π°Π΅Ρ‚ усилСниС . Π—Π½Π°Ρ‡Π΅Π½ΠΈΠ΅ Π΄Π‘ ΠΏΠΎΠ»ΠΎΠΆΠΈΡ‚Π΅Π»ΡŒΠ½ΠΎΠ΅. (+)
    V 2 < V 1 ΠΈΠ»ΠΈ V out < V in ΠΎΠ·Π½Π°Ρ‡Π°Π΅Ρ‚ дСмпфирования. Π—Π½Π°Ρ‡Π΅Π½ΠΈΠ΅ Π΄Π‘ ΠΎΡ‚Ρ€ΠΈΡ†Π°Ρ‚Π΅Π»ΡŒΠ½ΠΎΠ΅. (-)
    V 2 / V 1 ΠΈΠ»ΠΈ V out / V in ΠΎΠ·Π½Π°Ρ‡Π°Π΅Ρ‚ ΡΠΎΠΎΡ‚Π½ΠΎΡˆΠ΅Π½ΠΈΠ΅ .УсилСниС ΠΈΠ»ΠΈ Π·Π°Ρ‚ΡƒΡ…Π°Π½ΠΈΠ΅ Π² Π΄Π‘ составляСт:
    L = 20 Γ— log (ΠΎΡ‚Π½ΠΎΡˆΠ΅Π½ΠΈΠ΅ напряТСний Π’ 2 / Π’ 1 ) Π² Π΄Π‘. Π’ 1 = Π’ Π² являСтся справочным.


    Π‘Π»ΠΎΠ²ΠΎ Β«ΡƒΡΠΈΠ»ΠΈΡ‚Π΅Π»ΡŒ мощности» ΠΈΡΠΏΠΎΠ»ΡŒΠ·ΡƒΠ΅Ρ‚ΡΡ Π½Π΅ΠΏΡ€Π°Π²ΠΈΠ»ΡŒΠ½ΠΎ, особСнно Π² Π°ΡƒΠ΄ΠΈΠΎΡ‚Π΅Ρ…Π½ΠΈΠΊΠ΅.
    НапряТСниС ΠΈ Ρ‚ΠΎΠΊ ΠΌΠΎΠΆΠ½ΠΎ ΡƒΡΠΈΠ»ΠΈΡ‚ΡŒ. Π‘Ρ‚Ρ€Π°Π½Π½Ρ‹ΠΉ Ρ‚Π΅Ρ€ΠΌΠΈΠ½ Β«ΡƒΡΠΈΠ»ΠΈΡ‚Π΅Π»ΡŒ мощности» ΠΈΠΌΠ΅Π΅Ρ‚
    стали ΠΏΠΎΠ½ΠΈΠΌΠ°Ρ‚ΡŒΡΡ ΠΊΠ°ΠΊ ΡƒΡΠΈΠ»ΠΈΡ‚Π΅Π»ΡŒ, ΠΏΡ€Π΅Π΄Π½Π°Π·Π½Π°Ρ‡Π΅Π½Π½Ρ‹ΠΉ для управлСния Π½Π°Π³Ρ€ΡƒΠ·ΠΊΠΎΠΉ, Ρ‚Π°ΠΊΠΎΠΉ ΠΊΠ°ΠΊ
    ΠΊΠ°ΠΊ Π³Ρ€ΠΎΠΌΠΊΠΎΠ³ΠΎΠ²ΠΎΡ€ΠΈΡ‚Π΅Π»ΡŒ.
    ΠœΡ‹ Π½Π°Π·Ρ‹Π²Π°Π΅ΠΌ ΠΏΡ€ΠΎΠΈΠ·Π²Π΅Π΄Π΅Π½ΠΈΠ΅ усилСния ΠΏΠΎ Ρ‚ΠΎΠΊΡƒ ΠΈ ΡƒΡΠΈΠ»Π΅Π½ΠΈΡŽ ΠΏΠΎ Π½Π°ΠΏΡ€ΡΠΆΠ΅Π½ΠΈΡŽ «усилСниСм мощности».



    УсилСниС мощности Π² Π΄Π΅Ρ†ΠΈΠ±Π΅Π»Π°Ρ…

    ΠŸΡ€ΠΈ ΠΏΡ€ΠΎΠ΅ΠΊΡ‚ΠΈΡ€ΠΎΠ²Π°Π½ΠΈΠΈ ΠΈΠ»ΠΈ Ρ€Π°Π±ΠΎΡ‚Π΅ со схСмами усилитСля ΠΈ Ρ„ΠΈΠ»ΡŒΡ‚Ρ€Π° Π½Π΅ΠΊΠΎΡ‚ΠΎΡ€Ρ‹Π΅ числа, ΠΈΡΠΏΠΎΠ»ΡŒΠ·ΡƒΠ΅ΠΌΡ‹Π΅ Π² расчСтах, ΠΌΠΎΠ³ΡƒΡ‚ Π±Ρ‹Ρ‚ΡŒ ΠΎΡ‡Π΅Π½ΡŒ большими ΠΈΠ»ΠΈ ΠΎΡ‡Π΅Π½ΡŒ малСнькими.НапримСр, Ссли ΠΌΡ‹ каскадируСм Π΄Π²Π° каскада усилитСля вмСстС с коэффициСнтом усилСния мощности ΠΈΠ»ΠΈ напряТСния, скаТСм, 20 ΠΈ 36 соотвСтствСнно, Ρ‚ΠΎ ΠΎΠ±Ρ‰Π΅Π΅ усилСниС Π±ΡƒΠ΄Π΅Ρ‚ 720 (20 * 36).

    Π’ΠΎΡ‡Π½ΠΎ Ρ‚Π°ΠΊ ΠΆΠ΅, Ссли ΠΌΡ‹ ΠΏΠΎΠ΄ΠΊΠ»ΡŽΡ‡ΠΈΠΌ каскад ΠΊ цСпям RC-Ρ„ΠΈΠ»ΡŒΡ‚Ρ€ΠΎΠ² ΠΏΠ΅Ρ€Π²ΠΎΠ³ΠΎ порядка с затуханиями 0,7071 каТдая, ΠΎΠ±Ρ‰Π΅Π΅ Π·Π°Ρ‚ΡƒΡ…Π°Π½ΠΈΠ΅ Π±ΡƒΠ΄Π΅Ρ‚ 0,5 (0,7071 * 0,7071). Помня, ΠΊΠΎΠ½Π΅Ρ‡Π½ΠΎ, Ρ‡Ρ‚ΠΎ Ссли Π²Ρ‹Ρ…ΠΎΠ΄ схСмы ΠΏΠΎΠ»ΠΎΠΆΠΈΡ‚Π΅Π»ΡŒΠ½Ρ‹ΠΉ, Ρ‚ΠΎ ΠΎΠ½ ΠΏΡ€ΠΎΠΈΠ·Π²ΠΎΠ΄ΠΈΡ‚ усилСниС ΠΈΠ»ΠΈ усилСниС, Π° Ссли Π΅Π³ΠΎ Π²Ρ‹Ρ…ΠΎΠ΄ ΠΎΡ‚Ρ€ΠΈΡ†Π°Ρ‚Π΅Π»ΡŒΠ½Ρ‹ΠΉ, Ρ‚ΠΎ ΠΎΠ½ ΠΏΡ€ΠΎΠΈΠ·Π²ΠΎΠ΄ΠΈΡ‚ Π·Π°Ρ‚ΡƒΡ…Π°Π½ΠΈΠ΅ ΠΈΠ»ΠΈ ΠΏΠΎΡ‚Π΅Ρ€ΠΈ.

    ΠŸΡ€ΠΈ Π°Π½Π°Π»ΠΈΠ·Π΅ схСм Π² частотной области ΡƒΠ΄ΠΎΠ±Π½Π΅Π΅ ΡΡ€Π°Π²Π½ΠΈΠ²Π°Ρ‚ΡŒ ΠΎΡ‚Π½ΠΎΡˆΠ΅Π½ΠΈΠ΅ Π°ΠΌΠΏΠ»ΠΈΡ‚ΡƒΠ΄ Π²Ρ‹Ρ…ΠΎΠ΄Π½Ρ‹Ρ… ΠΈ Π²Ρ…ΠΎΠ΄Π½Ρ‹Ρ… Π·Π½Π°Ρ‡Π΅Π½ΠΈΠΉ Π² логарифмичСском ΠΌΠ°ΡΡˆΡ‚Π°Π±Π΅, Π° Π½Π΅ Π² Π»ΠΈΠ½Π΅ΠΉΠ½ΠΎΠΌ ΠΌΠ°ΡΡˆΡ‚Π°Π±Π΅.Π’Π°ΠΊΠΈΠΌ ΠΎΠ±Ρ€Π°Π·ΠΎΠΌ, Ссли ΠΌΡ‹ ΠΈΡΠΏΠΎΠ»ΡŒΠ·ΡƒΠ΅ΠΌ логарифмичСскоС ΠΎΡ‚Π½ΠΎΡˆΠ΅Π½ΠΈΠ΅ Π΄Π²ΡƒΡ… Π²Π΅Π»ΠΈΡ‡ΠΈΠ½, P 1 ΠΈ P 2 , ΠΌΡ‹ ΠΏΠΎΠ»ΡƒΡ‡ΠΈΠΌ Π½ΠΎΠ²ΡƒΡŽ Π²Π΅Π»ΠΈΡ‡ΠΈΠ½Ρƒ ΠΈΠ»ΠΈ ΡƒΡ€ΠΎΠ²Π΅Π½ΡŒ, ΠΊΠΎΡ‚ΠΎΡ€Ρ‹ΠΉ ΠΌΠΎΠΆΠ΅Ρ‚ Π±Ρ‹Ρ‚ΡŒ прСдставлСн с использованиСм Π΄Π΅Ρ†ΠΈΠ±Π΅Π» .

    Π’ ΠΎΡ‚Π»ΠΈΡ‡ΠΈΠ΅ ΠΎΡ‚ напряТСния ΠΈΠ»ΠΈ Ρ‚ΠΎΠΊΠ°, ΠΊΠΎΡ‚ΠΎΡ€Ρ‹Π΅ ΠΈΠ·ΠΌΠ΅Ρ€ΡΡŽΡ‚ΡΡ Π² Π²ΠΎΠ»ΡŒΡ‚Π°Ρ… ΠΈ ​​ампСрах соотвСтствСнно, Π΄Π΅Ρ†ΠΈΠ±Π΅Π» , ΠΈΠ»ΠΈ просто Π΄Π‘ для краткости, прСдставляСт собой просто ΠΎΡ‚Π½ΠΎΡˆΠ΅Π½ΠΈΠ΅ Π΄Π²ΡƒΡ… Π·Π½Π°Ρ‡Π΅Π½ΠΈΠΉ, Π½Π° самом Π΄Π΅Π»Π΅ ΠΎΡ‚Π½ΠΎΡˆΠ΅Π½ΠΈΠ΅ ΠΎΠ΄Π½ΠΎΠ³ΠΎ значСния ΠΊ Π΄Ρ€ΡƒΠ³ΠΎΠΌΡƒ извСстному ΠΈΠ»ΠΈ фиксированному Π·Π½Π°Ρ‡Π΅Π½ΠΈΡŽ, поэтому ΡΠ»Π΅Π΄ΠΎΠ²Π°Ρ‚Π΅Π»ΡŒΠ½ΠΎ, Π΄Π΅Ρ†ΠΈΠ±Π΅Π» являСтся Π±Π΅Π·Ρ€Π°Π·ΠΌΠ΅Ρ€Π½ΠΎΠΉ Π²Π΅Π»ΠΈΡ‡ΠΈΠ½ΠΎΠΉ, Π½ΠΎ ΠΈΠΌΠ΅Π΅Ρ‚ Π² качСствС Π΅Π΄ΠΈΠ½ΠΈΡ† измСрСния Β«Π‘Π΅Π»Β» Π² Ρ‡Π΅ΡΡ‚ΡŒ изобрСтатСля Ρ‚Π΅Π»Π΅Ρ„ΠΎΠ½Π° АлСксандра ГрэхСма Π‘Π΅Π»Π»Π°.

    ΠžΡ‚Π½ΠΎΡˆΠ΅Π½ΠΈΠ΅ Π»ΡŽΠ±Ρ‹Ρ… Π΄Π²ΡƒΡ… Π·Π½Π°Ρ‡Π΅Π½ΠΈΠΉ, Π³Π΄Π΅ ΠΎΠ΄Π½ΠΎ фиксированноС ΠΈΠ»ΠΈ извСстноС ΠΈ ΠΎΠ΄Π½ΠΎΠ³ΠΎ ΠΈ Ρ‚ΠΎΠ³ΠΎ ΠΆΠ΅ количСства ΠΈΠ»ΠΈ Π΅Π΄ΠΈΠ½ΠΈΡ†, Π±ΡƒΠ΄ΡŒ Ρ‚ΠΎ ΠΌΠΎΡ‰Π½ΠΎΡΡ‚ΡŒ, напряТСниС ΠΈΠ»ΠΈ Ρ‚ΠΎΠΊ, ΠΌΠΎΠΆΠ΅Ρ‚ Π±Ρ‹Ρ‚ΡŒ прСдставлСно Π² Π΄Π΅Ρ†ΠΈΠ±Π΅Π»Π°Ρ… (Π΄Π‘), Π³Π΄Π΅ Β«Π΄Π΅Ρ†ΠΈΒ» ΠΎΠ·Π½Π°Ρ‡Π°Π΅Ρ‚ ΠΎΠ΄Π½Ρƒ Π΄Π΅ΡΡΡ‚ΡƒΡŽ (1/10) ΠΎΡ‚ Π‘Π΅Π». ΠžΡ‡Π΅Π²ΠΈΠ΄Π½ΠΎ, Ρ‡Ρ‚ΠΎ Π² расчСтС Π½Π° ΠΎΠ΄ΠΈΠ½ Π±Π΅Π» приходится 10 Π΄Π΅Ρ†ΠΈΠ±Π΅Π» (10 Π΄Π‘), ΠΈΠ»ΠΈ 1 Π±Π΅Π» = 10 Π΄Π΅Ρ†ΠΈΠ±Π΅Π».

    Π”Π΅Ρ†ΠΈΠ±Π΅Π» ΠΎΠ±Ρ‹Ρ‡Π½ΠΎ ΠΈΡΠΏΠΎΠ»ΡŒΠ·ΡƒΠ΅Ρ‚ΡΡ для отобраТСния коэффициСнта измСнСния мощности (увСличСния ΠΈΠ»ΠΈ ΡƒΠΌΠ΅Π½ΡŒΡˆΠ΅Π½ΠΈΡ) ΠΈ ΠΎΠ±Ρ‹Ρ‡Π½ΠΎ опрСдСляСтся ΠΊΠ°ΠΊ Π·Π½Π°Ρ‡Π΅Π½ΠΈΠ΅, ΠΊΠΎΡ‚ΠΎΡ€ΠΎΠ΅ Π² Π΄Π΅ΡΡΡ‚ΡŒ Ρ€Π°Π· ΠΏΡ€Π΅Π²Ρ‹ΡˆΠ°Π΅Ρ‚ Π»ΠΎΠ³Π°Ρ€ΠΈΡ„ΠΌ ΠΏΠΎ основанию 10 Π΄Π²ΡƒΡ… ΡƒΡ€ΠΎΠ²Π½Π΅ΠΉ мощности.Π’Π°ΠΊ, Π½Π°ΠΏΡ€ΠΈΠΌΠ΅Ρ€, 1 Π’Ρ‚ ΠΊ 10 Π’Ρ‚ — это Ρ‚ΠΎ ΠΆΠ΅ ΡΠΎΠΎΡ‚Π½ΠΎΡˆΠ΅Π½ΠΈΠ΅ мощности, Ρ‡Ρ‚ΠΎ ΠΈ 10 Π’Ρ‚ ΠΊ 100 Π’Ρ‚, Ρ‚ΠΎ Π΅ΡΡ‚ΡŒ 10: 1, поэтому, хотя Π΅ΡΡ‚ΡŒ большая Ρ€Π°Π·Π½ΠΈΡ†Π° Π² количСствС Π²Π°Ρ‚Ρ‚, 9 ΠΏΠΎ ΡΡ€Π°Π²Π½Π΅Π½ΠΈΡŽ с 90, ΡΠΎΠΎΡ‚Π½ΠΎΡˆΠ΅Π½ΠΈΠ΅ Π΄Π΅Ρ†ΠΈΠ±Π΅Π» Π±ΡƒΠ΄Π΅Ρ‚ Ρ‚ΠΎΡ‡Π½ΠΎ Ρ‚Π°ΠΊ ΠΆΠ΅.

    НадСюсь, Ρ‚ΠΎΠ³Π΄Π° ΠΌΡ‹ смоТСм ΡƒΠ²ΠΈΠ΄Π΅Ρ‚ΡŒ, Ρ‡Ρ‚ΠΎ Π·Π½Π°Ρ‡Π΅Π½ΠΈΠ΅ Π² Π΄Π΅Ρ†ΠΈΠ±Π΅Π»Π°Ρ… (Π΄Π‘) — это ΠΎΡ‚Π½ΠΎΡˆΠ΅Π½ΠΈΠ΅, ΠΈΡΠΏΠΎΠ»ΡŒΠ·ΡƒΠ΅ΠΌΠΎΠ΅ для сравнСния ΠΈ расчСта ΡƒΡ€ΠΎΠ²Π½Π΅ΠΉ измСнСния мощности, Π° Π½Π΅ сама ΠΌΠΎΡ‰Π½ΠΎΡΡ‚ΡŒ. Π˜Ρ‚Π°ΠΊ, Ссли Ρƒ нас Π΅ΡΡ‚ΡŒ Π΄Π²Π΅ Π²Π΅Π»ΠΈΡ‡ΠΈΠ½Ρ‹ мощности, Π½Π°ΠΏΡ€ΠΈΠΌΠ΅Ρ€: P 1 ΠΈ P 2 , ΡΠΎΠΎΡ‚Π½ΠΎΡˆΠ΅Π½ΠΈΠ΅ этих Π΄Π²ΡƒΡ… Π²Π΅Π»ΠΈΡ‡ΠΈΠ½ Π±ΡƒΠ΄Π΅Ρ‚ прСдставлСно ΡƒΡ€Π°Π²Π½Π΅Π½ΠΈΠ΅ΠΌ:

    Π΄Π‘ = 10log 10 [P 2 / P 1 ]

    Π“Π΄Π΅ P 1 прСдставляСт Π²Ρ…ΠΎΠ΄Π½ΡƒΡŽ ΠΌΠΎΡ‰Π½ΠΎΡΡ‚ΡŒ, Π° P 2 прСдставляСт Π²Ρ‹Ρ…ΠΎΠ΄Π½ΡƒΡŽ ΠΌΠΎΡ‰Π½ΠΎΡΡ‚ΡŒ (P OUT / P IN ).

    ΠŸΠΎΡΠΊΠΎΠ»ΡŒΠΊΡƒ Π΄Π΅Ρ†ΠΈΠ±Π΅Π» прСдставляСт собой логарифмичСскоС ΠΈΠ·ΠΌΠ΅Π½Π΅Π½ΠΈΠ΅ ΠΏΠΎ основанию 10 для Π΄Π²ΡƒΡ… ΡƒΡ€ΠΎΠ²Π½Π΅ΠΉ мощности, ΠΌΡ‹ ΠΌΠΎΠΆΠ΅ΠΌ Ρ€Π°ΡΡˆΠΈΡ€ΠΈΡ‚ΡŒ это ΡƒΡ€Π°Π²Π½Π΅Π½ΠΈΠ΅, ΠΈΡΠΏΠΎΠ»ΡŒΠ·ΡƒΡ Π°Π½Ρ‚ΠΈΠ»ΠΎΠ³Π°Ρ€ΠΈΡ„ΠΌΡ‹, Ρ‡Ρ‚ΠΎΠ±Ρ‹ ΠΏΠΎΠΊΠ°Π·Π°Ρ‚ΡŒ, насколько Π½Π° самом Π΄Π΅Π»Π΅ ΠΈΠ·ΠΌΠ΅Π½Π΅Π½ΠΈΠ΅ ΠΎΠ΄Π½ΠΎΠ³ΠΎ Π΄Π΅Ρ†ΠΈΠ±Π΅Π»Π° (1 Π΄Π‘).

    Π΄Π‘ = 10log 10 [P 2 / P 1 ]

    Если P 2 / P 1 Ρ€Π°Π²Π½ΠΎ 1, Ρ‚ΠΎ Π΅ΡΡ‚ΡŒ P 1 = P 2 , Ρ‚ΠΎΠ³Π΄Π°:

    Π΄Π‘ = 10 log 10 [1] = log 10 [1/10] = log 10 [0.1] = Π°Π½Ρ‚ΠΈΠ»ΠΎΠ³ [0,1]

    Π’Π°ΠΊΠΈΠΌ ΠΎΠ±Ρ€Π°Π·ΠΎΠΌ, ΠΈΠ·ΠΌΠ΅Π½Π΅Π½ΠΈΠ΅ значСния Π² Π΄Π‘ Ρ€Π°Π²Π½ΠΎ: 10 0,1 = 1,259

    ΠžΡ‡Π΅Π²ΠΈΠ΄Π½ΠΎ, Ρ‡Ρ‚ΠΎ логарифмичСскоС ΠΈΠ·ΠΌΠ΅Π½Π΅Π½ΠΈΠ΅ Π΄Π²ΡƒΡ… стСпСнСй ΠΈΠΌΠ΅Π΅Ρ‚ ΠΎΡ‚Π½ΠΎΡˆΠ΅Π½ΠΈΠ΅ 1,259, Ρ‡Ρ‚ΠΎ ΠΎΠ·Π½Π°Ρ‡Π°Π΅Ρ‚, Ρ‡Ρ‚ΠΎ ΠΈΠ·ΠΌΠ΅Π½Π΅Π½ΠΈΠ΅ Π½Π° 1 Π΄Π‘ прСдставляСт собой ΡƒΠ²Π΅Π»ΠΈΡ‡Π΅Π½ΠΈΠ΅ (ΠΈΠ»ΠΈ ΡƒΠΌΠ΅Π½ΡŒΡˆΠ΅Π½ΠΈΠ΅) мощности Π½Π° 25,9% (ΠΈΠ»ΠΈ ΠΎΠΊΡ€ΡƒΠ³Π»Π΅Π½Π½ΠΎΠ΅ Π·Π½Π°Ρ‡Π΅Π½ΠΈΠ΅ Π½Π° 26%).

    Π˜Ρ‚Π°ΠΊ, Ссли схСма ΠΈΠ»ΠΈ систСма ΠΈΠΌΠ΅Π΅Ρ‚ коэффициСнт усилСния, скаТСм, 5 (7 Π΄Π‘), ΠΈ ΠΎΠ½ увСличиваСтся Π½Π° 26%, Ρ‚ΠΎ Π½ΠΎΠ²ΠΎΠ΅ ΠΎΡ‚Π½ΠΎΡˆΠ΅Π½ΠΈΠ΅ мощности схСмы Π±ΡƒΠ΄Π΅Ρ‚: 5 * 1,26 = 6,3, поэтому 10log 10 (6,3) = 8 Π΄Π‘. Π£Π²Π΅Π»ΠΈΡ‡Π΅Π½ΠΈΠ΅ усилСния Π½Π° + 1 Π΄Π‘, Ρ‡Ρ‚ΠΎ Π΅Ρ‰Π΅ Ρ€Π°Π· Π΄ΠΎΠΊΠ°Π·Ρ‹Π²Π°Π΅Ρ‚, Ρ‡Ρ‚ΠΎ ΠΈΠ·ΠΌΠ΅Π½Π΅Π½ΠΈΠ΅ Π½Π° + 1 Π΄Π‘ прСдставляСт собой логарифмичСскоС ΡƒΠ²Π΅Π»ΠΈΡ‡Π΅Π½ΠΈΠ΅ мощности Π½Π° 26%, Π° Π½Π΅ Π»ΠΈΠ½Π΅ΠΉΠ½ΠΎΠ΅ ΠΈΠ·ΠΌΠ΅Π½Π΅Π½ΠΈΠ΅.

    Π”Π΅Ρ†ΠΈΠ±Π΅Π» ΠŸΡ€ΠΈΠΌΠ΅Ρ€ No1

    ΠΡƒΠ΄ΠΈΠΎΡƒΡΠΈΠ»ΠΈΡ‚Π΅Π»ΡŒ Π²Ρ‹Π΄Π°Π΅Ρ‚ 100 Π’Ρ‚ Π½Π° Π½Π°Π³Ρ€ΡƒΠ·ΠΊΡƒ Π΄ΠΈΠ½Π°ΠΌΠΈΠΊΠ° 8 Ом ΠΏΡ€ΠΈ ΠΏΠΎΠ΄Π°Ρ‡Π΅ Π²Ρ…ΠΎΠ΄Π½ΠΎΠ³ΠΎ сигнала 100 ΠΌΠ’Ρ‚. РассчитайтС коэффициСнт усилСния усилитСля Π² Π΄Π΅Ρ†ΠΈΠ±Π΅Π»Π°Ρ….

    ΠœΡ‹ ΠΌΠΎΠΆΠ΅ΠΌ Π²Ρ‹Ρ€Π°Π·ΠΈΡ‚ΡŒ усилСниС мощности усилитСля Π² Π΄Π΅Ρ†ΠΈΠ±Π΅Π»Π°Ρ… нСзависимо ΠΎΡ‚ Π΅Π³ΠΎ Π²Ρ…ΠΎΠ΄Π½Ρ‹Ρ… ΠΈΠ»ΠΈ Π²Ρ‹Ρ…ΠΎΠ΄Π½Ρ‹Ρ… Π·Π½Π°Ρ‡Π΅Π½ΠΈΠΉ, ΠΏΠΎΡΠΊΠΎΠ»ΡŒΠΊΡƒ ΡƒΡΠΈΠ»ΠΈΡ‚Π΅Π»ΡŒ, ΠΎΠ±Π΅ΡΠΏΠ΅Ρ‡ΠΈΠ²Π°ΡŽΡ‰ΠΈΠΉ Π²Ρ‹Ρ…ΠΎΠ΄Π½ΡƒΡŽ ΠΌΠΎΡ‰Π½ΠΎΡΡ‚ΡŒ 40 Π’Ρ‚ для Π²Ρ…ΠΎΠ΄Π½ΠΎΠΉ 40 ΠΌΠ’Ρ‚, Ρ‚Π°ΠΊΠΆΠ΅ Π±ΡƒΠ΄Π΅Ρ‚ ΠΈΠΌΠ΅Ρ‚ΡŒ ΠΌΠΎΡ‰Π½ΠΎΡΡ‚ΡŒ усилСниС 30 Π΄Π‘ ΠΈ Ρ‚Π°ΠΊ Π΄Π°Π»Π΅Π΅.

    ΠœΡ‹ Ρ‚Π°ΠΊΠΆΠ΅ ΠΌΠΎΠ³Π»ΠΈ Π±Ρ‹, Ссли Π±Ρ‹ Π·Π°Ρ…ΠΎΡ‚Π΅Π»ΠΈ, ΠΏΡ€Π΅ΠΎΠ±Ρ€Π°Π·ΠΎΠ²Π°Ρ‚ΡŒ это Π·Π½Π°Ρ‡Π΅Π½ΠΈΠ΅ Π΄Π΅Ρ†ΠΈΠ±Π΅Π» усилитСля ΠΎΠ±Ρ€Π°Ρ‚Π½ΠΎ Π² Π»ΠΈΠ½Π΅ΠΉΠ½ΠΎΠ΅ Π·Π½Π°Ρ‡Π΅Π½ΠΈΠ΅, сначала ΠΏΡ€Π΅ΠΎΠ±Ρ€Π°Π·ΠΎΠ²Π°Π² Π΄Π΅Ρ†ΠΈΠ±Π΅Π»Ρ‹ (Π΄Π‘) Π² Π±Π΅Π», помня, Ρ‡Ρ‚ΠΎ Π΄Π΅Ρ†ΠΈΠ±Π΅Π» составляСт 1/10 Π±Π΅Π»Π°.НапримСр:

    Π£ΡΠΈΠ»ΠΈΡ‚Π΅Π»ΡŒ Π·Π²ΡƒΠΊΠ° ΠΌΠΎΡ‰Π½ΠΎΡΡ‚ΡŒΡŽ 100 Π’Ρ‚ ΠΈΠΌΠ΅Π΅Ρ‚ коэффициСнт усилСния 30 Π΄Π‘. КакоС Π±ΡƒΠ΄Π΅Ρ‚ Π΅Π³ΠΎ максимальноС Π²Ρ…ΠΎΠ΄Π½ΠΎΠ΅ Π·Π½Π°Ρ‡Π΅Π½ΠΈΠ΅.

    Π’Π°ΠΊΠΈΠΌ ΠΎΠ±Ρ€Π°Π·ΠΎΠΌ, Ρ€Π΅Π·ΡƒΠ»ΡŒΡ‚Π°Ρ‚ составляСт 100 ΠΌΠ’Ρ‚, ΠΊΠ°ΠΊ заявлСно Π² ΠΏΡ€ΠΈΠΌΠ΅Ρ€Π΅ β„–1.

    Одним ΠΈΠ· прСимущСств использования логарифмичСского ΠΎΡ‚Π½ΠΎΡˆΠ΅Π½ΠΈΡ Π΄Π²ΡƒΡ… мощностСй ΠΏΠΎ основанию 10 являСтся Ρ‚ΠΎ, Ρ‡Ρ‚ΠΎ ΠΏΡ€ΠΈ Ρ€Π°Π±ΠΎΡ‚Π΅ с каскадом Π½Π΅ΡΠΊΠΎΠ»ΡŒΠΊΠΈΡ… каскадов усилитСля, Ρ„ΠΈΠ»ΡŒΡ‚Ρ€Π° ΠΈΠ»ΠΈ Π°Ρ‚Ρ‚Π΅Π½ΡŽΠ°Ρ‚ΠΎΡ€Π°, ΠΌΡ‹ ΠΌΠΎΠΆΠ΅ΠΌ просто ΡΠ»ΠΎΠΆΠΈΡ‚ΡŒ ΠΈΠ»ΠΈ Π²Ρ‹Ρ‡Π΅ΡΡ‚ΡŒ ΠΈΡ… значСния Π² Π΄Π΅Ρ†ΠΈΠ±Π΅Π»Π°Ρ… вмСсто умноТСния ΠΈΠ»ΠΈ дСлСния ΠΈΡ… Π»ΠΈΠ½Π΅ΠΉΠ½Ρ‹Ρ… Π·Π½Π°Ρ‡Π΅Π½ΠΈΠΉ. Π”Ρ€ΡƒΠ³ΠΈΠΌΠΈ словами, ΠΎΠ±Ρ‰Π΅Π΅ усилСниС схСмы (+ Π΄Π‘) ΠΈΠ»ΠΈ Π·Π°Ρ‚ΡƒΡ…Π°Π½ΠΈΠ΅ (-Π΄Π‘) прСдставляСт собой сумму ΠΎΡ‚Π΄Π΅Π»ΡŒΠ½Ρ‹Ρ… усилСний ΠΈ ослаблСний для всСх каскадов, ΠΏΠΎΠ΄ΠΊΠ»ΡŽΡ‡Π΅Π½Π½Ρ‹Ρ… ΠΌΠ΅ΠΆΠ΄Ρƒ Π²Ρ…ΠΎΠ΄ΠΎΠΌ ΠΈ Π²Ρ‹Ρ…ΠΎΠ΄ΠΎΠΌ.

    НапримСр, Ссли одноступСнчатый ΡƒΡΠΈΠ»ΠΈΡ‚Π΅Π»ΡŒ ΠΈΠΌΠ΅Π΅Ρ‚ коэффициСнт усилСния ΠΏΠΎ мощности 20 Π΄Π‘ ΠΈ ΠΏΠΈΡ‚Π°Π΅Ρ‚ ΠΏΠ°ΡΡΠΈΠ²Π½ΡƒΡŽ Ρ€Π΅Π·ΠΈΡΡ‚ΠΈΠ²Π½ΡƒΡŽ ΡΠ΅Ρ‚ΡŒ с ослаблСниСм 2, ΠΏΡ€Π΅ΠΆΠ΄Π΅ Ρ‡Π΅ΠΌ сигнал Π±ΡƒΠ΄Π΅Ρ‚ снова усилСн с использованиСм Π²Ρ‚ΠΎΡ€ΠΎΠ³ΠΎ каскада усилитСля с коэффициСнтом усилСния 200. Π’ΠΎΠ³Π΄Π° общая ΠΌΠΎΡ‰Π½ΠΎΡΡ‚ΡŒ коэффициСнт усилСния Ρ†Π΅ΠΏΠΈ ΠΌΠ΅ΠΆΠ΄Ρƒ Π²Ρ…ΠΎΠ΄ΠΎΠΌ ΠΈ Π²Ρ‹Ρ…ΠΎΠ΄ΠΎΠΌ Π² Π΄Π΅Ρ†ΠΈΠ±Π΅Π»Π°Ρ… Π±ΡƒΠ΄Π΅Ρ‚:

    Для пассивной схСмы Π·Π°Ρ‚ΡƒΡ…Π°Π½ΠΈΠ΅, Ρ€Π°Π²Π½ΠΎΠ΅ 2, ΠΎΠ·Π½Π°Ρ‡Π°Π΅Ρ‚, Ρ‡Ρ‚ΠΎ схСма ΠΈΠΌΠ΅Π΅Ρ‚ ΠΏΠΎΠ»ΠΎΠΆΠΈΡ‚Π΅Π»ΡŒΠ½Ρ‹ΠΉ коэффициСнт усилСния 1/2 = 0,5, Ρ‚Π°ΠΊΠΈΠΌ ΠΎΠ±Ρ€Π°Π·ΠΎΠΌ, коэффициСнт усилСния ΠΏΠΎ мощности пассивной сСкции составляСт:

    Π΄Π‘ УсилСниС = 10log 10 [0.5] = -3 Π΄Π‘ (ΠΎΠ±Ρ€Π°Ρ‚ΠΈΡ‚Π΅ Π²Π½ΠΈΠΌΠ°Π½ΠΈΠ΅ Π½Π° ΠΎΡ‚Ρ€ΠΈΡ†Π°Ρ‚Π΅Π»ΡŒΠ½ΠΎΠ΅ Π·Π½Π°Ρ‡Π΅Π½ΠΈΠ΅)

    Π£ΡΠΈΠ»ΠΈΡ‚Π΅Π»ΡŒ Π²Ρ‚ΠΎΡ€ΠΎΠ³ΠΎ каскада ΠΈΠΌΠ΅Π΅Ρ‚ коэффициСнт усилСния 200, Ρ‚Π°ΠΊΠΈΠΌ ΠΎΠ±Ρ€Π°Π·ΠΎΠΌ, коэффициСнт усилСния ΠΏΠΎ мощности Π² этой сСкции составляСт:

    Π΄Π‘ УсилСниС = 10log 10 [200] = + 23 Π΄Π‘

    Π’ΠΎΠ³Π΄Π° ΠΎΠ±Ρ‰ΠΈΠΉ коэффициСнт усилСния схСмы Π±ΡƒΠ΄Π΅Ρ‚:

    20-3 + 23 = + 40 Π΄Π‘

    ΠœΡ‹ ΠΌΠΎΠΆΠ΅ΠΌ Π΄Π²Π°ΠΆΠ΄Ρ‹ ΠΏΡ€ΠΎΠ²Π΅Ρ€ΠΈΡ‚ΡŒ наш ΠΎΡ‚Π²Π΅Ρ‚ 40 Π΄Π‘, ΡƒΠΌΠ½ΠΎΠΆΠΈΠ² ΠΈΠ½Π΄ΠΈΠ²ΠΈΠ΄ΡƒΠ°Π»ΡŒΠ½Ρ‹Π΅ коэффициСнты усилСния ΠΊΠ°ΠΆΠ΄ΠΎΠΉ ступСни ΠΎΠ±Ρ‹Ρ‡Π½Ρ‹ΠΌ способом ΡΠ»Π΅Π΄ΡƒΡŽΡ‰ΠΈΠΌ ΠΎΠ±Ρ€Π°Π·ΠΎΠΌ:

    ΠŸΡ€ΠΈΡ€ΠΎΡΡ‚ мощности 20 Π΄Π‘ Π² Π΄Π΅Ρ†ΠΈΠ±Π΅Π»Π°Ρ… Ρ€Π°Π²Π΅Π½ ΡƒΡΠΈΠ»Π΅Π½ΠΈΡŽ 100, Ρ‚Π°ΠΊ ΠΊΠ°ΠΊ 10 (20/10) = 100.Π˜Ρ‚Π°ΠΊ:

    100 x 0,5 x 200 = 10 000 (ΠΈΠ»ΠΈ Π² 10 000 Ρ€Π°Π· большС)

    ΠŸΡ€Π΅ΠΎΠ±Ρ€Π°Π·ΠΎΠ²Π°Π½ΠΈΠ΅ этого ΠΎΠ±Ρ€Π°Ρ‚Π½ΠΎ Π² Π·Π½Π°Ρ‡Π΅Π½ΠΈΠ΅ Π² Π΄Π΅Ρ†ΠΈΠ±Π΅Π»Π°Ρ… Π΄Π°Π΅Ρ‚:

    Π΄Π‘ УсилСниС = 10log 10 [10,000] = 40 Π΄Π‘

    Π’ΠΎΠ³Π΄Π° ясно Π²ΠΈΠ΄Π½ΠΎ, Ρ‡Ρ‚ΠΎ коэффициСнт усилСния 10000 Ρ€Π°Π²Π΅Π½ коэффициСнту усилСния мощности +40 Π΄Π‘, ΠΊΠ°ΠΊ ΠΏΠΎΠΊΠ°Π·Π°Π½ΠΎ Π²Ρ‹ΡˆΠ΅, ΠΈ Ρ‡Ρ‚ΠΎ ΠΌΡ‹ ΠΌΠΎΠΆΠ΅ΠΌ ΠΈΡΠΏΠΎΠ»ΡŒΠ·ΠΎΠ²Π°Ρ‚ΡŒ Π·Π½Π°Ρ‡Π΅Π½ΠΈΠ΅ Π² Π΄Π΅Ρ†ΠΈΠ±Π΅Π»Π°Ρ… для выраТСния Π±ΠΎΠ»ΡŒΡˆΠΈΡ… ΠΎΡ‚Π½ΠΎΡˆΠ΅Π½ΠΈΠΉ мощностСй с Π³ΠΎΡ€Π°Π·Π΄ΠΎ мСньшими числами, ΠΏΠΎΡΠΊΠΎΠ»ΡŒΠΊΡƒ 40 Π΄Π‘ — это коэффициСнт мощности 10000. , Ρ‚ΠΎΠ³Π΄Π° ΠΊΠ°ΠΊ -40 Π΄Π‘ — это коэффициСнт мощности 0,0001. Π’Π°ΠΊΠΈΠΌ ΠΎΠ±Ρ€Π°Π·ΠΎΠΌ, использованиС Π΄Π΅Ρ†ΠΈΠ±Π΅Π» ΡƒΠΏΡ€ΠΎΡ‰Π°Π΅Ρ‚ вычислСния.

    Π”Π΅Ρ†ΠΈΠ±Π΅Π» напряТСния ΠΈ Ρ‚ΠΎΠΊΠ°

    Π›ΡŽΠ±ΠΎΠΉ ΡƒΡ€ΠΎΠ²Π΅Π½ΡŒ мощности ΠΌΠΎΠΆΠ½ΠΎ Π²Ρ‹Ρ€Π°Π·ΠΈΡ‚ΡŒ ΠΊΠ°ΠΊ напряТСниС ΠΈΠ»ΠΈ Ρ‚ΠΎΠΊ, Ссли Π½Π°ΠΌ извСстно сопротивлСниС. Богласно Π·Π°ΠΊΠΎΠ½Ρƒ Ома, P = V 2 / R ΠΈ P = I 2 R. ΠŸΠΎΡΠΊΠΎΠ»ΡŒΠΊΡƒ V ΠΈ I относятся ΠΊ проходящСму Ρ‡Π΅Ρ€Π΅Π· Π½Π΅Π³ΠΎ Ρ‚ΠΎΠΊΡƒ ΠΈ Π½Π°ΠΏΡ€ΡΠΆΠ΅Π½ΠΈΡŽ Π½Π° ΠΎΠ΄Π½ΠΎΠΌ ΠΈ Ρ‚ΠΎΠΌ ΠΆΠ΅ сопротивлСнии, Ссли (ΠΈ Ρ‚ΠΎΠ»ΡŒΠΊΠΎ Ссли) ΠΌΡ‹ сдСлаСм R = R = 1, Ρ‚ΠΎ значСния Π² Π΄Π‘ для ΠΎΡ‚Π½ΠΎΡˆΠ΅Π½ΠΈΠΉ напряТСния (V 1 ΠΈ V 2 ), Π° Ρ‚Π°ΠΊΠΆΠ΅ для Ρ‚ΠΎΠΊΠ° (I 1 ΠΈ I 2 ) Π±ΡƒΠ΄ΡƒΡ‚ прСдставлСны ΠΊΠ°ΠΊ:

    , Ρ‡Ρ‚ΠΎ составляСт 20log (усилСниС ΠΏΠΎ Π½Π°ΠΏΡ€ΡΠΆΠ΅Π½ΠΈΡŽ), Π° для усилСния ΠΏΠΎ Ρ‚ΠΎΠΊΡƒ Π±ΡƒΠ΄Π΅Ρ‚:

    Π’Π°ΠΊΠΈΠΌ ΠΎΠ±Ρ€Π°Π·ΠΎΠΌ, СдинствСнная Ρ€Π°Π·Π½ΠΈΡ†Π° ΠΌΠ΅ΠΆΠ΄Ρƒ расчСтами мощности, напряТСния ΠΈ Ρ‚ΠΎΠΊΠ° Π² Π΄Π΅Ρ†ΠΈΠ±Π΅Π»Π°Ρ… (Π΄Π‘) — это постоянныС 10 ΠΈ 20, ΠΈ Ρ‡Ρ‚ΠΎ для Ρ‚ΠΎΠ³ΠΎ, Ρ‡Ρ‚ΠΎΠ±Ρ‹ ΠΎΡ‚Π½ΠΎΡˆΠ΅Π½ΠΈΠ΅ Π΄Π‘ Π±Ρ‹Π»ΠΎ ΠΏΡ€Π°Π²ΠΈΠ»ΡŒΠ½Ρ‹ΠΌ Π²ΠΎ всСх случаях, Π΄Π²Π΅ Π²Π΅Π»ΠΈΡ‡ΠΈΠ½Ρ‹ Π΄ΠΎΠ»ΠΆΠ½Ρ‹ ΠΈΠΌΠ΅Ρ‚ΡŒ ΠΎΠ΄ΠΈΠ½Π°ΠΊΠΎΠ²Ρ‹Π΅ Π΅Π΄ΠΈΠ½ΠΈΡ†Ρ‹ измСрСния, Π»ΠΈΠ±ΠΎ Π²Π°Ρ‚Ρ‚Ρ‹, ΠΌΠΈΠ»Π»ΠΈΠ²Π°Ρ‚Ρ‚Ρ‹, Π²ΠΎΠ»ΡŒΡ‚Ρ‹, ΠΌΠΈΠ»Π»ΠΈΠ²ΠΎΠ»ΡŒΡ‚Ρ‹, Π°ΠΌΠΏΠ΅Ρ€Ρ‹ ΠΈΠ»ΠΈ ΠΌΠΈΠ»Π»ΠΈΠ°ΠΌΠΏΠ΅Ρ€Ρ‹ ΠΈΠ»ΠΈ Π»ΡŽΠ±Ρ‹Π΅ Π΄Ρ€ΡƒΠ³ΠΈΠ΅ Π΅Π΄ΠΈΠ½ΠΈΡ†Ρ‹ измСрСния.

    Π”Π΅Ρ†ΠΈΠ±Π΅Π» ΠŸΡ€ΠΈΠΌΠ΅Ρ€ No2

    Пассивная рСзистивная ΡΠ΅Ρ‚ΡŒ ΠΈΡΠΏΠΎΠ»ΡŒΠ·ΡƒΠ΅Ρ‚ΡΡ для обСспСчСния ослаблСния (ΠΏΠΎΡ‚Π΅Ρ€ΡŒ) 10 Π΄Π‘ ΠΏΡ€ΠΈ Π²Ρ…ΠΎΠ΄Π½ΠΎΠΌ напряТСнии 12 Π’. Каким Π±ΡƒΠ΄Π΅Ρ‚ Π·Π½Π°Ρ‡Π΅Π½ΠΈΠ΅ Π²Ρ‹Ρ…ΠΎΠ΄Π½ΠΎΠ³ΠΎ напряТСния сСти.

    ΠŸΠΎΡΠΊΠΎΠ»ΡŒΠΊΡƒ Π΄Π΅Ρ†ΠΈΠ±Π΅Π» прСдставляСт собой логарифмичСскоС ΠΈΠ·ΠΌΠ΅Π½Π΅Π½ΠΈΠ΅ мощности, напряТСния ΠΈΠ»ΠΈ Ρ‚ΠΎΠΊΠ°, ΠΌΡ‹ ΠΌΠΎΠΆΠ΅ΠΌ ΠΏΠΎΡΡ‚Ρ€ΠΎΠΈΡ‚ΡŒ Ρ‚Π°Π±Π»ΠΈΡ†Ρƒ, Ρ‡Ρ‚ΠΎΠ±Ρ‹ ΠΏΠΎΠΊΠ°Π·Π°Ρ‚ΡŒ ΠΊΠΎΠ½ΠΊΡ€Π΅Ρ‚Π½Ρ‹Π΅ коэффициСнты усилСния ΠΈ ΠΈΡ… эквивалСнтныС значСния Π² Π΄Π΅Ρ†ΠΈΠ±Π΅Π»Π°Ρ… Π½ΠΈΠΆΠ΅.

    Π”Π΅Ρ†ΠΈΠ±Π΅Π» Π’Π°Π±Π»ΠΈΡ†Π° прироста

    31,62 900
    Π΄Π‘ Π—Π½Π°Ρ‡Π΅Π½ΠΈΠ΅ ΠšΠΎΡΡ„Ρ„ΠΈΡ†ΠΈΠ΅Π½Ρ‚ мощности
    10log (A)
    ΠžΡ‚Π½ΠΎΡˆΠ΅Π½ΠΈΠ΅ напряТСниС / Ρ‚ΠΎΠΊ
    20log (A)
    -20dB 0.01 0,1
    -10 Π΄Π‘ 0,1 0,3162
    -6 Π΄Π‘ 0,25 1/2 = 0,5
    -3 Π΄Π‘ 1/2 = 0,5 1 / √ 2 = 0,707
    -1 Π΄Π‘ 0,79 0,89
    0 Π΄Π‘ 1 1
    1 Π΄Π‘ 1,26 1,1
    3dB 2 √2 = 1.414
    6 Π΄Π‘ 4 2
    10 Π΄Π‘ 10 √10 = 3,162
    20 Π΄Π‘ 100 10
    30 Π΄Π‘ 1000

    Из ΠΏΡ€ΠΈΠ²Π΅Π΄Π΅Π½Π½ΠΎΠΉ Π²Ρ‹ΡˆΠ΅ Ρ‚Π°Π±Π»ΠΈΡ†Ρ‹ Π² Π΄Π΅Ρ†ΠΈΠ±Π΅Π»Π°Ρ… Π²ΠΈΠ΄Π½ΠΎ, Ρ‡Ρ‚ΠΎ ΠΏΡ€ΠΈ 0 Π΄Π‘ коэффициСнт усилСния ΠΏΠΎ мощности, Π½Π°ΠΏΡ€ΡΠΆΠ΅Π½ΠΈΡŽ ΠΈ Ρ‚ΠΎΠΊΡƒ Ρ€Π°Π²Π΅Π½ Β«1Β» (Π΅Π΄ΠΈΠ½ΠΈΡ†Π°). Π­Ρ‚ΠΎ ΠΎΠ·Π½Π°Ρ‡Π°Π΅Ρ‚, Ρ‡Ρ‚ΠΎ схСма (ΠΈΠ»ΠΈ систСма) Π½Π΅ ΠΏΡ€ΠΎΠΈΠ·Π²ΠΎΠ΄ΠΈΡ‚ усилСния ΠΈΠ»ΠΈ ΠΏΠΎΡ‚Π΅Ρ€ΡŒ ΠΌΠ΅ΠΆΠ΄Ρƒ Π²Ρ…ΠΎΠ΄Π½Ρ‹ΠΌΠΈ ΠΈ Π²Ρ‹Ρ…ΠΎΠ΄Π½Ρ‹ΠΌΠΈ сигналами.Π’Π°ΠΊΠΈΠΌ ΠΎΠ±Ρ€Π°Π·ΠΎΠΌ, Π½ΡƒΠ»Π΅Π²ΠΎΠΉ Π΄Π‘ соотвСтствуСт Π΅Π΄ΠΈΠ½ΠΈΡ‡Π½ΠΎΠΌΡƒ ΡƒΡΠΈΠ»Π΅Π½ΠΈΡŽ, Ρ‚ΠΎ Π΅ΡΡ‚ΡŒ A = 1, Π° Π½Π΅ Π½ΡƒΠ»Π΅Π²ΠΎΠΌΡƒ ΡƒΡΠΈΠ»Π΅Π½ΠΈΡŽ.

    ΠœΡ‹ Ρ‚Π°ΠΊΠΆΠ΅ ΠΌΠΎΠΆΠ΅ΠΌ Π²ΠΈΠ΄Π΅Ρ‚ΡŒ, Ρ‡Ρ‚ΠΎ ΠΏΡ€ΠΈ + 3 Π΄Π‘ Π²Ρ‹Ρ…ΠΎΠ΄ схСмы (ΠΈΠ»ΠΈ систСмы) ΡƒΠ΄Π²ΠΎΠΈΠ» своС Π²Ρ…ΠΎΠ΄Π½ΠΎΠ΅ Π·Π½Π°Ρ‡Π΅Π½ΠΈΠ΅, Ρ‡Ρ‚ΠΎ ΠΎΠ·Π½Π°Ρ‡Π°Π΅Ρ‚ ΠΏΠΎΠ»ΠΎΠΆΠΈΡ‚Π΅Π»ΡŒΠ½ΠΎΠ΅ усилСниС (усилСниС) Π² Π΄Π‘, поэтому A> 1. Аналогично, ΠΏΡ€ΠΈ -3 Π΄Π‘ Π²Ρ‹Ρ…ΠΎΠ΄Π½ΠΎΠΉ сигнал схСмы Ρ€Π°Π²Π΅Π½ ΠΏΠΎΠ»ΠΎΠ²ΠΈΠ½Π΅ Π΅Π³ΠΎ. Π²Ρ…ΠΎΠ΄Π½ΠΎΠ΅ Π·Π½Π°Ρ‡Π΅Π½ΠΈΠ΅, ΠΎΠ·Π½Π°Ρ‡Π°ΡŽΡ‰Π΅Π΅ ΠΎΡ‚Ρ€ΠΈΡ†Π°Ρ‚Π΅Π»ΡŒΠ½ΠΎΠ΅ усилСниС (Π·Π°Ρ‚ΡƒΡ…Π°Π½ΠΈΠ΅) Π² Π΄Π‘, поэтому A <1. Π­Ρ‚ΠΎ Π·Π½Π°Ρ‡Π΅Π½ΠΈΠ΅ -3 Π΄Π‘ ΠΎΠ±Ρ‹Ρ‡Π½ΠΎ называСтся Ρ‚ΠΎΡ‡ΠΊΠΎΠΉ Β«ΠΏΠΎΠ»ΠΎΠ²ΠΈΠ½Π½ΠΎΠΉ мощности» ΠΈ опрСдСляСт Π³Ρ€Π°Π½ΠΈΡ‡Π½ΡƒΡŽ частоту Π² сСтях Ρ„ΠΈΠ»ΡŒΡ‚Ρ€ΠΎΠ².

    ВсС Ρ…ΠΎΡ€ΠΎΡˆΠΎ ΠΈ Ρ…ΠΎΡ€ΠΎΡˆΠΎ Ρ‚Π°Π±ΡƒΠ»ΠΈΡ€ΠΎΠ²Π°Ρ‚ΡŒ прирост мощности ΠΏΠΎ ΠΎΡ‚Π½ΠΎΡˆΠ΅Π½ΠΈΡŽ ΠΊ Π΄Π΅Ρ†ΠΈΠ±Π΅Π»Π°ΠΌ Π² справочной Ρ‚Π°Π±Π»ΠΈΡ†Π΅, Π½ΠΎ ΠΏΡ€ΠΈ Ρ€Π°Π±ΠΎΡ‚Π΅ с усилитСлями ΠΈ Ρ„ΠΈΠ»ΡŒΡ‚Ρ€Π°ΠΌΠΈ ΠΈΠ½ΠΆΠ΅Π½Π΅Ρ€Ρ‹-элСктрики ΠΏΡ€Π΅Π΄ΠΏΠΎΡ‡ΠΈΡ‚Π°ΡŽΡ‚ ΠΈΡΠΏΠΎΠ»ΡŒΠ·ΠΎΠ²Π°Ρ‚ΡŒ Π³Ρ€Π°Ρ„ΠΈΠΊΠΈ Π‘ΠΎΠ΄Π΅, Π΄ΠΈΠ°Π³Ρ€Π°ΠΌΠΌΡ‹ ΠΈΠ»ΠΈ Π³Ρ€Π°Ρ„ΠΈΠΊΠΈ Π² качСствС Π²ΠΈΠ·ΡƒΠ°Π»ΡŒΠ½ΠΎΠ³ΠΎ отобраТСния частотной характСристики Ρ†Π΅ΠΏΠ΅ΠΉ (ΠΈΠ»ΠΈ систСм). характСристики.Π—Π°Ρ‚Π΅ΠΌ, ΠΈΡΠΏΠΎΠ»ΡŒΠ·ΡƒΡ значСния Π΄Π°Π½Π½Ρ‹Ρ… Π² Ρ‚Π°Π±Π»ΠΈΡ†Π΅ Π²Ρ‹ΡˆΠ΅, ΠΌΡ‹ ΠΌΠΎΠΆΠ΅ΠΌ ΡΠΎΠ·Π΄Π°Ρ‚ΡŒ ΡΠ»Π΅Π΄ΡƒΡŽΡ‰ΠΈΠΉ Π³Ρ€Π°Ρ„ΠΈΠΊ Π‘ΠΎΠ΄Π΅ Β«Π΄Π΅Ρ†ΠΈΠ±Π΅Π»Β», ΠΏΠΎΠΊΠ°Π·Ρ‹Π²Π°ΡŽΡ‰ΠΈΠΉ Ρ€Π°Π·Π»ΠΈΡ‡Π½Ρ‹Π΅ полоТСния Ρ‚ΠΎΡ‡Π΅ΠΊ мощности.

    Π“Ρ€Π°Ρ„ΠΈΠΊ мощности Π‘ΠΎΠ΄Π΅ Π² Π΄Π΅Ρ†ΠΈΠ±Π΅Π»Π°Ρ…

    Π’ΠΎΠ³Π΄Π° ΠΌΡ‹ ΠΌΠΎΠΆΠ΅ΠΌ ясно Π²ΠΈΠ΄Π΅Ρ‚ΡŒ, Ρ‡Ρ‚ΠΎ кривая мощности Π½Π΅ являСтся Π»ΠΈΠ½Π΅ΠΉΠ½ΠΎΠΉ, Π° слСдуСт логарифмичСскому коэффициСнту 1,259.

    Π”Π΅Ρ†ΠΈΠ±Π΅Π» ΠšΡ€Π°Ρ‚ΠΊΠΎΠ΅ ΠΈΠ·Π»ΠΎΠΆΠ΅Π½ΠΈΠ΅ руководства

    ΠœΡ‹ Π²ΠΈΠ΄Π΅Π»ΠΈ Π² этом руководствС ΠΎ Π΄Π΅Ρ†ΠΈΠ±Π΅Π»Π°Ρ… (Π΄Π‘), Ρ‡Ρ‚ΠΎ это логарифмичСская Π΅Π΄ΠΈΠ½ΠΈΡ†Π° измСнСния мощности Base-10 ΠΈ Ρ‡Ρ‚ΠΎ Π΄Π΅Ρ†ΠΈΠ±Π΅Π» прСдставляСт собой Π±Π΅Π·Ρ€Π°Π·ΠΌΠ΅Ρ€Π½ΠΎΠ΅ Π·Π½Π°Ρ‡Π΅Π½ΠΈΠ΅ 1/10 ΠΎΡ‚ Π±Π΅Π» (1 Π±Π΅Π» = 10 Π΄Π΅Ρ†ΠΈΠ±Π΅Π» ΠΈΠ»ΠΈ 1 Π΄Π‘ = 0.1Π‘). Π”Π΅Ρ†ΠΈΠ±Π΅Π» позволяСт Π½Π°ΠΌ ΠΏΡ€Π΅Π΄ΡΡ‚Π°Π²ΠΈΡ‚ΡŒ большиС ΠΎΡ‚Π½ΠΎΡˆΠ΅Π½ΠΈΡ мощностСй с использованиСм Π½Π΅Π±ΠΎΠ»ΡŒΡˆΠΈΡ… чисСл, ΠΈ ΠΌΡ‹ Π²ΠΈΠ΄Π΅Π»ΠΈ Π²Ρ‹ΡˆΠ΅, Ρ‡Ρ‚ΠΎ 30 Π΄Π‘ эквивалСнтно ΡΠΎΠΎΡ‚Π½ΠΎΡˆΠ΅Π½ΠΈΡŽ мощностСй 1000 с Π½Π°ΠΈΠ±ΠΎΠ»Π΅Π΅ часто ΠΈΡΠΏΠΎΠ»ΡŒΠ·ΡƒΠ΅ΠΌΡ‹ΠΌΠΈ значСниями Π² Π΄Π΅Ρ†ΠΈΠ±Π΅Π»Π°Ρ…: 3 Π΄Π‘, 6 Π΄Π‘, 10 Π΄Π‘ ΠΈ 20 Π΄Π‘ (ΠΈ ΠΈΡ… ΠΎΡ‚Ρ€ΠΈΡ†Π°Ρ‚Π΅Π»ΡŒΠ½Ρ‹Π΅ эквивалСнты). . Однако 20 Π΄Π‘ Π½Π΅ Π²Π΄Π²ΠΎΠ΅ большС мощности 10 Π΄Π‘.

    Π”Π΅Ρ†ΠΈΠ±Π΅Π» Ρ‚Π°ΠΊΠΆΠ΅ ΠΏΠΎΠΊΠ°Π·Ρ‹Π²Π°Π΅Ρ‚ Π½Π°ΠΌ, Ρ‡Ρ‚ΠΎ любоС ΠΈΠ·ΠΌΠ΅Π½Π΅Π½ΠΈΠ΅ мощности Π½Π° Ρ‚ΠΎ ΠΆΠ΅ ΠΎΡ‚Π½ΠΎΡˆΠ΅Π½ΠΈΠ΅ Π±ΡƒΠ΄Π΅Ρ‚ ΠΈΠΌΠ΅Ρ‚ΡŒ Ρ‚Π°ΠΊΠΎΠ΅ ΠΆΠ΅ ΠΎΡ‚Π½ΠΎΡˆΠ΅Π½ΠΈΠ΅ Π΄Π΅Ρ†ΠΈΠ±Π΅Π». НапримСр, ΡƒΠ΄Π²ΠΎΠ΅Π½ΠΈΠ΅ мощности с 1 Π΄ΠΎ 2 Π’Ρ‚ — это Ρ‚ΠΎ ΠΆΠ΅ ΡΠΎΠΎΡ‚Π½ΠΎΡˆΠ΅Π½ΠΈΠ΅, Ρ‡Ρ‚ΠΎ ΠΈ с 10 Π’Ρ‚ Π΄ΠΎ 20 Π’Ρ‚, Ρ‚ΠΎ Π΅ΡΡ‚ΡŒ ΠΈΠ·ΠΌΠ΅Π½Π΅Π½ΠΈΠ΅ Π½Π° + 3 Π΄Π‘, Π² Ρ‚ΠΎ врСмя ΠΊΠ°ΠΊ ΠΈΠ·ΠΌΠ΅Π½Π΅Π½ΠΈΠ΅ Π½Π° -3 Π΄Π‘ ΠΎΠ·Π½Π°Ρ‡Π°Π΅Ρ‚, Ρ‡Ρ‚ΠΎ коэффициСнт мощности Π±ΡƒΠ΄Π΅Ρ‚ ΡƒΠΌΠ΅Π½ΡŒΡˆΠ΅Π½ Π²Π΄Π²ΠΎΠ΅.

    Если ΠΎΡ‚Π½ΠΎΡˆΠ΅Π½ΠΈΠ΅ Π΄Π‘ ΠΏΠΎΠ»ΠΎΠΆΠΈΡ‚Π΅Π»ΡŒΠ½ΠΎΠ΅ ΠΏΠΎ Π·Π½Π°Ρ‡Π΅Π½ΠΈΡŽ, это ΠΎΠ·Π½Π°Ρ‡Π°Π΅Ρ‚, Ρ‡Ρ‚ΠΎ усилСниС ΠΈΠ»ΠΈ усилСниС присутствуСт, Ρ‚Π°ΠΊ ΠΊΠ°ΠΊ выходная ΠΌΠΎΡ‰Π½ΠΎΡΡ‚ΡŒ ΠΏΡ€Π΅Π²Ρ‹ΡˆΠ°Π΅Ρ‚ Π²Ρ…ΠΎΠ΄Π½ΡƒΡŽ ΠΌΠΎΡ‰Π½ΠΎΡΡ‚ΡŒ (P OUT > P IN ). Однако, Ссли коэффициСнт мощности Π² Π΄Π‘ ΠΈΠΌΠ΅Π΅Ρ‚ ΠΎΡ‚Ρ€ΠΈΡ†Π°Ρ‚Π΅Π»ΡŒΠ½ΠΎΠ΅ Π·Π½Π°Ρ‡Π΅Π½ΠΈΠ΅, это ΠΎΠ·Π½Π°Ρ‡Π°Π΅Ρ‚, Ρ‡Ρ‚ΠΎ Π½Π° схСму влияСт Π·Π°Ρ‚ΡƒΡ…Π°Π½ΠΈΠ΅ ΠΈΠ»ΠΈ ΠΏΠΎΡ‚Π΅Ρ€ΠΈ, ΠΏΠΎΡΠΊΠΎΠ»ΡŒΠΊΡƒ выходная ΠΌΠΎΡ‰Π½ΠΎΡΡ‚ΡŒ Π±ΡƒΠ΄Π΅Ρ‚ мСньшС, Ρ‡Π΅ΠΌ входная ΠΌΠΎΡ‰Π½ΠΎΡΡ‚ΡŒ схСмы (P OUT

    IN ). ΠžΡ‡Π΅Π²ΠΈΠ΄Π½ΠΎ, Ρ‡Ρ‚ΠΎ 0 Π΄Π‘ ΠΎΠ·Π½Π°Ρ‡Π°Π΅Ρ‚, Ρ‡Ρ‚ΠΎ коэффициСнт мощности Ρ€Π°Π²Π΅Π½ Π±Π΅Π· ΡƒΠΌΠ΅Π½ΡŒΡˆΠ΅Π½ΠΈΡ ΠΈΠ»ΠΈ усилСния сигнала.

    ΠŸΡ€Π΅ΠΎΠ±Ρ€Π°Π·ΠΎΠ²Π°Π½ΠΈΠ΅

    Π΄Π΅Ρ†ΠΈΠ±Π΅Π»: коэффициСнт 10 ΠΈΠ»ΠΈ коэффициСнт 20?

    Π”Π΅Ρ†ΠΈΠ±Π΅Π» ΠΈΡΠΏΠΎΠ»ΡŒΠ·ΡƒΠ΅Ρ‚ΡΡ Π² ΡˆΠΈΡ€ΠΎΠΊΠΎΠΌ Π΄ΠΈΠ°ΠΏΠ°Π·ΠΎΠ½Π΅ ΠΏΡ€ΠΈΠ»ΠΎΠΆΠ΅Π½ΠΈΠΉ. Π”Π΅Ρ†ΠΈΠ±Π΅Π»Ρ‹ особСнно ΠΈΡΠΏΠΎΠ»ΡŒΠ·ΡƒΡŽΡ‚ΡΡ, ΠΊΠΎΠ³Π΄Π° Ρ€Π΅Ρ‡ΡŒ ΠΈΠ΄Π΅Ρ‚ ΠΎ мощности ΠΈΠ»ΠΈ ΠΏΡ€ΠΎΠΈΠ·Π²ΠΎΠ΄Π½ΠΎΠΉ ΠΌΠ΅Ρ€Π΅, значСния ΠΊΠΎΡ‚ΠΎΡ€Ρ‹Ρ… ΠΌΠΎΠ³ΡƒΡ‚ Π²Π°Ρ€ΡŒΠΈΡ€ΠΎΠ²Π°Ρ‚ΡŒΡΡ Π² ΡˆΠΈΡ€ΠΎΠΊΠΎΠΌ Π΄ΠΈΠ°ΠΏΠ°Π·ΠΎΠ½Π΅. НаиболСС часто Π΄Π΅Ρ†ΠΈΠ±Π΅Π»Ρ‹ ΠΈΡΠΏΠΎΠ»ΡŒΠ·ΡƒΡŽΡ‚ΡΡ для измСрСния громкости Π·Π²ΡƒΠΊΠ°. Π’Π°ΠΊ, Π½Π°ΠΏΡ€ΠΈΠΌΠ΅Ρ€, Π·Π²ΡƒΠΊ 0 Π΄Π‘ Π΅Π΄Π²Π° ΡΠ»Ρ‹ΡˆΠ΅Π½, Ρ‚ΠΎΠ³Π΄Π° ΠΊΠ°ΠΊ Ρƒ пылСсоса Π² срСднСм 75 Π΄Π‘, Π° Π½Π° Ρ€ΠΎΠΊ-ΠΊΠΎΠ½Ρ†Π΅Ρ€Ρ‚Π΅ — ΠΎΠΊΠΎΠ»ΠΎ 110 Π΄Π‘.

    Π’ классС Π²Ρ‹ часто Π½Π°ΠΉΠ΄Π΅Ρ‚Π΅ ΡΠ»Π΅Π΄ΡƒΡŽΡ‰ΠΈΠ΅ опрСдСлСния Ρ‡Π΅Π³ΠΎ-Π»ΠΈΠ±ΠΎ Π² Π΄Π΅Ρ†ΠΈΠ±Π΅Π»Π°Ρ…

    $$ X_ {dB} = 10 \ log_ {10} \ left (\ frac {X_ {lin}} {X_ {ref}} \ right). $$

    Π­Ρ‚ΠΎ ΡƒΡ€Π°Π²Π½Π΅Π½ΠΈΠ΅ ΠΏΡ€Π΅ΠΎΠ±Ρ€Π°Π·ΡƒΠ΅Ρ‚ Π²Π΅Π»ΠΈΡ‡ΠΈΠ½Ρƒ $ X_ {lin} $ ΠΈΠ· Π»ΠΈΠ½Π΅ΠΉΠ½ΠΎΠ³ΠΎ ΠΌΠ°ΡΡˆΡ‚Π°Π±Π° ΠΊ Π²Π΅Π»ΠΈΡ‡ΠΈΠ½Π΅ Π² шкалС Π΄Π‘ $ X_ {Π΄Π‘} $. Для этого сначала линСйная Π²Π΅Π»ΠΈΡ‡ΠΈΠ½Π° связана с эталонной Π²Π΅Π»ΠΈΡ‡ΠΈΠ½ΠΎΠΉ $ X_ {ref} $, ΠΈ ΠΈΡ… ΡΠΎΠΎΡ‚Π½ΠΎΡˆΠ΅Π½ΠΈΠ΅ прСобразуСтся Π² Π»ΠΎΠ³-ΠΎΠ±Π»Π°ΡΡ‚ΡŒ. По-Π²ΠΈΠ΄ΠΈΠΌΠΎΠΌΡƒ, Π² $ X_ {dB} $ Π½Π° самом Π΄Π΅Π»Π΅ Π½Π΅Ρ‚ Π΅Π΄ΠΈΠ½ΠΈΡ†Ρ‹ измСрСния, ΠΈ ΠΌΡ‹ искусствСнно добавляСм Π΄Π‘ , Ρ‡Ρ‚ΠΎΠ±Ρ‹ Π±Ρ‹Π»ΠΎ ясно, Ρ‡Ρ‚ΠΎ ΠΌΡ‹ находимся Π² логарифмичСской шкалС.Когда $ X_ {lin} $ равняСтся ΠΎΠΏΠΎΡ€Π½ΠΎΠΌΡƒ ΡƒΡ€ΠΎΠ²Π½ΡŽ, шкала Π΄Π‘ становится Ρ€Π°Π²Π½ΠΎΠΉ Π½ΡƒΠ»ΡŽ:

    $$ X_ {Π΄Π‘} = 10 \ log_ {10} \ left. \ Left (\ frac {X_ {lin}} {X_ {ref}} \ right) \ right | _ {X_ {lin} = X_ {ref} } = 0 Π΄Π‘. $$

    ΠšΡ€ΠΎΠΌΠ΅ Ρ‚ΠΎΠ³ΠΎ, ΠΊΠΎΠ³Π΄Π° $ X_ {lin}> X_ {ref} $, $ X_ {dB} $ ΠΏΠΎΠ»ΠΎΠΆΠΈΡ‚Π΅Π»ΡŒΠ½ΠΎ, Π° Ссли $ X_ {lin}

    Пока всС Ρ…ΠΎΡ€ΠΎΡˆΠΎ. Но, с Π΄Ρ€ΡƒΠ³ΠΎΠΉ стороны, Ρ‡Π°Ρ‰Π΅ всСго Π²Ρ‹ Ρ‚Π°ΠΊΠΆΠ΅ Π²ΠΈΠ΄ΠΈΡ‚Π΅ ΡΠ»Π΅Π΄ΡƒΡŽΡ‰Π΅Π΅ ΠΎΠΏΡ€Π΅Π΄Π΅Π»Π΅Π½ΠΈΠ΅:

    $$ Y_ {dB} = 20 \ log_ {10} \ left (\ frac {Y} {Y_ {ref}} \ right), $$

    Ρ‚.Π΅. ΠΌΠ½ΠΎΠΆΠΈΡ‚Π΅Π»ΡŒ ΠΏΠ΅Ρ€Π΅Π΄ Π»ΠΎΠ³Π°Ρ€ΠΈΡ„ΠΌΠΎΠΌ Ρ€Π°Π²Π΅Π½ 20 вмСсто 10. Какая вСрсия Π²Π΅Ρ€Π½Π° ? ОбС Ρ„ΠΎΡ€ΠΌΡƒΠ»Ρ‹ Π²Π΅Ρ€Π½Ρ‹, Π½ΠΎ ΠΏΡ€ΠΈ Π²Ρ‹Π±ΠΎΡ€Π΅ Ρ„ΠΎΡ€ΠΌΡƒΠ»Ρ‹ Π½Π΅ΠΎΠ±Ρ…ΠΎΠ΄ΠΈΠΌΠΎ ΡΠΎΠ±Π»ΡŽΠ΄Π°Ρ‚ΡŒ ΠΎΡΡ‚ΠΎΡ€ΠΎΠΆΠ½ΠΎΡΡ‚ΡŒ. Π˜Ρ‚Π°ΠΊ, Π²ΠΎΡ‚ ΠΏΡ€Π°Π²ΠΈΠ»ΠΎ:

    • Если Π²Ρ‹ ΠΏΡ€Π΅ΠΎΠ±Ρ€Π°Π·ΡƒΠ΅Ρ‚Π΅ Π²Π΅Π»ΠΈΡ‡ΠΈΠ½Ρƒ, которая относится ΠΊ мощности, ΠΈΠ»ΠΈ энСргии, коэффициСнт составит 10 .
    • Если ΠΏΡ€Π΅ΠΎΠ±Ρ€Π°Π·ΠΎΠ²Π°Ρ‚ΡŒ Π²Π΅Π»ΠΈΡ‡ΠΈΠ½Ρƒ, ΠΎΡ‚Π½ΠΎΡΡΡ‰ΡƒΡŽΡΡ ΠΊ Π°ΠΌΠΏΠ»ΠΈΡ‚ΡƒΠ΄Π΅ , коэффициСнт составит 20 .

    Как ΡƒΠ·Π½Π°Ρ‚ΡŒ, относится Π»ΠΈ Π²Π΅Π»ΠΈΡ‡ΠΈΠ½Π° ΠΊ мощности ΠΈΠ»ΠΈ Π°ΠΌΠΏΠ»ΠΈΡ‚ΡƒΠ΄Π΅? ΠŸΡ€ΠΎΠ²Π΅Ρ€ΡŒΡ‚Π΅, ΠΈΠΌΠ΅Π΅Ρ‚ Π»ΠΈ смысл Π²ΠΎΠ·Π²Π΅Π΄Π΅Π½ΠΈΠ΅ количСства Π² ΠΊΠ²Π°Π΄Ρ€Π°Ρ‚! Если Π²ΠΎΠ·Π²Π΅Π΄Π΅Π½ΠΈΠ΅ Π² ΠΊΠ²Π°Π΄Ρ€Π°Ρ‚ ΠΈΠΌΠ΅Π΅Ρ‚ смысл, ваша Π²Π΅Π»ΠΈΡ‡ΠΈΠ½Π°, скорСС всСго, Π±ΡƒΠ΄Π΅Ρ‚ Π°ΠΌΠΏΠ»ΠΈΡ‚ΡƒΠ΄ΠΎΠΉ. Если это Π½Π΅ ΠΈΠΌΠ΅Π΅Ρ‚ смысла, Π²Ρ‹, вСроятно, ΡΡ‚ΠΎΠ»ΠΊΠ½Π΅Ρ‚Π΅ΡΡŒ с ΠΌΠΎΡ‰Π½Ρ‹ΠΌ количСством.НапримСр, Π½Π΅Ρ‚ смысла Π²ΠΎΠ·Π²ΠΎΠ΄ΠΈΡ‚ΡŒ ΠΌΠΎΡ‰Π½ΠΎΡΡ‚ΡŒ сигнала Π² ΠΊΠ²Π°Π΄Ρ€Π°Ρ‚. Но ΠΈΠΌΠ΅Π΅Ρ‚ смысл Π²ΠΎΠ·Π²ΠΎΠ΄ΠΈΡ‚ΡŒ Π² ΠΊΠ²Π°Π΄Ρ€Π°Ρ‚ Π°ΠΌΠΏΠ»ΠΈΡ‚ΡƒΠ΄Ρƒ сигнала, ΠΏΠΎΡΠΊΠΎΠ»ΡŒΠΊΡƒ это Π΄Π°Π΅Ρ‚ ΠΌΠΎΡ‰Π½ΠΎΡΡ‚ΡŒ сигнала.

    Π”Ρ€ΡƒΠ³ΠΎΠΉ ΠΈΠ½Π΄ΠΈΠΊΠ°Ρ‚ΠΎΡ€, ΠΊΠΎΠ³Π΄Π° ΠΈΡΠΏΠΎΠ»ΡŒΠ·ΠΎΠ²Π°Ρ‚ΡŒ 10 ΠΈΠ»ΠΈ 20, ΡΠ»Π΅Π΄ΡƒΡŽΡ‰ΠΈΠΉ:

    Если вашС количСство $ X_ {lin} $ слСдуСт ΠΈΠ· Π½Π΅ΠΊΠΎΡ‚ΠΎΡ€ΠΎΠ³ΠΎ вычислСния, ΠΊΠΎΡ‚ΠΎΡ€ΠΎΠ΅ Π²ΠΊΠ»ΡŽΡ‡Π°Π΅Ρ‚ Π²ΠΎΠ·Π²Π΅Π΄Π΅Π½ΠΈΠ΅ Π² ΠΊΠ²Π°Π΄Ρ€Π°Ρ‚ Π½Π΅ΠΊΠΎΡ‚ΠΎΡ€ΠΎΠ³ΠΎ Π΄Ρ€ΡƒΠ³ΠΎΠ³ΠΎ количСства, ΠΈΡΠΏΠΎΠ»ΡŒΠ·ΡƒΠΉΡ‚Π΅ ΠΌΠ½ΠΎΠΆΠΈΡ‚Π΅Π»ΡŒ 10.

    НСкоторыС ΠΏΡ€ΠΈΠΌΠ΅Ρ€Ρ‹ΒΆ

    Π”Π°Π»Π΅Π΅ ΠΌΡ‹ вычисляСм Π½Π΅ΠΊΠΎΡ‚ΠΎΡ€Ρ‹Π΅ ΠΏΡ€ΠΈΠΌΠ΅Ρ€Ρ‹ прСобразования Π»ΠΈΠ½Π΅ΠΉΠ½ΠΎΠΉ ΡˆΠΊΠ°Π»Ρ‹ Π² ΡˆΠΊΠ°Π»Ρƒ Π΄Π‘:

    ΠŸΡ€Π΅ΠΎΠ±Ρ€Π°Π·ΡƒΠΉΡ‚Π΅ 100 Π’Ρ‚ Π² ΡˆΠΊΠ°Π»Ρƒ Π΄Π‘.Бсылка $ X_ {ref} = 1 ΠΌΠ’Ρ‚ $.

    {ref} $,>

    Π§Ρ‚ΠΎ Ρ‚Π°ΠΊΠΎΠ΅ Π΄Π΅Ρ†ΠΈΠ±Π΅Π» (Π΄Π‘)?

    ΠžΠΏΡ€Π΅Π΄Π΅Π»Π΅Π½ΠΈΠ΅ Π΄Π΅Ρ†ΠΈΠ±Π΅Π» (Π΄Π‘), ΠΊΠ°ΠΊ ΠΏΡ€Π΅ΠΎΠ±Ρ€Π°Π·ΠΎΠ²Π°Ρ‚ΡŒ, ΠΊΠ°Π»ΡŒΠΊΡƒΠ»ΡΡ‚ΠΎΡ€ ΠΈ Ρ‚Π°Π±Π»ΠΈΡ†Π° ΡΠΎΠΎΡ‚Π½ΠΎΡˆΠ΅Π½ΠΈΡ Π΄Π‘.

    ΠžΠΏΡ€Π΅Π΄Π΅Π»Π΅Π½ΠΈΠ΅ Π΄Π΅Ρ†ΠΈΠ±Π΅Π» (Π΄Π‘)

    Π”Π΅Ρ†ΠΈΠ±Π΅Π» (символ: Π΄Π‘) — логарифмичСская Π΅Π΄ΠΈΠ½ΠΈΡ†Π°, ΡƒΠΊΠ°Π·Ρ‹Π²Π°ΡŽΡ‰Π°Ρ коэффициСнт ΠΈΠ»ΠΈ коэффициСнт усилСния.

    Π”Π΅Ρ†ΠΈΠ±Π΅Π» ΠΈΡΠΏΠΎΠ»ΡŒΠ·ΡƒΠ΅Ρ‚ΡΡ для обозначСния уровня акустичСских Π²ΠΎΠ»Π½ ΠΈ элСктронных сигналов.

    ЛогарифмичСская шкала ΠΌΠΎΠΆΠ΅Ρ‚ ΠΎΠΏΠΈΡΡ‹Π²Π°Ρ‚ΡŒ ΠΎΡ‡Π΅Π½ΡŒ большиС ΠΈΠ»ΠΈ ΠΎΡ‡Π΅Π½ΡŒ малСнькиС числа Π² Π±ΠΎΠ»Π΅Π΅ ΠΊΠΎΡ€ΠΎΡ‚ΠΊΠΈΡ… обозначСниях.

    Π£Ρ€ΠΎΠ²Π΅Π½ΡŒ Π΄Π‘ ΠΌΠΎΠΆΠ½ΠΎ Ρ€Π°ΡΡΠΌΠ°Ρ‚Ρ€ΠΈΠ²Π°Ρ‚ΡŒ ΠΊΠ°ΠΊ ΠΎΡ‚Π½ΠΎΡΠΈΡ‚Π΅Π»ΡŒΠ½ΠΎΠ΅ усилСниС ΠΎΠ΄Π½ΠΎΠ³ΠΎ уровня ΠΏΠΎ ΡΡ€Π°Π²Π½Π΅Π½ΠΈΡŽ с Π΄Ρ€ΡƒΠ³ΠΈΠΌ ΡƒΡ€ΠΎΠ²Π½Π΅ΠΌ ΠΈΠ»ΠΈ ΠΊΠ°ΠΊ ΡƒΡ€ΠΎΠ²Π΅Π½ΡŒ Π°Π±ΡΠΎΠ»ΡŽΡ‚Π½ΠΎΠΉ логарифмичСской ΡˆΠΊΠ°Π»Ρ‹ для Ρ…ΠΎΡ€ΠΎΡˆΠΎ извСстных ΠΎΠΏΠΎΡ€Π½Ρ‹Ρ… ΡƒΡ€ΠΎΠ²Π½Π΅ΠΉ.

    Π”Π΅Ρ†ΠΈΠ±Π΅Π» — бСзразмСрная Π΅Π΄ΠΈΠ½ΠΈΡ†Π°.

    ΠžΡ‚Π½ΠΎΡˆΠ΅Π½ΠΈΠ΅ Π² Π±Π΅Π»Π°Ρ… — это дСсятичный Π»ΠΎΠ³Π°Ρ€ΠΈΡ„ΠΌ ΠΎΡ‚Π½ΠΎΡˆΠ΅Π½ΠΈΡ P 1 ΠΈ P 0 :

    Π‘ΠΎΠΎΡ‚Π½ΠΎΡˆΠ΅Π½ΠΈΠ΅ B = Π±Ρ€Π΅Π²Π½ΠΎ 10 ( P 1 / P 0 )

    Π”Π΅Ρ†ΠΈΠ±Π΅Π» составляСт ΠΎΠ΄Π½Ρƒ Π΄Π΅ΡΡΡ‚ΡƒΡŽ Π±Π΅Π»Π°, поэтому 1 Π±Π΅Π» Ρ€Π°Π²Π΅Π½ 10 Π΄Π΅Ρ†ΠΈΠ±Π΅Π»Π°ΠΌ:

    1B = 10 Π΄Π‘

    ΠŸΠ΅Ρ€Π΅Π΄Π°Ρ‚ΠΎΡ‡Π½ΠΎΠ΅ число

    ΠšΠΎΡΡ„Ρ„ΠΈΡ†ΠΈΠ΅Π½Ρ‚ мощности Π² Π΄Π΅Ρ†ΠΈΠ±Π΅Π»Π°Ρ… (Π΄Π‘) Ρ€Π°Π²Π΅Π½ дСсятикратному Π»ΠΎΠ³Π°Ρ€ΠΈΡ„ΠΌΡƒ ΠΏΠΎ основанию 10 ΠΎΡ‚Π½ΠΎΡˆΠ΅Π½ΠΈΡ P 1 ΠΈ P 0 :

    ΠšΠΎΡΡ„Ρ„ΠΈΡ†ΠΈΠ΅Π½Ρ‚ Π΄Π‘ = 10β‹…log 10 ( P 1 / P 0 )

    Π‘ΠΎΠΎΡ‚Π½ΠΎΡˆΠ΅Π½ΠΈΠ΅ Π°ΠΌΠΏΠ»ΠΈΡ‚ΡƒΠ΄

    Π‘ΠΎΠΎΡ‚Π½ΠΎΡˆΠ΅Π½ΠΈΠ΅ Ρ‚Π°ΠΊΠΈΡ… Π²Π΅Π»ΠΈΡ‡ΠΈΠ½, ΠΊΠ°ΠΊ напряТСниС, сила Ρ‚ΠΎΠΊΠ° ΠΈ ΡƒΡ€ΠΎΠ²Π΅Π½ΡŒ Π·Π²ΡƒΠΊΠΎΠ²ΠΎΠ³ΠΎ давлСния, рассчитываСтся ΠΊΠ°ΠΊ ΠΎΡ‚Π½ΠΎΡˆΠ΅Π½ΠΈΠ΅ ΠΊΠ²Π°Π΄Ρ€Π°Ρ‚ΠΎΠ².

    ΠžΡ‚Π½ΠΎΡˆΠ΅Π½ΠΈΠ΅ Π°ΠΌΠΏΠ»ΠΈΡ‚ΡƒΠ΄ Π² Π΄Π΅Ρ†ΠΈΠ±Π΅Π»Π°Ρ… (Π΄Π‘) составляСт 20 Π»ΠΎΠ³Π°Ρ€ΠΈΡ„ΠΌΠΎΠ² ΠΏΠΎ основанию 10 ΠΎΡ‚Π½ΠΎΡˆΠ΅Π½ΠΈΡ V 1 ΠΈ V 0 :

    ΠšΠΎΡΡ„Ρ„ΠΈΡ†ΠΈΠ΅Π½Ρ‚ Π΄Π‘ = 10β‹…log 10 ( V 1 2 / V 0 2 ) = 20β‹…log 10 ( V 1 / Π’ 0 )

    Π”Π΅Ρ†ΠΈΠ±Π΅Π» Π² Π²Π°Ρ‚Ρ‚Ρ‹, Π²ΠΎΠ»ΡŒΡ‚Ρ‹, Π³Π΅Ρ€Ρ†Ρ‹, ΠΊΠ°Π»ΡŒΠΊΡƒΠ»ΡΡ‚ΠΎΡ€ прСобразования паскалСй

    ΠŸΡ€Π΅ΠΎΠ±Ρ€Π°Π·ΠΎΠ²Π°Π½ΠΈΠ΅ Π΄Π‘, Π΄Π‘ΠΌ, Π΄Π‘Π’Ρ‚, Π΄Π‘Π’, Π΄Π‘ΠΌΠ’, Π΄Π‘ΠΌΠΊΠ’, Π΄Π‘Ρƒ, дБмкА, Π΄Π‘Π“Ρ†, Π΄Π‘Π£Π—Π”, дБА Π² Π²Π°Ρ‚Ρ‚Ρ‹, Π²ΠΎΠ»ΡŒΡ‚Ρ‹, Π°ΠΌΠΏΠ΅Ρ€Ρ‹, Π³Π΅Ρ€Ρ†Ρ‹, Π·Π²ΡƒΠΊΠΎΠ²ΠΎΠ΅ Π΄Π°Π²Π»Π΅Π½ΠΈΠ΅.

    1. УстановитС Ρ‚ΠΈΠΏ количСства ΠΈ Π΅Π΄ΠΈΠ½ΠΈΡ†Ρƒ Π΄Π΅Ρ†ΠΈΠ±Π΅Π».
    2. Π’Π²Π΅Π΄ΠΈΡ‚Π΅ значСния Π² ΠΎΠ΄Π½ΠΎ ΠΈΠ»ΠΈ Π΄Π²Π° тСкстовых поля ΠΈ Π½Π°ΠΆΠΌΠΈΡ‚Π΅ ΡΠΎΠΎΡ‚Π²Π΅Ρ‚ΡΡ‚Π²ΡƒΡŽΡ‰ΡƒΡŽ ΠΊΠ½ΠΎΠΏΠΊΡƒ ΠŸΡ€Π΅ΠΎΠ±Ρ€Π°Π·ΠΎΠ²Π°Ρ‚ΡŒ :

    ΠžΡ‚Π½ΠΎΡˆΠ΅Π½ΠΈΠ΅ мощности ΠΊ ΠΏΡ€Π΅ΠΎΠ±Ρ€Π°Π·ΠΎΠ²Π°Π½ΠΈΡŽ Π² Π΄Π‘

    ΠšΠΎΡΡ„Ρ„ΠΈΡ†ΠΈΠ΅Π½Ρ‚ усилСния G Π΄Π‘ Ρ€Π°Π²Π΅Π½ дСсятикратному Π»ΠΎΠ³Π°Ρ€ΠΈΡ„ΠΌΡƒ ΠΏΠΎ основанию 10 ΠΎΡ‚Π½ΠΎΡˆΠ΅Π½ΠΈΡ мощности P 2 ΠΈ ΠΎΠΏΠΎΡ€Π½ΠΎΠΉ мощности P 1 .

    G Π΄Π‘ = 10 log 10 ( P 2 / P 1 )

    P 2 — ΡƒΡ€ΠΎΠ²Π΅Π½ΡŒ мощности.

    P 1 — эталонный ΡƒΡ€ΠΎΠ²Π΅Π½ΡŒ мощности.

    G Π΄Π‘ — коэффициСнт мощности ΠΈΠ»ΠΈ усилСниС Π² Π΄Π‘.

    ΠŸΡ€ΠΈΠΌΠ΅Ρ€

    НайдитС коэффициСнт усилСния Π² Π΄Π‘ для систСмы с Π²Ρ…ΠΎΠ΄Π½ΠΎΠΉ ΠΌΠΎΡ‰Π½ΠΎΡΡ‚ΡŒΡŽ 5 Π’Ρ‚ ΠΈ Π²Ρ‹Ρ…ΠΎΠ΄Π½ΠΎΠΉ ΠΌΠΎΡ‰Π½ΠΎΡΡ‚ΡŒΡŽ 10 Π’Ρ‚.

    G Π΄Π‘ = 10 log 10 ( P Π½Π° Π²Ρ‹Ρ…ΠΎΠ΄Π΅ / P Π½Π° ) = 10 log 10 (10 Π’Ρ‚ / 5 Π’Ρ‚) = 3,01 Π΄Π‘

    Π΄Π‘ для прСобразования ΠΎΡ‚Π½ΠΎΡˆΠ΅Π½ΠΈΡ мощности

    ΠœΠΎΡ‰Π½ΠΎΡΡ‚ΡŒ P 2 Ρ€Π°Π²Π½Π° ΠΎΠΏΠΎΡ€Π½ΠΎΠΉ мощности P 1 , ΡƒΠΌΠ½ΠΎΠΆΠ΅Π½Π½ΠΎΠΉ Π½Π° 10, ΡƒΠ²Π΅Π»ΠΈΡ‡Π΅Π½Π½ΠΎΠΉ Π½Π° усилСниС Π² G Π΄Π‘ , Π΄Π΅Π»Π΅Π½Π½ΠΎΠ΅ Π½Π° 10.

    P 2 = P 1 β‹… 10 ( G Π΄Π‘ / 10)

    P 2 — ΡƒΡ€ΠΎΠ²Π΅Π½ΡŒ мощности.

    P 1 — эталонный ΡƒΡ€ΠΎΠ²Π΅Π½ΡŒ мощности.

    G Π΄Π‘ — коэффициСнт мощности ΠΈΠ»ΠΈ усилСниС Π² Π΄Π‘.

    ΠžΡ‚Π½ΠΎΡˆΠ΅Π½ΠΈΠ΅ Π°ΠΌΠΏΠ»ΠΈΡ‚ΡƒΠ΄Ρ‹ ΠΊ ΠΏΡ€Π΅ΠΎΠ±Ρ€Π°Π·ΠΎΠ²Π°Π½ΠΈΡŽ Π² Π΄Π‘

    Для Π°ΠΌΠΏΠ»ΠΈΡ‚ΡƒΠ΄Ρ‹ Π²ΠΎΠ»Π½, Ρ‚Π°ΠΊΠΈΡ… ΠΊΠ°ΠΊ напряТСниС, Ρ‚ΠΎΠΊ ΠΈ ΡƒΡ€ΠΎΠ²Π΅Π½ΡŒ Π·Π²ΡƒΠΊΠΎΠ²ΠΎΠ³ΠΎ давлСния:

    G Π΄Π‘ = 20 log 10 ( A 2 / A 1 )

    A 2 — ΡƒΡ€ΠΎΠ²Π΅Π½ΡŒ Π°ΠΌΠΏΠ»ΠΈΡ‚ΡƒΠ΄Ρ‹.

    A 1 — эталонный ΡƒΡ€ΠΎΠ²Π΅Π½ΡŒ Π°ΠΌΠΏΠ»ΠΈΡ‚ΡƒΠ΄Ρ‹.

    G Π΄Π‘ — ΠΎΡ‚Π½ΠΎΡˆΠ΅Π½ΠΈΠ΅ Π°ΠΌΠΏΠ»ΠΈΡ‚ΡƒΠ΄ ΠΈΠ»ΠΈ усилСниС Π² Π΄Π‘.

    Π΄Π‘ для прСобразования ΠΎΡ‚Π½ΠΎΡˆΠ΅Π½ΠΈΡ Π°ΠΌΠΏΠ»ΠΈΡ‚ΡƒΠ΄Ρ‹

    A 2 = A 1 β‹… 10 ( G Π΄Π‘ / 20)

    A 2 — ΡƒΡ€ΠΎΠ²Π΅Π½ΡŒ Π°ΠΌΠΏΠ»ΠΈΡ‚ΡƒΠ΄Ρ‹.

    A 1 — эталонный ΡƒΡ€ΠΎΠ²Π΅Π½ΡŒ Π°ΠΌΠΏΠ»ΠΈΡ‚ΡƒΠ΄Ρ‹.

    G Π΄Π‘ — ΠΎΡ‚Π½ΠΎΡˆΠ΅Π½ΠΈΠ΅ Π°ΠΌΠΏΠ»ΠΈΡ‚ΡƒΠ΄ ΠΈΠ»ΠΈ усилСниС Π² Π΄Π‘.

    ΠŸΡ€ΠΈΠΌΠ΅Ρ€

    НайдитС Π²Ρ‹Ρ…ΠΎΠ΄Π½ΠΎΠ΅ напряТСниС для систСмы с Π²Ρ…ΠΎΠ΄Π½Ρ‹ΠΌ напряТСниСм 5 Π’ ΠΈ усилСниСм ΠΏΠΎ Π½Π°ΠΏΡ€ΡΠΆΠ΅Π½ΠΈΡŽ 6 Π΄Π‘.

    Π’ Π²Ρ‹Ρ…ΠΎΠ΄ = Π’ дюйм 10 ( G Π΄Π‘ /20) = 5 Π’ β‹… 10 (6 Π΄Π‘ / 20) = 9,976 Π’ β‰ˆ 10 Π’

    ΠšΠΎΡΡ„Ρ„ΠΈΡ†ΠΈΠ΅Π½Ρ‚ усилСния ΠΏΠΎ Π½Π°ΠΏΡ€ΡΠΆΠ΅Π½ΠΈΡŽ

    ΠšΠΎΡΡ„Ρ„ΠΈΡ†ΠΈΠ΅Π½Ρ‚ усилСния ΠΏΠΎ Π½Π°ΠΏΡ€ΡΠΆΠ΅Π½ΠΈΡŽ ( G, , Π΄Π‘, ) Π² 20 Ρ€Π°Π· ΠΏΡ€Π΅Π²Ρ‹ΡˆΠ°Π΅Ρ‚ Π»ΠΎΠ³Π°Ρ€ΠΈΡ„ΠΌ ΠΏΠΎ основанию 10 ΠΎΡ‚Π½ΠΎΡˆΠ΅Π½ΠΈΡ Π²Ρ‹Ρ…ΠΎΠ΄Π½ΠΎΠ³ΠΎ напряТСния ( Π’, , , Π²Ρ‹Ρ…ΠΎΠ΄ ) ΠΈ Π²Ρ…ΠΎΠ΄Π½ΠΎΠ³ΠΎ напряТСния ( Π’, , , дюйм ):

    G Π΄Π‘ = 20β‹…log 10 ( Π’ Π½Π° Π²Ρ‹Ρ…ΠΎΠ΄Π΅ / Π’ дюйм )

    ВСкущая ΠΏΡ€ΠΈΠ±Ρ‹Π»ΡŒ

    ΠšΠΎΡΡ„Ρ„ΠΈΡ†ΠΈΠ΅Π½Ρ‚ усилСния ΠΏΠΎ Ρ‚ΠΎΠΊΡƒ ( G Π΄Π‘ ) Π² 20 Ρ€Π°Π· большС Π»ΠΎΠ³Π°Ρ€ΠΈΡ„ΠΌΠ° ΠΏΠΎ основанию 10 ΠΎΡ‚Π½ΠΎΡˆΠ΅Π½ΠΈΡ Π²Ρ‹Ρ…ΠΎΠ΄Π½ΠΎΠ³ΠΎ Ρ‚ΠΎΠΊΠ° ( I ΠΈΠ· ) ΠΈ Π²Ρ…ΠΎΠ΄Π½ΠΎΠ³ΠΎ Ρ‚ΠΎΠΊΠ° ( I Π² ):

    G Π΄Π‘ = 20β‹…log 10 ( I out / I дюйм )

    АкустичСскоС усилСниС

    АкустичСскоС усилСниС слухового Π°ΠΏΠΏΠ°Ρ€Π°Ρ‚Π° ( G Π΄Π‘ ) Π² 20 Ρ€Π°Π· большС Π»ΠΎΠ³Π°Ρ€ΠΈΡ„ΠΌΠ° ΠΏΠΎ основанию 10 ΠΎΡ‚Π½ΠΎΡˆΠ΅Π½ΠΈΡ Π²Ρ‹Ρ…ΠΎΠ΄Π½ΠΎΠ³ΠΎ уровня Π·Π²ΡƒΠΊΠ° ( L ΠΈΠ· ) ΠΈ Π²Ρ…ΠΎΠ΄Π½ΠΎΠΉ ΡƒΡ€ΠΎΠ²Π΅Π½ΡŒ Π·Π²ΡƒΠΊΠ° ( L, , , ).

    G Π΄Π‘ = 20β‹…log 10 ( L Π½Π° Π²Ρ‹Ρ…ΠΎΠ΄Π΅ / L дюйм )

    ΠžΡ‚Π½ΠΎΡˆΠ΅Π½ΠΈΠ΅ сигнал / ΡˆΡƒΠΌ (SNR)

    ΠžΡ‚Π½ΠΎΡˆΠ΅Π½ΠΈΠ΅ сигнал / ΡˆΡƒΠΌ ( SNR Π΄Π‘ ) Π² 10 Ρ€Π°Π· большС Π»ΠΎΠ³Π°Ρ€ΠΈΡ„ΠΌΠ° ΠΏΠΎ основанию 10 Π°ΠΌΠΏΠ»ΠΈΡ‚ΡƒΠ΄Ρ‹ сигнала ( A сигнал ) ΠΈ Π°ΠΌΠΏΠ»ΠΈΡ‚ΡƒΠ΄Π° ΡˆΡƒΠΌΠ° ( A noise ):

    SNR Π΄Π‘ = 10β‹…log 10 ( A сигнал / A ΡˆΡƒΠΌ )

    ΠΠ±ΡΠΎΠ»ΡŽΡ‚Π½Ρ‹Π΅ Π΅Π΄ΠΈΠ½ΠΈΡ†Ρ‹ Π² Π΄Π΅Ρ†ΠΈΠ±Π΅Π»Π°Ρ…

    ΠΠ±ΡΠΎΠ»ΡŽΡ‚Π½Ρ‹Π΅ Π΅Π΄ΠΈΠ½ΠΈΡ†Ρ‹ Π΄Π΅Ρ†ΠΈΠ±Π΅Π» относятся ΠΊ ΠΎΠΏΡ€Π΅Π΄Π΅Π»Π΅Π½Π½ΠΎΠΉ Π²Π΅Π»ΠΈΡ‡ΠΈΠ½Π΅ Π΅Π΄ΠΈΠ½ΠΈΡ†Ρ‹ измСрСния:

    Π‘Π»ΠΎΠΊ Имя НомСр ссылки Кол. Акций ΠšΠΎΡΡ„Ρ„ΠΈΡ†ΠΈΠ΅Π½Ρ‚
    Π΄Π‘ΠΌ Π΄Π΅Ρ†ΠΈΠ±Π΅Π» ΠΌΠΈΠ»Π»ΠΈΠ²Π°Ρ‚Ρ‚ 1 ΠΌΠ’Ρ‚ элСктричСская ΠΌΠΎΡ‰Π½ΠΎΡΡ‚ΡŒ ΠΏΠ΅Ρ€Π΅Π΄Π°Ρ‚ΠΎΡ‡Π½ΠΎΠ΅ число
    Π΄Π‘Π’Ρ‚ Π΄Π΅Ρ†ΠΈΠ±Π΅Π» Π²Π°Ρ‚Ρ‚ 1 Π’Ρ‚ элСктричСская ΠΌΠΎΡ‰Π½ΠΎΡΡ‚ΡŒ ΠΏΠ΅Ρ€Π΅Π΄Π°Ρ‚ΠΎΡ‡Π½ΠΎΠ΅ число
    Π΄Π‘rn ΠΎΠΏΠΎΡ€Π½Ρ‹ΠΉ ΡˆΡƒΠΌ Π² Π΄Π΅Ρ†ΠΈΠ±Π΅Π»Π°Ρ… 1ΠΏΠ’Ρ‚ элСктричСская ΠΌΠΎΡ‰Π½ΠΎΡΡ‚ΡŒ ΠΏΠ΅Ρ€Π΅Π΄Π°Ρ‚ΠΎΡ‡Π½ΠΎΠ΅ число
    Π΄Π‘ΠΌΠΊΠ’ Π΄Π΅Ρ†ΠΈΠ±Π΅Π» ΠΌΠΈΠΊΡ€ΠΎΠ²ΠΎΠ»ΡŒΡ‚ 1 ΠΌΠΊΠ’ Π‘ΠšΠ— напряТСниС ΠΎΡ‚Π½ΠΎΡˆΠ΅Π½ΠΈΠ΅ Π°ΠΌΠΏΠ»ΠΈΡ‚ΡƒΠ΄
    Π΄Π‘ΠΌΠ’ Π΄Π΅Ρ†ΠΈΠ±Π΅Π» ΠΌΠΈΠ»Π»ΠΈΠ²ΠΎΠ»ΡŒΡ‚ 1 ΠΌΠ’ RMS напряТСниС ΠΎΡ‚Π½ΠΎΡˆΠ΅Π½ΠΈΠ΅ Π°ΠΌΠΏΠ»ΠΈΡ‚ΡƒΠ΄
    Π΄Π‘Π’ Π΄Π΅Ρ†ΠΈΠ±Π΅Π» Π²ΠΎΠ»ΡŒΡ‚ 1 Π’ RMS напряТСниС ΠΎΡ‚Π½ΠΎΡˆΠ΅Π½ΠΈΠ΅ Π°ΠΌΠΏΠ»ΠΈΡ‚ΡƒΠ΄
    Π΄Π‘Π½ Π΄Π΅Ρ†ΠΈΠ±Π΅Π» Π±Π΅Π· Π½Π°Π³Ρ€ΡƒΠ·ΠΊΠΈ 0.775 Π’ RMS напряТСниС ΠΎΡ‚Π½ΠΎΡˆΠ΅Π½ΠΈΠ΅ Π°ΠΌΠΏΠ»ΠΈΡ‚ΡƒΠ΄
    Π΄Π‘Z Π΄Π΅Ρ†ΠΈΠ±Π΅Π» Z 1 ΠΌΠΊΠΌ 3 ΠΎΡ‚Ρ€Π°ΠΆΠ°Ρ‚Π΅Π»ΡŒΠ½Π°Ρ ΡΠΏΠΎΡΠΎΠ±Π½ΠΎΡΡ‚ΡŒ ΠΎΡ‚Π½ΠΎΡˆΠ΅Π½ΠΈΠ΅ Π°ΠΌΠΏΠ»ΠΈΡ‚ΡƒΠ΄
    дБмкА Π΄Π΅Ρ†ΠΈΠ±Π΅Π» ΠΌΠΈΠΊΡ€ΠΎΠ°ΠΌΠΏΠ΅Ρ€ 1 мкА Ρ‚Π΅ΠΊΡƒΡ‰ΠΈΠΉ ΠΎΡ‚Π½ΠΎΡˆΠ΅Π½ΠΈΠ΅ Π°ΠΌΠΏΠ»ΠΈΡ‚ΡƒΠ΄
    Π΄Π‘ΠžΠΌ Π΄Π΅Ρ†ΠΈΠ±Π΅Π» Ом 1 Ом сопротивлСниС ΠΎΡ‚Π½ΠΎΡˆΠ΅Π½ΠΈΠ΅ Π°ΠΌΠΏΠ»ΠΈΡ‚ΡƒΠ΄
    Π΄Π‘Π“Ρ† Π΄Π΅Ρ†ΠΈΠ±Π΅Π» Π³Π΅Ρ€Ρ† 1 Π“Ρ† частота ΠΏΠ΅Ρ€Π΅Π΄Π°Ρ‚ΠΎΡ‡Π½ΠΎΠ΅ число
    Π΄Π‘SPL Π΄Π΅Ρ†ΠΈΠ±Π΅Π» ΡƒΡ€ΠΎΠ²Π΅Π½ΡŒ Π·Π²ΡƒΠΊΠΎΠ²ΠΎΠ³ΠΎ давлСния 20 мкПа Π·Π²ΡƒΠΊΠΎΠ²ΠΎΠ΅ Π΄Π°Π²Π»Π΅Π½ΠΈΠ΅ ΠΎΡ‚Π½ΠΎΡˆΠ΅Π½ΠΈΠ΅ Π°ΠΌΠΏΠ»ΠΈΡ‚ΡƒΠ΄
    дБА Π΄Π΅Ρ†ΠΈΠ±Π΅Π» ΠΏΠΎ шкалС А 20 мкПа Π·Π²ΡƒΠΊΠΎΠ²ΠΎΠ΅ Π΄Π°Π²Π»Π΅Π½ΠΈΠ΅ ΠΎΡ‚Π½ΠΎΡˆΠ΅Π½ΠΈΠ΅ Π°ΠΌΠΏΠ»ΠΈΡ‚ΡƒΠ΄

    ΠžΡ‚Π½ΠΎΡΠΈΡ‚Π΅Π»ΡŒΠ½Ρ‹Π΅ Π΄Π΅Ρ†ΠΈΠ±Π΅Π»Ρ‹

    Π΄Π΅Ρ†ΠΈΠ±Π΅Π»
    Π‘Π»ΠΎΠΊ Имя НомСр ссылки Кол. Акций ΠšΠΎΡΡ„Ρ„ΠΈΡ†ΠΈΠ΅Π½Ρ‚
    Π΄Π‘ дСцибСл–– ΠΌΠΎΡ‰Π½ΠΎΡΡ‚ΡŒ / ΠΏΠΎΠ»Π΅
    Π΄Π‘Π½ нСсущая нСсущая ΠΌΠΎΡ‰Π½ΠΎΡΡ‚ΡŒ элСктричСская ΠΌΠΎΡ‰Π½ΠΎΡΡ‚ΡŒ ΠΏΠ΅Ρ€Π΅Π΄Π°Ρ‚ΠΎΡ‡Π½ΠΎΠ΅ число
    Π΄Π‘ΠΈ Π΄Π΅Ρ†ΠΈΠ±Π΅Π» ΠΈΠ·ΠΎΡ‚Ρ€ΠΎΠΏΠ½Ρ‹ΠΉ изотропная ΠΏΠ»ΠΎΡ‚Π½ΠΎΡΡ‚ΡŒ мощности Π°Π½Ρ‚Π΅Π½Π½Ρ‹ ΡƒΠ΄Π΅Π»ΡŒΠ½Π°Ρ ΠΌΠΎΡ‰Π½ΠΎΡΡ‚ΡŒ ΠΏΠ΅Ρ€Π΅Π΄Π°Ρ‚ΠΎΡ‡Π½ΠΎΠ΅ число
    Π΄Π‘FS Π΄Π΅Ρ†ΠΈΠ±Π΅Π» полная шкала полная цифровая шкала напряТСниС ΠΎΡ‚Π½ΠΎΡˆΠ΅Π½ΠΈΠ΅ Π°ΠΌΠΏΠ»ΠΈΡ‚ΡƒΠ΄
    Π΄Π‘rn ΠΎΠΏΠΎΡ€Π½Ρ‹ΠΉ ΡˆΡƒΠΌ Π² Π΄Π΅Ρ†ΠΈΠ±Π΅Π»Π°Ρ…

    Π¨ΡƒΠΌΠΎΠΌΠ΅Ρ€

    Π˜Π·ΠΌΠ΅Ρ€ΠΈΡ‚Π΅Π»ΡŒ уровня Π·Π²ΡƒΠΊΠ° ΠΈΠ»ΠΈ Π˜Π·ΠΌΠ΅Ρ€ΠΈΡ‚Π΅Π»ΡŒ SPL — это устройство, ΠΊΠΎΡ‚ΠΎΡ€ΠΎΠ΅ измСряСт ΡƒΡ€ΠΎΠ²Π΅Π½ΡŒ Π·Π²ΡƒΠΊΠΎΠ²ΠΎΠ³ΠΎ давлСния (SPL) Π·Π²ΡƒΠΊΠΎΠ²Ρ‹Ρ… Π²ΠΎΠ»Π½ Π² Π΄Π΅Ρ†ΠΈΠ±Π΅Π»Π°Ρ…. (Π΄Π‘-Π£Π—Π”) Π΅Π΄ΠΈΠ½ΠΈΡ†.

    Π˜Π·ΠΌΠ΅Ρ€ΠΈΡ‚Π΅Π»ΡŒ

    SPL ΠΈΡΠΏΠΎΠ»ΡŒΠ·ΡƒΠ΅Ρ‚ΡΡ для тСстирования ΠΈ измСрСния громкости Π·Π²ΡƒΠΊΠΎΠ²Ρ‹Ρ… Π²ΠΎΠ»Π½, Π° Ρ‚Π°ΠΊΠΆΠ΅ для ΠΌΠΎΠ½ΠΈΡ‚ΠΎΡ€ΠΈΠ½Π³Π° ΡˆΡƒΠΌΠΎΠ²ΠΎΠ³ΠΎ загрязнСния.

    Π•Π΄ΠΈΠ½ΠΈΡ†Π° измСрСния уровня Π·Π²ΡƒΠΊΠΎΠ²ΠΎΠ³ΠΎ давлСния — паскаль (Па), Π° Π² логарифмичСской шкалС ΠΈΡΠΏΠΎΠ»ΡŒΠ·ΡƒΠ΅Ρ‚ΡΡ Π΄Π‘-Π£Π—Π”.

    Π΄Π‘-SPL Ρ‚Π°Π±Π»ΠΈΡ†Π°

    Π’Π°Π±Π»ΠΈΡ†Π° ΠΎΠ±Ρ‰ΠΈΡ… ΡƒΡ€ΠΎΠ²Π½Π΅ΠΉ Π·Π²ΡƒΠΊΠΎΠ²ΠΎΠ³ΠΎ давлСния Π² Π΄Π‘SPL:

    Π’ΠΈΠΏ Π·Π²ΡƒΠΊΠ° Π£Ρ€ΠΎΠ²Π΅Π½ΡŒ ΡˆΡƒΠΌΠ° (Π΄Π‘-Π£Π—Π”)
    ΠŸΠΎΡ€ΠΎΠ³ ΡΠ»Ρ‹ΡˆΠΈΠΌΠΎΡΡ‚ΠΈ 0 Π΄Π‘SPL
    Π¨Π΅ΠΏΠΎΡ‚ 30 Π΄Π‘ SPL
    ΠšΠΎΠ½Π΄ΠΈΡ†ΠΈΠΎΠ½Π΅Ρ€ 50-70 Π΄Π‘ SPL
    Π Π°Π·Π³ΠΎΠ²ΠΎΡ€ 50-70 Π΄Π‘ SPL
    Π’Ρ€Π°Ρ„ΠΈΠΊ 60-85 Π΄Π‘ SPL
    Громкая ΠΌΡƒΠ·Ρ‹ΠΊΠ° 90-110 Π΄Π‘ SPL
    Π‘Π°ΠΌΠΎΠ»Π΅Ρ‚ 120-140 Π΄Π‘ SPL

    Π΄Π‘ Π² Ρ‚Π°Π±Π»ΠΈΡ†Π΅ прСобразования

    Π΄Π‘ Π‘ΠΎΠΎΡ‚Π½ΠΎΡˆΠ΅Π½ΠΈΠ΅ Π°ΠΌΠΏΠ»ΠΈΡ‚ΡƒΠ΄ ΠŸΠ΅Ρ€Π΅Π΄Π°Ρ‚ΠΎΡ‡Π½ΠΎΠ΅ число
    -100 Π΄Π‘ 10 -5 10 -10
    -50 Π΄Π‘ 0.00316 0,00001
    -40 Π΄Π‘ 0,010 0,0001
    -30 Π΄Π‘ 0,032 0,001
    -20 Π΄Π‘ 0,1 0,01
    -10 Π΄Π‘ 0,316 0,1
    -6 Π΄Π‘ 0,501 0,251
    -3 Π΄Π‘ 0,708 0,501
    -2 Π΄Π‘ 0.794 0,631
    -1 Π΄Π‘ 0,891 0,794
    0 Π΄Π‘ 1 1
    1 Π΄Π‘ 1,122 1,259
    2 Π΄Π‘ 1,259 1,585
    3 Π΄Π‘ 1,413 2 β‰ˆ 1,995
    6 Π΄Π‘ 2 β‰ˆ 1,995 3,981
    10 Π΄Π‘ 3.162 10
    20 Π΄Π‘ 10 100
    30 Π΄Π‘ 31,623 1000
    40 Π΄Π‘ 100 10000
    50 Π΄Π‘ 316,228 100000
    100 Π΄Π‘ 10 5 10 10

    Π΄Π‘ΠΌ β–Ί


    Π‘ΠΌ. Π’Π°ΠΊΠΆΠ΅

    УсилСниС

    ΠΈ Π΄Π΅Ρ†ΠΈΠ±Π΅Π»Ρ‹

    • Π˜Π·ΡƒΡ‡ΠΈΠ² этот Ρ€Π°Π·Π΄Π΅Π», Π²Ρ‹ смоТСтС:
    • ΠžΠΏΠΈΡˆΠΈΡ‚Π΅ усилСниС напряТСния ΠΊΠ°ΠΊ коэффициСнт.
    • Π‘Ρ€Π°Π²Π½ΠΈΡ‚Π΅ Π»ΠΈΠ½Π΅ΠΉΠ½ΡƒΡŽ ΠΈ Π»ΠΎΠ³Π°Ρ€ΠΈΡ„ΠΌΠΈΡ‡Π΅ΡΠΊΡƒΡŽ ΡˆΠΊΠ°Π»Ρ‹.
    • ΠžΠΏΠΈΡˆΠΈΡ‚Π΅ ΠΎΡ‚Π½ΠΎΡˆΠ΅Π½ΠΈΡ Π² Π΄Π΅Ρ†ΠΈΠ±Π΅Π»Π°Ρ….
    • β€’ ΠŸΠΎΠ»ΠΎΠΆΠΈΡ‚Π΅Π»ΡŒΠ½Ρ‹Π΅ ΠΈ ΠΎΡ‚Ρ€ΠΈΡ†Π°Ρ‚Π΅Π»ΡŒΠ½Ρ‹Π΅ значСния Π΄Π΅Ρ†ΠΈΠ±Π΅Π».
    • β€’ ΠŸΡ€Π΅ΠΎΠ±Ρ€Π°Π·ΠΎΠ²Π°Π½ΠΈΠ΅ усилСния мощности Π² Π΄Π΅Ρ†ΠΈΠ±Π΅Π»Ρ‹.
    • β€’ ΠŸΡ€Π΅ΠΎΠ±Ρ€Π°Π·ΡƒΠΉΡ‚Π΅ усилСниС напряТСния Π² Π΄Π΅Ρ†ΠΈΠ±Π΅Π»Ρ‹.
    • β€’ Π Π°ΡΠΏΠΎΠ·Π½Π°Π²Π°Ρ‚ΡŒ ΠΎΠ±Ρ‹Ρ‡Π½ΠΎ ΠΈΡΠΏΠΎΠ»ΡŒΠ·ΡƒΠ΅ΠΌΡ‹Π΅ значСния Π² Π΄Π‘.

    УсилСниС напряТСния

    ΠšΠΎΡΡ„Ρ„ΠΈΡ†ΠΈΠ΅Π½Ρ‚ усилСния напряТСния (Av) ΠΈΠ»ΠΈ усилСния усилитСля напряТСния опрСдСляСтся ΠΏΠΎ Ρ„ΠΎΡ€ΠΌΡƒΠ»Π΅:

    ΠŸΡ€ΠΈ ΠΎΠ±ΠΎΠΈΡ… напряТСниях, ΠΈΠ·ΠΌΠ΅Ρ€Π΅Π½Π½Ρ‹Ρ… ΠΎΠ΄ΠΈΠ½Π°ΠΊΠΎΠ²ΠΎ (Ρ‚.Π΅. ΠΎΠ±Π° RMS, ΠΎΠ±Π° Peak ΠΈΠ»ΠΈ ΠΎΠ±Π° Peak to Peak), Av — это ΠΎΡ‚Π½ΠΎΡˆΠ΅Π½ΠΈΠ΅ Ρ‚ΠΎΠ³ΠΎ, насколько Π²Ρ‹Π²ΠΎΠ΄ большС, Ρ‡Π΅ΠΌ Π²Π²ΠΎΠ΄, ΠΈ поэтому Π½Π΅ ΠΈΠΌΠ΅Π΅Ρ‚ Π΅Π΄ΠΈΠ½ΠΈΡ† измСрСния. Π­Ρ‚ΠΎ основная ΠΌΠ΅Ρ€Π° усилСния ΠΈΠ»ΠΈ эффСктивности усилитСля.

    ΠŸΠΎΡΠΊΠΎΠ»ΡŒΠΊΡƒ Π²Ρ‹Ρ…ΠΎΠ΄Π½ΠΎΠΉ сигнал усилитСля измСняСтся ΠΏΡ€ΠΈ Ρ€Π°Π·Π½Ρ‹Ρ… частотах сигнала, измСрСния Π²Ρ‹Ρ…ΠΎΠ΄Π½ΠΎΠΉ мощности ΠΈΠ»ΠΈ часто напряТСния, ΠΊΠΎΡ‚ΠΎΡ€ΠΎΠ΅ Π»Π΅Π³Ρ‡Π΅ ΠΈΠ·ΠΌΠ΅Ρ€ΠΈΡ‚ΡŒ, Ρ‡Π΅ΠΌ ΠΌΠΎΡ‰Π½ΠΎΡΡ‚ΡŒ, наносятся Π½Π° Π³Ρ€Π°Ρ„ΠΈΠΊ Π² зависимости ΠΎΡ‚ частоты Π½Π° Π³Ρ€Π°Ρ„ΠΈΠΊΠ΅ (кривая ΠΎΡ‚ΠΊΠ»ΠΈΠΊΠ°), Ρ‡Ρ‚ΠΎΠ±Ρ‹ ΠΏΠΎΠΊΠ°Π·Π°Ρ‚ΡŒ ΡΡ€Π°Π²Π½ΠΈΡ‚Π΅Π»ΡŒΠ½Ρ‹ΠΉ Π²Ρ‹Ρ…ΠΎΠ΄Π½ΠΎΠΉ сигнал Π² Ρ€Π°Π±ΠΎΡ‡Π΅ΠΌ Π΄ΠΈΠ°ΠΏΠ°Π·ΠΎΠ½Π΅ частот. усилитСля.

    ЛогарифмичСскиС вСсы

    ΠšΡ€ΠΈΠ²Ρ‹Π΅ ΠΎΡ‚ΠΊΠ»ΠΈΠΊΠ°

    ΠΎΠ±Ρ‹Ρ‡Π½ΠΎ ΠΈΡΠΏΠΎΠ»ΡŒΠ·ΡƒΡŽΡ‚ Π»ΠΎΠ³Π°Ρ€ΠΈΡ„ΠΌΠΈΡ‡Π΅ΡΠΊΡƒΡŽ ΡˆΠΊΠ°Π»Ρƒ частоты, Π½Π°Π½Π΅ΡΠ΅Π½Π½ΡƒΡŽ Π½Π° Π³ΠΎΡ€ΠΈΠ·ΠΎΠ½Ρ‚Π°Π»ΡŒΠ½ΡƒΡŽ ось абсцисс. Π­Ρ‚ΠΎ позволяСт ΠΈΡΠΏΠΎΠ»ΡŒΠ·ΠΎΠ²Π°Ρ‚ΡŒ Π±ΠΎΠ»Π΅Π΅ ΡˆΠΈΡ€ΠΎΠΊΠΈΠΉ Π΄ΠΈΠ°ΠΏΠ°Π·ΠΎΠ½ частот, Ρ‡Π΅ΠΌ ΠΏΡ€ΠΈ использовании Π»ΠΈΠ½Π΅ΠΉΠ½ΠΎΠΉ ΡˆΠΊΠ°Π»Ρ‹.

    Рис. 1.3.1 ЛогарифмичСская ΠΈ линСйная ΡˆΠΊΠ°Π»Ρ‹ Π² сравнСнии

    Π’Π΅Ρ€Ρ‚ΠΈΠΊΠ°Π»ΡŒΠ½Π°Ρ ось Y Ρ€Π°Π·ΠΌΠ΅Ρ‡Π΅Π½Π° Π² Π²ΠΈΠ΄Π΅ Π»ΠΈΠ½Π΅ΠΉΠ½Ρ‹Ρ… Π΄Π΅Π»Π΅Π½ΠΈΠΉ, Π½ΠΎ с использованиСм логарифмичСских Π΅Π΄ΠΈΠ½ΠΈΡ† Π² Π΄Π΅Ρ†ΠΈΠ±Π΅Π»Π°Ρ…, Ρ‡Ρ‚ΠΎ обСспСчиваСт Π³ΠΎΡ€Π°Π·Π΄ΠΎ больший Π΄ΠΈΠ°ΠΏΠ°Π·ΠΎΠ½ Π½Π° Ρ‚ΠΎΠΌ ΠΆΠ΅ расстоянии. Π˜ΡΠΏΠΎΠ»ΡŒΠ·ΡƒΠ΅ΠΌΠ°Ρ логарифмичСская Π΅Π΄ΠΈΠ½ΠΈΡ†Π° — Π΄Π΅Ρ†ΠΈΠ±Π΅Π», ΠΊΠΎΡ‚ΠΎΡ€Ρ‹ΠΉ составляСт ΠΎΠ΄Π½Ρƒ Π΄Π΅ΡΡΡ‚ΡƒΡŽ Π±Π΅Π»Π°, Π΅Π΄ΠΈΠ½ΠΈΡ†Π° измСрСния, ΠΈΠ·Π½Π°Ρ‡Π°Π»ΡŒΠ½ΠΎ прСдназначСнная для измСрСния ΠΏΠΎΡ‚Π΅Ρ€ΡŒ Π² Ρ‚Π΅Π»Π΅Ρ„ΠΎΠ½Π½Ρ‹Ρ… кабСлях, Π½ΠΎ ΠΏΠΎΡΠΊΠΎΠ»ΡŒΠΊΡƒ Π‘Π΅Π» ΠΎΠ±Ρ‹Ρ‡Π½ΠΎ слишком Π²Π΅Π»ΠΈΠΊ для Π±ΠΎΠ»ΡŒΡˆΠΈΠ½ΡΡ‚Π²Π° элСктронных ΠΏΡ€ΠΈΠΌΠ΅Π½Π΅Π½ΠΈΠΉ, Π΄Π΅Ρ†ΠΈΠ±Π΅Π» (Π΄Π‘) являСтся Π΅Π΄ΠΈΠ½ΠΈΡ†Π΅ΠΉ Π²Ρ‹Π±ΠΎΡ€Π°. .Помимо обСспСчСния Π±ΠΎΠ»Π΅Π΅ ΡƒΠ΄ΠΎΠ±Π½ΠΎΠΉ ΡˆΠΊΠ°Π»Ρ‹, Π΄Π΅Ρ†ΠΈΠ±Π΅Π» ΠΈΠΌΠ΅Π΅Ρ‚ Π΅Ρ‰Π΅ ΠΎΠ΄Π½ΠΎ прСимущСство ΠΏΡ€ΠΈ ΠΎΡ‚ΠΎΠ±Ρ€Π°ΠΆΠ΅Π½ΠΈΠΈ Π·Π²ΡƒΠΊΠΎΠ²ΠΎΠΉ ΠΈΠ½Ρ„ΠΎΡ€ΠΌΠ°Ρ†ΠΈΠΈ, чСловСчСскоС ΡƒΡ…ΠΎ Ρ‚Π°ΠΊΠΆΠ΅ Ρ€Π΅Π°Π³ΠΈΡ€ΡƒΠ΅Ρ‚ Π½Π° Π³Ρ€ΠΎΠΌΠΊΠΎΡΡ‚ΡŒ Π·Π²ΡƒΠΊΠ° Π°Π½Π°Π»ΠΎΠ³ΠΈΡ‡Π½ΠΎ логарифмичСской шкалС, поэтому использованиС ΡˆΠΊΠ°Π»Ρ‹ Π΄Π΅Ρ†ΠΈΠ±Π΅Π» Π΄Π°Π΅Ρ‚ Π±ΠΎΠ»Π΅Π΅ Π·Π½Π°Ρ‡ΠΈΠΌΠΎΠ΅ прСдставлСниС ΡƒΡ€ΠΎΠ²Π½Π΅ΠΉ Π·Π²ΡƒΠΊΠ°.

    Рис. 1.3.2 ΠšΡ€ΠΈΠ²Π°Ρ ΠΎΡ‚ΠΊΠ»ΠΈΠΊΠ° мощности Π·Π²ΡƒΠΊΠ°

    ΠŸΡ€ΠΈΡ€ΠΎΡΡ‚ мощности Π² дБс

    Π§Ρ‚ΠΎΠ±Ρ‹ ΠΎΠΏΠΈΡΠ°Ρ‚ΡŒ ΠΈΠ·ΠΌΠ΅Π½Π΅Π½ΠΈΠ΅ Π²Ρ‹Ρ…ΠΎΠ΄Π½ΠΎΠΉ мощности Π²ΠΎ всСм частотном Π΄ΠΈΠ°ΠΏΠ°Π·ΠΎΠ½Π΅ усилитСля, кривая ΠΎΡ‚ΠΊΠ»ΠΈΠΊΠ° Π² Π΄Π΅Ρ†ΠΈΠ±Π΅Π»Π°Ρ… ΠΈΡΠΏΠΎΠ»ΡŒΠ·ΡƒΠ΅Ρ‚ΡΡ для отобраТСния ΠΈΠ·ΠΌΠ΅Π½Π΅Π½ΠΈΠΉ Π²Ρ‹Ρ…ΠΎΠ΄Π½ΠΎΠΉ мощности.ΠœΠΎΡ‰Π½ΠΎΡΡ‚ΡŒ Π½Π° Ρ€Π°Π·Π»ΠΈΡ‡Π½Ρ‹Ρ… частотах Π²ΠΎ всСм Π΄ΠΈΠ°ΠΏΠ°Π·ΠΎΠ½Π΅ сравниваСтся с ΠΎΠΏΡ€Π΅Π΄Π΅Π»Π΅Π½Π½ΠΎΠΉ ΠΎΠΏΠΎΡ€Π½ΠΎΠΉ частотой (срСднСй частотой полосы). Π Π°Π·Π½ΠΈΡ†Π° Π² мощности Π½Π° срСднСй частотС полосы ΠΈ мощности Π½Π° любой Π΄Ρ€ΡƒΠ³ΠΎΠΉ измСряСмой частотС выраТаСтся Π² количСствС Π΄Π΅Ρ†ΠΈΠ±Π΅Π» большС (+ Π΄Π‘) ΠΈΠ»ΠΈ мСньшС (-Π΄Π‘), Ρ‡Π΅ΠΌ частота срСднСй полосы, ΠΊΠΎΡ‚ΠΎΡ€ΠΎΠΉ присвоСно Π·Π½Π°Ρ‡Π΅Π½ΠΈΠ΅ 0 Π΄Π‘. ΠžΠ±Ρ€Π°Ρ‚ΠΈΡ‚Π΅ Π²Π½ΠΈΠΌΠ°Π½ΠΈΠ΅, Ρ‡Ρ‚ΠΎ Π½Π° логарифмичСской шкалС частот Π½Π° рис. 1.3.1 сСрСдина Π΄ΠΈΠ°ΠΏΠ°Π·ΠΎΠ½Π° ΠΎΡ‚ 10 Π“Ρ† Π΄ΠΎ 100 ΠΊΠ“Ρ† составляСт 1 ΠΊΠ“Ρ†, ΠΈ частоты ΠΎΠΊΠΎΠ»ΠΎ этого числа (Π³Π΄Π΅ Π²Ρ‹Ρ…ΠΎΠ΄Π½ΠΎΠΉ сигнал ΠΎΠ±Ρ‹Ρ‡Π½ΠΎ максималСн) ΠΎΠ±Ρ‹Ρ‡Π½ΠΎ Π²Ρ‹Π±ΠΈΡ€Π°ΡŽΡ‚ΡΡ Π² качСствС ΠΎΠΏΠΎΡ€Π½ΠΎΠΉ частоты.

    ΠŸΡ€Π΅ΠΎΠ±Ρ€Π°Π·ΠΎΠ²Π°Π½ΠΈΠ΅ коэффициСнта усилСния мощности Π² Π΄Π‘ вычисляСтся ΠΏΡƒΡ‚Π΅ΠΌ умноТСния Π»ΠΎΠ³Π°Ρ€ΠΈΡ„ΠΌΠ° ΠΎΡ‚Π½ΠΎΡˆΠ΅Π½ΠΈΡ Π½Π° 10:

    Π“Π΄Π΅ P 1 — ΠΌΠΎΡ‰Π½ΠΎΡΡ‚ΡŒ Π² срСднСй полосС, Π° P 2 — измСряСмая ΠΌΠΎΡ‰Π½ΠΎΡΡ‚ΡŒ.

    ΠŸΡ€ΠΈΠΌΠ΅Ρ‡Π°Π½ΠΈΠ΅:

    ΠŸΡ€ΠΈ использовании этой Ρ„ΠΎΡ€ΠΌΡƒΠ»Ρ‹ Π² ΠΊΠ°Π»ΡŒΠΊΡƒΠ»ΡΡ‚ΠΎΡ€Π΅ Π²Π°ΠΆΠ½ΠΎ ΠΈΡΠΏΠΎΠ»ΡŒΠ·ΠΎΠ²Π°Ρ‚ΡŒ скобки, Ρ‚Π°ΠΊ Ρ‡Ρ‚ΠΎ ΠΈΡΠΏΠΎΠ»ΡŒΠ·ΡƒΠ΅Ρ‚ΡΡ 10-ΠΊΡ€Π°Ρ‚Π½Ρ‹ΠΉ Π»ΠΎΠ³Π°Ρ€ΠΈΡ„ΠΌ (P 1 / P 2 ), Π° Π½Π΅ 10-ΠΊΡ€Π°Ρ‚Π½Ρ‹ΠΉ Π»ΠΎΠ³Π°Ρ€ΠΈΡ„ΠΌ P 1 , Π΄Π΅Π»Π΅Π½Π½Ρ‹ΠΉ Π½Π° П 2 .

    Π½Π°ΠΏΡ€ΠΈΠΌΠ΅Ρ€ Ссли P 1 = 6 ΠΈ P 2 = 3

    10 x log (6/3) = 3 Π΄Π‘ (ΠΏΡ€Π°Π²ΠΈΠ»ΡŒΠ½Ρ‹ΠΉ ΠΎΡ‚Π²Π΅Ρ‚), Π½ΠΎ 10 x log 6/3 = 2.6 Π΄Π‘ (Π½Π΅ΠΏΡ€Π°Π²ΠΈΠ»ΡŒΠ½Ρ‹ΠΉ ΠΎΡ‚Π²Π΅Ρ‚).

    ΠšΠΎΡΡ„Ρ„ΠΈΡ†ΠΈΠ΅Π½Ρ‚ усилСния напряТСния Π² дБс

    Π₯отя ΠΎΠ±Ρ‹Ρ‡Π½ΠΎ коэффициСнт усилСния усилитСля ΠΏΠΎ Π½Π°ΠΏΡ€ΡΠΆΠ΅Π½ΠΈΡŽ описываСтся ΠΊΠ°ΠΊ ΠΎΡ‡Π΅Π½ΡŒ ΠΌΠ½ΠΎΠ³ΠΎ Π΄Π΅Ρ†ΠΈΠ±Π΅Π», это Π½Π΅ совсСм Ρ‚ΠΎΡ‡Π½ΠΎΠ΅ использованиС устройства. МоТно ΠΈΡΠΏΠΎΠ»ΡŒΠ·ΠΎΠ²Π°Ρ‚ΡŒ Π΄Π΅Ρ†ΠΈΠ±Π΅Π»Ρ‹ для сравнСния Π²Ρ‹Ρ…ΠΎΠ΄Π° усилитСля Π½Π° Ρ€Π°Π·Π½Ρ‹Ρ… частотах, ΠΏΠΎΡΠΊΠΎΠ»ΡŒΠΊΡƒ всС измСрСния Π²Ρ‹Ρ…ΠΎΠ΄Π½ΠΎΠΉ мощности ΠΈΠ»ΠΈ напряТСния проводятся Π½Π° ΠΎΠ΄Π½ΠΎΠΌ ΠΈ Ρ‚ΠΎΠΌ ΠΆΠ΅ импСдансС (Π½Π°Π³Ρ€ΡƒΠ·ΠΊΠ° усилитСля), Π½ΠΎ ΠΏΡ€ΠΈ описании усилСния напряТСния (ΠΌΠ΅ΠΆΠ΄Ρƒ Π²Ρ…ΠΎΠ΄ΠΎΠΌ ΠΈ Π²Ρ‹Ρ…ΠΎΠ΄ΠΎΠΌ) Π’ усилитСлС Π²Ρ…ΠΎΠ΄Π½ΠΎΠ΅ ΠΈ Π²Ρ‹Ρ…ΠΎΠ΄Π½ΠΎΠ΅ напряТСния ΠΈΠΌΠ΅ΡŽΡ‚ ΡΠΎΠ²Π΅Ρ€ΡˆΠ΅Π½Π½ΠΎ Ρ€Π°Π·Π½Ρ‹Π΅ импСдансы.Однако довольно ΡˆΠΈΡ€ΠΎΠΊΠΎ принято Ρ‚Π°ΠΊΠΆΠ΅ ΠΎΠΏΠΈΡΡ‹Π²Π°Ρ‚ΡŒ усилСниС напряТСния Π² Π΄Π΅Ρ†ΠΈΠ±Π΅Π»Π°Ρ….

    Рис. 1.3.3 ΠšΡ€ΠΈΠ²Π°Ρ ΠΎΡ‚ΠΊΠ»ΠΈΠΊΠ° Π·Π²ΡƒΠΊΠΎΠ²ΠΎΠ³ΠΎ напряТСния

    Когда коэффициСнт усилСния ΠΏΠΎ Π½Π°ΠΏΡ€ΡΠΆΠ΅Π½ΠΈΡŽ (A v ) ΠΈΠ»ΠΈ коэффициСнт усилСния ΠΏΠΎ Ρ‚ΠΎΠΊΡƒ (A i ) отобраТаСтся Π² зависимости ΠΎΡ‚ частоты, Π² Ρ‚ΠΎΡ‡ΠΊΠ°Ρ… –3 Π΄Π‘ коэффициСнт усилСния ΠΏΠ°Π΄Π°Π΅Ρ‚ Π΄ΠΎ 0,707 ΠΎΡ‚ максимального (срСдняя полоса) усилСния.

    ΠžΠ±Ρ€Π°Ρ‚ΠΈΡ‚Π΅ Π²Π½ΠΈΠΌΠ°Π½ΠΈΠ΅, Ρ‡Ρ‚ΠΎ для прСобразования ΠΎΡ‚Π½ΠΎΡˆΠ΅Π½ΠΈΠΉ напряТСний Π² Π΄Π‘ ΠΈΡΠΏΠΎΠ»ΡŒΠ·ΡƒΠ΅Ρ‚ΡΡ Π»ΠΎΠ³Π°Ρ€ΠΈΡ„ΠΌ 20 (Π’ Π½Π° Π²Ρ‹Ρ…ΠΎΠ΄Π΅ / Π’ Π½Π° )

    ОписаниС коэффициСнта усилСния ΠΏΠΎ Π½Π°ΠΏΡ€ΡΠΆΠ΅Π½ΠΈΡŽ усилитСля с Π²Ρ‹Ρ…ΠΎΠ΄Π½Ρ‹ΠΌ напряТСниСм 3.ΠΈ Ρ‚Π°ΠΊΠΆΠ΅ ΠΎΠ±Ρ‹Ρ‡Π½ΠΎ Shift + log. Π˜ΡΠΏΠΎΠ»ΡŒΠ·ΡƒΠΉΡ‚Π΅ Ρ‚Ρƒ ΠΆΠ΅ Ρ„ΠΎΡ€ΠΌΡƒΠ»Ρƒ для коэффициСнта усилСния Π² Π΄Π‘, Π° для прСобразования Π΄Π‘ Π² коэффициСнт мощности просто Π·Π°ΠΌΠ΅Π½ΠΈΡ‚Π΅ 20 Π² Ρ„ΠΎΡ€ΠΌΡƒΠ»Π΅ Π½Π° 10.

    ΠŸΡ€Π΅ΠΈΠΌΡƒΡ‰Π΅ΡΡ‚Π²ΠΎ использования Π΄Π‘ для ΠΈΠ½Π΄ΠΈΠΊΠ°Ρ†ΠΈΠΈ усилСния усилитСлСй состоит Π² Ρ‚ΠΎΠΌ, Ρ‡Ρ‚ΠΎ Π² многокаскадных усилитСлях ΠΎΠ±Ρ‰ΠΈΠΉ коэффициСнт усилСния сСрии усилитСлСй, Π²Ρ‹Ρ€Π°ΠΆΠ΅Π½Π½Ρ‹ΠΉ Π² простых ΡΠΎΠΎΡ‚Π½ΠΎΡˆΠ΅Π½ΠΈΡΡ…, Π±ΡƒΠ΄Π΅Ρ‚ ΠΏΡ€ΠΎΠΈΠ·Π²Π΅Π΄Π΅Π½ΠΈΠ΅ΠΌ ΠΎΡ‚Π΄Π΅Π»ΡŒΠ½Ρ‹Ρ… коэффициСнтов усилСния:

    Av1 x Av2 x Av3 x Av4 … ΠΈ Ρ‚. Π”.

    Π­Ρ‚ΠΎ ΠΌΠΎΠΆΠ΅Ρ‚ привСсти ΠΊ ΠΎΡ‡Π΅Π½ΡŒ большим числам, Π½ΠΎ ΠΎΠ±Ρ‰Π΅Π΅ ΠΈΠ½Π΄ΠΈΠ²ΠΈΠ΄ΡƒΠ°Π»ΡŒΠ½ΠΎΠ΅ усилСниС, Π²Ρ‹Ρ€Π°ΠΆΠ΅Π½Π½ΠΎΠ΅ Π² Π΄Π΅Ρ†ΠΈΠ±Π΅Π»Π°Ρ…, Π±ΡƒΠ΄Π΅Ρ‚ суммой ΠΈΠ½Π΄ΠΈΠ²ΠΈΠ΄ΡƒΠ°Π»ΡŒΠ½Ρ‹Ρ… усилСний:

    Av1 + Av2 + Av3 + Av4…Ρ‚Π°ΠΊ Π΄Π°Π»Π΅Π΅.

    Аналогичным ΠΎΠ±Ρ€Π°Π·ΠΎΠΌ Π²Ρ‹Ρ‡ΠΈΡ‚Π°ΡŽΡ‚ΡΡ ΠΏΠΎΡ‚Π΅Ρ€ΠΈ ΠΈΠ·-Π·Π° Ρ‚Π°ΠΊΠΈΡ… Ρ†Π΅ΠΏΠ΅ΠΉ, ΠΊΠ°ΠΊ Ρ„ΠΈΠ»ΡŒΡ‚Ρ€Ρ‹, Π°Ρ‚Ρ‚Π΅Π½ΡŽΠ°Ρ‚ΠΎΡ€Ρ‹ ΠΈ Ρ‚. Π”., Π§Ρ‚ΠΎΠ±Ρ‹ ΠΏΠΎΠ»ΡƒΡ‡ΠΈΡ‚ΡŒ ΠΎΠ±Ρ‰ΠΈΠ΅ ΠΏΠΎΡ‚Π΅Ρ€ΠΈ.

    Часто Π²ΡΡ‚Ρ€Π΅Ρ‡Π°ΡŽΡ‰ΠΈΠ΅ΡΡ значСния Π² Π΄Π‘

    0 Π΄Π‘ ΠžΠΏΠΎΡ€Π½Ρ‹ΠΉ ΡƒΡ€ΠΎΠ²Π΅Π½ΡŒ, ΠΊ ΠΊΠΎΡ‚ΠΎΡ€ΠΎΠΌΡƒ относятся всС значСния + Π΄Π‘ ΠΈ –дБ.

    Β± 1 Π΄Π‘ НаимСнСС Π·Π°ΠΌΠ΅Ρ‚Π½ΠΎΠ΅ ΠΈΠ·ΠΌΠ΅Π½Π΅Π½ΠΈΠ΅ ΡƒΡ€ΠΎΠ²Π½Π΅ΠΉ Π·Π²ΡƒΠΊΠ°, Ρ‚Π°ΠΊΠΆΠ΅ ΠΈΡΠΏΠΎΠ»ΡŒΠ·ΡƒΠ΅ΠΌΠΎΠ΅ для ограничСния полосы пропускания Π½Π° высококачСствСнных усилитСлях Π·Π²ΡƒΠΊΠ°.

    βˆ’3 Π΄Π‘ ΠžΠ±Ρ‹Ρ‡Π½ΠΎ ΠΈΡΠΏΠΎΠ»ΡŒΠ·ΡƒΠ΅Ρ‚ΡΡ для ограничСния полосы пропускания Π² усилитСлях с ΡƒΠΊΠ°Π·Π°Π½ΠΈΠ΅ΠΌ Ρ‚ΠΎΡ‡Π΅ΠΊ, Π³Π΄Π΅:

    Π°.Π’Ρ‹Ρ…ΠΎΠ΄Π½ΠΎΠ΅ напряТСниС ΡƒΠΏΠ°Π»ΠΎ Π΄ΠΎ 0,707 максимального (срСднСго Π΄ΠΈΠ°ΠΏΠ°Π·ΠΎΠ½Π°) Π²Ρ‹Ρ…ΠΎΠ΄Π½ΠΎΠ³ΠΎ сигнала.

    Π³. Выходная ΠΌΠΎΡ‰Π½ΠΎΡΡ‚ΡŒ ΡƒΠΏΠ°Π»Π° Π΄ΠΎ ΠΏΠΎΠ»ΠΎΠ²ΠΈΠ½Ρ‹ максимальной ΠΈΠ»ΠΈ срСднСй мощности.

    (ΠΏΠΎΠ»ΠΎΠ²ΠΈΠ½Π° Π°ΠΌΠΏΠ»ΠΈΡ‚ΡƒΠ΄Ρ‹ ΠΠΠŸΠ Π―Π–Π•ΠΠ˜Π― составляСт βˆ’6 Π΄Π‘ )

    Часто приводятся Ρ†ΠΈΡ„Ρ€Ρ‹ Π°Ρ‚Ρ‚Π΅Π½ΡŽΠ°Ρ‚ΠΎΡ€ΠΎΠ², ΠΏΡ€Π΅Π΄Π½Π°Π·Π½Π°Ρ‡Π΅Π½Π½Ρ‹Ρ… для ΡƒΠΌΠ΅Π½ΡŒΡˆΠ΅Π½ΠΈΡ Π²Ρ‹Ρ…ΠΎΠ΄Π½ΠΎΠΉ мощности Π³Π΅Π½Π΅Ρ€Π°Ρ‚ΠΎΡ€ΠΎΠ² сигналов Π½Π° измСряСмыС Π²Π΅Π»ΠΈΡ‡ΠΈΠ½Ρ‹.

    βˆ’20 Π΄Π‘ Амплитуда напряТСния сигнала, дСлСнная Π½Π° 10

    βˆ’40 Π΄Π‘ Амплитуда напряТСния сигнала, дСлСнная Π½Π° 100

    ΠŸΡ€Π΅ΠΎΠ±Ρ€Π°Π·ΠΎΠ²Π°Π½ΠΈΠ΅ ΠΌΠ΅ΠΆΠ΄Ρƒ Π΄Π‘ ΠΈ усилСниСм мощности ΠΈΠ»ΠΈ напряТСния

    Π’Π°Π±Π»ΠΈΡ†Π° 1.3.1 ΠšΠΎΡΡ„Ρ„ΠΈΡ†ΠΈΠ΅Π½Ρ‚ мощности / Π΄Π‘

    ΠšΠΎΡΡ„Ρ„ΠΈΡ†ΠΈΠ΅Π½Ρ‚ мощности Π΄Π‘
    P Π²Ρ‹Ρ…ΠΎΠ΄ / P дюйм 10 Π»ΠΎΠ³ (P ΠΈΠ· / P Π² )
    10 000 000 000 100
    1 000 000 000 90
    100 000 000 80
    10 000 000 70
    1 000 000 60
    100 000 50
    10 000 40
    1 000 30
    100 20
    10 10
    4 6
    2 3
    1.26 1
    1 0
    0,79 -1
    0,5 βˆ’3
    0,25 βˆ’6
    0,1 βˆ’10
    0,01 βˆ’20
    0,001 βˆ’30
    0,000 1 βˆ’40
    0.000 01 βˆ’50
    0,000 001 βˆ’60
    0,000 000 1 βˆ’70
    0,000 000 01 βˆ’80
    0,000 000 001 βˆ’90
    0,000 000 000 1 βˆ’100

    Π’Π°Π±Π»ΠΈΡ†Π° 1.3.2 Π‘ΠΎΠΎΡ‚Π½ΠΎΡˆΠ΅Π½ΠΈΠ΅ напряТСния ΠΈΠ»ΠΈ Ρ‚ΠΎΠΊΠ° / Π΄Π‘

    ΠšΠΎΡΡ„Ρ„ΠΈΡ†ΠΈΠ΅Π½Ρ‚ напряТСния ΠΈΠ»ΠΈ Ρ‚ΠΎΠΊΠ° Π΄Π‘
    Π’ Π½Π° Π²Ρ‹Ρ…ΠΎΠ΄Π΅ / Π’ Π½Π° Π²Ρ‹Ρ…ΠΎΠ΄Π΅ ΠΈΠ»ΠΈ Π½Π° Π½Π° Π²Ρ‹Ρ…ΠΎΠ΄Π΅ / Π½Π° Π½Π° Π²Ρ‹Ρ…ΠΎΠ΄Π΅ 20log (V ΠΈΠ· / V ΠΈΠ· ) 20log (I ΠΈΠ· / I ΠΈΠ· )
    100 000 100
    31 623 90
    10 000 80
    3 162 70
    1 000 60
    316.23 50
    100 40
    31,623 30
    10 20
    3,162 10
    2 6
    1,414 3
    1,122 1
    1 0
    0,891 -1
    0.707 βˆ’3
    0,5 βˆ’6
    0,316 2 βˆ’10
    0,1 βˆ’20
    0,031 62 βˆ’30
    0,01 βˆ’40
    0,003 162 βˆ’50
    0,001 βˆ’60
    0,000 316 2 βˆ’70
    0.000 1 βˆ’80
    0,000 0316 2 βˆ’90
    0,000 01 βˆ’100

    Π΄Π΅Ρ†ΠΈΠ±Π΅Π» (Π΄Π‘) — IPS

    ЧСловСчСскоС ΡƒΡ…ΠΎ Π½Π΅ Ρ€Π΅Π°Π³ΠΈΡ€ΡƒΠ΅Ρ‚ Π½Π° измСнСния Π·Π²ΡƒΠΊΠ° простым, Π»ΠΈΠ½Π΅ΠΉΠ½Ρ‹ΠΌ ΠΎΠ±Ρ€Π°Π·ΠΎΠΌ. Π§Ρ‚ΠΎΠ±Ρ‹ ΡƒΠ΄Π²ΠΎΠΈΡ‚ΡŒ Π³Ρ€ΠΎΠΌΠΊΠΎΡΡ‚ΡŒ Π·Π²ΡƒΠΊΠ°, Π½Π°ΠΏΡ€ΠΈΠΌΠ΅Ρ€, ΡƒΠ²Π΅Π»ΠΈΡ‡ΠΈΠ² Π³Ρ€ΠΎΠΌΠΊΠΎΡΡ‚ΡŒ Π½Π° аудиоусилитСлС, Π±ΠΎΠ»ΡŒΡˆΠΈΠ½ΡΡ‚Π²ΠΎ людСй ΠΎΠ±Π½Π°Ρ€ΡƒΠΆΠΈΠ²Π°ΡŽΡ‚, Ρ‡Ρ‚ΠΎ ΠΈΠΌ Π½Π΅ΠΎΠ±Ρ…ΠΎΠ΄ΠΈΠΌΠΎ ΡƒΠ²Π΅Π»ΠΈΡ‡ΠΈΡ‚ΡŒ ΠΌΠΎΡ‰Π½ΠΎΡΡ‚ΡŒ, ΠΏΡ€ΠΎΠΈΠ·Π²ΠΎΠ΄ΠΈΠΌΡƒΡŽ Π³Ρ€ΠΎΠΌΠΊΠΎΠ³ΠΎΠ²ΠΎΡ€ΠΈΡ‚Π΅Π»Π΅ΠΌ, ΠΏΡ€ΠΈΠΌΠ΅Ρ€Π½ΠΎ Π² Π΄Π΅ΡΡΡ‚ΡŒ Ρ€Π°Π·.Если Π±Ρ‹ ΠΌΡ‹ ΡƒΠ²Π΅Π»ΠΈΡ‡ΠΈΠ»ΠΈ ΠΌΠΎΡ‰Π½ΠΎΡΡ‚ΡŒ Π΅Ρ‰Π΅ Π² Π΄Π΅ΡΡΡ‚ΡŒ Ρ€Π°Π·, Ρ‚ΠΎ Π΅ΡΡ‚ΡŒ Π² 100 Ρ€Π°Π· большС мощности, с ΠΊΠΎΡ‚ΠΎΡ€ΠΎΠΉ ΠΌΡ‹ Π½Π°Ρ‡Π°Π»ΠΈ, Π·Π²ΡƒΠΊ снова Π±Ρ‹ Ρ‚ΠΎΠ»ΡŒΠΊΠΎ удвоился ΠΏΠΎ громкости — Ρ‚Π°ΠΊ ΡΡƒΠ±ΡŠΠ΅ΠΊΡ‚ΠΈΠ²Π½ΠΎ ΠΎΠ½ Π±Ρ‹Π» Π±Ρ‹ ΠΏΡ€ΠΈΠΌΠ΅Ρ€Π½ΠΎ Π² Ρ‡Π΅Ρ‚Ρ‹Ρ€Π΅ Ρ€Π°Π·Π° Π³Ρ€ΠΎΠΌΡ‡Π΅, Ρ‡Π΅ΠΌ исходный Π·Π²ΡƒΠΊ. .

    ДСсятикратноС ΡΠΎΠΎΡ‚Π½ΠΎΡˆΠ΅Π½ΠΈΠ΅ мощности опрСдСляСтся ΠΊΠ°ΠΊ 1 Π±Π΅Π» (ΠΎΡ‚ АлСксандра ГрэхСма Π‘Π΅Π»Π»Π° (1847-1922)). Π’Π°ΠΊΠΈΠΌ ΠΎΠ±Ρ€Π°Π·ΠΎΠΌ, шкала Π±Π΅Π» являСтся логарифмичСской ΠΏΠΎ ΠΎΡ‚Π½ΠΎΡˆΠ΅Π½ΠΈΡŽ ΠΊ основанию 10. Π’Π°ΠΊΠΈΠΌ ΠΎΠ±Ρ€Π°Π·ΠΎΠΌ, ΠΎΡ‚Π½ΠΎΡˆΠ΅Π½ΠΈΠ΅ мощности, Π²Ρ‹Ρ€Π°ΠΆΠ΅Π½Π½ΠΎΠ΅ Π² Π±Π΅Π»Π°Ρ… (Π½Π°Π·Ρ‹Π²Π°Π΅ΠΌΠΎΠ΅ Β«Ρ€Π°Π·Π½ΠΈΡ†Π΅ΠΉ ΡƒΡ€ΠΎΠ²Π½Π΅ΠΉΒ»), находится ΠΏΡƒΡ‚Π΅ΠΌ взятия Π»ΠΎΠ³Π°Ρ€ΠΈΡ„ΠΌΠ° (основаниС 10) ΠΎΡ‚Π½ΠΎΡˆΠ΅Π½ΠΈΡ ΡƒΡ€ΠΎΠ²Π½Π΅ΠΉ мощности.Π­Ρ‚Π° Π΅Π΄ΠΈΠ½ΠΈΡ†Π° измСрСния слишком Π²Π΅Π»ΠΈΠΊΠ° для практичСских Ρ†Π΅Π»Π΅ΠΉ, поэтому ΠΌΡ‹ ΠΈΡΠΏΠΎΠ»ΡŒΠ·ΡƒΠ΅ΠΌ ΠΎΠ΄Π½Ρƒ Π΄Π΅ΡΡΡ‚ΡƒΡŽ Π±Π΅Π» — Π΄Π΅Ρ†ΠΈΠ±Π΅Π»:

    .

    Π Π°Π·Π½ΠΎΡΡ‚ΡŒ ΡƒΡ€ΠΎΠ²Π½Π΅ΠΉ = 10log (10) (P2 / P1) Π΄Π‘.

    ΠžΠ±Ρ€Π°Ρ‚ΠΈΡ‚Π΅ Π²Π½ΠΈΠΌΠ°Π½ΠΈΠ΅ Π½Π° ΠΏΡ€Π°Π²ΠΈΠ»ΡŒΠ½ΡƒΡŽ Π½ΠΎΡ‚Π°Ρ†ΠΈΡŽ: малСнькая b ΠΎΠ·Π½Π°Ρ‡Π°Π΅Ρ‚ Π±Π΅Π», ΠΊΠΎΠ³Π΄Π° написано ΠΏΠΎΠ»Π½ΠΎΡΡ‚ΡŒΡŽ (Ρ‚Π°ΠΊ ΠΊΠ°ΠΊ это ΠΎΠ±Ρ‹Ρ‡Π½ΠΎΠ΅ слово), большая B — сокращСниС (Π² Ρ‡Π΅ΡΡ‚ΡŒ мистСра Π‘Π΅Π»Π»Π°) ΠΈ малСнькая d — Π΄Π΅Ρ†ΠΈ.

    ΠŸΡ€ΠΈ Ρ…ΠΎΡ€ΠΎΡˆΠΈΡ… условиях ΠΏΡ€ΠΎΡΠ»ΡƒΡˆΠΈΠ²Π°Π½ΠΈΡ минимальноС ΠΈΠ·ΠΌΠ΅Π½Π΅Π½ΠΈΠ΅ уровня Π·Π²ΡƒΠΊΠ°, ΠΊΠΎΡ‚ΠΎΡ€ΠΎΠ΅ ΠΌΠΎΠΆΠ΅Ρ‚ ΡƒΠ»ΠΎΠ²ΠΈΡ‚ΡŒ чСловСчСскоС ΡƒΡ…ΠΎ, составляСт ΠΎΠ΄ΠΈΠ½ Π΄Π΅Ρ†ΠΈΠ±Π΅Π» (1 Π΄Π‘).

    Π‘ этого ΠΌΠΎΠΌΠ΅Π½Ρ‚Π° всС Π»ΠΎΠ³Π°Ρ€ΠΈΡ„ΠΌΡ‹ ΠΏΡ€ΠΈΠ½ΠΈΠΌΠ°ΡŽΡ‚ΡΡ ΠΏΠΎ основанию 10.

    Π΄Π‘, ΠΌΠΎΡ‰Π½ΠΎΡΡ‚ΡŒ ΠΈ напряТСниС

    Π§Ρ‚ΠΎΠ±Ρ‹ ΠΈΡΠΏΠΎΠ»ΡŒΠ·ΠΎΠ²Π°Ρ‚ΡŒ систСму Π΄Π΅Ρ†ΠΈΠ±Π΅Π» для ΡƒΡ€ΠΎΠ²Π½Π΅ΠΉ напряТСния, ΠΌΡ‹ Π΄ΠΎΠ»ΠΆΠ½Ρ‹ ΠΈΡΠΏΠΎΠ»ΡŒΠ·ΠΎΠ²Π°Ρ‚ΡŒ Π·Π°ΠΊΠΎΠ½ Ома для прСобразования мощности Π² напряТСниС. ΠœΠΎΡ‰Π½ΠΎΡΡ‚ΡŒ W (Π² Π²Π°Ρ‚Ρ‚Π°Ρ…) — это напряТСниС V (Π² Π²ΠΎΠ»ΡŒΡ‚Π°Ρ…), ΡƒΠΌΠ½ΠΎΠΆΠ΅Π½Π½ΠΎΠ΅ Π½Π° Ρ‚ΠΎΠΊ I (Π² Π°ΠΌΠΏΠ΅Ρ€Π°Ρ…): W = VI, Π° Π·Π°ΠΊΠΎΠ½ Ома Π΄Π°Π΅Ρ‚ I = V / R, Π³Π΄Π΅ R — сопротивлСниС (Π² Ом). ΠŸΡ€ΠΎΡΡ‚Π°Ρ Π·Π°ΠΌΠ΅Π½Π° ΠΏΠΎΠΊΠ°Π·Ρ‹Π²Π°Π΅Ρ‚, Ρ‡Ρ‚ΠΎ ΠΌΠΎΡ‰Π½ΠΎΡΡ‚ΡŒ измСняСтся ΠΊΠ°ΠΊ ΠΊΠ²Π°Π΄Ρ€Π°Ρ‚ напряТСния. Π’ΠΎΠ·Π²Π΅Π΄Π΅Π½ΠΈΠ΅ числа Π² ΠΊΠ²Π°Π΄Ρ€Π°Ρ‚ соотвСтствуСт ΡƒΠ΄Π²ΠΎΠ΅Π½ΠΈΡŽ Π΅Π³ΠΎ Π»ΠΎΠ³Π°Ρ€ΠΈΡ„ΠΌΠ°, поэтому:

    Π Π°Π·Π½ΠΎΡΡ‚ΡŒ ΡƒΡ€ΠΎΠ²Π½Π΅ΠΉ = 20log (V1 / V2) Π΄Π‘

    Π£Ρ‚Π²Π΅Ρ€ΠΆΠ΄Π΅Π½ΠΈΠ΅, Ρ‡Ρ‚ΠΎ ΠΌΠΎΡ‰Π½ΠΎΡΡ‚ΡŒ измСняСтся ΠΏΡ€ΠΎΠΏΠΎΡ€Ρ†ΠΈΠΎΠ½Π°Π»ΡŒΠ½ΠΎ ΠΊΠ²Π°Π΄Ρ€Π°Ρ‚Ρƒ напряТСния, Π²Π΅Ρ€Π½ΠΎ Ρ‚ΠΎΠ»ΡŒΠΊΠΎ Π² Ρ‚ΠΎΠΌ случаС, Ссли сравниваСмыС напряТСния ΠΈΠΌΠ΅ΡŽΡ‚ ΠΎΠ΄ΠΈΠ½Π°ΠΊΠΎΠ²Ρ‹ΠΉ импСданс.Π’ Π°ΡƒΠ΄ΠΈΠΎ нас Π² основном интСрСсуСт напряТСниС, Π° Π½Π΅ ΠΌΠΎΡ‰Π½ΠΎΡΡ‚ΡŒ (Π½Π΅ говоря ΡƒΠΆΠ΅ ΠΎΠ± усилитСлях Π³Ρ€ΠΎΠΌΠΊΠΎΠ³ΠΎΠ²ΠΎΡ€ΠΈΡ‚Π΅Π»Π΅ΠΉ), поэтому ΠΎΡ‡Π΅Π½ΡŒ ΡƒΠ΄ΠΎΠ±Π½ΠΎ Π²ΠΎΠΎΠ±Ρ‰Π΅ Π·Π°Π±Ρ‹Ρ‚ΡŒ ΠΎΠ± импСдансах ΠΈ нСзависимо ΡΡ€Π°Π²Π½ΠΈΠ²Π°Ρ‚ΡŒ напряТСния, ΠΈΡΠΏΠΎΠ»ΡŒΠ·ΡƒΡ ΠΏΡ€ΠΈΠ²Π΅Π΄Π΅Π½Π½ΡƒΡŽ Π²Ρ‹ΡˆΠ΅ Ρ„ΠΎΡ€ΠΌΡƒΠ»Ρƒ; ΠΈ это Ρ‚ΠΎ, Ρ‡Ρ‚ΠΎ ΠΌΡ‹ Π΄Π΅Π»Π°Π΅ΠΌ.

    Π­Ρ‚ΠΎ ΠΎΠ·Π½Π°Ρ‡Π°Π΅Ρ‚, Ρ‡Ρ‚ΠΎ ΠΌΡ‹ Π½Π΅ ΠΌΠΎΠΆΠ΅ΠΌ ΠΏΡ€Π΅ΠΎΠ±Ρ€Π°Π·ΠΎΠ²Π°Ρ‚ΡŒ ΠΎΠ±Ρ€Π°Ρ‚Π½ΠΎ Π² ΠΎΡ‚Π½ΠΎΡˆΠ΅Π½ΠΈΠ΅ мощностСй Ρ€Π°Π·Π½ΠΎΡΡ‚ΡŒ ΡƒΡ€ΠΎΠ²Π½Π΅ΠΉ, ΠΏΠΎΠ»ΡƒΡ‡Π΅Π½Π½ΡƒΡŽ ΠΈΠ· ΠΎΡ‚Π½ΠΎΡˆΠ΅Π½ΠΈΡ напряТСний, Ссли Ρ‚ΠΎΠ»ΡŒΠΊΠΎ ΠΌΡ‹ Π½Π΅ Π·Π½Π°Π΅ΠΌ, Ρ‡Ρ‚ΠΎ напряТСния ΠΈΠΌΠ΅ΡŽΡ‚ ΠΎΠ΄ΠΈΠ½Π°ΠΊΠΎΠ²Ρ‹ΠΉ импСданс.

    ВычислСниС Π² Π΄Π΅Ρ†ΠΈΠ±Π΅Π»Π°Ρ…

    Когда Π²Ρ‹ складываСтС Π»ΠΎΠ³Π°Ρ€ΠΈΡ„ΠΌΡ‹ Π΄Π²ΡƒΡ… чисСл вмСстС, Ρ€Π΅Π·ΡƒΠ»ΡŒΡ‚Π°Ρ‚ΠΎΠΌ являСтся Π»ΠΎΠ³Π°Ρ€ΠΈΡ„ΠΌ произвСдСния Π΄Π²ΡƒΡ… чисСл: logA + logB = log (AB)

    Π’Π°ΠΊΠΈΠΌ ΠΎΠ±Ρ€Π°Π·ΠΎΠΌ, эффСкт умноТСния напряТСния Π½Π° ряд ΡΠΎΠΎΡ‚Π½ΠΎΡˆΠ΅Π½ΠΈΠΉ ΠΌΠΎΠΆΠ½ΠΎ Ρ€Π°ΡΡΡ‡ΠΈΡ‚Π°Ρ‚ΡŒ, слоТив ΡΠΎΠΎΡ‚Π²Π΅Ρ‚ΡΡ‚Π²ΡƒΡŽΡ‰ΠΈΠ΅ Π΄Π΅Ρ†ΠΈΠ±Π΅Π»Ρ‹.НапримСр, ΡΠΎΠΎΡ‚Π½ΠΎΡˆΠ΅Π½ΠΈΠ΅ мощностСй 2: 1 — это Ρ€Π°Π·Π½ΠΎΡΡ‚ΡŒ ΡƒΡ€ΠΎΠ²Π½Π΅ΠΉ 3 Π΄Π‘, Π° ΡΠΎΠΎΡ‚Π½ΠΎΡˆΠ΅Π½ΠΈΠ΅ мощностСй 3: 1 — Ρ€Π°Π·Π½ΠΎΡΡ‚ΡŒ ΡƒΡ€ΠΎΠ²Π½Π΅ΠΉ 4,8 Π΄Π‘, поэтому ΡΠΎΠΎΡ‚Π½ΠΎΡˆΠ΅Π½ΠΈΠ΅ мощностСй 6: 1 (2 x 3) являСтся ΡƒΡ€ΠΎΠ²Π½Π΅ΠΌ Ρ€Π°Π·Π½ΠΈΡ†Π° (3 + 4,8) = 7,8 Π΄Π‘.

    Π’ΠΎΡ‡Π½ΠΎ Ρ‚Π°ΠΊ ΠΆΠ΅ Π΄Π΅Π»Π΅Π½ΠΈΠ΅ Π½Π° коэффициСнт соотвСтствуСт Π²Ρ‹Ρ‡ΠΈΡ‚Π°Π½ΠΈΡŽ ΡΠΎΠΎΡ‚Π²Π΅Ρ‚ΡΡ‚Π²ΡƒΡŽΡ‰Π΅ΠΉ Ρ€Π°Π·Π½ΠΈΡ†Ρ‹ ΡƒΡ€ΠΎΠ²Π½Π΅ΠΉ Π² Π΄Π΅Ρ†ΠΈΠ±Π΅Π»Π°Ρ….

    Π§Ρ‚ΠΎΠ±Ρ‹ ΠΏΡ€Π΅ΠΎΠ±Ρ€Π°Π·ΠΎΠ²Π°Ρ‚ΡŒ Ρ€Π°Π·Π½ΠΈΡ†Ρƒ ΡƒΡ€ΠΎΠ²Π½Π΅ΠΉ ΠΎΠ±Ρ€Π°Ρ‚Π½ΠΎ Π² ΡΠΎΠΎΡ‚Π½ΠΎΡˆΠ΅Π½ΠΈΠ΅ мощности ΠΈΠ»ΠΈ напряТСния: Ρ€Π°Π·Π΄Π΅Π»ΠΈΡ‚Π΅ Π½Π° 10, Ρ‡Ρ‚ΠΎΠ±Ρ‹ ΠΏΡ€Π΅ΠΎΠ±Ρ€Π°Π·ΠΎΠ²Π°Ρ‚ΡŒ Π΅Π΅ Π² Π±Π΅Π»; (Ρ‚ΠΎΠ»ΡŒΠΊΠΎ Ссли ΠΏΡ€Π΅ΠΎΠ±Ρ€Π°Π·ΠΎΠ²Π°Ρ‚ΡŒ Π² коэффициСнт напряТСния, Ρ€Π°Π·Π΄Π΅Π»ΠΈΡ‚Π΅ Π½Π° Π΄Π²Π°;) Π²ΠΎΠ·ΡŒΠΌΠΈΡ‚Π΅ 10 Π² ΡΡ‚Π΅ΠΏΠ΅Π½ΡŒ ΠΏΠΎΠ»ΡƒΡ‡Π΅Π½Π½ΠΎΠ³ΠΎ числа (Ρ‚Π°ΠΊΠΆΠ΅ Π½Π°Π·Ρ‹Π²Π°Π΅ΠΌΠΎΠ³ΠΎ Π°Π½Ρ‚ΠΈΠ»ΠΎΠ³Π°Ρ€ΠΈΡ„ΠΌΠΎΠΌ ΠΈΠ»ΠΈ Π°Π½Ρ‚ΠΈΠ»ΠΎΠ³Π°Ρ€ΠΈΡ„ΠΌΠΎΠΌ).

    ИспользованиС

    Π›ΡŽΠ±Π°Ρ Ρ„ΠΎΡ€ΠΌΠ° коэффициСнта мощности ΠΌΠΎΠΆΠ΅Ρ‚ Π±Ρ‹Ρ‚ΡŒ Π²Ρ‹Ρ€Π°ΠΆΠ΅Π½Π° Π² Π΄Π΅Ρ†ΠΈΠ±Π΅Π»Π°Ρ…; Π½Π°ΠΏΡ€ΠΈΠΌΠ΅Ρ€, ΠΎΡ‚Π½ΠΎΡˆΠ΅Π½ΠΈΠ΅ акустичСской мощности, ΠΏΡ€ΠΎΠΈΠ·Π²ΠΎΠ΄ΠΈΠΌΠΎΠΉ Π³Ρ€ΠΎΠΌΠΊΠΎΠ³ΠΎΠ²ΠΎΡ€ΠΈΡ‚Π΅Π»Π΅ΠΌ, ΠΊ Π΅Π³ΠΎ потрСбляСмой элСктричСской мощности. На ΠΏΡ€Π°ΠΊΡ‚ΠΈΠΊΠ΅ Π΄Π΅Ρ†ΠΈΠ±Π΅Π»Ρ‹ находят наибольшСС ΠΏΡ€ΠΈΠΌΠ΅Π½Π΅Π½ΠΈΠ΅ со Π·Π²ΡƒΠΊΠΎΠΌ ΠΈ сигналами Π² тСлСкоммуникациях ΠΈ Ρ€Π°Π΄ΠΈΠΎΠ²Π΅Ρ‰Π°Π½ΠΈΠΈ.

    Помимо мощности, ΠΎΠ½ΠΈ ΠΎΠ±Ρ‹Ρ‡Π½ΠΎ ΠΈΡΠΏΠΎΠ»ΡŒΠ·ΡƒΡŽΡ‚ΡΡ со ΠΌΠ½ΠΎΠ³ΠΈΠΌΠΈ Π²Π΅Π»ΠΈΡ‡ΠΈΠ½Π°ΠΌΠΈ, ΠΊΠΎΡ‚ΠΎΡ€Ρ‹Π΅ Π½Π°ΠΏΡ€ΡΠΌΡƒΡŽ связаны с ΠΌΠΎΡ‰Π½ΠΎΡΡ‚ΡŒΡŽ, Ρ‚Π°ΠΊΠΈΠΌΠΈ ΠΊΠ°ΠΊ напряТСниС, Π·Π²ΡƒΠΊΠΎΠ²ΠΎΠ΅ Π΄Π°Π²Π»Π΅Π½ΠΈΠ΅ ΠΈ Π½Π°ΠΏΡ€ΡΠΆΠ΅Π½Π½ΠΎΡΡ‚ΡŒ радиочастотного поля. ΠŸΠΎΡΠΊΠΎΠ»ΡŒΠΊΡƒ Π΄Π΅Ρ†ΠΈΠ±Π΅Π»Ρ‹ ΠΎΠΏΠΈΡΡ‹Π²Π°ΡŽΡ‚ ΠΎΡ‚Π½ΠΎΡˆΠ΅Π½ΠΈΡ, ΠΈΡ… Ρ‚Π°ΠΊΠΆΠ΅ ΠΌΠΎΠΆΠ½ΠΎ ΠΈΡΠΏΠΎΠ»ΡŒΠ·ΠΎΠ²Π°Ρ‚ΡŒ для опрСдСлСния Ρ‚Π°ΠΊΠΈΡ… Ρ„Π°ΠΊΡ‚ΠΎΡ€ΠΎΠ², ΠΊΠ°ΠΊ усилСниС элСктронных усилитСлСй ΠΈ ΠΏΠΎΡ‚Π΅Ρ€ΠΈ Π² Π°Ρ‚Ρ‚Π΅Π½ΡŽΠ°Ρ‚ΠΎΡ€Π°Ρ…, элСктронных ΠΈΠ»ΠΈ акустичСских.

    ΠŸΠΎΡ…ΠΎΠΆΠΈΠ΅ записи

    1.2: Π”Π΅Ρ†ΠΈΠ±Π΅Π» — Engineering LibreTexts

    Π‘ΠΎΠ»ΡŒΡˆΠΈΠ½ΡΡ‚Π²ΠΎ людСй Π·Π½Π°ΠΊΠΎΠΌΡ‹ с Ρ‚Π΅Ρ€ΠΌΠΈΠ½ΠΎΠΌ Β«Π΄Π΅Ρ†ΠΈΠ±Π΅Π»Β» Π² ΠΎΡ‚Π½ΠΎΡˆΠ΅Π½ΠΈΠΈ Π·Π²ΡƒΠΊΠΎΠ²ΠΎΠ³ΠΎ давлСния. НСрСдко ΠΌΠΎΠΆΠ½ΠΎ ΡƒΡΠ»Ρ‹ΡˆΠ°Ρ‚ΡŒ, ΠΊΠ°ΠΊ ΠΊΡ‚ΠΎ-Ρ‚ΠΎ Π³ΠΎΠ²ΠΎΡ€ΠΈΡ‚ Ρ‡Ρ‚ΠΎ-Ρ‚ΠΎ Π²Ρ€ΠΎΠ΄Π΅ Β«Π’Ρ‡Π΅Ρ€Π° Π²Π΅Ρ‡Π΅Ρ€ΠΎΠΌ Π½Π° ΠΊΠΎΠ½Ρ†Π΅Ρ€Ρ‚Π΅ Π±Ρ‹Π»ΠΎ 110 Π΄Π΅Ρ†ΠΈΠ±Π΅Π», Π° Π² ΡƒΡˆΠ°Ρ… всС Π΅Ρ‰Π΅ Π·Π²Π΅Π½ΠΈΡ‚Β». Π­Ρ‚ΠΎ популярноС использованиС нСсколько Π½Π΅Ρ‚ΠΎΡ‡Π½ΠΎ, Π½ΠΎ ΠΏΠΎΠΊΠ°Π·Ρ‹Π²Π°Π΅Ρ‚, Ρ‡Ρ‚ΠΎ Π΄Π΅Ρ†ΠΈΠ±Π΅Π»Ρ‹ ΡƒΠΊΠ°Π·Ρ‹Π²Π°ΡŽΡ‚ Π½Π° ΠΊΠ°ΠΊΡƒΡŽ-Ρ‚ΠΎ Π²Π΅Π»ΠΈΡ‡ΠΈΠ½Ρƒ — Π² Π΄Π°Π½Π½ΠΎΠΌ случаС ΡƒΡ€ΠΎΠ²Π΅Π½ΡŒ Π·Π²ΡƒΠΊΠΎΠ²ΠΎΠ³ΠΎ давлСния.

    1.2.1: ΠŸΡ€Π΅Π΄ΡΡ‚Π°Π²Π»Π΅Π½ΠΈΠ΅ мощности ΠΈ напряТСния Π² Π΄Π΅Ρ†ΠΈΠ±Π΅Π»Π°Ρ…

    Π’ ΠΏΡ€ΠΎΡΡ‚Π΅ΠΉΡˆΠ΅ΠΉ Ρ„ΠΎΡ€ΠΌΠ΅ Π΄Π΅Ρ†ΠΈΠ±Π΅Π» ΠΈΡΠΏΠΎΠ»ΡŒΠ·ΡƒΠ΅Ρ‚ΡΡ для измСрСния Π½Π΅ΠΊΠΎΡ‚ΠΎΡ€ΠΎΠ³ΠΎ коэффициСнта усилСния, Π½Π°ΠΏΡ€ΠΈΠΌΠ΅Ρ€ усилСния мощности ΠΈΠ»ΠΈ напряТСния.Π’ ΠΎΡ‚Π»ΠΈΡ‡ΠΈΠ΅ ΠΎΡ‚ ΠΎΠ±Ρ‹Ρ‡Π½Ρ‹Ρ… ΠΈΠ·ΠΌΠ΅Ρ€Π΅Π½ΠΈΠΉ усилСния, с ΠΊΠΎΡ‚ΠΎΡ€Ρ‹ΠΌΠΈ Π²Ρ‹, Π²ΠΎΠ·ΠΌΠΎΠΆΠ½ΠΎ, Π·Π½Π°ΠΊΠΎΠΌΡ‹, Ρ„ΠΎΡ€ΠΌΠ° Π΄Π΅Ρ†ΠΈΠ±Π΅Π» являСтся логарифмичСской. Из-Π·Π° этого ΠΎΠ½ ΠΌΠΎΠΆΠ΅Ρ‚ Π±Ρ‹Ρ‚ΡŒ ΠΎΡ‡Π΅Π½ΡŒ ΠΏΠΎΠ»Π΅Π·Π΅Π½ для отобраТСния ΡΠΎΠΎΡ‚Π½ΠΎΡˆΠ΅Π½ΠΈΠΉ ΠΈΠ·ΠΌΠ΅Π½Π΅Π½ΠΈΠΉ, Π° Ρ‚Π°ΠΊΠΆΠ΅ Π°Π±ΡΠΎΠ»ΡŽΡ‚Π½Ρ‹Ρ… ΠΈΠ·ΠΌΠ΅Π½Π΅Π½ΠΈΠΉ. Базовая Π΅Π΄ΠΈΠ½ΠΈΡ†Π° — Π‘Π΅Π». Π§Ρ‚ΠΎΠ±Ρ‹ ΠΏΡ€Π΅ΠΎΠ±Ρ€Π°Π·ΠΎΠ²Π°Ρ‚ΡŒ ΠΎΠ±Ρ‹Ρ‡Π½ΠΎΠ΅ усилСниС Π² Π΅Π³ΠΎ Π°Π½Π°Π»ΠΎΠ³ Bel, просто Π²ΠΎΠ·ΡŒΠΌΠΈΡ‚Π΅ ΠΎΠ±Ρ‰ΠΈΠΉ Π»ΠΎΠ³Π°Ρ€ΠΈΡ„ΠΌ (основаниС 10) усилСния. Π’ Ρ„ΠΎΡ€ΠΌΠ΅ уравнСния:

    \ [\ text {усилСниС Π‘Π΅Π»} = \ log_ {10} (ΠΎΠ±Ρ‹Ρ‡Π½ΠΎΠ΅ \ усилСниС) \ nonumber \]

    ΠžΠ±Ρ€Π°Ρ‚ΠΈΡ‚Π΅ Π²Π½ΠΈΠΌΠ°Π½ΠΈΠ΅, Ρ‡Ρ‚ΠΎ Π² Π±ΠΎΠ»ΡŒΡˆΠΈΠ½ΡΡ‚Π²Π΅ ΠΊΠ°Π»ΡŒΠΊΡƒΠ»ΡΡ‚ΠΎΡ€ΠΎΠ² ΠΎΠ±Ρ‰ΠΈΠΉ ΠΆΡƒΡ€Π½Π°Π» обозначаСтся ΠΊΠ°ΠΊ Β«\ (\ log \)Β», Π° Π½Π°Ρ‚ΡƒΡ€Π°Π»ΡŒΠ½Ρ‹ΠΉ ΠΆΡƒΡ€Π½Π°Π» — ΠΊΠ°ΠΊ Β«\ (\ ln \)Β».К соТалСнию, Π½Π΅ΠΊΠΎΡ‚ΠΎΡ€Ρ‹Π΅ языки программирования ΠΈΡΠΏΠΎΠ»ΡŒΠ·ΡƒΡŽΡ‚ Β«\ (\ log \)Β» для обозначСния Π½Π°Ρ‚ΡƒΡ€Π°Π»ΡŒΠ½ΠΎΠ³ΠΎ ΠΆΡƒΡ€Π½Π°Π»Π° ΠΈ Β«\ (\ log 10 \)Β» для ΠΎΠ±Ρ‰Π΅Π³ΠΎ ΠΆΡƒΡ€Π½Π°Π»Π°. Π­Ρ‚ΠΎΡ‚ Π±Π°Π³ укусил Π½Π΅ ΠΎΠ΄ΠΈΠ½ ΡƒΡ‡Π΅Π½ΠΈΠΊ, Π±ΡƒΠ΄ΡŒΡ‚Π΅ остороТны! Π’ качСствС ΠΏΡ€ΠΈΠΌΠ΅Ρ€Π°, Ссли схСма Π²Ρ‹Π΄Π°Π΅Ρ‚ Π²Ρ‹Ρ…ΠΎΠ΄Π½ΡƒΡŽ ΠΌΠΎΡ‰Π½ΠΎΡΡ‚ΡŒ 200 ΠΌΠΈΠ»Π»ΠΈΠ²Π°Ρ‚Ρ‚ для Π²Ρ…ΠΎΠ΄Π½ΠΎΠΉ 10 ΠΌΠΈΠ»Π»ΠΈΠ²Π°Ρ‚Ρ‚, ΠΌΡ‹ ΠΎΠ±Ρ‹Ρ‡Π½ΠΎ Π³ΠΎΠ²ΠΎΡ€ΠΈΠΌ, Ρ‡Ρ‚ΠΎ ΠΎΠ½Π° ΠΈΠΌΠ΅Π΅Ρ‚ коэффициСнт усилСния ΠΏΠΎ мощности:

    \ [G = \ frac {P_ {out}} {P_ {in}} \\ G = \ frac {200 \ mW} {10 \ mW} \\ G = 20 \ nonumber \]

    Для вСрсии Bel просто Π²ΠΎΠ·ΡŒΠΌΠΈΡ‚Π΅ ΠΆΡƒΡ€Π½Π°Π» этого Ρ€Π΅Π·ΡƒΠ»ΡŒΡ‚Π°Ρ‚Π°.{‘} = 1,301 \ nonumber \]

    ΠšΠΎΡΡ„Ρ„ΠΈΡ†ΠΈΠ΅Π½Ρ‚ усилСния ΠΏΠΎ Π±Π΅Π»Ρƒ составляСт 1,3 Π±Π΅Π». Π’Π΅Ρ€ΠΌΠΈΠ½ Β«Π±Π΅Π»Β» Π½Π΅ являСтся Π΅Π΄ΠΈΠ½ΠΈΡ†Π΅ΠΉ Π² строгом смыслС слова (ΠΊΠ°ΠΊ Π² Β«Π²Π°Ρ‚Ρ‚Π°Ρ…Β»), Π° просто ΠΈΡΠΏΠΎΠ»ΡŒΠ·ΡƒΠ΅Ρ‚ΡΡ для обозначСния Ρ‚ΠΎΠ³ΠΎ, Ρ‡Ρ‚ΠΎ это Π½Π΅ ΠΎΠ±Ρ‹Ρ‡Π½ΠΎΠ΅ усилСниС. Напротив, ΠΎΠ±Ρ‹Ρ‡Π½Ρ‹Π΅ коэффициСнты усилСния мощности ΠΈ напряТСния ΠΈΠ½ΠΎΠ³Π΄Π° ΡƒΠΊΠ°Π·Ρ‹Π²Π°ΡŽΡ‚ΡΡ Π² Π΅Π΄ΠΈΠ½ΠΈΡ†Π°Ρ… Π’Ρ‚ / Π’Ρ‚ ΠΈ Π’ / Π’, Ρ‡Ρ‚ΠΎΠ±Ρ‹ ΠΎΡ‚Π»ΠΈΡ‡ΠΈΡ‚ΡŒ ΠΈΡ… ΠΎΡ‚ коэффициСнтов усилСния Bel. Π’Π°ΠΊΠΆΠ΅ ΠΎΠ±Ρ€Π°Ρ‚ΠΈΡ‚Π΅ Π²Π½ΠΈΠΌΠ°Π½ΠΈΠ΅, Ρ‡Ρ‚ΠΎ символ усилСния мощности Bel — это \ (G ‘\), Π° Π½Π΅ \ (G \). Π’ΠΎ ΠΈΠ·Π±Π΅ΠΆΠ°Π½ΠΈΠ΅ ΠΏΡƒΡ‚Π°Π½ΠΈΡ†Ρ‹ всС коэффициСнты усилСния ΠΏΠΎ Π±Π΅Π»Ρƒ ΠΎΠ±ΠΎΠ·Π½Π°Ρ‡Π°ΡŽΡ‚ΡΡ ΡΠ»Π΅Π΄ΡƒΡŽΡ‰ΠΈΠΌ ΡˆΡ‚Ρ€ΠΈΡ…ΠΎΠΌ (’). ΠŸΠΎΡΠΊΠΎΠ»ΡŒΠΊΡƒ Π±Π΅Π»ΠΊΠΈ, ΠΊΠ°ΠΊ ΠΏΡ€Π°Π²ΠΈΠ»ΠΎ, довольно большиС, ΠΌΡ‹ ΠΎΠ±Ρ‹Ρ‡Π½ΠΎ ΠΈΡΠΏΠΎΠ»ΡŒΠ·ΡƒΠ΅ΠΌ ΠΎΠ΄Π½Ρƒ Π΄Π΅ΡΡΡ‚ΡƒΡŽ Π±Π΅Π»Π° ΠΊΠ°ΠΊ Π½ΠΎΡ€ΠΌΡƒ.{‘} = 10 \ log_ {10} G \ label {1.1} \]

    Π“Π΄Π΅ Ρ€Π΅Π·ΡƒΠ»ΡŒΡ‚Π°Ρ‚ Π² Π΄Π‘.

    На этом этапС Π²Π°ΠΌ ΠΌΠΎΠΆΠ΅Ρ‚ Π±Ρ‹Ρ‚ΡŒ интСрСсно ΡƒΠ·Π½Π°Ρ‚ΡŒ, Π² Ρ‡Π΅ΠΌ большоС прСимущСство Π΄Π΅Ρ†ΠΈΠ±Π΅Π»ΡŒΠ½ΠΎΠΉ систСмы. Π§Ρ‚ΠΎΠ±Ρ‹ ΠΎΡ‚Π²Π΅Ρ‚ΠΈΡ‚ΡŒ Π½Π° этот вопрос, вспомнитС нСсколько ΠΈΠ΄Π΅Π½Ρ‚ΠΈΡ„ΠΈΠΊΠ°Ρ‚ΠΎΡ€ΠΎΠ² ΠΆΡƒΡ€Π½Π°Π»ΠΎΠ². ΠΠΎΡ€ΠΌΠ°Π»ΡŒΠ½ΠΎΠ΅ ΡƒΠΌΠ½ΠΎΠΆΠ΅Π½ΠΈΠ΅ становится слоТСниСм Π² систСмС ΠΆΡƒΡ€Π½Π°Π»ΠΎΠ², Π° Π΄Π΅Π»Π΅Π½ΠΈΠ΅ становится Π²Ρ‹Ρ‡ΠΈΡ‚Π°Π½ΠΈΠ΅ΠΌ. Π’ΠΎΡ‡Π½ΠΎ Ρ‚Π°ΠΊ ΠΆΠ΅ силы ΠΈ ΠΊΠΎΡ€Π½ΠΈ становятся ΡƒΠΌΠ½ΠΎΠΆΠ΅Π½ΠΈΠ΅ΠΌ ΠΈ Π΄Π΅Π»Π΅Π½ΠΈΠ΅ΠΌ. Из-Π·Π° этого ΠΎΠ±Π½Π°Ρ€ΡƒΠΆΠΈΠ²Π°ΡŽΡ‚ΡΡ Π΄Π²Π΅ Π²Π°ΠΆΠ½Ρ‹Π΅ Π²Π΅Ρ‰ΠΈ. Π’ΠΎ-ΠΏΠ΅Ρ€Π²Ρ‹Ρ…, коэффициСнты измСнСния становятся постоянными смСщСниями Π² систСмС Π΄Π΅Ρ†ΠΈΠ±Π΅Π», Π° Π²ΠΎ-Π²Ρ‚ΠΎΡ€Ρ‹Ρ…, вСсь Π΄ΠΈΠ°ΠΏΠ°Π·ΠΎΠ½ Π·Π½Π°Ρ‡Π΅Π½ΠΈΠΉ ΡƒΠΌΠ΅Π½ΡŒΡˆΠ°Π΅Ρ‚ΡΡ Π² Ρ€Π°Π·ΠΌΠ΅Ρ€Π΅.{‘} = 10 \ log_ {10} G \) 1 0 Π΄Π‘ 2 3,01 Π΄Π‘ 4 6,02 Π΄Π‘ 8 9,03 Π΄Π‘ 10 10 Π΄Π‘

    ΠœΡ‹ Ρ‚Π°ΠΊΠΆΠ΅ ΠΌΠΎΠΆΠ΅ΠΌ Ρ€Π°ΡΡΠΌΠΎΡ‚Ρ€Π΅Ρ‚ΡŒ Π΄Ρ€ΠΎΠ±Π½Ρ‹Π΅ коэффициСнты (Ρ‚.Π΅. ΠΏΠΎΡ‚Π΅Ρ€ΠΈ вмСсто ΠΏΡ€ΠΈΠ±Ρ‹Π»ΠΈ).

    Π€Π°ΠΊΡ‚ΠΎΡ€ Π΄Π‘ Π—Π½Π°Ρ‡Π΅Π½ΠΈΠ΅
    0.5 βˆ’3,01 Π΄Π‘
    0,25 βˆ’6,02 Π΄Π‘
    0,125 βˆ’9,03 Π΄Π‘
    0,1 βˆ’10 Π΄Π‘

    Если Π²Ρ‹ посмотритС Π²Π½ΠΈΠΌΠ°Ρ‚Π΅Π»ΡŒΠ½ΠΎ, Π²Ρ‹ Π·Π°ΠΌΠ΅Ρ‚ΠΈΡ‚Π΅, Ρ‡Ρ‚ΠΎ ΡƒΠ΄Π²ΠΎΠ΅Π½ΠΈΠ΅ прСдставлСно ΡƒΠ²Π΅Π»ΠΈΡ‡Π΅Π½ΠΈΠ΅ΠΌ ΠΏΡ€ΠΈΠΌΠ΅Ρ€Π½ΠΎ Π½Π° 3 Π΄Π‘. Π€Π°ΠΊΡ‚ΠΎΡ€ 4 — это, ΠΏΠΎ сути, Π΄Π²Π° удвоСния. Π‘Π»Π΅Π΄ΠΎΠ²Π°Ρ‚Π΅Π»ΡŒΠ½ΠΎ, это эквивалСнтно 3 Π΄Π‘ + 3 Π΄Π‘ ΠΈΠ»ΠΈ 6 Π΄Π‘.ΠŸΠΎΠΌΠ½ΠΈΡ‚Π΅, ΠΏΠΎΡΠΊΠΎΠ»ΡŒΠΊΡƒ ΠΌΡ‹ ΠΈΡΠΏΠΎΠ»ΡŒΠ·ΡƒΠ΅ΠΌ ΠΆΡƒΡ€Π½Π°Π»Ρ‹, ΡƒΠΌΠ½ΠΎΠΆΠ΅Π½ΠΈΠ΅ прСвращаСтся Π² простоС слоТСниС. Аналогичным ΠΎΠ±Ρ€Π°Π·ΠΎΠΌ ΡƒΠΌΠ΅Π½ΡŒΡˆΠ΅Π½ΠΈΠ΅ Π²Π΄Π²ΠΎΠ΅ прСдставлСно ΠΏΡ€ΠΈΠΌΠ΅Ρ€Π½ΠΎ Π½Π° -3 Π΄Π‘. Π—Π½Π°ΠΊ минус ΡƒΠΊΠ°Π·Ρ‹Π²Π°Π΅Ρ‚ Π½Π° ΡƒΠΌΠ΅Π½ΡŒΡˆΠ΅Π½ΠΈΠ΅. Π§Ρ‚ΠΎΠ±Ρ‹ Π½Π΅ΠΌΠ½ΠΎΠ³ΠΎ ΡƒΠΏΡ€ΠΎΡΡ‚ΠΈΡ‚ΡŒ ΡΠΈΡ‚ΡƒΠ°Ρ†ΠΈΡŽ, ΠΏΡ€Π΅Π΄ΡΡ‚Π°Π²ΡŒΡ‚Π΅ ΠΌΠ½ΠΎΠΆΠΈΡ‚Π΅Π»ΠΈ 2 ΠΊΠ°ΠΊ \ (\ pm \) 3 Π΄Π‘, Π·Π½Π°ΠΊ, ΡƒΠΊΠ°Π·Ρ‹Π²Π°ΡŽΡ‰ΠΈΠΉ, ΡƒΠ²Π΅Π»ΠΈΡ‡ΠΈΠ²Π°Π΅Ρ‚Π΅ Π»ΠΈ Π²Ρ‹ (ΡƒΠΌΠ½ΠΎΠΆΠ°Π΅Ρ‚Π΅) ΠΈΠ»ΠΈ ΡƒΠΌΠ΅Π½ΡŒΡˆΠ°Π΅Ρ‚Π΅ (Π΄Π΅Π»Π΅Π½ΠΈΠ΅). Как Π²ΠΈΠ΄ΠΈΡ‚Π΅, коэффициСнт 10 Π΄Π°Π΅Ρ‚ ΠΎΡ‡Π΅Π½ΡŒ ΡƒΠ΄ΠΎΠ±Π½Ρ‹Π΅ 10 Π΄Π‘. {‘} = 10 \ times 2.{-1} \ frac {23} {10} \\ G = 199,5 \ nonumber \]

    Π‘Π»Π΅Π΄ΠΎΠ²Π°Ρ‚Π΅Π»ΡŒΠ½ΠΎ, \ (P_ {out} = 199,5 \ cdot 1 \) ΠΌΠ’Ρ‚, ΠΈΠ»ΠΈ 199,5 ΠΌΠ’Ρ‚

    Π’Ρ‹ Ρ‚Π°ΠΊΠΆΠ΅ ΠΌΠΎΠΆΠ΅Ρ‚Π΅ ΠΈΡΠΏΠΎΠ»ΡŒΠ·ΠΎΠ²Π°Ρ‚ΡŒ Ρ‚Π΅Ρ…Π½ΠΈΠΊΡƒ аппроксимации Π² ΠΎΠ±Ρ€Π°Ρ‚Π½ΠΎΠΌ порядкС. Для этого Ρ€Π°Π·Π΄Π΅Π»ΠΈΡ‚Π΅ усилСниС Π΄Π‘ Π½Π° Π±Π»ΠΎΠΊΠΈ ΠΏΠΎ 10 ΠΈ 3 Π΄Π‘:

    \ [23 \ Π΄Π‘ = 3 \ Π΄Π‘ + 10 \ Π΄Π‘ + 10 \ Π΄Π‘ \ nonumber \]

    Π’Π΅ΠΏΠ΅Ρ€ΡŒ Π·Π°ΠΌΠ΅Π½ΠΈΡ‚Π΅ ΠΊΠ°ΠΆΠ΄Ρ‹ΠΉ Ρ„Ρ€Π°Π³ΠΌΠ΅Π½Ρ‚ ΡΠΎΠΎΡ‚Π²Π΅Ρ‚ΡΡ‚Π²ΡƒΡŽΡ‰ΠΈΠΌ ΠΌΠ½ΠΎΠΆΠΈΡ‚Π΅Π»Π΅ΠΌ ΠΈ ΡƒΠΌΠ½ΠΎΠΆΡŒΡ‚Π΅ ΠΈΡ… вмСстС (ΠΏΠΎΠΌΠ½ΠΈΡ‚Π΅, Ρ‡Ρ‚ΠΎ ΠΏΡ€ΠΈ ΠΏΠ΅Ρ€Π΅Ρ…ΠΎΠ΄Π΅ ΠΎΡ‚ ΠΆΡƒΡ€Π½Π°Π»Π° ΠΊ ΠΎΠ±Ρ‹Ρ‡Π½ΠΎΠΉ Ρ„ΠΎΡ€ΠΌΠ΅ слоТСниС прСвращаСтся Π² ΡƒΠΌΠ½ΠΎΠΆΠ΅Π½ΠΈΠ΅).

    \ [3 Π΄Π‘ = 2X \ 10 \ Π΄Π‘ = 10X, \ text {so,} \\ G = 2 \ times 10 \ times 10 \\ G = 200 \ nonumber \]

    Π₯отя ΠΌΠ΅Ρ‚ΠΎΠ΄ аппроксимации каТСтся ΠΌΠ΅Π΄Π»Π΅Π½Π½Π΅Π΅, Ρ‡Π΅ΠΌ ΠΊΠ°Π»ΡŒΠΊΡƒΠ»ΡΡ‚ΠΎΡ€, ΠΏΡ€Π°ΠΊΡ‚ΠΈΠΊΠ° ΠΏΠΎΠΊΠ°ΠΆΠ΅Ρ‚ ΠΎΠ±Ρ€Π°Ρ‚Π½ΠΎΠ΅.{‘} _ {total} = 40 \ Π΄Π‘ \ nonumber \]

    Как Π²Ρ‹ ΠΌΠΎΠ³Π»ΠΈ Π·Π°ΠΌΠ΅Ρ‚ΠΈΡ‚ΡŒ, Π²ΠΎ всСх ΠΏΡ€ΠΈΠΌΠ΅Ρ€Π°Ρ… Π΄ΠΎ этого ΠΌΠΎΠΌΠ΅Π½Ρ‚Π° использовалось усилСниС ΠΏΠΎ мощности, Π° Π½Π΅ ΠΏΠΎ Π½Π°ΠΏΡ€ΡΠΆΠ΅Π½ΠΈΡŽ. Π£ вас ΠΌΠΎΠΆΠ΅Ρ‚ Π²ΠΎΠ·Π½ΠΈΠΊΠ½ΡƒΡ‚ΡŒ соблазн ΠΈΡΠΏΠΎΠ»ΡŒΠ·ΠΎΠ²Π°Ρ‚ΡŒ Ρ‚Π΅ ΠΆΠ΅ уравнСния для усилСния напряТСния. Одним словом, НЕ. Если Π²Ρ‹ Π½Π° ΠΌΠ³Π½ΠΎΠ²Π΅Π½ΠΈΠ΅ вспомнитС, Π²Ρ‹ вспомнитС, Ρ‡Ρ‚ΠΎ ΠΌΠΎΡ‰Π½ΠΎΡΡ‚ΡŒ измСняСтся ΠΊΠ°ΠΊ ΠΊΠ²Π°Π΄Ρ€Π°Ρ‚ напряТСния. Π”Ρ€ΡƒΠ³ΠΈΠΌΠΈ словами, ΡƒΠ΄Π²ΠΎΠ΅Π½ΠΈΠ΅ напряТСния ΠΏΡ€ΠΈΠ²Π΅Π΄Π΅Ρ‚ ΠΊ Ρ‡Π΅Ρ‚Ρ‹Ρ€Π΅Ρ…ΠΊΡ€Π°Ρ‚Π½ΠΎΠΌΡƒ ΡƒΠ²Π΅Π»ΠΈΡ‡Π΅Π½ΠΈΡŽ мощности. Если Π±Ρ‹ Π²Ρ‹ использовали Ρ‚Π΅ ΠΆΠ΅ прСобразования Π΄Π‘, ΡƒΠ΄Π²ΠΎΠ΅Π½ΠΈΠ΅ напряТСния составило Π±Ρ‹ 3 Π΄Π‘, Π½ΠΎ, ΠΏΠΎΡΠΊΠΎΠ»ΡŒΠΊΡƒ ΠΌΠΎΡ‰Π½ΠΎΡΡ‚ΡŒ ΡƒΠ²Π΅Π»ΠΈΡ‡ΠΈΠ»Π°ΡΡŒ Π² Ρ‡Π΅Ρ‚Ρ‹Ρ€Π΅ Ρ€Π°Π·Π°, это ΡƒΠΊΠ°Π·Ρ‹Π²Π°Π»ΠΎ Π±Ρ‹ Π½Π° ΠΏΠΎΠ²Ρ‹ΡˆΠ΅Π½ΠΈΠ΅ Π½Π° 6 Π΄Π‘.{‘} _v = 20 \ \ log_ {10} \ A_v \ label {1.2} \]

    Однако Π±ΡƒΠ΄ΡŒΡ‚Π΅ остороТны, усилСниС напряТСния Bel равняСтся ΡƒΡΠΈΠ»Π΅Π½ΠΈΡŽ мощности Bel Ρ‚ΠΎΠ»ΡŒΠΊΠΎ Π² Ρ‚ΠΎΠΌ случаС, Ссли Π²Ρ…ΠΎΠ΄Π½ΠΎΠ΅ ΠΈ Π²Ρ‹Ρ…ΠΎΠ΄Π½ΠΎΠ΅ сопротивлСниС систСмы согласованы (Π²Ρ‹, Π²ΠΎΠ·ΠΌΠΎΠΆΠ½ΠΎ, ΠΏΠΎΠΌΠ½ΠΈΡ‚Π΅ ΠΈΠ· своСй ΠΏΡ€Π΅Π΄Ρ‹Π΄ΡƒΡ‰Π΅ΠΉ Ρ€Π°Π±ΠΎΡ‚Ρ‹, Ρ‡Ρ‚ΠΎ Π²ΠΏΠΎΠ»Π½Π΅ Π²ΠΎΠ·ΠΌΠΎΠΆΠ½ΠΎ ΡΠΏΡ€ΠΎΠ΅ΠΊΡ‚ΠΈΡ€ΠΎΠ²Π°Ρ‚ΡŒ схСму с сильно Ρ€Π°Π·Π»ΠΈΡ‡Π°ΡŽΡ‰ΠΈΠΌΡΡ коэффициСнтом усилСния напряТСния ΠΈ мощности. . ΠŸΠΎΠ²Ρ‚ΠΎΡ€ΠΈΡ‚Π΅Π»ΡŒ ΠΏΠΎ Π½Π°ΠΏΡ€ΡΠΆΠ΅Π½ΠΈΡŽ, Π½Π°ΠΏΡ€ΠΈΠΌΠ΅Ρ€, дСмонстрируСт ΡƒΠΌΠ΅Ρ€Π΅Π½Π½Ρ‹ΠΉ прирост мощности с коэффициСнтом усилСния ΠΏΠΎ Π½Π°ΠΏΡ€ΡΠΆΠ΅Π½ΠΈΡŽ, Ρ€Π°Π²Π½Ρ‹ΠΌ Π΅Π΄ΠΈΠ½ΠΈΡ†Π΅. Π’ΠΏΠΎΠ»Π½Π΅ вСроятно, Ρ‡Ρ‚ΠΎ ΠΏΠΎΠ²Ρ‚ΠΎΡ€ΠΈΡ‚Π΅Π»ΡŒ Π½Π΅ Π±ΡƒΠ΄Π΅Ρ‚ Π΄Π΅ΠΌΠΎΠ½ΡΡ‚Ρ€ΠΈΡ€ΠΎΠ²Π°Ρ‚ΡŒ согласованныС импСдансы.) Если Π±Ρ‹ ΠΌΡ‹ пСрСсчитали Π½Π°ΡˆΡƒ ΠΏΡ€Π΅Π΄Ρ‹Π΄ΡƒΡ‰ΡƒΡŽ Ρ‚Π°Π±Π»ΠΈΡ†Ρƒ ΠΎΠ±Ρ‰ΠΈΡ… коэффициСнтов, ΠΌΡ‹ Π±Ρ‹ ΠΎΠ±Π½Π°Ρ€ΡƒΠΆΠΈΠ»ΠΈ, Ρ‡Ρ‚ΠΎ ΡƒΠ΄Π²ΠΎΠ΅Π½ΠΈΠ΅ усилСния ΠΏΠΎ Π½Π°ΠΏΡ€ΡΠΆΠ΅Π½ΠΈΡŽ эквивалСнтно ΡƒΠ²Π΅Π»ΠΈΡ‡Π΅Π½ΠΈΡŽ Π½Π° 6 Π΄Π‘, Π° дСсятикратноС ΡƒΠ²Π΅Π»ΠΈΡ‡Π΅Π½ΠΈΠ΅ эквивалСнтно ΡƒΠ²Π΅Π»ΠΈΡ‡Π΅Π½ΠΈΡŽ Π½Π° 20 Π΄Π‘, Ρ‡Ρ‚ΠΎ Π²Π΄Π²ΠΎΠ΅ ΠΏΡ€Π΅Π²Ρ‹ΡˆΠ°Π΅Ρ‚ Ρ€Π°Π·ΠΌΠ΅Ρ€ ΠΈΡ… Π°Π½Π°Π»ΠΎΠ³ΠΎΠ² ΠΏΠΎ ΡƒΡΠΈΠ»Π΅Π½ΠΈΡŽ мощности.{-1} \ frac {26} {20} \\ A_v = 19.95 \\ V_ {out} = A_v \ V_ {in} \\ V_ {out} = 19.95 \ times 10 \ mV \ nonumber \]

    ПослСдний ΠΌΠΎΠΌΠ΅Π½Ρ‚, ΠΊΠΎΡ‚ΠΎΡ€Ρ‹ΠΉ слСдуСт ΠΎΡ‚ΠΌΠ΅Ρ‚ΠΈΡ‚ΡŒ Π² этом Ρ€Π°Π·Π΄Π΅Π»Π΅, Π·Π°ΠΊΠ»ΡŽΡ‡Π°Π΅Ρ‚ΡΡ Π² Ρ‚ΠΎΠΌ, Ρ‡Ρ‚ΠΎ, ΠΊΠ°ΠΊ ΠΈ Π² случаС усилСния мощности, ΠΎΡ‚Ρ€ΠΈΡ†Π°Ρ‚Π΅Π»ΡŒΠ½ΠΎΠ΅ Π·Π½Π°Ρ‡Π΅Π½ΠΈΠ΅ Π² Π΄Π‘ ΡƒΠΊΠ°Π·Ρ‹Π²Π°Π΅Ρ‚ Π½Π° ΠΏΠΎΡ‚Π΅Ρ€ΡŽ. Π‘Π»Π΅Π΄ΠΎΠ²Π°Ρ‚Π΅Π»ΡŒΠ½ΠΎ, Π΄Π΅Π»ΠΈΡ‚Π΅Π»ΡŒ напряТСния 2: 1 Π±ΡƒΠ΄Π΅Ρ‚ ΠΈΠΌΠ΅Ρ‚ΡŒ коэффициСнт усилСния -6 Π΄Π‘.

    1.2.2: ΠŸΡ€Π΅Π΄ΡΡ‚Π°Π²Π»Π΅Π½ΠΈΠ΅ сигнала Π² Π΄Π‘Π’Ρ‚ ΠΈ Π΄Π‘Π’

    Как Π²ΠΈΠ΄Π½ΠΎ ΠΈΠ· ΠΏΡ€Π΅Π΄Ρ‹Π΄ΡƒΡ‰Π΅Π³ΠΎ Ρ€Π°Π·Π΄Π΅Π»Π°, ΠΌΠΎΠΆΠ½ΠΎ ΠΏΠΎΡ‚Ρ€Π°Ρ‚ΠΈΡ‚ΡŒ Π·Π½Π°Ρ‡ΠΈΡ‚Π΅Π»ΡŒΠ½ΠΎΠ΅ врСмя Π½Π° ΠΏΡ€Π΅ΠΎΠ±Ρ€Π°Π·ΠΎΠ²Π°Π½ΠΈΠ΅ ΠΌΠ΅ΠΆΠ΄Ρƒ усилСниСм Π΄Π΅Ρ†ΠΈΠ±Π΅Π» ΠΈ ΠΎΠ±Ρ‹Ρ‡Π½Ρ‹ΠΌΠΈ напряТСниями ΠΈ мощностями.ΠŸΠΎΡΠΊΠΎΠ»ΡŒΠΊΡƒ Ρ„ΠΎΡ€ΠΌΠ° Π² Π΄Π΅Ρ†ΠΈΠ±Π΅Π»Π°Ρ… Π΄Π΅ΠΉΡΡ‚Π²ΠΈΡ‚Π΅Π»ΡŒΠ½ΠΎ Π΄Π°Π΅Ρ‚ прСимущСства ΠΏΡ€ΠΈ ΠΈΠ·ΠΌΠ΅Ρ€Π΅Π½ΠΈΠΈ усилСния, Π±Ρ‹Π»ΠΎ Π±Ρ‹ Ρ€Π°Π·ΡƒΠΌΠ½ΠΎ ΠΈΡΠΏΠΎΠ»ΡŒΠ·ΠΎΠ²Π°Ρ‚ΡŒ Ρ„ΠΎΡ€ΠΌΡƒ Π² Π΄Π΅Ρ†ΠΈΠ±Π΅Π»Π°Ρ… Ρ‚Π°ΠΊΠΆΠ΅ для ΡƒΡ€ΠΎΠ²Π½Π΅ΠΉ мощности ΠΈ напряТСния. Π­Ρ‚ΠΎ ΠΎΡ‚Π½ΠΎΡΠΈΡ‚Π΅Π»ΡŒΠ½ΠΎ простой процСсс. НСт ΠΏΡ€ΠΈΡ‡ΠΈΠ½, ΠΏΠΎ ΠΊΠΎΡ‚ΠΎΡ€Ρ‹ΠΌ ΠΌΡ‹ Π½Π΅ ΠΌΠΎΠΆΠ΅ΠΌ Π²Ρ‹Ρ€Π°Π·ΠΈΡ‚ΡŒ ΠΌΠΎΡ‰Π½ΠΎΡΡ‚ΡŒ ΠΈΠ»ΠΈ напряТСниС Π² логарифмичСской Ρ„ΠΎΡ€ΠΌΠ΅. ΠŸΠΎΡΠΊΠΎΠ»ΡŒΠΊΡƒ Π·Π½Π°Ρ‡Π΅Π½ΠΈΠ΅ Π² Π΄Π‘ просто ΡƒΠΊΠ°Π·Ρ‹Π²Π°Π΅Ρ‚ Π½Π° ΡΠΎΠΎΡ‚Π½ΠΎΡˆΠ΅Π½ΠΈΠ΅, всС, Ρ‡Ρ‚ΠΎ Π½Π°ΠΌ Π½ΡƒΠΆΠ½ΠΎ ΡΠ΄Π΅Π»Π°Ρ‚ΡŒ, это Π²Ρ‹Π±Ρ€Π°Ρ‚ΡŒ эталон (Ρ‚ΠΎ Π΅ΡΡ‚ΡŒ ΡΡ€Π°Π²Π½ΠΈΡ‚Π΅Π»ΡŒΠ½ΡƒΡŽ Π±Π°Π·Ρƒ для ΠΎΡ‚Π½ΠΎΡˆΠ΅Π½ΠΈΡ). Для измСрСния мощности вСроятным Π²Ρ‹Π±ΠΎΡ€ΠΎΠΌ Π±ΡƒΠ΄Π΅Ρ‚ 1 Π²Π°Ρ‚Ρ‚. Π”Ρ€ΡƒΠ³ΠΈΠΌΠΈ словами, ΠΌΡ‹ ΠΌΠΎΠΆΠ΅ΠΌ ΠΎΠΏΠΈΡΠ°Ρ‚ΡŒ ΠΌΠΎΡ‰Π½ΠΎΡΡ‚ΡŒ ΠΊΠ°ΠΊ ΠΎΠΏΡ€Π΅Π΄Π΅Π»Π΅Π½Π½ΠΎΠ΅ количСство Π΄Π‘ Π²Ρ‹ΡˆΠ΅ ΠΈΠ»ΠΈ Π½ΠΈΠΆΠ΅ 1 Π²Π°Ρ‚Ρ‚Π°.{‘} = 20,8 \ Π΄Π‘Π’Ρ‚ \ nonumber \]

    НСт Π½ΠΈΡ‡Π΅Π³ΠΎ святого Π² 1-Π²Π°Ρ‚Ρ‚Π½ΠΎΠΌ эталонС, Ссли Π½Π΅ ΡΡ‡ΠΈΡ‚Π°Ρ‚ΡŒ Π΅Π³ΠΎ удобства. ΠœΡ‹ ΠΌΠΎΠ³Π»ΠΈ Π±Ρ‹ Ρ‚Π°ΠΊ ΠΆΠ΅ Π»Π΅Π³ΠΊΠΎ Π²Ρ‹Π±Ρ€Π°Ρ‚ΡŒ Π΄Ρ€ΡƒΠ³ΡƒΡŽ ссылку. Π”Ρ€ΡƒΠ³ΠΈΠ΅ ΠΎΠ±Ρ‰ΠΈΠ΅ ΠΊΠΎΠ½Ρ‚Ρ€ΠΎΠ»ΡŒΠ½Ρ‹Π΅ Ρ‚ΠΎΡ‡ΠΊΠΈ — 1 ΠΌΠΈΠ»Π»ΠΈΠ²Π°Ρ‚Ρ‚ (Π΄Π‘ΠΌ) ΠΈ 1 Ρ„Π΅ΠΌΡ‚ΠΎΠ²Π°Ρ‚Ρ‚ (Π΄Π‘Ρ„). ΠžΡ‡Π΅Π²ΠΈΠ΄Π½ΠΎ, Ρ‡Ρ‚ΠΎ dBf ΠΈΡΠΏΠΎΠ»ΡŒΠ·ΡƒΠ΅Ρ‚ΡΡ для ΠΎΡ‡Π΅Π½ΡŒ Π½ΠΈΠ·ΠΊΠΈΡ… ΡƒΡ€ΠΎΠ²Π½Π΅ΠΉ сигнала, Π½Π°ΠΏΡ€ΠΈΠΌΠ΅Ρ€, исходящих ΠΎΡ‚ Π°Π½Ρ‚Π΅Π½Π½Ρ‹. Π΄Π‘ΠΌ ΠΎΡ‡Π΅Π½ΡŒ ΡˆΠΈΡ€ΠΎΠΊΠΎ ΠΈΡΠΏΠΎΠ»ΡŒΠ·ΡƒΠ΅Ρ‚ΡΡ Π² индустрии связи. Π§Ρ‚ΠΎΠ±Ρ‹ ΠΈΡΠΏΠΎΠ»ΡŒΠ·ΠΎΠ²Π°Ρ‚ΡŒ эти Π΄Ρ€ΡƒΠ³ΠΈΠ΅ ссылки, просто Ρ€Π°Π·Π΄Π΅Π»ΠΈΡ‚Π΅ Π΄Π°Π½Π½ΡƒΡŽ ΡΡ‚Π΅ΠΏΠ΅Π½ΡŒ Π½Π° Π½ΠΎΠ²ΡƒΡŽ ссылку.

    ΠŸΡ€ΠΈΠΌΠ΅Ρ€ \ (\ PageIndex {8} \)

    НСбольшой ΠΏΠ΅Ρ€ΡΠΎΠ½Π°Π»ΡŒΠ½Ρ‹ΠΉ Π°ΡƒΠ΄ΠΈΠΎΠΏΠ»Π΅Π΅Ρ€ Π²Ρ‹Π΄Π°Π΅Ρ‚ Π½Π° Π½Π°ΡƒΡˆΠ½ΠΈΠΊΠΈ ΠΌΠΎΡ‰Π½ΠΎΡΡ‚ΡŒ 200 ΠΌΠ’Ρ‚.{‘} _ {out} = -7 \ dBV \ nonumber \]

    ΠžΠ±Ρ€Π°Ρ‚ΠΈΡ‚Π΅ Π²Π½ΠΈΠΌΠ°Π½ΠΈΠ΅, Ρ‡Ρ‚ΠΎ ΠΊΠΎΠ½Π΅Ρ‡Π½Ρ‹ΠΌΠΈ Π΅Π΄ΠΈΠ½ΠΈΡ†Π°ΠΌΠΈ измСрСния ΡΠ²Π»ΡΡŽΡ‚ΡΡ Π΄Π‘Π’, Π° Π½Π΅ Π΄Π‘, Ρ‡Ρ‚ΠΎ ΡƒΠΊΠ°Π·Ρ‹Π²Π°Π΅Ρ‚ Π½Π° напряТСниС, Π° Π½Π΅ Ρ‚ΠΎΠ»ΡŒΠΊΠΎ Π½Π° усилСниС.

    ΠŸΡ€ΠΈΠΌΠ΅Ρ€ \ (\ PageIndex {12} \)

    Π“ΠΈΡ‚Π°Ρ€Π½ΠΎΠΌΡƒ ΡƒΡΠΈΠ»ΠΈΡ‚Π΅Π»ΡŽ мощности трСбуСтся входная ΠΌΠΎΡ‰Π½ΠΎΡΡ‚ΡŒ 20 Π΄Π‘ΠΌΠ’Ρ‚ для достиТСния Π²Ρ‹Ρ…ΠΎΠ΄Π½ΠΎΠΉ мощности 25 Π΄Π‘Π’Ρ‚. Какой коэффициСнт усилСния усилитСля Π² Π΄Π‘?

    Π’ΠΎ-ΠΏΠ΅Ρ€Π²Ρ‹Ρ…, Π½Π΅ΠΎΠ±Ρ…ΠΎΠ΄ΠΈΠΌΠΎ ΠΏΡ€Π΅ΠΎΠ±Ρ€Π°Π·ΠΎΠ²Π°Ρ‚ΡŒ показания мощности Ρ‚Π°ΠΊ, Ρ‡Ρ‚ΠΎΠ±Ρ‹ ΠΎΠ½ΠΈ использовали ΠΎΠ΄Π½Ρƒ ΠΈ Ρ‚Ρƒ ΠΆΠ΅ ΡΡ‚Π°Π»ΠΎΠ½Π½ΡƒΡŽ Π΅Π΄ΠΈΠ½ΠΈΡ†Ρƒ. ΠŸΠΎΡΠΊΠΎΠ»ΡŒΠΊΡƒ Π΄Π‘ΠΌ прСдставляСт собой эталон Π½Π° 30 Π΄Π‘ мСньшС эталонного Π΄Π‘Π’Ρ‚, просто Π²Ρ‹Ρ‡Ρ‚ΠΈΡ‚Π΅ 30 Π΄Π‘ для компСнсации.{‘} = 35 \ Π΄Π‘ \ nonumber \]

    ΠžΠ±Ρ€Π°Ρ‚ΠΈΡ‚Π΅ Π²Π½ΠΈΠΌΠ°Π½ΠΈΠ΅, Ρ‡Ρ‚ΠΎ Π΅Π΄ΠΈΠ½ΠΈΡ†Ρ‹ измСрСния — Π΄Π‘, Π° Π½Π΅ Π΄Π‘Π’Ρ‚. Π­Ρ‚ΠΎ ΠΎΡ‡Π΅Π½ΡŒ Π²Π°ΠΆΠ½ΠΎ! Π‘ΠΊΠ°Π·Π°Ρ‚ΡŒ, Ρ‡Ρ‚ΠΎ усилСниС составляСт Β«ΡΡ‚ΠΎΠ»ΡŒΠΊΠΎΒ» Π΄Π‘Π’Ρ‚, Ρ‚ΠΎ ΠΆΠ΅ самоС, Ρ‡Ρ‚ΠΎ ΡΠΊΠ°Π·Π°Ρ‚ΡŒ, Ρ‡Ρ‚ΠΎ усилСниС Β«ΡΡ‚ΠΎΠ»ΡŒΠΊΠΎΒ» Π²Π°Ρ‚Ρ‚. ΠžΡ‡Π΅Π²ΠΈΠ΄Π½ΠΎ, Ρ‡Ρ‚ΠΎ прирост — это «чистыС» числа ΠΈ Π½Π΅ содСрТат Ρ‚Π°ΠΊΠΈΡ… Π΅Π΄ΠΈΠ½ΠΈΡ†, ΠΊΠ°ΠΊ Π²Π°Ρ‚Ρ‚Ρ‹ ΠΈΠ»ΠΈ Π²ΠΎΠ»ΡŒΡ‚Ρ‹.

    ИспользованиС систСмы Π½Π° основС Π΄Π‘ графичСски ΠΏΠΎΠΊΠ°Π·Π°Π½ΠΎ Π½Π° рисункС \ (\ PageIndex {1} \). ΠžΠ±Ρ€Π°Ρ‚ΠΈΡ‚Π΅ Π²Π½ΠΈΠΌΠ°Π½ΠΈΠ΅, ΠΊΠ°ΠΊ усилСниС каскада добавляСтся ΠΊ Π²Ρ…ΠΎΠ΄Π½ΠΎΠΌΡƒ сигналу для формирования Π²Ρ‹Ρ…ΠΎΠ΄Π½ΠΎΠ³ΠΎ сигнала. Π’ этой Ρ„ΠΎΡ€ΠΌΠ΅ ΠΌΠΎΠΆΠ½ΠΎ быстро ΠΏΡ€ΠΎΠ°Π½Π°Π»ΠΈΠ·ΠΈΡ€ΠΎΠ²Π°Ρ‚ΡŒ Π΄Π°ΠΆΠ΅ большиС схСмы.Π§Ρ‚ΠΎΠ±Ρ‹ ΡΠ΄Π΅Π»Π°Ρ‚ΡŒ Тизнь Π² Π»Π°Π±ΠΎΡ€Π°Ρ‚ΠΎΡ€ΠΈΠΈ Π΅Ρ‰Π΅ ΠΏΡ€ΠΎΡ‰Π΅, ΠΌΠΎΠΆΠ½ΠΎ ΠΏΡ€ΠΎΠ²ΠΎΠ΄ΠΈΡ‚ΡŒ измСрСния нСпосрСдствСнно Π² Ρ„ΠΎΡ€ΠΌΠ΅ Π΄Π‘. ΠŸΠΎΡΡ‚ΡƒΠΏΠ°Ρ Ρ‚Π°ΠΊΠΈΠΌ ΠΎΠ±Ρ€Π°Π·ΠΎΠΌ, Π²Π°ΠΌ Π½ΠΈΠΊΠΎΠ³Π΄Π° Π½Π΅ придСтся Π²Ρ‹ΠΏΠΎΠ»Π½ΡΡ‚ΡŒ ΠΊΠΎΠ½Π²Π΅Ρ€Ρ‚Π°Ρ†ΠΈΡŽ ΠΏΡ€ΠΈ поискС ΠΈ устранСнии нСисправностСй Π² Π΄ΠΈΠ·Π°ΠΉΠ½Π΅.

    Π”ΠΎΠ±Π°Π²ΠΈΡ‚ΡŒ ΠΊΠΎΠΌΠΌΠ΅Π½Ρ‚Π°Ρ€ΠΈΠΉ

    Π’Π°Ρˆ адрСс email Π½Π΅ Π±ΡƒΠ΄Π΅Ρ‚ ΠΎΠΏΡƒΠ±Π»ΠΈΠΊΠΎΠ²Π°Π½. ΠžΠ±ΡΠ·Π°Ρ‚Π΅Π»ΡŒΠ½Ρ‹Π΅ поля ΠΏΠΎΠΌΠ΅Ρ‡Π΅Π½Ρ‹ *